DDSEP 9 MCQs
DDSEP 9 MCQs
CHAPTER 1
Esophageal disorders
C. Prakash Gyawali, MD, MRCP, AGAF and
Vani Konda, MD
1
2 Digestive Diseases Self-Education Program®
al. Esophageal sensitivity to acid in patients with ence of reflux or motor disorders as a mechanism
Barrett’s esophagus is not related to preserved for symptoms. A barium esophagram does not add
esophageal mucosal integrity. Neurogastroenterol much to the evaluation of chest pain in the ab-
Motil 2017;29. sence of dysphagia.
Fass R, Naliboff BD, Fass SS, et al. The effect
of auditory stress on perception of intraesophageal REFERENCES
acid in patients with gastroesophageal reflux dis- Roman S, Gyawali CP, Savarino E, et al. Ambula-
ease. Gastroenterology. 2008 Mar;134(3):696-705. tory reflux monitoring for diagnosis of gastro-
esophageal reflux disease: Update of the Porto
consensus and recommendations from an interna-
Question 5 tional consensus group. Neurogastroenterol Motil.
A 50-year-old obese man presents for further 2017 Oct;29(10):1-15.
evaluation of a three month history of retrosternal Fass R, Achem SR. Noncardiac chest pain:
chest pain. He reports a heavy, pressure-like pain diagnostic evaluation. Dis Esophagus. 2012
in the chest that sometimes improves with belch- Feb;25(2):89-101.
ing or with an oral antacid. He has not noticed
food triggers, reports it is not temporally related
to meals, and has no nocturnal pain. He has no Question 6
regurgitation. He has tried an H2 receptor antago- Which one of the following medications can in-
nist without benefit. Prior to referral, he under- crease likelihood of symptomatic gastroesophageal
went an upper endoscopy with biopsy which was reflux?
normal. Physical examination is normal.
A. Alendronate
Which of the following is the most appropriate B. Baclofen
next step? C. Metoprolol
D. Naproxen
A. Initiate PPI trial with omeprazole 40 mg bid E. Theophylline
B. Perform 96 hour wireless pH monitoring
C. Perform a barium esophagram CORRECT ANSWER: E
D. Perform esophageal manometry
E. Refer to a cardiologist RATIONALE
Theophylline can reduce basal LES pressure,
CORRECT ANSWER: E and can promote symptomatic gastroesophageal
reflux. Baclofen, on the other hand, reduces tran-
RATIONALE sient LES relaxations, and reduces reflux episodes.
The patient is male, middle aged, and obese, which Beta blockers do not impact reflux, but beta adren-
are all risk factors for cardiac disease. Symptoms ergic agonists can increase reflux. Naproxen and
are atypical for reflux disease, and typical triggers alendronate are irritants to the esophageal mucosa
for reflux are not reported. In individuals at risk if retained in the esophagus, but do not increase
for heart disease, cardiac evaluation takes pre- reflux tendency.
cedence over evaluation for reflux disease. A PPI
trial would delay cardiac evaluation and therefore REFERENCE
would not be appropriate as an initial approach. If Mungan Z, Pınarbaşı Şimşek B. Which drugs are
cardiac disease were to be excluded, prolonged pH risk factors for the development of gastroesopha-
monitoring off PPI and esophageal manometry are geal reflux disease? Turk J Gastroenterol. 2017
appropriate tests to establish or refute the pres- Dec;28(Suppl 1):S38-S43.
4 Digestive Diseases Self-Education Program®
Question 7 D. Obesity
Which of the following is a consequence of re- E. Older age
duced saliva production following radiation for
head and neck cancer? CORRECT ANSWER: A
non-cardiac chest pain, and requests that he be ern diagnosis of GERD: the Lyon Consensus. Gut.
evaluated for the presence of GERD. He reports 2018 Jul;67(7):1351-1362.
postprandial heartburn three to five times a week.
He has no other esophageal symptoms, and no
alarm features. Upper endoscopy demonstrates Question 10
a 3 cm segment with salmon colored mucosa, A 25-year-old gentleman has heartburn and acid
and biopsies demonstrate intestinal metaplasia regurgitation several times a week. An upper
without dysplasia. There is a three cm axial hiatus endoscopy is completely normal, and no hiatus
hernia. An esophageal high resolution manometry hernia is seen. Esophageal biopsies from the distal
confirms the presence of the hiatus hernia, and esophagus reveal papillary elongation and basal
also demonstrates ineffective esophageal motil- cell hyperplasia. A prolonged wireless pH study
ity, without contraction reserve. A pool of yellow- demonstrates acid exposure time of six to eight
ish fluid is noted in the gastric fundus, which the percent on each day of the four day study. A bar-
endoscopist reports as the presence of bile. ium swallow demonstrates ‘reflux to the thoracic
inlet’. He is concerned about taking medications,
Which of the following features is most suggestive and wants to know if he absolutely needs to start
of a diagnosis of GERD? PPI therapy.
A. Bile in the gastric fundus Among patients with non-erosive reflux disease,
B. Hiatus hernia which of the following factors predicts symptom
C. Ineffective esophageal manometry improvement with proton pump inhibitor
D. Intestinal metaplasia therapy?
E. Post-prandial heartburn
A. Abnormal esophageal biopsy
CORRECT ANSWER: D B. Abnormal pH study
C. Absence of hiatus hernia
RATIONALE D. Presence of acid regurgitation
Biopsy proven intestinal metaplasia on biopsies E. Reflux seen on barium swallow
taken from the distal esophagus is conclusive for a
diagnosis of GERD, and correlates well with distal CORRECT ANSWER: B
esophageal acid exposure time. While hiatus her-
nia and ineffective esophageal motility may both RATIONALE
predict a higher esophageal acid burden, these are Abnormal acid exposure on pH monitoring inde-
not pathognomonic for GERD, and can be seen pendently predicts response to antireflux therapy,
without significant GERD as well as in healthy including PPI therapy. Microscopic esophagitis
asymptomatic controls. Heartburn as a symptom on routine esophageal biopsies is not predictive
has only modest sensitivity and specificity for of PPI response in nonerosive reflux disease,
GERD. Bile is seen normally in the gastric fundus, as there is overlap with reflux hypersensitivity,
and does not establish the presence of GERD. which is a functional disorder. The presence of
While each of these features (other than intestinal a hiatus hernia can be associated with higher
metaplasia) by itself does not establish GERD, esophageal acid exposure times. Regurgitation as
these features in combination increase confidence a symptom does not improve as well as heartburn
in a GERD diagnosis. with acid suppression. Barium swallow finding
of reflux does not correlate with symptoms, acid
REFERENCE burden on pH testing, or symptom response to
Gyawali CP, Kahrilas PJ, Savarino E, et al. Mod- anti-reflux therapy.
6 Digestive Diseases Self-Education Program®
Question 11
Which of the following can falsely elevate acid ex- Question 12
posure time with wireless pH monitoring studies? A 33-year-old lady presents with retrosternal
burning for the past five years. She has been taking
A. Acidic food/beverage swallows a PPI once a day before breakfast for two years,
B. Alkaline reflux but continues to have burning two to three times
C. Antacid use a day. Taking an antacid does not provide benefit.
D. Esophageal shortening She has no alarm symptoms. She underwent an
E. Post nasal drip upper endoscopy while on PPI a year ago, which
was normal, including biopsies.
CORRECT ANSWER: A
Which of the following is the most appropriate
RATIONALE next step?
Esophageal pH monitoring is often used to clarify
esophageal reflux burden, especially when esoph- A. Barium esophagram with a solid barium bolus
ageal symptoms persist despite acid suppression. B. Dual phase gastric emptying study
Wireless pH monitoring has a single recording C. High resolution esophageal impedance
site on a pH capsule placed in the esophagus manometry
six cm proximal to the endoscopically identified D. PH-impedance monitoring on her current
squamocolumnar junction. Since there is a single PPI dose
site of pH recording, acidic swallows cannot be E. Repeat endoscopy with wireless pH
differentiated from acid reflux. A careful diary is monitoring off PPI
kept by the patient, which is reconciled with the
pH recording. An oral intake is excluded from CORRECT ANSWER: E
analysis to avoid confounding of the acid expo-
sure time from acidic swallows. Alkaline reflux RATIONALE
is not detected with wireless pH monitoring, and The patient has retrosternal burning that has not
would not increase acid exposure time. Antacids responded to daily PPI therapy. She has no prior
neutralize gastric acid, and can potentially reduce conclusive evidence of GERD, and therefore has
acid exposure time. Esophageal shortening does unproven GERD. This is an indication for pH
not impact pH assessment using a wireless cap- monitoring performed off PPI therapy, to estab-
sule, since the capsule is attached to the esopha- lish or refute the presence of GERD. Among the
geal wall, but can impact catheter based pH options provided, a repeat endoscopy with pro-
recordings, where an improperly placed catheter longed wireless pH monitoring off PPI is the most
Chapter 1 — Esophageal disorders 7
appropriate option, where abnormal acid exposure five. In contrast, the number needed to treat for a
time will reinforce the need for PPI therapy, and heartburn response is less than two. Regurgitation
physiologic acid exposure time will suggest an al- does not respond well to acid suppression, and can
ternate mechanism for symptoms. pH-impedance persist despite adequate heartburn response in
monitoring on PPI therapy is indicated in patients patients with typical reflux symptoms. Less than
with proven GERD (prior esophagitis, peptic stric- 20 percent of patients with cough and hoarseness
ture or Barrett’s esophagus on endoscopy, prior report response to PPI therapy, and response is
abnormal pH study), where the intent of reflux even lower with globus sensation. Consequently,
monitoring is to establish mechanism of ongoing the more atypical the symptom, the higher the
symptoms. Barium radiography with solid barium need for ambulatory reflux monitoring off PPI to
bolus is useful in the evaluation of esophageal rule in or rule out reflux disease, since positive pH
dysphagia. High resolution impedance manom- testing can predict treatment response to PPI.
etry is performed for unexplained dysphagia, and
sometimes for persisting esophageal symptoms, REFERENCE
but an ambulatory pH study will have higher yield Gyawali CP, Fass R. Management of Gastroesoph-
in the evaluation of heartburn. A gastric emptying ageal Reflux Disease. Gastroenterology. 2018
study will not help initial evaluation of retrosternal Jan;154(2):302-318.
burning.
REFERENCE Question 14
Savarino E, Bredenoord AJ, Fox M, et al. Advanc- A 45-year-old man presents with post prandial
es in the physiologic assessment and diagnosis regurgitation. Initially, he had heartburn and
of gastroesophageal reflux disease: A consensus regurgitation, and an endoscopy performed prior
statement. Nature Reviews Gastroenterol Hepatol to PPI initiation demonstrated erosive esophagitis
2017;14(11):665-676. (LA grade C). He was given PPI twice a day in ad-
dition to lifestyle modification, which improved his
heartburn, but regurgitation persists. He has no
Question 13 dysphagia. Repeat endoscopy three months later
Among the following symptoms, which one demonstrates resolution of his esophagitis, and no
is most likely to respond to a trial of PPI evidence of Barrett’s esophagus. He has a three cm
therapy? intermittent sliding hiatus hernia.
persist despite PPI therapy. Ambulatory reflux with a suboptimal response of reflux symptoms
monitoring is indicated to evaluate the mechanism to standard reflux therapy. Affective disorders
of persisting regurgitation, and is performed on including anxiety and depression are also associ-
maximal PPI therapy. If regurgitation correlates ated with an incomplete response to PPI therapy,
with reflux episodes, if a high number of reflux possibly related to visceral hypersensitivity and
episodes are identified, or if acid exposure time is hypervigilance. In contrast, high esophageal acid
elevated, escalation of management can be consid- exposure times, and presence of erosive esophagi-
ered, and the patient can be offered antireflux sur- tis are associated with better symptom responses
gery, especially since he also has a hiatus hernia. compared to lack of conclusive evidence for reflux
Behavioral therapy (e.g. diaphragmatic breathing) on esophageal testing. PPIs work best when ad-
can be offered if rumination is suspected. Esopha- ministered 30-45 min before the first meal of the
geal manometry may be performed to assess peri- day. The S-isomer of omeprazole (esomeprazole)
staltic performance prior to antireflux surgery, and has better bioavailability because of slower elimi-
barium swallow can define anatomic relationships nation, and may provide better symptom response
at the esophagogastric junction, but these tests in equivalent doses.
would not evaluate whether his current symp-
toms are due to incomplete response to therapy. REFERENCES
A gastric emptying study can determine if gastro- Weijenborg PW, Cremonini F, Smout AJ, Brede-
paresis could be contributing to regurgitation if noord AJ. PPI therapy is equally effective in well-
pH impedance testing is negative. An endoscopic defined non-erosive reflux disease and in reflux
ultrasound is not useful in this patient. esophagitis: a meta-analysis. Neurogastroenterol
Motil. 2012 Aug;24(8):747-57, e350.
REFERENCE Wang AJ, Wang H, Xu L, et al. Predictors of
Roman S, Gyawali CP, Savarino E, et al. Ambula- clinical response of acid suppression in Chinese
tory reflux monitoring for diagnosis of gastro- patients with gastroesophageal reflux disease. Dig
esophageal reflux disease: Update of the Porto Liver Dis. 2013 Apr;45(4):296-300.
consensus and recommendations from an inter-
national consensus group. Neurogastroenterol
Motil. 2017 Oct;29(10):1-15. Question 16
Which of the following is a known consequence of
successful antireflux surgery?
Question 15
Which of the following is a predictor of suboptimal A. Delayed solid phase gastric emptying
response of heartburn to PPI therapy? B. Belching and regurgitation
C. Dysphagia
A. Administration of PPI before breakfast D. Esophageal hypomotility
B. Administration of S-isomer of omeprazole E. Small bowel bacterial overgrowth
C. High acid exposure time on pH testing
D. Presence of irritable bowel syndrome CORRECT ANSWER: C
E. Presence of LA Grade B reflux esophagitis
RATIONALE
CORRECT ANSWER: D Dysphagia is a recognized consequence of anti-
reflux surgery. Early postoperative dysphagia is
RATIONALE common, likely related to postoperative edema,
The presence of functional bowel disorders, and typically improves without need for interven-
including irritable bowel syndrome, is associated tion. Late postoperative dysphagia persisting
Chapter 1 — Esophageal disorders 9
RATIONALE RATIONALE
Untreated reflux disease can influence interpreta- Young males with solid food dysphagia need to be
tion of biopsies obtained from Barrett’s esopha- evaluated for eosinophilic esophagitis, and this is
gus, and increases diagnosis of low-grade dyspla- best performed with an endoscopy with proximal
sia or indefinite dysplasia. Over-diagnosis of low and distal esophageal biopsies. The histologi-
grade dysplasia can significantly impact health cal hallmark consists of demonstration of more
care costs. In these instances, repeating biopsies than 15 eosinophils/high power field. Since PPI
following initiation of PPI therapy can provide therapy can normalize eosinophil counts, endos-
a more representative interpretation. Esopha- copy with biopsy should be performed prior to
geal mucosal resection is performed for nodules initiation of PPI therapy when this diagnosis is
identified within the Barrett’s segment. Distal suspected. Although barium radiography pro-
esophagectomy is an option for limited esopha- vides complementary information, and has a
geal cancer without metastases and high-grade high sensitivity for identification of subtle stric-
dysplasia, although endoscopic ablation is the tures, endoscopy with biopsy is preferred for
preferred management option. A fundoplication initial diagnosis when eosinophilic esophagitis is
is an option for management of reflux disease, but suspected. Esophageal manometry is performed
would not be offered to an asymptomatic patient if endoscopy and/or barium radiography do not
without esophagitis. If repeat biopsies demon- establish a mechanism for dysphagia. pH-imped-
strate non-dysplastic Barrett’s esophagus, a three ance monitoring is performed off PPI therapy if
to five-year surveillance interval is appropriate. esophageal acid burden needs to be quantified,
especially in patients with reflux symptoms where
REFERENCES a PPI trial does not improve symptoms.
Wani S, Rubenstein JH, Vieth M, Bergman
J. Diagnosis and Management of Low-Grade REFERENCE
Dysplasia in Barrett’s Esophagus: Expert Review Furuta GT, Katzka DA. Eosinophilic Esophagitis.
From the Clinical Practice Updates Committee N Engl J Med. 2015 Oct 22;373(17):1640-8.
of the American Gastroenterological Association.
Gastroenterology. 2016 Nov;151(5):822-835.
Question 20
An 18-year-old college freshman is referred for
Question 19 endoscopy with a 12 month history of intermit-
A 22-year-old male comes to see you with a two tent retrosternal pain, and one episode of food
year history of heartburn responsive to antacids, impaction that spontaneously resolved. Endos-
and intermittent solid food dysphagia, particu- copy demonstrates normal esophageal mucosa,
larly with bread and meat. He has no regurgi- and no stricture. Distal esophageal biopsies
tation or weight loss. Physical examination is demonstrate 20 eosinophils per high power field.
normal. Which of the following is an appropriate Which of the following is the most appropriate
investigative approach? next step?
Chapter 1 — Esophageal disorders 11
Question 22 REFERENCE
A 65-year-old lady underwent endoscopy for Kim SH, Jeong JB, Kim JW, et al. Clinical and
a three week history of retrosternal chest pain endoscopic characteristics of drug-induced
during swallowing. She has hypertension and esophagitis. World J Gastroenterol. 2014 Aug
diabetes, treated with amlodipine and metformin 21;20(31):10994-9.
respectively. She has rosacea, for which she takes
doxycycline. She has celiac disease, treated with a
gluten free diet. She smokes a packet of cigarettes Question 23
a day. Physical examination is normal. A 22-year-old male with schizoaffective disorder is
Endoscopy was performed, and revealed brought to the emergency room at 1a.m. after hav-
an erosion in the proximal esophagus. Biopsies ing swallowed a button battery at 11p.m.. He has a
revealedan acute neutrophilic infiltrate, but no history of GERD, and takes a daily PPI with con-
viral inclusions. . Distal esophageal biopsies were trol of his heartburn. He reports infrequent dys-
normal. She was started on a daily PPI, but reports phagia to solids. He currently has no symptoms,
no improvement. and is able to handle secretions without problems.
He last ate at 7p.m. the previous evening.
Which of the following is a likely mechanism for Chest x-ray shows a radio-opaque foreign body
her condition? below the carina but above the diaphragm.
diagnosis of GERD or in the management of his an antireflux surgery. The optimal timing of
intermittent breakthrough symptoms. While some PPI therapy is 30 minutes prior to meals on an
patients do require escalation to BID PPI therapy empty stomach in order for the best absorption
or surgery (Answer C, D), he may be better served and activation of drug. Taking the medication
to focus on directed lifestyle modifications for his after meals (Answer B) would reduce absorp-
intermittent symptoms that are related to larger tion of drug and make it less effective. She does
dinner time meals. not have dysphagia or stricture and a dilation
(Answer D) is not needed. She does not meet
REFERENCE criteria for eosinophilic esophagitis and topi-
Ness-Jensen E, Hveem K, El-Serag H, Lagergren cal steroids (Answer E) are not indicated. A CT
J. Lifestyle Intervention inGastroesophageal scan (Answer C) is not part of the standard anti-
Reflux Disease. Clin Gastroenterol Hepatol. 2016 reflux procedure work up.
Feb;14(2):175-82.
REFERENCES
Bello B, Zoccali M, Gullo R, Allaix ME, Herbella
Question 28 FA, Gasparaitis A, Patti MG. Gastroesophageal
A 35-year-old female has had heartburn for reflux disease and antireflux surgery-what is the
seven years and has had to escalate PPI therapy proper preoperative work-up? J Gastrointest Surg.
from QD to BID prior to meals, and recently 2013 Jan;17(1):14-20.
added H2 blockers at bedtime. She denies any Jobe BA, Richter JE, Hoppo T, et al. Pre-
dysphagia. She has to maintain a very strict diet operative diagnostic work-up prior to antireflux
in order to be without symptoms. She is inter- surgery: an “evidence and experience-based”
ested in surgery to be able to liberalize her diet consensus of the Esophageal Diagnostic Advisory
and spare the amount of medication she needs Panel. J Am Coll Surg 2013;217(4):586-97.
to take. Her EGD demonstrates a small hiatal
hernia and biopsies had 3 eosinophils per hpf
in the distal esophagus. A wireless pH study Question 29
performed at the time off therapy demonstrates A 56-year-old female with a BMI 42, diabetes, and
percent time with pH less than four at seven hyperlipidemia presents with a 5 cm hiatal hernia.
percent and DeMeester score of 22. She has symptoms of heartburn during the day
and significant nocturnal regurgitation such that
What should be done prior to a Nissen she is sleeping in a recliner at night.
fundoplication in this patient?
What surgical option would be most effective in
A. High resolution manometry this patient?
B. Change the time of her PPI to after meals
C. CT scan of the chest A. Hiatal hernia repair with Nissen
D. EGD with empiric dilation fundoplication
E. Trial of swallowed fluticasone B. Hiatal hernia repair with Dor fundoplication
C. Magnetic sphincter augmentation
CORRECT ANSWER: A D. Hiatal hernia repair with gastric bypass
E. Gastric sleeve
RATIONALE
A high resolution manometry study (Answer CORRECT ANSWER: D
A) would be needed to exclude the presence
of a major motor disorder prior to pursuing
16 Digestive Diseases Self-Education Program®
RATIONALE RATIONALE
This patient is potentially experiencing rebound The patient had an isolated trigger of repeated
acid hypersecretion and may require a tapered emesis that caused a Mallory Weiss tear.. This
step down approach (Answer C) to H2 blocker patient no longer has an indication for continued
maintenance or on-demand PPI therapy. Symp- PPI therapy and it should be discontinued (An-
toms upon cessation of PPI therapy are not an swers A, B). Unlike erosive esophagitis or gastric
indication for indefinite therapy (Answers A). ulcers which would prompt a repeat EGD to
SSRIs (Answer D) would be indicated in esopha- check for healing (Answer C), a healing Mallory
geal hypersensitivity, and Baclofen to inhibit Weiss tear does not require endoscopic monitor-
TLESRs during the postprandial period. Metoclo- ing. He does not have symptoms that would war-
pramide (Answer B) is not recommended in the rant additional work up (Answers D and E).
treatment of GERD, due to adverse effects
of tardive dyskinesia. REFERENCE
Heidelbaugh JJ, Kim AH, Chang R, Walker PC.
REFERENCES Overutilization of proton-pump inhibitors: what
Hunfeld NG, Geus WP, Kuipers EJ. Systematic the clinician needs to know. Therap Adv Gastroen-
review: Rebound acid hypersecretion after therapy terol. 2012 Jul;5(4):219-32.
with proton pump inhibitors. Aliment Pharmacol
Ther. 2007 Jan 1;25(1):39-46.
Kim J, Blackett JW, Jodorkovsky D. Strate- Question 33
gies for Effective Discontinuation of Proton Pump A 60-year-old male had voice hoarseness and
Inhibitors. Curr Gastroenterol Rep. 2018 May presented to ENT. A direct laryngoscopy demon-
16;20(6):27. strated redness consistent with laryngopharyngeal
reflux, and he was started on BID PPI. He denies
heartburn,dysphagia or any symptoms related to
Question 32 meals. He has worsening voice hoarseness and
A 60-year-old male was discharged from the hospi- otherwise no change in any symptoms after three
tal three weeks ago after an Upper GI bleed. He had months of PPI therapy. EGD was negative for
eaten a seafood salad and had repeated episodes of esophageal pathology.
vomiting, the last of which contained coffee ground
material. His hemoglobin and hemodynamic were What is the next best step?
stable during the whole hospitalization. An EGD
showed no active bleeding and demonstrated a A. Continue PPI BID
healing Mallory-Weiss tear. He was discharged B. Refer for antireflux surgery
home on Omeprazole. He is asymptomatic. C. Wireless pH monitoring, over 96 hours,
on therapy
What is the next best step? D. Wireless pH monitoring, over 96 hours,
off therapy
A. Discontinue PPI E. Repeat EGD with esophageal
B. Repeat EGD to check for healing biopsies
C. Check for H. Pylori infection
D. Barium swallow CORRECT ANSWER: D
RATIONALE Question 38
There is high interobserver variability among A 60-year-old male has a 3 cm segment of Bar-
pathologists in the diagnosis of dysplasia in Bar- rett’s esophagus with a visible lesion.
rett’s esophagus. Therefore, a pathologist with
gastrointestinal expertise should confirm the Which of these lesions is the least likely to be
diagnosis of dysplasia (Answer C). Confirmed low cured by endoscopic therapy?
grade dysplasia may be managed with either en-
doscopic therapy such as radiofrequency ablation A. A raised 1.5 cm area with a regular mucosal
(Answer A) or with close surveillance. A surveil- pattern and increased vascular pattern
lance interval of three to five years is appropriate B. A 1 cm ulcerated area with loss of mucosal
for non-dysplastic Barrett’s, but not in the setting pattern and surrounding irregular vascular
of current low grade dysplasia (Answer E). If the pattern.
patient does not undergo endoscopic eradica- C. A 1 cm flat area with irregular mucosal and
tion therapy, surveillance endoscopy should be vascular pattern
performed every 6 months for one year and then D. An 8 mm polypoid lesion with a villous
annually until there is reversion to non-dysplastic mucosal pattern
Barrett’s. EUS is useful for evaluation of lymph E. A 2 cm slightly elevated area with a
node metastasis, but not in the staging of superfi- cerebriform mucosal pattern
cial metaplasia (Answer B). The decision to pursue
anti-reflux surgery should be based on symptoms, CORRECT ANSWER: B
and the dysplasia would need to be addressed first
(Answer D). RATIONALE
A depressed lesion has the highest risk of sub-
REFERENCES mucosal invasion, which is associated with a 20
Duits L, van der Wel M, Cotton C, et al. Patients percent or greater risk of lymph node metastasis
with Barrett’s esophagus and confirmed persis- and may not be amenable to endoscopic therapy
tent low-grade dysplasia are at increased risk (Answer B). Additionally, irregular mucosal pat-
for progression to neoplasia. Gastroenterology terns and irregular vascular patterns are more
2017;152:993-1001. consistent with neoplasia then those that are
Kestens C, Offerhaus GJ, van Baal JW, et al. round, tubular, villous, or cerebriform.
Chapter 1 — Esophageal disorders 21
REFERENCES months for one year, and then yearly (Answer B).
Peters FP, Brakenhoff KP, Curvers WL, Rosmolen EUS is not warranted in confirmed high-grade dys-
WD, Fockens P, ten Kate FJ, Krishnadath KK, plasia without any evidence of cancer (Answer E).
Bergman JJ. Histologic evaluation of resection
specimens obtained at 293 endoscopic resections REFERENCE
in Barrett’s esophagus. Gastrointest Endosc. 2008 Desai M, Saligram S, Gupta N, Vennalaganti P,
Apr;67(4):604-9. Bansal A, Choudhary A, Vennelaganti S, He J, Titi
Sharma P, Bergman JJ, Goda K, Kato M, M, Maselli R, Qumseya B, Olyaee M, Waxman I,
Messmann H, Alsop BR, Gupta N, Vennalaganti P, Repici A, Hassan C, Sharma P. Efficacy and safety
Hall M, Konda V, Koons A, Penner O, Goldblum outcomes of multimodal endoscopic eradication
JR, Waxman I. Development and Validation of therapy in Barrett’s esophagus-related neoplasia:
a Classification System to Identify High-Grade a systematic review and pooled analysis. Gastroin-
Dysplasia and Esophageal Adenocarcinoma in test Endosc. 2017 Mar;85(3):482-495.e4.
Barrett’s Esophagus Using Narrow-Band Imaging.
Gastroenterology. 2016 Mar;150(3):591-8. doi:
10.1053/j.gastro.2015.11.037. Epub 2015 Nov 25. Question 40
PubMed PMID: 26627609. A 70-year-old male was treated for a nodule in the
setting of a 5 cm segment of Barrett’s esophagus.
The nodule was high grade dysplasia and resected
Question 39 in entirety with negative margins. The remainder
A 57-year-old male had a 4 cm segment of Bar- of the segment underwent radiofrequency ablation
rett’s with a nodule identified and resected by for a total of five sessions, with still 50 percent of
endoscopic mucosal resection. The pathology was the Barrett’s segment present. Biopsies of the pre-
consistent with high-grade dysplasia and margins viously treated areas were negative for dysplasia,
were negative. There was no evidence of cancer. but confirmed persistent metaplasia.
What is the next step? What therapeutic options for the Barrett’s mucosa
may be considered at this point?
A. Continue PPI BID indefinitely
B. Repeat EGD and biopsies yearly A. Completion endoscopic submucosal dissection
C. Biopsy the rest of the segment and treat if B. Photodynamic therapy
any residual dysplasia C. Repeat radiofrequency ablation regimen
D. Radiofrequency ablation D. Esophagectomy
E. Endoscopic ultrasound E. Cryotherapy
RATIONALE RATIONALE
Due to the risk of metachronous and synchronous After five sessions, the patient is a radiofrequency
lesions, the standard approach is to resect any ablation (RFA) failure, thusrepeated RFA is un-
visible lesion in the setting of dysplasia and then likely to eradicate the Barrett’s segment (Answer
treat the remainder of the at-risk epithelium with C). Cryotherapy has been reported as a salvage
ablative modes such as radiofrequency ablation treatment after RFA failures (Answer E) for the
(Answer D). After therapy, surveillance is typically goal of total Barrett’s eradication. It does not have
more rigorous for high-grade dysplasia with some the long-term data as present with RFA and does
proposed intervals of three months for one year, six carry a more favorable risk profile than radical en-
22 Digestive Diseases Self-Education Program®
doscopic mucosal resection or endoscopic submu- yearly (Answer A) and not yet yearly or as needed
cosal dissection (answer A). Photodynamic therapy (Answers C, E). EUS in confirmed high grade dys-
(Answer E) has largely fallen out of favor due to its plasia without any evidence of cancer is of limited
adverse event profile. Given the high grade dysplai- yield given the zero rate of lymph node metastasis
sa is cleared and there is no presence of neoplasia, in high grade dysplasia (Answer D). PET/CT (An-
esophagectomy (Answer D) is not indicated. swer B) is useful for detection of metastatic disease
for staging and monitoring cancer.
REFERENCE
Sengupta N, Ketwaroo GA, Bak DM, Kedar V, REFERENCES
Chuttani R, Berzin TM, Sawhney MS,Pleskow DK. Krishnamoorthi R, Singh S, Ragunathan K, A
Salvage cryotherapy after failed radiofrequency Katzka D, K Wang K, G Iyer P. Risk of recur-
ablation for Barrett’s esophagus-related dysplasia rence of Barrett’s esophagus after successful
is safe and effective. Gastrointest Endosc. 2015 endoscopic therapy. Gastrointest Endosc. 2016
Sep;82(3):443-8. doi: 10.1016/j.gie.2015.01.033. Jun;83(6):1090-1106.e3.
Epub 2015 Apr 14. Stier MW, Konda VJ, Hart J, Waxman I.
Post-ablation surveillance in Barrett’s esophagus:
A review of the literature. World J Gastroenterol.
Question 41 2016 May 7;22(17):4297-306.
A 65-year-old male with a history of Barrett’s Shaheen NJ, Falk GW, Iyer PG, Gerson
esophagus with a nodule of high grade dysplasia LB; American College of Gastroenterology.ACG
was treated successfully by endoscopic mucosal Clinical Guideline: Diagnosis and Management
resection and subsequent radiofrequency ablation. of Barrett’s Esophagus. Am J Gastroenterol. 2016
Surveillance endoscopy reveals complete eradica- Jan;111(1):30-50.
tion of intestinal metaplasia.
REFERENCE REFERENCE
Gurvits GE, Cherian K, Shami MN, Korabathina Zaninotto G, Bennett C, Boeckxstaens G, et al. The
R, El-Nader EM, Rayapudi K,Gandolfo FJ, Alsh- 2018 ISDE achalasia guidelines. Dis Esophagus.
umrany M, Patel H, Chowdhury DN, Tsiakos A. 2018 Sep 1;31(9).
Black esophagus: new insights and multicenter
international experience in 2014. Dig Dis Sci. 2015
Feb;60(2):444-53. Question 46
A 67-year-old female has had long standing dys-
phagia. She is on proton pump inhibitor therapy.
Question 45 She has several rings in her esophagus and narrow
A 70-year-old male presents with progressive dys- caliber throughout her esophagus. Biopsies do
phagia over the past four months and 30 pound not have any eosinophils and have intraepithelial
weight loss. A barium swallow demonstrates a lymphocytes around the papilla (up to 30 per hpf)
dilated esophagus with a bird’s beak appearance. with spongiosis.
What is the next best step?
What treatment do you recommend?
A. Referral for per-oral endoscopic myotomy
B. high resolution esophageal manometry A. Topical steroids and serial dilation
C. Calcium channel blocker B. Pneumatic balloon dilation
D. EGD C. Fluconazole
E. Placement of G tube D. Ganciclovir
E. Calcium Channel blockers
CORRECT ANSWER: D
CORRECT ANSWER: A
RATIONALE
RATIONALE
Achalasia and pseudoachalasia are on the dif- This patient has lymphocytic esophagitis charac-
ferential. Given the advanced age, progressive terized by peripapillary lymphocytosis and spon-
course, and significant weight loss, an endoscopy giosis. The exact definition has not been standard-
with careful attention to GEJ should be performed ized in terms of number of lymphocytes per hpf.
to rule out malignancy causing a pseudoachalasia Treatment is with proton pump inhibitor, steroids,
presentation (Answer D). Manometry should be and dilation when there are structural lesions (An-
done after the endoscopy to confirm and subtype swer A). The other choices are for primary motility
the achalasia. If achalasia is confirmed and malig- disorders (Answer B and E) or infectious etiologies
nancy is rule out, myotomy either with a modified (Answer C and D).
Chapter 1 — Esophageal disorders 25
REFERENCE REFERENCES
Rouphael C, Gordon IO, Thota PN. Lymphocytic Jobe BA, Kahrilas PJ, Vernon AH, et al. Endo-
esophagitis: Still an enigma a decade later. World scopic appraisal of the gastroesophageal valve
J Gastroenterol. 2017 Feb 14;23(6):949-956. after antireflux surgery. Am J Gastroenterol.
2004;99(2):233–43.
Abdelmoaty WF, Swanstrom LL. Endoscopic
Question 47 Evaluation of Post-Fundoplication Anatomy. Curr
A 47-year-old male patient had long standing Gastroenterol Rep. 2017 Aug 24;19(10):51.
GERD and is status post Nissen fundoplication Juhasz A, Sundaram A, Hoshino M, Lee TH,
five years ago. He was doing well until three Filipi CJ, Mittal SK. Endoscopic assessment of
months ago when he had frequent regurgitation failed fundoplication: a case for standardization.
and chest pain. An EGD demonstrates the top of Surg Endosc. 2011 Dec;25(12):3761-6.
the gastric folds at 35 cm and the diaphragmatic
impression at 40 cm. The scope traversed easily
into the stomach. An initial retroflexed view does Question 48
not show a visible wrap at the hiatus and closer A 57-year-old female with Raynaud’s syndrome
inspection demonstrates a wrap proximal to and scleroderma complains of dysphagia. Barium
the hiatus. swallow demonstrates a dilated, fluid-filled
esophagus. A high-resolution manometry is
What would you describe this as in your ordered.
endoscopy report?
What are the most likely findings on this study?
A. Slipped Nissen
B. Herniated fundoplication A. High IRP, Absent contractility
C. Tight Wrap B. High IRP, Premature contractions
D. Malpostioned wrap C. Normal IRP, Premature contractions
E. Complete disruption D. Normal IRP, Hypercontractile contractions
E. Low IRP, Absent contractility
CORRECT ANSWER: B
CORRECT ANSWER: E
RATIONALE
An endoscopy and barium imaging are useful in RATIONALE
identifying fundoplication failures. This is a case Patients with scleroderma are likely to have major
where the wrap has migrated above the hiatus and motility disorders of the esophagus. They can
caused a trans-diaphragmatic wrap herniation have a patulous lower esophageal sphincter and
(Answer B). Given the patient is symptomatic, this dilated, fluid-illed esophagus in later stages of the
may warrant redo fundoplication. Other failures disease. The high-resolution manometry findings
include slipped Nissen or malpositioned wrap, in in scleroderma esophagus include absent contrac-
which the gastric folds are seen proximal to the tility with a low IRP sphincter (Answer E). Acha-
wrap impression and the wrap is seen around lasia subtypes would have a high IRP and absent
the stomach rather than the GEJ (Answer A, D), contractility (Answer A, classic Type I subtype) or
a tight wrap, in which passage of the scope may premature contractions (Answer B, spastic type
encounter resistance, and proximal dilation may III subtype). Answer C reflects possible diffuse
be seen (Answer C), or complete loss of the folds of esophageal spasm and Answer D may reflect Jack-
the fundoplication (Answer E). hammer esophagus.
26 Digestive Diseases Self-Education Program®
Question 49
A 64-year-old female patient with mild COPD Question 50
underwent a Heller myotomy 20 years ago for the A 35-year-old female has been having daily dyspha-
treatment of achalasia. She did well until three gia, regurgitation, and chest pain for six months.
years ago, when she had daily dysphagia, regurgi- She had no previous heartburn. She lost five
tation and drooling at night. An endoscopy shows pounds. She had no response to daily proton pump
a fluid-filled, tortuous esophagus with mild resis- inhibitor therapy. She was being referred for work
tance to scope passage at the gastroesophageal up to consider a Nissen fundoplication. EGD with
junction. She had no improvement with a balloon wireless pH monitoring off therapy demonstrates
dilation to 20 mm. A barium swallow demon- no obvious strictures or masses. The LES was not
strates a severely dilated esophagus and manome- patulous, and there was no hiatal hernia. Biopsies
try demonstrates an above normal IRP with absent of distal and mid esophagus were normal. The time
contractility. spent with pH 4 or less was seven percent and the
Demeester score was 25. There was no correlation
What is the next best step? between reflux and symptoms. Manometry findings
demonstrated an Integrated relaxation pressure
A. Botox injection (IRP) of 22 (above normal), distal contractile in-
B. Pneumatic dilation tegral (DCI) with normal vigor, and no premature
C. Esophagectomy contractions. There were abnormal appearing swal-
D. Redo Heller myotomy lows with repetitive contractions and compartmen-
E. Nitrates talized pressurization with all swallows. A barium
swallow showed no masses or strictures and a
CORRECT ANSWER: B barium tablet hung up at the GEJ.
CORRECT ANSWER: B
RATIONALE
REFERENCE
Kahrilas PJ, Bredenoord AJ, Fox M, Gyawali CP,
Roman S, Smout AJ, Pandolfino JE; International
High Resolution Manometry Working Group.
The Chicago Classification of esophageal motility
disorders, v3.0. Neurogastroenterol Motil. 2015
Feb;27(2):160-74.
28 Digestive Diseases Self-Education Program®
Answers & critiques
CHAPTER 2
29
30 Digestive Diseases Self-Education Program®
Question 3
A 64-year-old white woman with hypothyroidism Question 4
and insulin-dependent diabetes is referred to A 77-year-old man with a history of peptic ulcer
gastroenterology for megaloblastic, normocytic disease underwent partial gastrectomy with a
anemia. She underwent an unremarkable gastrojejunostomy anastomosis (Billroth II)
colonoscopy. An upper endoscopy is notable for without vagotomy 30 years ago for refractory ulcer
pallorous mucosa and biopsies from the gastric disease. He has no other significant past medical
body revealed a paucity of oxyntic mucosa with history. Several years after surgery he developed
pseudopyloric metaplasia. There are no H. pylori dyspepsia, which was successfully treated with
organisms seen with routine or special stains. over-the-counter antacids. Given persistence
Serologic evaluation is expected to reveal B12 of symptoms he was placed on 20 mg daily
deficiency and the most sensitive test to confirm omeprazole. He continued to have intermittent
pernicious anemia is: episodes of severe epigastric abdominal pain
with occasional admissions for gastrointestinal
A. Schilling test bleeding therefore was placed on total cumulative
B. Anti-parietal cell antibody dose of 80 mg omeprazole daily. He undergoes
C. Anti-intrinsic factor antibody an upper endoscopy, revealing anastomotic and
D. Serum homocysteine levels jejunal ulcerations. A fasting serum gastrin level
was 195 pg/ml. The best next step in evaluation is:
CORRECT ANSWER: B
A. Secretin stimulation test
RATIONALE B. Octreotide nuclear medicine scan
Autoimmune metaplastic atrophic gastritis C. Empirically increase omeprazole to three
(AMAG) can occur in the context of times daily dosing
polyglandular autoimmune syndrome. When D. Exploratory laparotomy
present, AMAG can progress to pernicious
anemia that is characterized by low levels of CORRECT ANSWER: A
vitamin B12. Parietal cell dropout
and resultant loss of intrinsic factor (IF) is RATIONALE
required for B12absorption. As a result, both This patient most likely has persistent peptic
anti-parietal cell and anti-intrinsic factor ulcers due to retained antrum as a result of
antibodies are used to determine the cause of B12 his prior Billroth II operation. In this scenario,
deficiency. However, the sensitivity for the former a portion of the antrum is inadvertently left
is 80 percent compared to 50 percent for the behind after surgery and, chronically bathed in
latter. The Schilling test can provide insight into alkaline fluid in the excluded portion, leads to
the etiology of B12 deficiency, including problems unopposed gastrin hypersecretion and gastric
with IF production versus malabsorption; acid production. A secretin stimulation test can
however, it is rarely available and therefore is differentiate between hypergastrinemia from
not the ideal test. Serum homocysteine levels gastrinoma or retained antrum. If an octreotide
are highly sensitive to detect B12 deficiency but nuclear medicine scan were performed and
cannot identify a cause and may be affected by the negative, it would neither confirm the etiology nor
presence of renal insufficiency. exclude other diagnoses within the differential.
Chapter 2 — Acid diseases of the stomach 31
A. Ibuprofen A. Type 1
B. Naproxen B. Type 2
C. Meloxicam C. Type 3
D. Diclofenac D. Type 4
RATIONALE RATIONALE
The use of NSAIDs is independently associated Gastric carcinoids arise in enterochromaffin-
with an increased risk of gastrointestinal like (ECL) cells. Type 1 gastric carcinoids are the
ulceration and bleeding. The risk of ibuprofen is most common type of gastric carcinoids and are
lowest whereas the risk of naproxen is highest most commonly associated with chronic atrophic
among this group. gastritis. They generally have the lowest metastatic
risk at less than five percent. Type 2 gastric
REFERENCES carcinoids are the least common type of gastric
García-Rodríguez LA and Hernandez-Diaz carcinoids and associated with Zollinger-Ellison
S. “Relative risk of upper gastrointestinal syndrome and MEN-1 syndrome. They have
complications among users of acetaminophen a metastatic risk of approximately 10 percent.
and nonsteroidal anti-inflammatory drugs.” Sporadic, or type 3, gastric carcinoids have the
Epidemiology. 2001;12:570-576. highest malignant potential with a metastatic
Hernández-Díaz S and Rodríguez L. risk greater than 50 percent. There are no type 4
“Association between nonsteroidal anti- gastric carcinoids.
inflammatory drugs and upper gastrointestinal
bleeding/perforation: an overview of REFERENCES
epidemiologic studies published in the 1990s.” Delle Fave G, O’Toole, D, et al. “ENETS
Arch Intern Med. 2000;160:2093-2099. consensus guidelines update for gastroduodenal
de Abajo FJ and García-Rodríguez LA. neuroendocrine neoplasms.” Neuroendocrinology
“Risk of upper gastrointestinal bleeding and 2016;103:119–124.
perforation associated with low-dose aspirin as Basuroy R, Srirajaskanthan R et al. “Review
plan and enteric-coated formulations.” BMC Clin article: the investigation and management
Pharmacol. 2001;1:1. of gastric neuroendocrine tumors.” Aliment
Pharmacol Ther. 2014;39:1071-1084.
Question 8
A 63-year-old man without a significant past Question 9
medical history is admitted to the hospital A 35-year-old man without a significant past
with melena and acute anemia. He is placed on medical history is admitted to the trauma
intravenous proton pump inhibitor therapy and intensive care unit after sustaining multiple
Chapter 2 — Acid diseases of the stomach 33
REFERENCE RATIONALE
Cook, DJ, Fuller, HD, et al. “Risk factors for Gastric acid production is stimulated by multiple
gastrointestinal bleeding in critically ill patients.” mechanisms, including via vagal inputs even at the
N Eng J Med 1994;330:377-381. thought of ingesting a meal. This acid production
is buffered during meal consumption. However, a
layer of unbuffered contents referred to as the acid
Question 10 pocket sits within the proximal stomach in the
Which of the following gastroprotective area of the cardia and fundus. Buffering of, and
mechanisms is affected by the use of non-steroid potentially physically inhibiting, this layer with
anti-inflammatory drugs (NSAIDs)? the use of alginates can reduce the symptoms of
gastroesophageal reflux after meals.
A. Decreased prostaglandin synthesis
B. Down-regulation of gastric acid production REFERENCE
C. Increased production of gastric mucous layer Rohof WO, Bennink, RJ et al. “An alginate-antacid
D. Improved gastric blood flow formulation localizes to the acid pocket to reduce acid
reflux in patients with gastroesophageal reflux disease.”
CORRECT ANSWER: A Clin Gastroenterol Hepatol. 2013;11:1585-1591.
34 Digestive Diseases Self-Education Program®
Question 12 Question 13
A 44-year-old woman with a family history A 30-year-old woman with a history of anxiety
of MEN-1 reports significant abdominal pain disorder presents to the gastroenterology clinic
and heartburn. She reports a 45 pound weight with epigastric abdominal bloating discomfort and
loss over the prior six months. She is tried on early satiety. Her pain has been present on most
standard dose proton pump inhibitor without days for the previous year, during which time
any substantive benefit and therefore undergoes she has been working to complete a PhD thesis
upper endoscopy. She is found to have erosive in astrophysics. There is mild nausea without
esophagitis, several cratered gastric and multiple vomiting, no relationship with food or bowel
shallow ulcerations in the distal duodenum movements and she has no other chronic medical
beyond the ampulla. She is diagnosed with problems. Her primary care provider treated her
Zollinger-Ellison syndrome (ZES) and is referred with proton pump inhibitors without success
for further management. and she has had a normal metabolic profile and
complete blood count. She was then referred for
Which of the following is a cause of weight gain open-access upper endoscopy and biopsies of
frequently seen after medical gastric acid control the small bowel and stomach were normal. She
in the management of ZES? now presents for a follow-up appointment. What
is the mechanism by which buspirone reduces
A. Appropriate fat and nutrient absorption symptoms in functional dyspepsia?
B. Direct cytoreductive properties of PPI
C. Increase in bowel movement frequency A. Reduction in gastric emptying time
D. Improvement in dietary intake B. Fundic relaxation and accommodation
C. Direct appetite stimulation
CORRECT ANSWER: A D. Direct anxiolytic properties
a fundus relaxing drug, in patients with dismiss these without discussion would not be
functional dyspepsia. Clin Gastroenterol Hepatol. appropriate.
2012;10:1239-1245.
Miwa H, Nagahara A, Tominaga K, et al. REFERENCE
Efficacy of the 5-HT1A agonist tandospirone Freedberg DE, Kim LS, Yang YX. “The risks
citrate in improving symptoms of patients with and benefits of long-term use of proton pump
functional dyspepsia: a randomized controlled inhibitors: expert review and best practice
trial. Am J Gastroenterol 2009;104:2779 –2787. advice from the American Gastroenterological
Association.” Gastroenterology. 2017;152:706-715.
Question 14
A 69-year-old woman with a history of Question 15
hypothyroidism, treated breast cancer and In patients with bleeding from a peptic ulcer,
gastroesophageal reflux disease presents to your which is the best management approach?
clinic. Her heartburn occurred most days of
the week and previously responded to 40mg of A. Epinephrine injection only to clean based
omeprazole, which she has been using daily for ulcer
the last 10 years. Her mother died of a myocardial B. Bipolar cautery only to pigmented spot
infarction in her 70s and she reports seeing a news C. Application of hemostatic clips only to
story implicating proton pump inhibitors with adherent clot
the development of cardiovascular disease. Your D. Combination therapy of epinephrine and
initial recommendation should be: hemostatic clips to non-bleeding visible vessel
Laine, L and Jensen, DM. “Management of presentation: a systematic review. Obes Surg
patients with ulcer bleeding.” Am J Gastroenterol. 2014;24:299–309.
2012;107:345-360.
Question 17
Question 16 In the above patient, ulcer healing time can be
A 55-year-old man with type 2 diabetes, accelerated by which of the following?
hypertension and osteoarthritis underwent Roux-
en-Y gastric bypass for morbid obesity five months A. Re-do operation now with attempt and
ago. He reports losing 100 pounds from his altering pouch size
maximal weight but recently developed abdominal B. Ingestion of opened PPI capsules
pain and fatigue. A CT abdomen pelvis with C. Use of high dose oral PPI
contrast reveals expected post-surgical anatomy D. Use of sucralfate
but no other significant findings. He subsequently
undergoes an upper endoscopy revealing a linear CORRECT ANSWER: B
ulcer along the margin of the gastroenterostomy.
RATIONALE
Which of the following is a risk factor for the The rate of marginal ulcer after Roux-en-Y gastric
development of marginal ulcers? bypass can be as high as 16 percent. The risk
of developing these ulcers at the gastrojejunal
A. Weight loss of greater than 50 pounds after anastomosis is likely multifactorial, including
surgery size and location of the pouch, the presence of
B. Hypertension a gastrogastric fistula, the presence of a foreign
C. Long gastric pouch (suture) material or ischemia. There is also
D. PPI use evidence for acidity playing a role, with the
jejunum receiving acid chyme directly from the
CORRECT ANSWER: C stomach. Even when controlling for the use of
sucralfate and PPI dose, the ingestion of opened
RATIONALE PPI capsules was associated with a reduction
Relative ischemia to the gastroenterostomy is in healing time compared to standard oral
a primary risk factor for the development of PPI. Surgery to revise this patient’s bypass is
marginal ulcer in patients with a history of Roux- premature.
en-Y gastric bypass. This risk is increased in
patients who smoke, use NSAIDs or have diabetes. REFERENCE:
A short gastric pouch is associated with a reduced Schulman AR, Chan WW, et al. “Opened proton
risk of marginal ulcer. pump inhibitor capsules reduce time to healing
compared with intact capsules for marginal
REFERENCES ulceration follow Roux-en-Y gastric bypass.” Clin
Azagury DE, Aby Dayyeh BK, Greenwalt, IT, Gastroenterol Hepatol. 2017;15:494-500.
Thompson CC. “Marginal ulceration after Roux-
en-Y gastric bypass surgery: characteristics, risk
factors, treatment, and outcomes.” Endoscopy. Question 18
2011;43:950-954. A 60-year-old man with long-standing heartburn
Coblijn UK, Goucham AB, Lagarde SM, et al. undergoes upper endoscopy to screen for Barrett’s
Development of ulcer disease after Roux-en-Y esophagus. His symptoms have been well-
gastric bypass, incidence, risk factors, and patient controlled on proton pump inhibitors (PPIs) for 15
Chapter 2 — Acid diseases of the stomach 37
years. On examination of the stomach, there are have a clear relationship with eating or bowel
multiple pearly white polyps with a lacy reticular movements. It has not changed in character over
vascular pattern in varying sizes from three this time. An abdominal ultrasound was negative.
millimeters to two centimeters. She denies any heartburn or regurgitation but
reports frequent nausea. She had a colonoscopy
Which is a risk factor in the development of and upper endoscopy for iron deficiency three
dysplasia in fundic gland polyps? years ago, but these exams were normal without
any biopsies being taken.
A. Hereditary nonpolyposis colorectal cancer
(Lynch) syndrome Which testing modality would you recommend to
B. Small fundic gland polyp size assess for Helicobacter pylori infection?
C. The presence of antral gastritis
D. Barrett’s esophagus A. Urease breath test
B. Stool antigen
CORRECT ANSWER: C C. Serum antibody
D. Upper endoscopy with biopsies
RATIONALE
Fundic gland polyps are commonly encountered CORRECT ANSWER: A
during upper endoscopy, especially among
users of proton pump inhibitors. These polyps RATIONALE
are generally considered to have low risk for Histology is the gold standard for documenting
malignant transformation, which may be on the H. pylori infection but this patient already had
basis of chemopreventive properties of PPIs. an upper endoscopy and it would not be the most
However, dysplasia is associated with increasing cost-effective approach. Serum antibodies are
size of the largest fundic gland polyps, the non-specific for active infection. Urease breath test
presence of antral gastritis and in patients has marginally better performance characteristics
with familial adenomatous polyposis (FAP) (95 percent sensitivity and 95 percent specificity)
syndrome. Barrett’s esophagus is not associated compared to stool antigen (93 percent sensitivity
with the development of dysplastic fundic and 90 percent specificity). Although the
gland polyps. likelihood of celiac disease in this patient is low,
given the presence of iron deficiency it could be
REFERENCES concurrently evaluated; use of serologic screening
Bianchi LK, Burke CA, et al. “Fundic gland polyp would be more cost effective than repeating upper
dysplasia is common in familial adenomatous endoscopy with duodenal biopsies.
polyposis.” Clin Gastroenterol Hepatol.
2008;6:180-185. REFERENCES
Svoboda A, Tillisch-Svoboda S. “Barrett’s Gisbert JP and Pajares JM. “Review article:
patients developing gastric polyps during acid 13C-urea breath test in the diagnosis of
suppression are at greater risk of esophageal Helicobacter pylori infection—a critical review.”
dysplasia.” Endoscopy. 1996;28:S20. Aliment Pharmacol Ther 2004;20:1001-17.
Wright CL and Kelly JK. “The use of
routine special stains for upper gastrointestinal
Question 19 biopsies.” Am J Surg Pathol. 2006;30:357-61.
A 53-year-old African American woman presents Best LM, Takwoingi Y, et al. “Non-invasive
with epigastric abdominal pain. Her pain has diagnostic tests for Helicobacter pylori infection.”
been present for the last 12 months but does not Cochrane Database Syst Rev. 2018;3:Cd012080.
38 Digestive Diseases Self-Education Program®
Question 20 Question 21
A 63-year-old female patient has a urease A 73-year-old man with a previously placed
breath test that confirms active Helicobacter cardiac stent is on dual antiplatelet therapy
pylori infection. She reports a recent urinary with aspirin and ticagrelor. He recently strained
tract infection treated with ciprofloxacin and was his back playing tennis and has been using
prescribed azithromycin for bronchitis last year ibuprofen for pain relief. He presents to the
but reports no drug allergies. emergency department after the acute onset
of hematochezia and weakness but denies
What regimen would you recommend? any passage of blood clots, hematemesis or
abdominal pain. What is the best justification for
A. PPI + amoxicillin + clarithromycin nasogastric lavage?
B. PPI + bismuth + metronidazole +
tetracycline A. It is associated with reduced mortality
C. PPI + levofloxacin + amoxicillin B. It can reduce the need for additional
D. PPI + amoxicillin + rifabutin endoscopy
C. It is a well-tolerated test
CORRECT ANSWER: B D. It is the most effective way to discriminate
upper versus lower source of bleeding
RATIONALE
There are several potential reasons for treatment CORRECT ANSWER: B
failure in patients H. pylori, with antibiotic
resistance an increasingly important factor. RATIONALE
Amoxicillin resistance is estimated to be less The placement of nasogastric (NG) tubes is
than one percent, but this patient has multiple uncomfortable but if used in the appropriate
recent antibiotic drug exposures reducing the context can provide benefit. In patients with
likelihood of success with triple therapy. An suspected large volume upper GI bleeding, it
optimal up-front strategy would include a 14- can be used to clear the gastric fundus of blood
day regimen of quadruple therapy and saving and improve diagnostic visualization. Studies
a regimen with rifabutin in the setting of evaluating the performance characteristics of
treatment failure or salvage. NG lavage reveal a significant false negative rate
that can be up to 15 percent, limiting its role as
REFERENCES an optimal discriminator between upper and
Fallone CA, Chiba N, et al. “The Toronto lower GI bleeding. The use of NG lavage has not
Consensus for the Treatment of Helicobacter been shown to reduce mortality in the setting of
pylori Infection in Adults. Gastroenterology. GI bleeding.
2016;151:51-69 e14.
Malfertheiner P, Megraud F et al. REFERENCES
“Management of Helicobacter pylori infection- Scott, L and Kearney, D. “A Randomized
the Maastricht V/Florence Consensus Report.” Controlled Trial of Gastric Lavage Prior
Gut. 2017;66:6-30. to Endoscopy for Acute Upper Gastrointestinal
Chey WD, Leontiadis GI, Howden CW, Bleeding.” J Clin Gastro. 2004;38:861-865.
Moss SF. “ACG Clinical Guideline: Treatment Huang ES, Karsan S et al. “Impact of
of Helicobacter pylori Infection.” Am J nasogastric lavage on outcomes in acute GI
Gastroenterol. 2017;112:212-39. bleeding.” Gastrointest Endosc. 2011;74:971-980.
Pallin DJ and Saltzman JR. “Is nasogastric
tube lavage in patients with acute upper GI
Chapter 2 — Acid diseases of the stomach 39
immediate hemorrhage noted from a large attributes to food poisoning. She noted bright red
mass in the gastric cardia. What is the best next blood in her emesis over the last several episodes
endoscopic approach? of vomiting, which prompted presentation.
Routine vital signs and basic lab work is ordered.
A. Use of argon plasma coagulation (APC) Which scoring system will provide the single
B. Placement of hemoclips to assist best estimate of whether she would benefit from
interventional radiology in locating bleeding hospital admission and early endoscopy (within
source 24 hours)?
C. Use of hemostatic powder (TC-325)
D. Multimodal therapy with epinephrine A. The Glasgow-Blatchford score
injection and bipolar cautery B. The Rockall risk score
C. The Albumin, INR, mental status, systolic
CORRECT ANSWER: C blood pressure, and older age (AIMS-65) score
D. The Baylor bleeding score
RATIONALE
This patient has multiple risk factors for esophageal CORRECT ANSWER: A
cancer and it is reasonable to expect the observed
mass in the gastric cardia is malignant. This likely RATIONALE
represents a hemodynamically significant brisk Multiple scoring tools to stratify patients with
bleed and transferring the patient at this time upper GI bleeding exist. Unlike the Rockall
would be suboptimal. Instead, endoscopic wide score, which is better for assessing mortality risk,
area treatment would be ideal. Hemostatic powder the Glasgow-Blatchford score utilizes routinely
was recently approved in the United States and has obtained laboratory parameters and clinical history.
been shown to be effective in the management of It has been shown to have the best performance
bleeding from gastrointestinal tumors. There are no characteristics (99 percent negative predictive
trials comparing the effectiveness of argon plasma value) to exclude the need for early endoscopy in
coagulation (APC), which has been used to control patients found to have low risk scores.
tumor bleeding, with hemostatic spray. However,
the latter modality would likely be associated with REFERENCE
a smaller effective treatment area. Focal therapy, Laursen SB, Hansen JM, Schaffalitzky de
including dual modality treatment, is unlikely to Muckadell OB. “The Glasgow Blatchford Score
provide effective control of the bleeding. is the most accurate assessment of patients
with upper gastrointestinal hemorrhage.” Clin
REFERENCE Gastroenterol Hepatol. 2012;10:1130-1135.
Haddara S., Jacques J, et al. “A novel hemostatic
powder for upper gastrointestinal bleeding (the
‘GRAPHE’ registry).” Endoscopy. 2016:48:1084- Question 26
1095. A patient with pernicious anemia will be expected
to have the following:
REFERENCE
Chey WD, Leontiadis GI, Howden CW, Moss Question 31
SF. ACG Clinical Guideline: Treatment of A 40-year-old obese lady presents with six weeks
Helicobacter pylori Infection. Am J Gastroenterol. of severe post prandial abdominal pain. She
2017;112:212-239. underwent a gastric bypass two years ago and had
lost 60 pounds before starting to gain it back.
She is now back to her preoperative weight.
Question 30
A 62-year-old tennis instructor presents with You perform a detailed workup and find:
chronic post prandial abdominal pain. He had
been on PPI for two weeks with minimal relief. A. H. pylori gastritis
You perform an upper endoscopy and see multiple B. Atrophic gastritis
antral ulcers and erosions. A post prandial serum C. Elevated methyl-malonic acid levels
gastrin level is ordered and found to be 210pg/ml. D. Low folate
RATIONALE RATIONALE
H pylori may very well be seen as this may be It is important to note that enteric coated
encountered in asymptomatic patients but this is preparations of NSAIDs have not been shown to
not specific to bypass. It is important to note that reduce PUD. An important mechanism behind
after roux-en y bypass, vitamin B12 deficiency is this disease is the inhibition of prostaglandin
common (given the important role of the stomach synthesis (through inhibition of COX-1) which
in its absorption—see DDSEP study material). In play an important role in maintaining the
fact, it is often difficult to manage, not responding protective mucosal barrier. This effect is systemic
to standard multivitamin regimens. Folate and therefore enteric preparations would not help.
deficiency is not seen. While the co-incidence of H pylori and NSAID
intake has been shown to increase the risk of
REFERENCES PUD, the incidence of this infection is decreasing,
Blume CA, Boni CC, Casagrande DS, et al. not increasing. The patient does not have alarm
Nutritional profile of patients before and after features and is young which would make an
Roux-en-Y gastric bypass: 3-year follow-up. Obes “urgent” upper endoscopy premature and not
Surg 2012;22:1676–85. indicated.
von Drygalski A, Andris DA. Anemia after
bariatric surgery: more than just iron deficiency. REFERENCES
Nutr Clin Pract 2009;24:217–26. Talley NJ, Vakil NB, Moayyedi P. American
Gasteyger C, Suter M, Gaillard RC, et al. Gastroenterological Association technical review
Nutritional deficiencies after Roux-en-Y gastric on the evaluation of dyspepsia. Gastroenterol.
bypass for morbid obesity often cannot be prevented 2005;129:1756-80.
by standard multivitamin supplementation. Am J Kemmerly, T, Kaunitz, JD. Gastroduodenal
Clin Nutr 2008;87:1128–33. mucosal defense. Curr Opin Gastroenterol 2014.
Malinowski SS. Nutritional and metabolic Laine L, Jensen DM. Management of patients
complications of bariatric surgery. Am J Med Sci with ulcer bleeding. Am J Gastroenterol 2012;
2006;331:219–25. 107:345-360.
Question 32 Question 33
A 32-year-old accountant presents with two A 79-year-old gentleman on apixaban for atrial
months of gnawing abdominal pain. He has a fibrillation presents to the emergency department
history of migraines for which he takes ibuprofen with one day of light headedness and dark stools.
with good effect. Suspecting NSAID induced His initial vital signs include a supine blood
gastritis, erosions or ulcers you tell him: pressure of 110/60 mm Hg, heart rate of 98 bpm,
and respiratory rate of 12 per minute. His labs
A. He should discuss with his primary care show a BUN of 30 mg/dL and a hgb of 9g/dl. After
a switch to a different (non-NSAID) agent to obtaining adequate IV access your next step is to:
manage his migraines
B. He should switch to an enteric coated NSAID A. Transfuse platelets
C. The incidence of H pylori is increasing so he B. Transfuse prbc to a goal of 10g/dl
should be tested C. Start IV fluids
D. He needs an urgent upper endoscopy D. Transfuse FFP to reverse the anticoagulation
RATIONALE REFERENCES
The patient likely has a gastric outlet obstruction Fisher JC, Masiakos PT, Oviedo J, et al. Gastric
(GOO) secondary to the large ulcer which may outlet obstruction as a consequence of a duodenal
have caused edema and associated deformity. web masquerading as gastrinoma in an adult. Curr
While PPI should be started the next step would Surg 2003;60:602–6.
Chapter 2 — Acid diseases of the stomach 45
Malfertheiner P, Bayerdorffer E, Diete U, Gil stool.” You decide not to correct him and instead
J, Lind T, Misiuna P, et al. The GU-MACH study: inquire about the patient.
the effect of 1-week omeprazole triple therapy
on Helicobacter pylori infection in patients with Which piece of information will confirm your
gastric ulcer. Aliment Pharmacol Ther. 1999;13: suspicion of a stress ulcer?
703–712.
A. The patient was on a heparin drip for a DVT
B. She was intubated and placed on a ventilator
Question 36 last night for “respiratory failure”
A 59-year-old gentleman presents with one month C. This evening, she was started on
of abdominal pain. He notes a “few pounds” of hydrocortisone 50mg IV Q8H for suspected
weight loss but is “trying to eat better.” Your next adrenal insufficiency
step is: D. Her platelets are 21,000
A. Obtain a CT scan to evaluate for other causes Which of her medications puts her at the highest
of her pain. risk for PUD?
B. Surgical referral
C. Add on Carafate A. Iron
D. Start a neuromodulator B. Sumatriptan
C. Alendronate
CORRECT ANSWER: B D. Tamoxifen
undertaken to confirm healing (non-healing is the side effects of these medications and is asking
suspicious for malignancy). Ulcers in ZES are you your opinion: “will I go crazy or have a heart
typically post bulbar and serum gastrin level, at attack?!” You tell her:
this point, will not yet inform management.
A. There are some evidence that PPI therapy may
REFERENCE lead to dementia in certain populations
Kim HU. Diagnostic and treatment approaches B. Long term PPI therapy has been shown to
for refractory peptic ulcers. Clin Endosc. 2015;48: mildly increase the risk of heart disease
285-90. C. PPI therapy can cause C difficile infections in
critically ill patients (not her)
D. She may have some benign polyps in her
Question 43 stomach but nothing else
A 39-year-old lady presents with four months of E. There are potential risks with any long term
epigastric abdominal pain that are not responding medication and we will taper you down to the
to daily PPI, prescribed by her primary care. She lowest effective dose
undergoes an upper endoscopy which shows a
normal esophagus, stomach, and duodenum (to CORRECT ANSWER: E
the second part). What is the next best step?
RATIONALE
A. Refer for pH (reflux) testing PPI therapy has not been shown to be causative
B. increase PPI to twice daily for heart disease, dementia, pneumonia, C
C. Add on a night time H2 blocker difficile, or various other disorders and diseases
D. Start amitriptyline mentioned in the media. While there may be an
association in some cases the evidence is weak
CORRECT ANSWER: D to moderate at best. In general, these medicines
should only be used when necessary (this patient
RATIONALE has erosive esophagitis) and tapered to the lowest
This patient likely has non ulcer dyspepsia as effective (and necessary) dose.
demonstrated by her lack of response to PPI and
normal appearing endoscopy. It is important to note REFERENCE
that these patients typically do not respond to PPI (as Complications of Proton Pump Inhibitor Therapy.
compared to those with erosive esophagitis or PUD). Vaezi M.F., Yang Y.-X., Howden C.W. (2017)
Therefore, a neuromodulator should be considered. Gastroenterology, 153 (1), pp. 35-48.
REFERENCE
Savarino E, Pohl D, Zentilin P, et al. Functional Question 45
heartburn has more in common with functional A 32-year-old, otherwise healthy, gentleman
dyspepsia than with non-erosive reflux disease. presents with abdominal pain and diarrhea.
Gut. 2009;58:1185-91. His primary care provider orders stool studies
and an H pylori breath test, both of which are
negative. He undergoes an upper endoscopy
Question 44 which shows severe erosive esophagitis and
You see a 45-year-old lady in follow up. She has multiple duodenal ulcers. He is started on PPI
been on 40mg of omeprazole daily for six months therapy for six weeks and undergoes a repeat
for reflux complicated by erosive esophagitis (seen upper endoscopy when he sees no improvement.
on upper endoscopy). She is very nervous about Your next best test is:
Chapter 2 — Acid diseases of the stomach 49
Question 51
A 52-year-old female comes to GI clinic reporting
a sour acid taste in her mouth and burning
sensation that occurs after eating. She has
noted some dysphagia to solids. but not liquids.
No history of food impaction. She undergoes
EGD and is found to have mild esophagitis. The
following is true regarding sex differences related
to GERD for men and women:
CORRECT ANSWER: B
52 Digestive Diseases Self-Education Program®
Answers & critiques
CHAPTER 3
53
54 Digestive Diseases Self-Education Program®
CORRECT ANSWER: B the etiology is unclear and/or who are having com-
plications from acute pancreatitis.
RATIONALE
Secretin is a hormone secreted into the blood REFERENCE
stream by the S cells of the small intestine in Tenner S, Baillie J, DeWitt J, Vege SS. American
response to duodenal pH. Secretin binds to the College of Gastroentrology Guideline: Manage-
basolateral membrane of the duct cells, which ment of Acute Pancreatitis. Am J Gastroenterol.
increases intracellular cAMP and calcium resulting 2013; 108: 1400-15.
in a bicarbonate rich fluid secretion into the pan-
creas ducts. Measurement of peak bicarbonate
concentration (Normal: greater than 80 meq/L) Question 4
in pancreas fluid is used to assess for pancreas Cholecystokinin is a hormone secreted from the en-
function. terocytes of the small intestine in response to a meal.
RATIONALE
In the search for etiology, the American College of Question 5
Gastroenterology guidelines recommend that trans- There are three major phases of pancreas
abdominal ultrasound be performed in all patients secretion: cephalic, gastric and intestinal.
with acute pancreatitis (strong recommendation, The intestinal phase of pancreas secretion
low quality of evidence). It is recommended that CT associated with a meal accounts for what degree
and MRI imaging be reserved for patients in whom of pancreas secretion?
Chapter 3 — Pancreatic physiology & disease 55
A. 10 percent RATIONALE
B. 20 percent In a RCT, Lactated Ringer’s has been shown to de-
C. 80 percent crease systemic inflammation in acute pancreatitis.
D. 100 percent Lactated ringers has been shown to decrease blood
E. 30 percent CRP levels when given as a resuscitation fluid in
acute pancreatitis. The mechanism involves altera-
CORRECT ANSWER: C tions in the GPR81-receptor. The American College
of Gastroenterology, as well as American Gastroen-
RATIONALE terological Association Guidelines, recommend goal
The cephalic phase (10 percent of total pancreas directed therapy with lactated ringers as first line
secretion) occurs due to the sight, smell or taste treatment for acute pancreatitis.
of food. The gastric phase (~20 to 25 percent of
total pancreas secretion) occurs as food enters REFERENCES
and distends the stomach. The intestinal phase, Wu, B et al., Lactated Ringer’s solution reduces
which accounts for 70 to 90 percent of total secre- systemic inflammation compared with saline in
tion results when food particles and acidic gastric patients with acute pancreatitis. Clin Gastroen-
contents are delivered into the small intestine. terol Hepatol. 2011 Aug;9(8):710-717.
This stimulates Secretin and CCK release from the Lerch MM, et al., The anti-inflammasome
small bowel enterocytes resulting in a bicarbonate effect of lactate and the lactate GPR81-receptor in
and protein rich pancreas fluid. pancreatic and liver inflammation. Gastroenterol-
ogy. 2014 Jun;146(7):1602-5.
REFERENCE
Pandol, Stephen J. Sleisinger and Fordtran’s Gas-
trointestinal and Liver Disease, 10th ed. Chapter Question 7
56, 934-943. A 25-year-old female with abdominal pain under-
goes a CT scan in the emergency room. An inciden-
tal 2.5 cm pancreas cyst is seen. There is no history
Question 6 of pancreatitis and no family history of pancreas
A 53-year-old male presents with epigastric pain cancer. EUS reveals a “central stellate scar” and
radiating to his back, nausea and vomiting for 12 multiple small cysts in a honeycomb appearance.
hours. His BP is 100/60 mm Hg, heart rate 110 Aspiration of the cyst is negative for malignancy
bpm. Labs reveal a WBC of 12,000 (normal 4,000- with low CEA level and high glycogen levels.
10,000), Hemoglobin 17 g/dL (normal 13-17 g/
dL), normal hepatic profile, lipase 3,000 U/L What is the next best step in management?
(normal 0-150 U/L).
A. Observation
Which of the following resuscitation fluids should B. Surgical resection
be initiated? C. Endoscopic ablation
D. ERCP with Spyglass
A. Lactated Ringers E. Repeat EUS in one year
B. Normal Saline
C. Albumin bolus CORRECT ANSWER: A
D. Five percent dextrose in half normal saline
E. Hydroxyethyl starch RATIONALE
This is an asymptomatic cyst with EUS features
CORRECT ANSWER: A of a “classic” serous cystadenoma. These are NOT
56 Digestive Diseases Self-Education Program®
premalignant and do not require surveillance. a systematic review and meta-analysis. Clin
They also have been termed glycogen rich Gastroenterol Hepatol. 2014 Feb;12(2):219-28.
adenomas. Observation is indicated and manage-
ment will be dictated by symptoms associated
with cyst enlargement. Question 9
Secretin release by duodenal S-cells is required to
REFERENCE stimulate pancreatic bicarbonate secretion. Which
Elta, GH et al., ACG Clinical Guideline: Diagnosis of the following is the most important regulatory
and Management of Pancreatic Cysts. The American mechanism of duodenal secretin release?
Journal of Gastroenterology 113, 464-479 (2018).
A. PH less than 4.5
B. Short chain fatty acids
Question 8 C. Amino acids
Which of the following is frequently found in D. Disaccharides
patients with chronic pancreatitis? E. Cholecystokinin
Question 12
Question 11 Pancreatic acinar cell agonists stimulate digestive
A 21-year-old female underwent ERCP for enzyme secretion via several receptor-mediated
choledocholithiasis. Biliary cannulation was mechanisms.
difficult and a needle-knife sphincterotomy
was performed. Stones were removed and Which of the following intracellular events result
occlusion cholangiogram did not reveal any f from pancreatic acinar cell stimulation?
illing defects. The next morning, the patient
presents with severe epigastric abdominal A. GRP decreases cyclic AMP
pain, nausea and vomiting. Her lipase is B. CCK increases cyclic AMP
4000 IU/L (normal 0-60 IU/L). C. Secretin increases cyclic AMP
D. VIP increases calcium
What is the next best step in management? E. Acetylcholine decreases calcium
Question 13 Question 14
A 54-year-old alcoholic male with a history of recur- A 64-year-old male with a recent history of acute
rent acute pancreatitis undergoes a CT scan for pancreatitis has a dilated main pancreatic duct
abdominal pain that reveals pancreatic parenchymal with prominent side branch lesions seen on CT
calcifications and a dilated main pancreatic duct of scan. Endoscopic evaluation reveals mucus extrud-
11 mm. There is no evidence of a pancreatic mass or ing from a dilated ampulla.
evidence of duodenal obstructive symptoms.
What is the next best step in management of this
What is the best long term management of pain in patient?
this patient with chronic pancreatitis?
A. EUS +/- FNA
A. Pancreaticoduodenectomy B. ERCP with pancreatic stent
(Whipple procedure) C. Surgical resection
B. Lateral pancreatico-jejunostomy D. Repeat MRI in three years
(Peustow) procedure E. ERCP with Spyglass
C. Total pancreatectomy with islet
autotransplantation (TPIAT) CORRECT ANSWER: C
D. Distal pancreatectomy
E. ERCP with pancreatic duct stent placement RATIONALE
This patient has a main duct IPMN, which has
CORRECT ANSWER: B a high potential for malignant transformation
and should be resected if possible. Resection is
RATIONALE also indicated for branch-duct IPMN’s which are
Surgical resection / drainage is the best long- symptomatic (e.g. pancreatitis), associated with
term option for symptomatic “large duct” chronic obstructive jaundice or main duct involvement,
pancreatitis. Patients with a diffusely dilated duct have a solid component within the cyst, or have
(as above), are the best candidates for a drain- concerning features on EUS-FNA.
age procedure, such as the lateral pancreatico-
jejunostomy. A randomized controlled trial REFERENCE
compared lateral pancreatico-jejunostomy to Elta, GH et al, ACG Clinical Guideline: Diagnosis
ERCP with pancreatic stenting (+/- lithotripsy), and Management of Pancreatic Cysts. The Ameri-
and found that surgical treatment led to improved can Journal of Gastroenterology 113, 464-479
pain control at 24 months follow-up. Patients (2018).
with focal ductal disease and pain can be managed
by surgical resection of the diseased ductal seg-
ment. (Whipple, distal pancreatectomy). Chronic Question 15
pancreatitis patients with “small duct” disease are A 23-year-old female has a childhood history of
often times considered for TPIAT. recurrent acute pancreatitis and family history
of chronic pancreatitis in several relatives. The
REFERENCES patient does not have any evidence of diabetes
Cahen, DL et al. Endoscoic versus surgical drain- and has chronic abdominal pain. Genetic test-
age of the pancreatic duct in chronic pancreatitis. ing reveals a PRSS1 gene mutation. Radiologic
N Engl J Med 2007; 356(7):676-84. imaging is consistent with small duct chronic
Trikudanathan G, et al. Modern treatment of pancreatitis.
patients with chronic pancreatitis. Gastroenterol
Clin North Am. 2012; Mar;41(1):63-76. What is the next best step in management?
Chapter 3 — Pancreatic physiology & disease 59
REFERENCES Question 20
Jensen, Robert T et al, Sleisinger and Fordtran’s A 62-year-old female presents to the emergency
Gastrointestinal and Liver Disease, 10th ed. Chap- department for left lower quadrant pain and
ter 33, 501-541. fever. A CT scan reveals sigmoid diverticulitis,
Metz DC, Jensent RT. Gastrointestinal neuro- and a cyst in the body of the pancreas. The cyst
endocrine tumors: pancreatic endocrine tumors. measures 2.5cm and the pancreatic duct is dilated
Gastroenterology 2008;135(5):1469–92. ` to 7mm in the body and tail.
Chapter 3 — Pancreatic physiology & disease 61
REFERENCE Question 26
Glazer ES, et al., Sytematic Review and Case A 31-year-old female with history of alcohol
Series Report of Acinar Cell Carcinoma of the abuse presented at an outside hospital two
Pancreas. Cancer control 2016; 23 (4): 446-454. months ago with acute pancreatitis. Her hospital
course was complicated by necrotizing pancre-
atitis and questionable pancreatic duct leak. She
Question 25 now presents to see you for persistent abdominal
A 56-year-old male with known chronic pancre- pain, early satiety and nausea, vomiting. Labs
atitis presents with progressive abdominal pain, reveal WBC 9,000 cells/mm3, AST 52 U/L, ALT
weight loss and obstructive jaundice and a bili- 30 U/L, alk phos 90 U/L, TB 1.5 mg/dl
rubin of eight. A CT scan with contrast reveals
a four cm mass in the pancreas head. There is You get a repeat CT with contrast as below.
no lymphadenopathy and vascular architecture
is maintained. What is the next best step in
management?
A. Pancreaticoduodenectomy
(Whipple procedure)
B. Lateral pancreaticojejunostomy procedure
(Peustow) procedure
C. EUS +/- FNA
D. MRI/MRCP
E. ERCP with bile duct brushing and stent
CORRECT ANSWER: A
64 Digestive Diseases Self-Education Program®
A. Pancreatic ascites
B. Pancreatic pseudocyst
C. Walled off pancreatic necrosis
D. Acute pancreatic fluid collection
E. Acute pancreatic necrosis
CORRECT ANSWER: C
RATIONALE
The revised Atlanta classification delineates the
pancreatic cyst collections into a) acute peripan-
creatic fluid collection seen in the first four weeks,
non-encapsulated peripancreatic fluid collections MRI:
b) Pseudocysts develop after four weeks and well
encapsulated c) acute necrotic collection occurs
less than four weeks from the acute pancreatitis d)
walled off necrosis is after four weeks, encapsulated
necrosis as in this patient.
REFERENCE
Banks PA, Bollen TL, Dervenis C, et al. Classifica-
tion of acute pancreatitis - 2012: revisions of the
Atlanta classification and definitions by interna-
tional consensus. Gut 2013; 62:102.
Question 29
A 60-year-old healthy male undergoes a non-
contrast CT for further evaluation of flank pain.
CT confirms nephrolithiasis, but also reveals
multiple possible cysts in the pancreas and MRI
is recommended for further evaluation. MRI as
below reveals multiple subcentimeter cysts in the
pancreatic body, with a 3.5 cm cyst in the head of
the pancreas, with a 1.5 cm solid mural nodule in
the cyst. The PD is prominent in the head of the
pancreas and measured four mm in diameter.
(See figure on the following page.)
What is the most likely diagnosis?
A. Serous cystadenoma
B. Mucinous cystic neoplasm
C. Intraductal papillary mucinous neoplasm
66 Digestive Diseases Self-Education Program®
Question 30
A 60-year-old female undergoes a CT for further
evaluation of hematuria. She is incidentally found
to have a 15 mm cyst in the tail of the pancreas.
A. EUS FNA
B. ERCP
C. Surveillance MRI in one year
D. Surgery
E. Surveillance MRI in three months
CORRECT ANSWER: A
She denies any history of pancreatitis or family
RATIONALE history of pancreas cancer. She is a non-smoker
Size greater than three cm, mural nodule and dilated and is otherwise healthy without any comorbidi-
pancreatic duct are all high risk characteristics for ties. She denies any weight loss and the CBD and
mucinous cyst. Thus, this patient should be referred PD on the CT scan are normal. The cyst does not
to a multidisciplinary group and undergo EUS-FNA have a solid component on the scan. Which is the
for further characterization of the cyst, with analysis most appropriate next step?
of the cyst fluid cytology and chemistry (fluid CEA
and amylase levels). Cyst internal septations, pres- A. EUS FNA now
ence of debris, papillary projections and mural nod- B. ERCP now
ules can be visualized effectively with EUS. Presence C. MRI in one year
of vascular invasion, pancreatic duct communication D. CT in one year
and lymph node metastases can be evaluated. EUS E. Surgery referral
also allows for FNA of the high risk cystic lesions
for biochemical, cytological and DNA analysis that CORRECT ANSWER: C
might further aid diagnosis, characterization and
differentiation of the pancreatic cysts. RATIONALE
Per the 2018 ACG guidelines, incidentally found
REFERENCES one to two cm cysts without a solid component,
Grace H. Elta, Brintha K. Enestvedt, Bryan G. Sauer non-dilated PD, less than three cm can be sur-
and Anne Marie Lennon. ACG Clinical Guideline: veyed with MRI in one year. If no change, then
Diagnosis and Management of Pancreatic Cysts. MRI every one years for a total of three years is
Am J Gastroenterol. 2018 Apr;113(4):464-479. recommended. If cyst remains stable, then MRI
Chapter 3 — Pancreatic physiology & disease 67
REFERENCE
Krishna SG, Rao BB, Ugbarugba E, et al. Diag-
nostic performance of endoscopic ultrasound for
detection of pancreatic malignancy following an
indeterminate multidetector CT scan: a systemic
review and meta-analysis. Surg Endosc. 2017
Nov;31(11):4558-4567.
Question 33
60-year-old male with known history of alcohol in-
A. Refer to surgery for duced chronic pancreatitis with chronic abdominal
pancreaticoduodenectomy pain well controlled on pancreatic enzyme supple-
(Whipple’s surgery) ment presents for follow up visit. He has been
B. Check CA 19-9 abstinent from drinking for past 10 years. He was
C. EUS FNA well until three months ago, when he started to
D. MRI lose weight and now reports a 20 lb unintentional
E. Refer to oncology weight loss. He reports a 20 pack-year smoking
history. He denies any abdominal pain, but reports
CORRECT ANSWER: C polyuria. His labs reveal normal CBC and basic
chemistry, except a fasting blood sugar of 180 mg/
RATIONALE dL. He has a past medical history of HTN.
Pancreatic cancer is the fourth leading cause of
cancer-related death in the United States, with a What is the next best step?
dismal five-year relative survival rate of approxi-
mately eight percent. This is mostly due to delay A. Increase pancreatic enzymes dose
in diagnosis. Despite improvement in the cross- B. Add PPI to enzymes
Chapter 3 — Pancreatic physiology & disease 69
REFERENCES RATIONALE
Kirkegård J, Mortensen FV, Cronin-Fenton D. Patients with gallstone pancreatitis with clini-
Chronic Pancreatitis and Pancreatic Cancer Risk: cal deterioration and clinical picture suggestive
A Systematic Review and Meta-analysis. Am J of ascending cholangitis should undergo urgent
Gastroenterol. 2017 Sep;112(9):1366-1372. ERCP. Patients with mild biliary pancreatitis with
Raimondi S1, Lowenfels AB, Morselli-Labate no signs of biliary obstruction should undergo
AM, Maisonneuve P, Pezzilli R. Pancreatic cancer interval cholecystectomy as soon as possible
in chronic pancreatitis; aetiology, incidence, and once patient improves, ideally during the same
early detection. Best Pract Res Clin Gastroenterol. hospitalization. ERCP in patients with biliary
2010 Jun;24(3):349-58. pancreatitis without evidence of biliary obstruc-
Conwell DL1, Lee LS, Yadav D, Longnecker tion or cholangitis is not necessary. This patient
DS, Miller FH, Mortele KJ, Levy MJ, Kwon R, Lieb has developed ascending cholangitis and thus will
JG, Stevens T, Toskes PP, Gardner TB, Gelrud A, benefit from an urgent ERCP.
Wu BU, Forsmark CE, Vege SS. American Pan-
creatic Association Practice Guidelines in Chronic REFERENCES
Pancreatitis: evidence-based report on diagnostic Folsch UR, Nitsche R, Ludtke R, Hilgers RA,
guidelines. Pancreas. 2014 Nov;43(8):1143-62. Creutzfeldt W. Early ERCP and papillotomy com-
70 Digestive Diseases Self-Education Program®
Question 36
A 29-year-old male presents with severe epigastric
pain, nausea and vomiting for the past eight hours.
His heart rate is 110 beats/min, BP 105/60 mm
Hg. Labs reveal lipase 4,000 U/L, AST 80 U/L,
ALT 1000 U/L, TB 0.6 mg/dL, alkaline phospha-
tase 90 U/L, WBC 12,000 cells/mm3, Hemoglobin
16.6 mg/dL, Hematocrit 50 percent, BUN 55 mg/
dL and creatinine 1.2 mg/dL. Ultrasound showed
multiple gallstones in the gallbladder, with a non-
dilated common bile duct.
What is the next best step in the management of
the patient’s pseudocyst? What is the best next step?
Chapter 3 — Pancreatic physiology & disease 71
Treatment with cyanoacrylate injection is Atrophic gastritis). In order for intrinsic factor
a treatment option for bleeding gastric varices; to bind vitamin B12, B12 must first be released
while this patient does have small gastric varices, from binding with the R-protein, which occurs via
there are no stigmata and another source was pancreatic protease breakdown of the R-protein.
found, so it would not be appropriate to treat Patients with chronic pancreatitis are not able to
them. Nuclear medicine tagged red blood cell break down the R-protein as efficiently, and thus
scan has no role when the site of bleeding is found can develop vitamin B12 deficiency.
endoscopically.
REFERENCES
REFERENCE Green R. Vitamin B12 deficiency from the perspec-
Yu, P and Gong, J. Hemosuccus pancreaticus: A tive of a practicing hematologist. Blood. 2017; May;
mini-review. Ann Med Surg (Lond). 2018 Apr; 28: 129 (19):2603-2611.
45–48. Gueant GL at al. Malabsorption of vitamin B12
in pancreatic insufficiency of the adult and of the
child. Pancreas 1990 Sep;5(5):559-67.
Question 43
A 65-year-old male with chronic pancreatitis
related to long-standing alcohol use comes to see Question 44
you for a second opinion. He has been abstinent Which of the following conditions carries the
from alcohol for twenty years. He reports a one greatest risk for the development of pancreatic
year history of six loose, oily stools per day, but adenocarcinoma?
minimal abdominal pain. He was recently found
to have vitamin B12 deficiency by his PCP. What is A. Peutz-Jeghers syndrome
the likely mechanism for this patient’s vitamin B12 B. BRCA-2 mutation
deficiency? C. Lynch syndrome
D. PRSS1 mutation
A. Antibodies to intrinsic factor E. Cystic fibrosis
B. Atrophy of gastric lining due to chronic
alcohol use CORRECT ANSWER: D
C. Reduced breakdown of the R-protein by
pancreatic proteases RATIONALE
D. Decreased absorption of vitamin B12 in the The PRSS1 gene encodes cationic trypsinogen.
ileum due to chronic diarrhea A mutation in this gene causes hereditary pan-
E. Loss of vitamin B12 in stool, due to steatorrhea creatitis, which carries an approximately 50-fold
increased risk of developing pancreatic adeno-
CORRECT ANSWER: C carcinoma (approximately 50 percent chance).
Peutz-Jeghers patients have an 11 to 36 percent
RATIONALE risk of developing pancreatic adenocarcinoma in
Vitamin B12 absorption requires intrinsic fac- their lifetime. BRCA-2 carriers have about a five
tor to bind to B12 to facilitate absorption in the percent risk of pancreatic cancer. Lynch Syndrome
terminal ileum. Any interruption of terminal ileal patients have an approximately two to 18 percent
absorptive capacity can thus lead to vitamin B12 lifetime risk of pancreatic cancer, depending on
deficiency (eg. Crohn’s disease, ileal resection). which mismatch repair gene is mutated. Patients
Intrinsic factor is produced by parietal cells, so any with cystic fibrosis have a very high likelihood (80
condition that leads to decreased parietal cell mass to 85 percent) of developing pancreatic insuf-
or function can lead to vitamin B12 deficiency (eg. ficiency, and may develop overt pancreatitis, but
76 Digestive Diseases Self-Education Program®
there is no specific association with pancreatic limited data regarding the use of steroid-sparing
adenocarcinoma. agents in patients with Type 2 AIP.
REFERENCES REFERENCES
Klein AP. Genetic susceptibility to pancreatic can- Sah RP,Chari ST, Pannala R et al. Differences in
cer. Mol carcinog 2012;51: 14. clinical profile and relapse rate of type 1 versus
Lowenfels AB, Maisonneuve P, DiMagno EP et type 2 autoimmune pancreatitis. Gastroenterol-
al. Hereditary pancreatitis and the risk of pancre- ogy. 2010;139 (1): 140.
atic cancer. International Hereditary Pancreatitis O’Reilly DA, Malde DJ, Duncan T, et al. Re-
Study Group. J Natl Cancer Inst. 1997;89 (6):442. view of the diagnosis, classification and manage-
ment of autoimmune pancreatitis. World J Gastro-
intest Pathophysiol. 2014 May 15; 5(2): 71–81.
Question 45
A 24-year-old man with ulcerative colitis in remis-
sion on vedolizumab presents with acute pancre- Question 46
atitis. Labs reveal a lipase 875 U/mL (normal 0-60 A 27-year-old man with Crohn’s ileocolitis on inf-
U/L), AST 35 U/L, ALT 45 U/L, total bilirubin 2.5 liximab and azathioprine for the past two months
mg/dL, alkaline phosphatase 200 U/L, WBC 9,000 presents with severe epigastric pain radiating to
cells/mm3. CT scan reveals a mass in the pancre- his back and nausea. He does not smoke or drink
atic head with mild biliary and pancreatic ductal any alcohol. He has no family history of pancreati-
dilation. EUS-FNA reveals a granulocytic epithelial tis or inflammatory bowel disease. His vital signs
lesion without any evidence of malignancy. What is are normal. Lipase is 3,000 U/L and hepatic panel
the next step in the management of this patient? is normal. Ultrasound shows a normal gallbladder
without stones and no biliary dilation. What is the
A. Whipple procedure likely etiology of his acute pancreatitis?
B. PET-CT scan
C. Laparoscopy A. Alcohol
D. Initiation of glucocorticoids B. Gallstones
E. Switch to infliximab C. Azathioprine
D. Duodenal Crohn’s disease
CORRECT ANSWER: D E. SPINK1 mutation
REFERENCES REFERENCE
Badalov N, Baradarian R, Kadirawel I, et al. Drug- Modlin IM, Oberg K et al. Gastroenteropancreatic
induced acute pancreatitis: an evidence-based neuroendocrine tumours. Lancet Oncology 2008;
review. Clin Gastroenterol Hepatol 2007;5: 648. 9: 61-72.
Question 47 Question 48
A 63-year-old man with recently diagnosed A 36-year-old man presents with severe acute
diabetes presents with diarrhea and a new rash. pancreatitis. His initial vital signs are: tempera-
The rash is located around his mouth and on his ture 37 degrees C, heart rate of 115 beats/minute
buttocks and groin. The rash appears erythema- and blood pressure of 89/50 mm Hg. Labs reveal
tous and there are multiple vesicles; some fresh a white blood cell count of 18,000 cells/mm3,
and others crusting over. He has lost 26 pounds in Hct of 50 percent, lipase 3000 U/L, creatinine
the past two months unintentionally, so a CT scan of 1.5 mg/dL, normal hepatic function panel. A
was done, which reveals a three cm solid lesion in CT scan reveals significant fluid and stranding
the body of the pancreas. What is the most likely around the pancreas, with an area in the pancre-
diagnosis? atic body that is not enhancing. The gallbladder
is normal and there is no biliary dilation. He is
A. Insulinoma started on aggressive IV fluid hydration with
B. Pancreatic adenocarcinoma lactated ringers.
C. Glucagonoma
D. VIPoma What is the next best step?
E. Pancreatic Lymphoma
A. Initiate broad spectrum antibiotics and
CORRECT ANSWER: C aggressive intravenous fluid hydration
B. Start TPN via central venous access
RATIONALE C. Keep NPO, monitor hematocrit and BUN/
Glucagonoma is a rare neuroendocrine tumor aris- creatinine and adjust fluid accordingly
ing from pancreatic alpha-2 cells of the islets. This D. Perform ERCP
tumor secretes high levels of glucagon, which lead E. Perform endoscopic ultrasound followed by
to insulin resistance, diarrhea, and weight loss. ERCP if stone is confirmed in bile duct
Necrolytic migratory erythema is the characteris-
tic rash located on the limbs, around the lips and CORRECT ANSWER: C
groin/perineum/buttocks, which is seen in 70 to
80 percent of patients with this particular tumor. RATIONALE
NME is described as a crusting, erythematous, ve- This patient has evidence of acute necrotizing pan-
sicular rash, and is often the presenting symptom. creatitis. Antibiotics are not recommended unless
Insulinoma causes hypoglycemia and can there is evidence of acute cholangitis or infection
often present with seizures or other sequelae of elsewhere. This patient does meet SIRS criteria
hypoglycemia. It is the most common pancreatic due to the severity of his pancreatitis, he is afebrile
endocrine tumor. VIPoma causes significant secre- and there is no evidence of cholangitis, so antibi-
tory diarrhea, often leading to dehydration and otics are unnecessary.
hypokalemia. Pancreatic lymphoma can often be The mainstay of therapy for acute pancre-
confused for adenocarcinoma, but does not cause atitis is goal-directed fluid therapy, with close
any symptoms of hormonal hypersecretion and monitoring of hemoglobin/hematocrit and BUN/
would not present as this patient did. Creatinine to ensure adequate fluid hydration and
78 Digestive Diseases Self-Education Program®
REFERENCES
Teich N, Rosendahl J, Toth M, et al. Mutations of
human cationic trypsinogen (PRSS1) and chronic
pancreatitis. Hum Mutat 2006;27(8):721-730.
NIH Genetics Home REFERENCE-https://
ghr.nlm.nih.gov/condition/hereditary-pancreatitis
80 Digestive Diseases Self-Education Program®
Answers & critiques
CHAPTER 4
Question 1 RATIONALE
A 45-year-old man undergoes an upper endos- The patient was found to have an incidental
copy for further evaluation of reflux. At the time sporadic ampullary adenoma. These adenomas
of his EGD, an abnormality is seen at his am- can progress to adenocarcinoma and should be
pulla. No additional lesions are seen within the resected. Historically pancreaticoduodenectomy
examined duodenum. Further evaluation with a or a trans duodenal ampullectomy were the
side viewer reveals the abnormality noted in the treatment of choice. However, these procedures
image. Biopsies are obtained and return positive are associated with significant morbidity and
for adenoma without high grade dysplasia. Labs mortality, and endoscopic removal for non-in-
reveal a total bilirubin of 1.0 mg/dL, alkaline vasive ampullary adenomas is preferred. In this
phosphatase of 67 IU/L, AST 15 IU/L, ALT 18 scenario, the adenoma is greater than or equal
IU/L. An EUS examination confirms a 15 mm to one cm but not large (less than 4cm) and is
ampullary lesion without invasion of the mus- without ductal involvement, endoscopic resec-
cularis propria, common bile duct or pancreas tion should be considered as first line therapy.
duct. The next best step in management is: If, at the time of attempted endoscopic resec-
tion, the lesion does not lift, surgical resection
could be considered. Given that the adenoma
could harbor underlying carcinoma, which may
not have been diagnosed by pinch biopsies,
and the relatively young age and health of the
patient, upfront resection would be favored over
surveillance.
REFERENCES
Ardengh, J.C., et al., Endoscopic papillectomy:
The limits of the indication, technique and re-
sults. World J Gastrointest Endosc, 2015. 7(10):
A. Surveillance with side viewing exam in p. 987-94.
one year Committee, A.S.o.P., et al., The role of en-
B. Transduodenal ampullectomy doscopy in ampullary and duodenal adenomas.
C. Endoscopic resection Gastrointest Endosc, 2015. 82(5): p. 773-81.
D. No further intervention is required El, H., II and G.A. Cote, Endoscopic diag-
E. Pancreaticoduodenectomy nosis and management of ampullary lesions.
Gastrointest Endosc Clin N Am, 2013. 23(1): p.
CORRECT ANSWER: C 95-109.
81
82 Digestive Diseases Self-Education Program®
Question 3
A. Surveillance with side viewing exam in A 54-year-old woman presents to your clinic for
one year further nausea, vomiting and jaundice. On exam,
B. Transduodenal (surgical) ampullectomy her abdomen is soft, non-tender and not distend-
C. Endoscopic resection ed. Her sclerae are anicteric. Laboratory studies
D. No further intervention is required are obtained and
E. Pancreaticoduodenectomy reveal a total
bilirubin of 1.5
CORRECT ANSWER: E mg/dL, alkaline
phosphatase of
RATIONALE 24 IU/L, AST
Given the rarity of ampullary neuroendocrine 18 IU/L, ALT 19
tumors limited data exists regarding the best ap- IU/L. Her MRCP
proach to this type of tumor. Pancreaticoduode- image is shown
nectomy is recommended for all ampullary neuro- here.
endocrine tumors given the high rate of metastases
seen even with small lesions. Transduodenal For definitive
ampullectomy is felt to offer an inadequate onco- management,
logical resection and generally has not been recom- you recommend:
mended as the first line approach to ampullary
Chapter 4 — Diseases of the biliary tract 83
Khashab, M.A., et al., Delayed and unsuc- for confirmation of the diagnosis, if the fluid is
cessful endoscopic retrograde cholangiopancrea- analyzed for bile, as well as source control. While
tography are associated with worse outcomes in ERCP has been shown to be highly effective for
patients with acute cholangitis. Clin Gastroenterol treatment of bile leaks, it will do little to hasten an
Hepatol, 2012. 10(10): p. 1157-61. already formed symptomatic fluid collection that
Kiriyama, S., et al., Tokyo Guidelines 2018: may or may not be communicating with the duct.
diagnostic criteria and severity grading of acute Surgical revision should be considered if percu-
cholangitis (with videos). J Hepatobiliary Pancreat taneous drainage and or ERCP fail to resolve the
Sci, 2018. 25(1): p. 17-30. leak, or if a transection injury occurs, as this is
unlikely to be amenable to endoscopic therapy.
Question 5 REFERENCES
A 35-year-old woman who underwent cholecystec- Ahmad, F., et al., An algorithm for the manage-
tomy three days previously presents to the ED for ment of bile leak following laparoscopic cholecys-
further evaluation of RUQ abdominal pain and fe- tectomy. Ann R Coll Surg Engl, 2007. 89(1):
vers. On physical exam her abdomen is tender in the p. 51-6.
RUQ with guarding and rebound. Labs reveal a total Baillie, J., Endoscopic approach to the patient
bilirubin of 2 mg/dL, alkaline phosphatase of 150 with bile duct injury. Gastrointest Endosc Clin N
IU/L, AST 60 IU/L, ALT 82 IU/L, lipase 50 U/L (50 Am, 2013. 23(2): p. 461-72.
ULN), WBC 16.4 10^9/L. Cross sectional imaging is Nunez, D., Jr., J.L. Becerra, and L.C. Martin,
performed and reveals this finding seen here. Subhepatic collections complicating laparoscopic
cholecystectomy: percutaneous management
Abdom Imaging, 1994. 19(3): p. 248-50.
Saraswat, V.A., et al., Endoscopic manage-
ment of postoperative bile leak. J Gastroenterol
Hepatol, 1996. 11(2): p. 148-51.
Question 6
A 53-year-old woman presents to the ED for further
management of persistent right upper quadrant
pain. She is normotensive and afebrile. On exam,
The next best step in management is: she is tender to palpation in the RUQ with guarding
but no rebound. Labs reveal a total bilirubin of 1.7
A. ERCP mg/dL, alkaline phosphatase of 80 IU/L, AST 35
B. Percutaneous drainage tube placement IU/L, ALT of 36 IU/L, lipase of 15 U (ULN 50) and
C. Surgical revision WBC of 11.2 10^9/L. A RUQ ultrasound is obtained
D. No further intervention is needed and reveals a CBD of 5 mm, a thickened gallbladder
E. EUS wall, pericholecystic fluid and cholelithiasis. The
next best step in management is:
CORRECT ANSWER: B
A. ERCP
RATIONALE B. EUS
This patient has a symptomatic bile leak post cho- C. MRCP
lecystectomy and percutaneous drainage should D. No further management necessary
be performed. Percutaneous drainage will allow E. Surgical referral for cholecystectomy
Chapter 4 — Diseases of the biliary tract 85
Freeman, M.L., Complications of endoscopic biliary sphincter of Oddi disorder. Her pain is
retrograde cholangiopancreatography: avoidance consistent with biliary type pain however she does
and management. Gastrointest Endosc Clin N Am, not have abnormal liver function tests or biliary
2012. 22(3): p. 567-86. ductal dilation. This patient should not undergo
Geenen, J.E., et al., The efficacy of endoscopic an ERCP for suspected sphincter of Oddi disorder.
sphincterotomy after cholecystectomy in patients The EPISOD trial demonstrated a 10-15 percent
with sphincter-of-Oddi dysfunction. N Engl J rate of post ERCP pancreatitis in expert hands.
Med, 1989. 320(2): p. 82-7. Prior studies have suggested rates of post ERCP
Toouli, J., et al., Manometry based randomised pancreatitis as high as 30 percent in those with
trial of endoscopic sphincterotomy for sphincter of suspected sphincter of Oddi dysfunction.
Oddi dysfunction. Gut, 2000. 46(1): p. 98-102.
REFERENCES
Cotton, P.B., et al., Effect of endoscopic sphincter-
Question 8 otomy for suspected sphincter of Oddi dysfunction
A 40-year-old woman presents to your clinic for on pain-related disability following cholecystecto-
further evaluation of RUQ abdominal pain. The my: the EPISOD randomized clinical trial. JAMA,
patient reports intermittent RUQ pain occurring 2014. 311(20): p. 2101-9.
several times per month. At its most severe, it is Cotton, P.B., et al., Rome IV. Gallbladder and
a 10/10, and typically lasts two to three hours. Sphincter of Oddi Disorders. Gastroenterology,
She has not noted an improvement with bowel 2016.
movements or acid suppression. She underwent Freeman, M.L., Complications of endoscopic
cholecystectomy for similar pain one year prior retrograde cholangiopancreatography: avoidance
without improvement. LFTs have been obtained and management. Gastrointest Endosc Clin N Am,
during her pain episodes and reveal a total biliru- 2012. 22(3): p. 567-86.
bin of 1 mg/dL, alkaline phosphatase of 50 IU/L, Yaghoobi, M., et al., Incidence and predic-
AST 15 IU/L, ALT 25 IU/L, and lipase of 10 U tors of post-ERCP pancreatitis in patients with
(ULN 50 U). A CT scan was previously obtained suspected sphincter of Oddi dysfunction undergo-
and revealed a CBD which measured six mm. ing biliary or dual sphincterotomy: results from
EUS was performed and was unremarkable. The the EPISOD prospective multicenter randomized
patient reports she has researched online and she sham-controlled study. Endoscopy, 2015. 47(10):
is sure she has sphincter of Oddi dysfunction. She p. 884-90.
is adamant that you perform an ERCP. You inform
her that she does not fulfill criteria for sphincter of
Oddi dysfunction and that her risk of pancreatitis Question 9
following an ERCP is approximately: A 25-year-old man with a history of migraines
presents for further evaluation of intermittent
A. 0-five percent RUQ abdominal pain associated with meals for
B. 10-30 percent several years. He had undergone an abdominal
C. 45-60 percent ultrasound, CT scan of his abdomen and upper
D. 70-85 percent endoscopy without a clear etiology. Given concern
E. 90-100 percent for possible acalculous cholecystitis, he under-
went a cholecystectomy several years ago. His
CORRECT ANSWER: B AST and ALT have been mildly elevated in the
past but are currently normal. alkaline phospha-
RATIONALE tase is elevated at 134 IU/L. IgG4, AMA and im-
This patient does not fulfill criteria for functional munoglobulins are not elevated. HIV is negative.
Chapter 4 — Diseases of the biliary tract 87
REFERENCES
European Association for the Study of the, L.,
EASL Clinical Practice Guidelines: management of
cholestatic liver diseases. J Hepatol, 2009. 51(2):
p. 237-67.
Lindor, K.D., et al., ACG Clinical Guideline:
Primary Sclerosing Cholangitis. Am J Gastroen-
terol, 2015. 110(5): p. 646-59; quiz 660.
Palmela, C., et al., Inflammatory Bowel Dis-
ease and Primary Sclerosing Cholangitis: A Review
of the Phenotype and Associated Specific Features.
Gut Liver, 2018. 12(1): p. 17-29.
Razumilava, N., G.J. Gores, and K.D. Lin-
He should undergo what additional test as part of dor, Cancer surveillance in patients with primary
his work up of this condition? sclerosing cholangitis. Hepatology, 2011. 54(5): p.
1842-52.
A. Colonoscopy with biopsies Rossi, R.E., D. Conte, and S. Massironi,
B. Colonoscopy without biopsies Primary sclerosing cholangitis associated with
C. EUS inflammatory bowel disease: an update. Eur J
D. Upper endoscopy with biopsies Gastroenterol Hepatol, 2016. 28(2): p. 123-31.
E. Upper endoscopy without biopsies
You recommend: similar pain in the past which were not persistent.
On exam, she is tender to palpation in the RUQ
A. ERCP without rebound or guarding. Her sclerae are
B. Upper GI series icteric. Labs reveal a total bilirubin of 3.5 mg/dL,
C. EUS guided cystgastrostomy alkaline phosphatase of 200 IU/L, AST 200 IU/L,
D. Upper endoscopy ALT 190 IU/L, and lipase of 41 U (ULN 50 U) and
E. Supportive care with antibiotics and fluids WBC of 13.5 10^9/L. Her medical history is other-
wise notable for hypertension. You recommend:
CORRECT ANSWER: B
A. Right upper quadrant ultrasound
RATIONALE B. MRCP
This patient has a duodenal perforation and C. EUS
should undergo an upper GI series with gastro- D. ERCP
grafin to localize the site of the leak. Duodenal E. Cholecystectomy
perforation is a rare complication of cholecystec-
tomy occurring in 0.07 percent to 0.09 percent CORRECT ANSWER: A
of cholecystectomy cases. While rare, failure to
recognize this complication is associated with RATIONALE
significant mortality. The location of the fluid col- The differential for a patient with right up-
lection within the retroperitoneal space, as well per quadrant pain and abnormal liver function
as markedly elevated amylase level and elevated tests is broad and includes cholecystitis, biliary
bilirubin level raise concern for a possible perfo- obstruction, acute hepatitis, and viral gastro-
ration. Perforation is a relative contraindication enteritis. In this patient with prior symptoms
to endoscopic evaluation and additional imaging suggestive of biliary colic, cholecystitis or cho-
with an upper GI series should be performed prior ledocholithiasis are a concern. The next best test,
to an endoscopic evaluation. As this is a perfora- in a patient with right upper quadrant pain and
tion and not a bile leak, ERCP is of no benefit. elevated aminotransferase levels, bilirubin and
EUS guided cystgastrostomy would be appropriate alkaline phosphatase, would be a right upper
in the setting of a matured pseudocyst or WON quadrant ultrasound. While the sensitivity for
but, in the setting of a recent perforation, drainage detection of choledocholithiasis is approximately
into the stomach would not be of benefit. 50 percent, ultrasound has a high sensitivity
for cholelithiasis, bile duct dilation and GB wall
REFERENCES thickness. In addition, if the biliary tree appears
Croce, E., et al., Duodenal perforations after normal, doppler studies can be performed to
laparoscopic cholecystectomy. Surg Endosc, 1999. evaluate for possible underlying vascular etiolo-
13(5): p. 523-5. gies of pain and abnormal aminotransferases,
Machado, N.O., Duodenal injury post laparo- ALP or bilirubin. MRCP and EUS have a much
scopic cholecystectomy: Incidence, mechanism, higher sensitivity for choledocholithiasis, approx-
management and outcome. World J Gastrointest imately 92 percent and 95 percent, respectively,
Surg, 2016. 8(4): p. 335-44. than US or CT scan. These should be considered
in those patients who are at an intermediate risk
for choledocholithiasis rather than as a first line
Question 11 test for right upper quadrant pain with abnor-
A 46-year-old woman presents to the ED for fur- mal liver tests. ERCP and cholecystectomy may
ther evaluation of persistent right upper quadrant be necessary in this patient, but the underlying
pain. The patient reports intermittent episodes of etiology should first be determined.
Chapter 4 — Diseases of the biliary tract 89
Question 14 REFERENCES
A 28-year-old woman with short gut syndrome Lammert, F., et al., Gallstones. Nat Rev Dis
due to multiple small bowel resections for Primers, 2016. 2: p. 16024.
Crohn’s disease is found to have cholelithiasis Quigley, E.M., et al., Hepatobiliary compli-
on a right upper quadrant ultrasound performed cations of total parenteral nutrition. Gastroen-
for abdominal pain. She is TPN dependent due terology, 1993. 104(1): p. 286-301.
to her short gut syndrome. She receives inflix- Stokes, C.S., M. Krawczyk, and F. Lammert,
imab for her Crohn’s disease. The patient also Gallstones: environment, lifestyle and genes. Dig
takes an oral contraceptive. All of these are risk Dis, 2011. 29(2): p. 191-201.
Chapter 4 — Diseases of the biliary tract 91
Question 15 Question 16
A 36-year-old man with sickle cell disease A 40-year-old woman presents to the emergency
presented to the emergency department with department for further evaluation of right upper
abdominal pain. He previously underwent a cho- quadrant pain. She is noted to be febrile to 39C,
lecystectomy. On RUQ US, he is noted to have hypotensive with a BP of 80/40 mmHg, and
a dilated bile duct to 12 mm. His total bilirubin tachycardic with a HR of 120 bpm. Initial labs
is six mg/dL (indirect 4 mg/dL), ALP is 200 reveal a bilirubin of six mg/dL, ALP 250 IU/L,
IU/L, AST 84 IU/L, ALT 90 IU/L. He undergoes AST 500 IU/L, ALT 430 IU/L and lipase of 1500
ERCP. At the time of his ERCP, he is found to U (ULN 50). A right upper quadrant shows a
have choledocholithiasis. A single black stone is dilated bile duct to 11 mm with an eight millime-
removed. This stone is most likely composed of ter hyperechoic focus within the CBD as well as
what percentage of cholesterol: several hyperechoic foci within the gallbladder.
She is stabilized and undergoes ERCP with stone
A. 90-100 percent extraction. She does well post procedure. What
B. 60-80 percent is her risk of a recurrent biliary event if she does
C. 40-50 percent not undergo cholecystectomy?
D. 20-30 percent
A. 0-10 percent
CORRECT ANSWER: D B. 20-40 percent
C. 50-70 percent
RATIONALE D. 80-100 percent
This patient has formed a pigmented stone most
likely secondary to hemolysis due to his underly- CORRECT ANSWER: B
ing sickle cell disease. In this scenario hemolysis
leads to an excess of systemic bilirubin. Uncon- RATIONALE
jugated bilirubin is not water soluble and com- Currently no clear guidelines exist regarding
bines with calcium to form calcium bilirubinate optimal timing of cholecystectomy for choledo-
within the mucin glycoproteins that are secreted cholithiasis. In patients who have gallstone
by the gallbladder mucosa. Gallstones are clas- pancreatitis or choledocholithiasis, recurrent
sified into cholesterol gallstones and pigmented biliary events (biliary colic, recurrent gallstone
gallstones. Cholesterol gallstones contain more pancreatitis, choledocholithiasis) have been
than 50 percent cholesterol by weight. Pig- observed to occur in 18-40 percent patients if
mented gallstones are further subdivided into cholecystectomy is delayed beyond the initial
black and brown stones and contain less than 30 hospitalization. Interestingly, a recent retrospec-
percent cholesterol by weight. tive cohort study found that just over 50 percent
of patients with choledocholithiasis requiring
REFERENCES ERCP underwent cholecystectomy. Given the
Cariati, A., Gallstone Classification in Western clear risk and possibility of loss of follow up,
Countries. Indian J Surg, 2015. 77(Suppl 2): p. early cholecystectomy in surgically fit candidates
376-80. (prior to hospital discharge or within six weeks
Lammert, F., et al., Gallstones. Nat Rev Dis of discharge) should be recommended in this
Primers, 2016. 2: p. 16024. patient population.
Vitek, L. and M.C. Carey, New pathophysi-
ological concepts underlying pathogenesis of REFERENCES
pigment gallstones. Clin Res Hepatol Gastroen- Hernandez, V., et al., Recurrence of acute gall-
terol, 2012. 36(2): p. 122-9. stone pancreatitis and relationship with chole-
92 Digestive Diseases Self-Education Program®
cystectomy or endoscopic sphincterotomy. Am J small as three mm. In patients with polyps that
Gastroenterol, 2004. 99(12): p. 2417-23. are greater than or equal to 10 mm, cholecys-
Huang RJ, Barakat MT, Girotra M, Banerjee tectomy is recommended. If the polyp is smaller
S. Practice Patterns for Cholecystectomy After En- than 10 mm and the patient is felt to have symp-
doscopic Retrograde Cholangiopancreatography toms attributable to the polyp, cholecystectomy
for Patients With Choledocholithiasis. Gastroen- is suggested. If polyps are smaller than 10 mm
terology 2017;153:762-771.e2. without symptoms, ultrasound surveillance is
Reinders, J.S., et al., Early laparoscopic cho- recommended. In this patient, the polyp is larger
lecystectomy improves outcomes after endoscopic than 10 mm and he should undergo a cholecys-
sphincterotomy for choledochocystolithiasis. tectomy. Given the size, surveillance with US is
Gastroenterology, 2010. 138(7): p. 2315-20. not recommended. MRCP or CT is likely to be of
van Baal, M.C., et al., Timing of cholecystec- limited utility in this scenario. The patient does
tomy after mild biliary pancreatitis: a systematic have a gallbladder in place so ERCP for function-
review. Ann Surg, 2012. 255(5): p. 860-6. al biliary sphincter of Oddi disorder should not
be performed, as the EPISOD trial showed that
there is no justification to perform manometry
Question 17 or sphincterotomy in patients with normal labs
A 36-year-old man with recurrent RUQ pain as- and imaging (prior SOD type III patients), which
sociated with meals is seeing you in clinic. Ami- as of Rome IV are considered to have “functional
notransferase levels have been measured with biliary-type pain.”
these episodes and have been normal. He under-
went a RUQ US which did not reveal cholelithia- REFERENCES
sis; however, an 11-mm non-mobile echogenic Chou, S.C., et al., Polypoid lesions of the gallblad-
lesion was noted within the gallbladder wall. You der: analysis of 1204 patients with long-term
recommend: follow-up. Surg Endosc, 2017. 31(7): p. 2776-2782.
McCain, R.S., et al., Current practices and fu-
A. Cholecystectomy ture prospects for the management of gallbladder
B. ERCP with manometry for functional polyps: A topical review. World J Gastroenterol,
biliary sphincter of Oddi disorder 2018. 24(26): p. 2844-2852.
C. CT scan Wiles, R., et al., Management and follow-up of
D. MR/MRCP gallbladder polyps : Joint guidelines between the
E. A repeat US in one year European Society of Gastrointestinal and Abdomi-
nal Radiology (ESGAR), European Association
CORRECT ANSWER: A for Endoscopic Surgery and other Interventional
Techniques (EAES), International Society of Di-
RATIONALE gestive Surgery - European Federation (EFISDS)
Polypoid gallbladder lesions are seen in up to and European Society of Gastrointestinal Endos-
five percent of adults in Western populations. copy (ESGE). Eur Radiol, 2017. 27(9): p. 3856-
Polypoid lesions can be pseudopolyps including 3866.
cholesterol polyps, inflammatory polyps, adeno- Zielinski, M.D., et al., Comparison of surgi-
myoma which are benign or true polyps includ- cally resected polypoid lesions of the gallbladder
ing adenocarcinomas as well as adenomas (which to their pre-operative ultrasound characteristics. J
have malignant potential). A prior series noted Gastrointest Surg, 2009. 13(1): p. 19-25.
that only polyps six mm or larger in size were Cotton P.B., et al. Gallbladder and Sphinc-
found to have malignancy however in a more ter of Oddi Disorders. Gastroenterology
recent series malignancy was noted in polyps as 2016;150:1420–1429.
Chapter 4 — Diseases of the biliary tract 93
Question 18 19(1): p. 2.
A 52-year-old man with known PSC is found to Esnaola, N.F., et al., Evaluation and manage-
have rising aminotransferase levels. He undergoes ment of intrahepatic and extrahepatic cholangio-
an MR/MRCP and is found to have a new irregu- carcinoma. Cancer, 2016. 122(9): p. 1349-69.
lar hilar mass and intrahepatic ductal dilation. His Njei, B., et al., Systematic review with meta-
bilirubin is eight mg/dL, ALP 450 U/L, AST 200 analysis: endoscopic retrograde cholangiopancre-
U/L, ALT 230 IU/L. CA 19-9 is 900 U/mL, CEA atography-based modalities for the diagnosis of
is 8 ng/mL and AFP is four ng/mL. There is no cholangiocarcinoma in primary sclerosing cholan-
evidence of metastatic disease on cross sectional gitis. Aliment Pharmacol Ther, 2016. 44(11-12): p.
imaging of his chest abdomen and pelvis. Which 1139-1151.
of these should be avoided in this patient?
RATIONALE
This patient presentation is most consistent with
HIV cholangiopathy. This condition has become
rare in the United States following initiation of
HARRT therapy. Approximately 80 percent of
cases are in patients with advanced disease (CD4
less than 100). Almost all patients have a marked
increase in ALP with only mild increase in AST,
ALT and bilirubin. The most common presenta-
tion is with papillary stenosis. In the case of our
patient, she is a refugee from a country with a rate
of HIV infection of approximately 20-30 percent,
placing her at an increased risk for underlying
HIV/AIDS. The new rash consistent with Kaposi
sarcoma provides further support for underlying A. Cholangiocarcinoma
AIDS. The patient does not have a pancreas head B. HCC
mass and it is unlikely given the findings on her C. Pancreatic adenocarcinoma
ERCP that there is a primary cholangiocarcinoma, D. Gallbladder cancer
making CEA and CA 19-9 unlikely to point to an E. Colon Cancer
underlying malignancy. AMA is associated with
PBC; however, this presentation is not consistent CORRECT ANSWER: D
with PBC. Autoimmune cholangiopathy is a con-
sideration as well but, given the patient’s young RATIONALE
age and female sex and isolated papillary stenosis, The MRCP shows a pancreaticobiliary maljunc-
it is unlikely that this is an underlying autoim- tion. These patients are at an increased risk of
mune condition. developing gallbladder cancer, with one series
report cancer arising in 15 percent of patients with
REFERENCES pancreaticobiliary maljunction and congenital bili-
Devarbhavi, H., et al., HIV/AIDS cholangiopathy: ary dilation and in 36 percent without bile duct
clinical spectrum, cholangiographic features and dilation. There is also an increased risk of devel-
outcome in 30 patients. J Gastroenterol Hepatol, oping cholangiocarcinoma, in seven percent with
2010. 25(10): p. 1656-60. congenital biliary dilation and 3.1 percent without
Tonolini, M. and R. Bianco, HIV-related/AIDS bile duct dilation. More recent studies report a
cholangiopathy: pictorial review with emphasis nine percent rate of pancreatic cancer in patients
on MRCP findings and differential diagnosis. Clin with maljunction without bile duct dilation. It is
Imaging, 2013. 37(2): p. 219-26. believed that reflux of pancreatic juice into the
biliary tree is responsible for the increased risk for
gallbladder and bile duct cancers. In patients with
Question 20 congenital biliary dilation, resection of the extra-
A 21-year-old woman presents for further evalua- hepatic duct and gallbladder is recommended. The
tion of intermittent abdominal pain. On exam, she management of patients without bile duct dilation
is tender to palpation without rebound or guard- is more controversial. While cholecystectomy is
ing. AST is 78 IU/L, ALT 90 IU/L, ALP 158 IU/L, recommended in these patients, there is disagree-
Bilirubin 1.5 mg/dL. An MRCP is performed and is ment regarding the need for extrahepatic ductal
shown here. This patient is most likely to develop resection. These patients are not at an increased
which malignancy? risk for colon cancer or HCC.
Chapter 4 — Diseases of the biliary tract 95
REFERENCES A. Cholangiocarcinoma
Funabiki, T., et al., Pancreaticobiliary maljunction B. Pancreatic adenocarcinoma
and carcinogenesis to biliary and pancreatic ma- C. An ampullary adenoma
lignancy. Langenbecks Arch Surg, 2009. 394(1): p. D. IgG4 disease
159-69. E. PSC
Kamisawa, T., et al., Diagnostic criteria for F. Chronic pancreatitis
pancreaticobiliary maljunction 2013. J Hepatobili-
ary Pancreat Sci, 2014. 21(3): p. 159-61. CORRECT ANSWER: D
Kamisawa, T., et al., Biliary carcinogenesis in
pancreaticobiliary maljunction. J Gastroenterol, RATIONALE
2017. 52(2): p. 158-163. This is a young woman with recurrent pancreatitis
Ohuchida, J., et al., Long-term results of treat- of unclear etiology and a new distal biliary stricture.
ment for pancreaticobiliary maljunction without The MR shows findings that are consistent with au-
bile duct dilatation. Arch Surg, 2006. 141(11): p. toimmune pancreatitis (a sausage shaped pancreas
1066-70. with capsular enhancement). In this setting, the
Rungsakulkij, N. and P. Boonsakan, Synchro- stricture is likely related to underlying IgG4 disease.
nous gallbladder and pancreatic cancer associ- It is unclear in this imaging if the stricture is caused
ated with pancreaticobiliary maljunction. World J by pancreatic head swelling from her pancreatitis
Gastroenterol, 2014. 20(39): p. 14500-4. or if IgG4 cholangiopathy is most likely responsible.
In a young patient, PSC and HIV cholangiopathy
are other considerations. While PSC can present as
Question 21 isolated extra hepatic duct stricturing, it is uncom-
An 18-year-old woman presents for further mon and, in the setting of imaging suggesting
evaluation of jaundice. Her past medical his- autoimmune pancreatitis, it is less likely. She has no
tory is notable for 2 prior episodes of idiopathic risk factors for cholangiocarcinoma and, given her
pancreatitis. Her AST is 50, ALT 60, ALP 140, age, this is not felt to be a likely etiology. The same
bilirubin 4 and lipase is 150. On exam her sclerae applies for pancreatic adenocarcinoma and, ad-
are icteric. She is not tender to palpation over ditionally, no pancreas head mass is seen. Changes
her abdomen. A RUQ ultrasound is obtained and of chronic pancreatitis would be unlikely after two
extra and intrahepatic ductal dilation are noted. episodes of pancreatitis, and chronic pancreatitis is
A MR/MRCP is pursued and shows abrupt taper- not felt to be the most likely etiology.
ing of the CBD within the pancreas head, as well
as delayed pancreatic enhancement with a thin REFERENCES
rim of peripheral enhancement as seen here. Her Geary, K., et al., IgG4-Related Cholangiopathy and
CBD stricture is most likely secondary to which Its Mimickers: A Case Report and Review High-
underlying condition? lighting the Importance of Early Diagnosis. Int J
Surg Pathol, 2018. 26(2): p. 165-173.
Joshi, D. and G.J. Webster, Biliary and hepat-
ic involvement in IgG4-related disease. Aliment
Pharmacol Ther, 2014. 40(11-12): p. 1251-61.
Okazaki, K., et al., IgG4 cholangiopathy:
current concept, diagnosis, and pathogenesis. J
Hepatol, 2014. 61(3): p. 690-5.
Watanabe, T., et al., Mechanisms of lower bile
duct stricture in autoimmune pancreatitis. Pan-
creas, 2014. 43(2): p. 255-60.
96 Digestive Diseases Self-Education Program®
REFERENCES
Baharith, H. and K. Khan, Bouveret syndrome:
When there are no options. Can J Gastroenterol
Hepatol, 2015. 29(1): p. 17-8. A. Cholangitis
Cappell, M.S. and M. Davis, Characterization B. Pancreatitis
of Bouveret’s syndrome: a comprehensive review C. Obstructive jaundice
of 128 cases. Am J Gastroenterol, 2006. 101(9): p. D. Diarrhea
2139-46. E. GI bleeding
Haddad, F.G., W. Mansour, and L. Deeb,
Bouveret’s Syndrome: Literature Review. Cureus, CORRECT ANSWER: B
2018. 10(3): p. e2299.
Nickel, F., et al., Bouveret’s syndrome: pre- RATIONALE
sentation of two cases with review of the literature This patient has a choledochocele. While many
and development of a surgical treatment strategy. are asymptomatic, the most common clinical
BMC Surg, 2013. 13: p. 33. presentation is pancreatitis, which is seen in
38-70 percent of patients with a choledochocele.
Jaundice is seen in 11-25 percent of patients and
Question 25 cholangitis is seen in 0-10 percent of patients.
A 68-year-old man presents to your clinic for fur- The most common presentation of choledochal
ther evaluation of abdominal pain. On exam, the cysts type I, II, IV, V is cholangitis, followed by
patient’s abdomen is soft, not tender and not dis- obstructive jaundice, then pancreatitis. Diarrhea
tended. You note icteric sclerae. Labs are obtained and GI bleeding are not typical presentations of
and demonstrate an AST of 92 IU/L, ALT 87 IU/L, choledochal cysts. This is the only type of chole-
Bili 3 mg/dL, ALP 180 IU/L, and lipase is 23 U/L. dochal cyst for which ERCP can offer definitive
A RUQ ultrasound demonstrates ductal dilation management. The risk of malignancy is very low,
but no cholelithiasis or sludge. MRCP is subse- especially when compared to other choledochal
quently performed and is shown here. ERCP is cysts, and the need for surveillance following en-
performed with the image shown above. The most doscopic management (snare resection or sphinc-
common clinical presentation of this condition is: terotomy) remains controversial.
Chapter 4 — Diseases of the biliary tract 99
Barr Fritcher EG, Kipp BR, Voss JS, et al. CORRECT ANSWER: B
Primary sclerosing cholangitis patients with serial
polysomy flouresence in situ hybridization results RATIONALE
are at increased risk of cholangiocarcinoma. Am J This patient has an early stage intrahepatic CCA
Gastroenterol 2011;106:2023-2028. (iCCA) arising in the setting of cirrhosis second-
ary to nonalcoholic steatohepatitis. While surgical
resection offers potentially curative therapy for
Question 27 iCCA, the presence of cirrhosis and portal hyper-
A 48-year-old man presents for routine evalu- tension make the patient high risk for hepatic
ation of cirrhosis secondary to nonalcoholic resection. While iCCA was conventionally con-
steatohepatitis. He has a history of non-bleeding sidered a contraindication for liver transplanta-
varices and medically controlled ascites. His tion due to high risk of recurrence, recent stud-
medical history is otherwise significant for obe- ies demonstrate good survival in patients with
sity with a BMI of 31. His laboratory evaluation cirrhosis and highly selective patients with very
is as follows: early iCCA. A retrospective, multi-center study
revealed a low five-year recurrence of iCCA among
AST 148 U/L patients with lesions smaller than two centime-
ALT 213 U/L ters (12 percent compared to 77 percent among
Alkaline phosphatase 391 U/L patients with lesions larger than two centimeters
Total bilirubin 3.3 mg/dL or multifocal disease). Overall survival among
CA 19-9 444 U/mL patients with lesions smaller than two centime-
Alpha feto-protein 2.8 ng/mL ters was 65 percent. Thus, patients with cirrhosis
Hemoglobin 12.5 g/dL and iCCA who otherwise qualify as candidates for
Platelets 91 x 109/L liver transplantation can be considered for trans-
Sodium 140 mmol/L plantation at specialized transplant centers if they
INR 1.3 have a single lesion less than or equal to three
Albumin 3.0 g/dL centimeters in diameter, no evidence of vascular
Creatinine 1.1 mg/dL invasion or extrahepatic disease, and no history of
prior attempted resection which might violate the
Ultrasound reveals a nodule in the left lobe of tumor plane.
the liver. CT scan is done to further evaluate this Locoregional therapies such as TACE and
lesion and reveals a 1.8 cm heterogeneous mass radiofrequency ablation are a reasonable approach
with portal venous enhancement in segment two in patients with more advanced iCCA who are
without vascular invasion. Ultrasound guided not candidates for resection or transplantation.
biopsy confirms well differentiated adenocarci- While waiting for transplantation, local regional
noma. Chest CT and endoscopic ultrasound with therapies may be employed as a bridge to trans-
perihilar lymph node sampling is negative for plantation. Gemcitabine and cisplatin constitute
malignancy. What is the next best step in manage- first line chemotherapy in patients with advanced
ment of this patient? stage CCA who are not candidates for locoregional
therapy or surgery.
A. Refer to oncology for gemcitabine and
cisplatin. REFERENCES
B. Refer to liver transplantation center Rizvi S, Khan SA, Hallemeier CL, et al. Chol-
C. Refer for surgical resection angiocarcinoma- evolving concepts and thera-
D. Refer for transarterial chemoembolization peutic strategies. Nat Rev Clin Oncol. 2018
E. Refer for radiofrequency ablation. Feb;15(2):95-111.
Chapter 4 — Diseases of the biliary tract 101
Li YY, Li H, Lv P, et al. Prognostic value of cir- over time have been identified as risk factors for
rhosis for intrahepatic cholangiocarcinoma after malignancy. Although this patient is asymptom-
surgical treatment. J Gastrointest Surg. 2011 Apr; atic from her gallbladder polyp and gallstones,
15(4): 608-613. laparoscopy cholecystectomy is recommended for
Sapisochin G, Rodriguez de Lope C, Gastaca polyps larger than one centimeter. Ultrasound
M, et al. “Very early” intrahepatic cholangiocar- surveillance is recommended for gallbladder pol-
cinoma in cirrhotic patients: should liver trans- yps without high-risk features.
plantation be reconsidered in these patients? Am J Oral dissolution therapy with ursodeoxycholic
Transplant. 2014 Mar;14(3):660-7. acid can be used in a select subset of patients with
Sapisochin G, Rodriguez de Lope C, Gastaca cholelithiasis. Studies suggest that this is most
M, et al. Intrahepatic cholangiocarcinoma or effective in patients with gallstones < 1cm in diam-
mixed hepatocellular-cholangiocarcinoma in pa- eter however.
tients undergoing liver transplantation: a Spanish
matched cohort multicenter study. Ann Surg. 2014 REFERENCES
May;259(5):944-52. Gallahan WC, Conway JD. Diagnosis and manage-
ment of gallbladder polyps. Gastroenterol Clin
North Am. 2010;39:359-367.
Question 28 Terzi C, Sokmen S, Seckin S, et al. Polypoid
A 58-year-old woman presents for evaluation of lesions of the gallbladder: report of 100 cases with
a gallbladder lesion which was noted incidentally special reference to operative indications. Surgery.
on a CT scan done to evaluate kidney stones. An 2000 Jun;127(6):622-7.
ultrasound is recommended and reveals choleli- Myers RP, Shaffer EA, Beck PL. Gallbladder
thiasis and a hyperechoic polyp which measures polyps: epidemiology, natural history and man-
12 mm in diameter. Her bile ducts appear nor- agement. Can J Gastroenterol. 2002,16:187-94.
mal. She is asymptomatic and denies abdominal Cairns V, Neal CP, Dennison AR, et al. Risk
discomfort. What is the next best step in man- and cost-effectiveness of surveillance followed by
agement? cholecystectomy for gallbladder polyps. Arch Surg.
2012:1-6.
A. Recommend laparoscopic cholecystectomy. Tomida S, Abei M, Yamaguchi T, et al. Long-
B. No follow up needed. term ursodeoxycholic acid therapy is associated
C. Recommend laparoscopic cholecystectomy if with reduced risk of biliary pain and acute cho-
she develops symptoms suggestive of biliary lecystitis in patients with gallbladder stones: a
colic. cohort analysis. Hepatology. 1999;30(1):6.
D. Ultrasound in six months.
E. Ursodeoxycholic acid 13-15 mg/kg/day.
Question 29
A 35 year-old-woman presents to the clinic for
CORRECT ANSWER: A
evaluation of elevated liver tests. Her liver tests
were checked as part of her insurance policy one
RATIONALE
year ago and they have subsequently remained
Gallbladder polyps are common and can be benign abnormal. Her medical history is otherwise sig-
or neoplastic. Polyp size is an important factor in nificant for ulcerative colitis which was diagnosed
determining risk of malignancy. Age greater than five years ago. Her last colonoscopy was done one
60, coexisting gallstones, lower polyp number, year ago and revealed mild chronic colitis. Biop-
polyp size more than one centimeter, primary scle- sies had no evidence of dysplasia. Her laboratory
rosing cholangitis and evidence of polyp growth tests are as follows:
102 Digestive Diseases Self-Education Program®
Total bilirubin 1.2 mg/dL is a concern for an overlap syndrome with autoim-
Alkaline phosphatase 360 U/L mune hepatitis. Anti-mitochondrial antibodies are
ALT 45 U/L useful to establish a diagnosis of primary biliary
AST 62 U/L cholangitis. While PSC patients are at an increased
risk of gallbladder cancer, a prophylactic cholecys-
MRCP is performed and the results are shown
tectomy would not be indicated.
below:
REFERENCES
Thackeray EW, Charatcharoenwitthaya P, Elfaki
D, et al. Colon neoplasms develop early in the
course of inflammatory bowel disease and primary
sclerosing cholangitis. Clin Gastroenterol Hepatol.
2011;9:52-56.
Chapman R, Fevery J, Kalloo A, et al. Diagno-
sis and management of primary sclerosing cholan-
gitis. Hepatology 2010;51:660-678.
Question 30
A 40-year-old man presents to the emergency
What is the next best step in management?
room with abdominal pain, recurrent fevers, and
worsening jaundice. He has a past medical history
A. Liver biopsy
significant for recurrent episodes of cholangitis,
B. Check anti-mitochondrial antibody titers
ascites, and esophageal varices. A CT scan (below)
C. Cholecystectomy
is performed in the emergency room. MRCP per-
D. Colonoscopy with surveillance biopsies in
formed several weeks earlier is shown below:
4 years
E. Colonoscopy with surveillance biopsies now
CORRECT ANSWER: E
RATIONALE
The above patient has asymptomatic large-duct
primary sclerosing cholangitis (PSC). The diag-
nosis is secured with the cholangiogram, which
shows typical features of PSC, consisting of mul-
tifocal segmental biliary strictures and dilations.
Up to 80 percent of PSC patients have underlying
inflammatory bowel disease, particularly ulcer-
ative colitis (UC). Individuals with PSC-UC are
four times more likely to develop colorectal cancer
compared to those with UC alone. Therefore,
annual colonoscopy with surveillance biopsies is
recommended for patients with PSC-UC.
A liver biopsy is not necessary to confirm a
diagnosis of PSC unless the cholangiogram is
normal and small-duct PSC is suspected, or there
Chapter 4 — Diseases of the biliary tract 103
Which of the following is true regarding this While Caroli’s syndrome predisposes to cholangi-
patient’s diagnosis? tis, the pathogenesis of this disorder is thought to
be due to genetic mutation in ciliary proteins and
A. Autosomal dominant polycystic kidney not due to parasitic infection.
disease is the most common renal
manifestation. REFERENCES
B. Cyst aspiration and sclerotherapy are Ozlem Y, Bayraktar Y. Clinical characteristics
indicated. of Caroli’s syndrome. World J Gastroenterol.
C. This patient is at increased risk for 2007;13:1934-1937.
cholangiocarcinoma.
D. Liver biopsy is required to establish the
diagnosis. Question 31
E. This patient’s liver disease is secondary A 49-year-old man presents to transplant clinic for
to parasitic infection. evaluation of fevers and jaundice. He underwent
an orthotopic liver transplant two months ago for
CORRECT ANSWER: C decompensated cirrhosis secondary to primary
sclerosing cholangitis (PSC). His post-operative
RATIONALE course was notable for mild elevations in his al-
This patient has Caroli’s syndrome which is a kaline phosphatase, AST, and ALT. A liver biopsy
congenital disorder characterized by multifocal, done one month post-transplant did not show
segmental dilation of large intrahepatic bile ducts rejection. Laboratory tests reveal the following:
associated with congenital hepatic fibrosis. The
diagnosis of Caroli’s syndrome is established by WBC count 14,000/L
imaging studies. This patient’s cholangiogram Total bilirubin 6.2 mg/dL
illustrates characteristic features of Caroli’s syn- Direct bilirubin 5.2 mg/dL
drome including diffuse dilation of the biliary tree Alkaline phosphatase 450 U/L
with extensive fusiform and saccular dilation of the AST 100 U/L
intrahepatic bile ducts. Other characteristic radio- ALT 108 U/L
graphic features (indicated by arrowheads on CT ERCP reveals multiple intrahepatic strictures.
scan) include the “central dot sign” where intralu-
What is the next best step in management of this
minal portal vein radicals appear as foci of contrast
patient?
enhancement within the dilated bile ducts.
Biliary cysts in Caroli’s syndrome are irregu-
A. Refer for retransplantation.
larly shaped and communicate with the bile ducts.
B. Increase immunosuppression.
In contrast, cysts in polycystic liver disease are
C. Hepatitis C viral load.
more regularly shaped and do not communicate
D. Repeat liver biopsy to rule out PSC.
with bile ducts.
E. Doppler ultrasound of the hepatic arteries.
Renal manifestations occur in up to 60
percent of patients with Caroli’s syndrome and
include medullary sponge kidney, cortical cysts, CORRECT ANSWER: E
autosomal-recessive polycystic kidney disease, or
very rarely, autosomal-dominant polycystic kidney RATIONALE
disease. Patients with Caroli’s syndrome are at Biliary tract complications are among the most
increased risk for cholangiocarcinoma which is common post-transplant complications and can
thought to be due to bile stasis and high concen- occur in five to 15 percent of patients following a
trations of unconjugated secondary bile salts. deceased-donor transplant and up to 32 percent in
104 Digestive Diseases Self-Education Program®
REFERENCES
Kochhar G, Parungao JM, Hanouneh IA, et al.
Biliary complications following liver transplanta-
tion. World J Gastroenterol. 2013;19:2841-2846.
Fosby B, Karlsen TH, Melum E. Recurrence
and rejection in liver transplantation for primary
sclerosing cholangitis. World J Gastroenterol.
Which of the following is true regarding her liver
2012;18:1-15.
disease?
within the first few months of life. Second, GGT is AST 42 U/L
elevated in PFIC type III, and normal in the other ALT 31 U/L
subtypes. Third, ductular proliferation can be seen Alkaline phosphatase 118 U/L
in liver biopsy specimens of patients with PFIC Total bilirubin 1.8 mg/dL
type III, and this finding is absent in PFIC type I Hemoglobin A1c 6.5 percent
& II. Last, PFIC III is associated with mutations in
the ABCB4 gene, which affects a protein involved Abdominal ultrasound reveals cholelithiasis.
in biliary phospholipid excretion (MDR3). Dimin-
Which of the following is the most likely cause of
ished biliary phospholipid excretion leads to an
impaired gallbladder motility in this patient?
increase in bile lithogenicity. PFIC II is associated
with mutation of the ACBC11 gene which encodes
A. Estrogen
an ATP-dependent canalicular bile salt export
B. Progesterone
pump (BSEP). PFIC I is associated with mutation
C. Elevated triglycerides
of ATP8B1 which encodes the canalicular mem-
D. Clofibrate
brane protein FIC1.
E. Ileal resection
Ursodeoxycholic acid is the initial medical
treatment of choice for all PFIC subtypes. CORRECT ANSWER: B
Mutations in the JAG1 gene are associated with
Alagille Syndrome. This presents with ductope- RATIONALE
nia, cholestasis, and liver failure and is associated There are multiple factors which can predispose
with multiple extrahepatic features including to gallstone formation including gallbladder
dysmorphic facies and cardiac and renal abnor- dysmotility leading to bile stasis, secretion of bile
malities. Mutations in the ATP7B gene on chro- supersaturated with cholesterol, and cholesterol
mosome 13 impact a copper transport protein and nucleation leading to crystal formation. Gallblad-
are associated with Wilson’s disease. der motility is stimulated by the fat content in
ingested meals, vagal (cholinergic) nerve stimu-
REFERENCES lation, cholecystokinin (CCK) and motilin. In
Davit-Spraul A, Gonzales E, Baussan C, et al. contrast, the physiologic increases in progesterone
Progressive familial intrahepatic cholestasis. Or- during pregnancy decrease gallbladder motility.
phanet J Rare Dis. 2009;4:1750-1172. Clofibrate is a lipid-lowering drug that re-
Erlinger S, Arias IM, Dhumeaux D. Inherited duces the activity of cholesterol 7α-hydroxylase
disorders of bilirubin transport and conjugation: (the rate limiting enzyme in bile salt synthesis in
new insights into molecular mechanisms and con- the classical pathway). This promotes cholesterol
sequences. Gastroenterology. 2014;46:1625-1638. supersaturation in the bile and decreases bile salt
concentrations but is not known to impact gall-
bladder motility.
Question 34 Estrogen, obesity, and hyperlipidemia predis-
A 35-year-old woman, G2P1, who is 25 weeks pose to gallstone formation by promoting choles-
pregnant presents for evaluation of right upper terol supersaturation. In addition to its impact on
quadrant pain. Her medical history is significant gallbladder motility, progesterone can also promote
for obesity, hyperlipidemia which was previ- cholesterol saturation by decreasing bile salts.
ously treated with clofibrate, and ileal resection Ileal Crohn’s disease and ileal resection pre-
for Crohn’s disease. Her pregnancy thus far has dispose to gallstone formation by interrupting
been complicated by gestational hypertension re- the enterohepatic circulation of bile acids. This
quiring treatment with labetolol. Her lab evalua- results in reduced hepatic bile acid secretion and
tion is as follows: supersaturation of bile with cholesterol.
Chapter 4 — Diseases of the biliary tract 107
Which of the following is the most likely and acquired, which result in cholestasis due
diagnosis? to progressive destruction of intrahepatic bile
ducts. Caroli’s syndrome is a congenital disorder
A. Vanishing bile duct syndrome characterized by multifocal, segmental dilation
B. Caroli’s syndrome of large intrahepatic bile ducts associated with
C. Alagille syndrome congenital hepatic fibrosis. Biliary atresia refers
D. Progressive familial intrahepatic to a fibro-obliterative disease of the extrahepatic
cholestasis type III bile ducts which presents with biliary obstruction
E. Biliary atresia in the neonatal period. Although biliary atresia
usually presents in isolation, it can occur with
CORRECT ANSWER: C laterality malformation or other congenital mal-
formations such as intestinal atresia and cardiac
RATIONALE abnormalities.
Alagille syndrome is an autosomal dominant
condition that can present as neonatal jaundice, REFERENCES
characterized by a conjugated hyperbilirubinemia Hartley JL, Gissen P, Kelly DA. Alagille syndrome
and ductopenia. Cirrhosis and complications sec- and other hereditary causes of cholestasis. Clin
ondary to portal hypertension can develop in a Liver Dis. 2013;17:279-300.
subset of patients later in life. Alagille syndrome Ziol M, Barbu V, Rosmorduc O, et al. ABCB4
has several key extrahepatic manifestations that heterozygous gene mutations associated with
are illustrated in this case. Cardiac defects are fibrosing cholestatic liver disease in adults. Gas-
present in 90 percent of patients, with peripheral troenterology. 2008;135:131-141.
pulmonary stenosis being the most common. Haber BA, Russo P. Biliary atresia. Gastroen-
Skeletal features, classically abnormal fusion terol Clin North Am. 2003;32(3):891.
of the spine leading to a butterfly shape of the Woolf GM, Vierling JM. Disappearing
thoracic vertebrae, are also common. A posterior intrahepatic bile ducts: the syndromes and
embryotoxon (a corneal abnormality notable on a their mechanisms. Semin Liver Dis. 1993
slit-lamp examination) can be seen in 90 percent Aug;13(3):261-75.
of patients with Alagille syndrome, in addition
to 15 percent of the normal population. Lastly,
facial features characterized by a triangular face Question 38
with deep-set eyes and a pronounced forehead A 48-year-old man presents to clinic for evalu-
are typical. Genetic testing plays an important ation of abdominal pain, fever, and jaundice.
role in confirming the diagnosis. Mutations in the His medical history is significant for gallstone
JAG1 gene (encodes the NOTCH signaling path- pancreatitis for which he underwent laparoscopic
way ligand Jagged-1) are found in 95 percent of cholecystectomy one month prior. He is mildly
cases, while defects in the NOTCH2 gene can be tachycardic with a heart rate of 103 beats per
detected in the small subset of patients without minute. Temperature is 37.6° C. Physical exami-
JAG1 mutations. nation is remarkable for scleral icterus and right
Progressive familial intrahepatic cholesta- upper quadrant pain on palpation. (See imaging
sis type III (PFIC III) is a cholestatic disease on the following page.)
caused by mutations in the ABCB4 gene. This
gene encodes the MDR3 protein which partici- Total bilirubin 4.4 mg/dL
pates in phosphatidycholine translocation in the Alkaline phosphatase 381 U/L
bile canaliculus. Vanishing bile duct syndrome ALT 106 U/L
describes a group of disorders, both congenital AST 98 U/L
110 Digestive Diseases Self-Education Program®
A. Cholecystectomy Question 40
B. MRCP A 52-year-old man is referred in order to evaluate
C. ERCP with sphincterotomy an incidental finding detected on a CT scan. Re-
D. ERCP with sphincterotomy followed cently, he underwent a CT of his abdomen during
by cholecystectomy an evaluation for hematuria. This showed a cystic
E. Cholecystectomy with intra-operative structure on the medial wall of the duodenum.
cholangiogram You perform an ERCP (shown below), which re-
veals a focal dilation of the intra-duodenal portion
CORRECT ANSWER: D of the bile duct.
RATIONALE
The above patient is presenting with symp-
tomatic gallstone disease. His bilirubin
greater than 4.0 mg/dL, combined with
the presence of a dilated common bile duct
(CBD), are concerning for choledocholithiasis.
Risk stratification for choledocholithiasis in
patients presenting with symptomatic cho-
lelithiasis should be performed. Very strong
predictors of a CBD stone include a bilirubin
greater than 4 mg/dL, ascending cholangitis,
and a CBD stone visualized on an ultrasound.
It is recommended that a preoperative ERCP Which of the following is the correct diagnosis?
be performed if any of these very strong
predictors are present. Strong predictors of a A. Type I biliary cyst
CBD stone include a dilated CBD (greater than B. Type II biliary cyst
six millimeters with intact gallbladder) and a C. Type III biliary cyst
modest elevation in bilirubin (1.8-4 mg/dL). D. Type IVA biliary cyst
If both of the strong predictors are present, a E. Type IVB biliary cyst
pre-operative ERCP is suggested. Moderate
CORRECT ANSWER: C
predictors of a CBD stone include abnormal
liver tests other than bilirubin, age greater RATIONALE
than 55 years and gallstone pancreatitis. If The cholangiogram illustrates a choledochocele
none of these predictors are present then (also referred to as a Type III choledochal cyst),
patients should proceed directly to a cholecys- which is a cystic dilation of the intra-duodenal
tectomy. However, for the remaining patients portion of the common bile duct. The character-
at an intermediate-risk for a CBD stone, one istic features of the other congenital biliary cyst
of the following is suggested: preoperative subtypes are as follows: Type I=dilation of extra-
MRCP, preoperative endoscopic ultrasound or hepatic bile duct with normal intrahepatic duct
intraoperative cholangiogram. (most common subtype); Type II=diverticulum
of the extrahepatic bile duct; Type IVa multiple
REFERENCE dilation of the intra and extrahepatic biliary tree;
ASGE Standards of Practice Committee, Maple JT, Type IVb=multiple dilations of the extrahepatic
Ben-Menachem T, et al. The role of endoscopy in bile duct; Type V (Caroli’s disease)=multiple cystic
the evaluation of suspected choledocholithiasis. dilations of the intrahepatic bile ducts with normal
Gastrointest Endosc. 2010;71:1-9. extrahepatic duct.
112 Digestive Diseases Self-Education Program®
While choledochoceles can be asymptomatic, one year ago with nonspecific findings on patholo-
abdominal pain is present in 90 percent of symp- gy. He is up to date with cancer screening and had
tomatic cases, and jaundice develops in a smaller an unremarkable colonoscopy two years ago. His
subset of patients (~20 percent). There are several physical examination is remarkable for jaundice.
key features that distinguish choledochoceles from His laboratory evaluation is as follows:
other choledochal cysts. First, pancreatitis is the
most common complication in choledochoceles, Hemoglobin 13.4 g/dL
and it can occur in 70 percent of individuals. Pan- Leukocytes 7,500/L
creatitis is less common among other subtypes of ESR 62
biliary cysts. Second, choledochoceles (in addition AST 35 U/L
to Type II cysts) have a very low malignant poten- ALT 39 U/L
tial (~two percent), particularly when they present ALP 555 U/L
without an anomalous pancreatobiliary junction, Total bilirubin 5.4 mg/dL
with only several cases of cholangiocarcinoma or CA 19-9 18 U/L
ampullary cancer being reported. This stands in
stark contrast to the other cyst subtypes (Types I, You recommend MRCP which reveals stricturing
IV and V), which carry a high risk for biliary tract of the extrahepatic common bile duct, a prominent
cancers. Lastly, symptomatic choledochoceles distal bile duct stricture, and enlargement of the
can be treated less invasively through endoscopic pancreas. What is the next best step in evaluation
cyst excision or sphincterotomy. Conversely, the of this patient?
treatment of Type I and IV cysts involves complete
extrahepatic bile duct cyst excision (Roux-en- A. Check serum IgG4 level
Y hepaticojejunostomy), Type II is treated by a B. ERCP with biliary brushings for cytology
diverticulotomy followed by a primary common and FISH
bile duct closure, and Type V (Caroli’s disease) can C. Endoscopic ultrasound to assess pancreatic
be treated with either a hepatic resection or liver head and distal common bile duct
transplantation (when indicated). D. PET scan
E. Refer for hepaticojejunostomy
REFERENCES
CORRECT ANSWER: A
Law R, Topazian M. Diagnosis and treatment
of choledochoceles. Clin Gastroenterol Hepatol.
RATIONALE
2014;12:196-203.
This patient presents with jaundice and imaging
Soares KC, Arnaoutakis DJ, Kamel I, et al.
features suggestive of sclerosing cholangitis. The
Choledochal cysts: presentation, clinical dif-
presence of a prominent pancreas on imaging and
ferentiation, and management. J Am Coll Surg.
history of lymphadenopathy is suggestive of IgG4-
2014;219:1167-1180.
related disease. IgG4 related sclerosing disease
is a systemic disease characterized by extensive
Question 41 tissue infiltration by IgG4 positive plasma cells
A 62-year-old man is referred for a second opin- and T cells, which cause fibrotic lesions associated
ion regarding primary sclerosing cholangitis. He with obliterative phlebitis. IgG4-related disease
presented with jaundice and vague abdominal can affect the pancreas, bile duct, gallbladder,
discomfort three months prior, and PSC was diag- salivary glands, kidney, lung, prostate, and retro-
nosed after characteristic features were noted on peritoneum. Lymphadenopathy is also common.
ERCP. His medical history is otherwise remark- The disease occurs more often in older men and is
able for coronary artery disease and excisional often characterized by elevated IgG4 levels.
biopsy of an enlarged supraclavicular lymph node The patient’s history of lymphadenopathy with
Chapter 4 — Diseases of the biliary tract 113
imaging features of sclerosing cholangitis and has a layer of cuboidal cells with underlying ovari-
enlargement of the pancreas make IgG4 related an-type stroma. What is the most likely diagnosis?
sclerosing disease the most likely diagnosis.
A. Intrahepatic cholangiocarcinoma
REFERENCES B. Echinococcal cyst
Ghazale A, Chari ST, Zhang L, et al. Immuno- C. Type I biliary cyst
globulin G4-associated cholangitis: clinical profile D. Biliary cystadenoma
and response to therapy. Gastroenterology. E. Type II biliary cyst
2008;134:706-715.
Hart PA, Topazian MD, Witzig TE, et al. Treat- CORRECT ANSWER: D
ment of relapsing autoimmune pancreatitis with
immunomodulators and rituximab: the Mayo RATIONALE
Clinic experience. Gut. 2013;62:1605-1615. Biliary cystadenomas are irregular cystic lesions
that are slow-growing tumors originating from the
intrahepatic biliary tree (rarely the extrahepatic
Question 42 biliary tree). These lesions are found in middle-age
A 72-year-old woman presents to establish care after women (90 percent). Typical imaging features are
recently moving from Central America. She does shown above: a multiloculated cyst with internal
note abdominal fullness and early satiety. She is oth- septations and a thickened irregular wall. Calci-
erwise healthy. On physical examination, her abdo- fications and papillary wall nodules may also be
men is distended with a palpable mass in her upper present. While not a universal finding, it is classic
abdomen. Her laboratory evaluation is as follows: for these cysts to have an ovarian-like stroma that
expresses receptors for estrogen and progesterone.
Total bilirubin 1.1 mg/dL While the true natural history of biliary cystadeno-
Direct bilirubin 0.4 mg/dL mas is unclear, in some reports, up to 20 percent
Alkaline phosphatase 120 U/L undergo malignant transformation into biliary cyst-
ALT 41 U/L adenocarcinomas. An elevated CA 19-9 can be seen
AST 33 U/L in either condition biliary cystadenoma or cystad-
CA 19-9 315 U/mL enocarcinoma. Consequently, complete surgical
removal is generally recommended once detected.
You recommend MRI of the abdomen which is The rate of recurrence after surgery is low.
shown below:
It would be atypical for a cholangiocarcinoma
She subsequently undergoes a surgical resection of to have the above radiographic features. Echino-
the lesion. The pathology shows that the structure coccal cysts can present as a complex cyst. While
the patient has risk factors for
exposure to Echinococcus, the
absence of daughter cysts and
the histologic features in this
case make this option less likely.
Type I biliary cysts involve the
extrahepatic duct and are asso-
ciated with an increased risk of
biliary tract malignancies. Type
II biliary cysts are diverticula
connected to the extrahepatic
duct through a narrow stalk.
114 Digestive Diseases Self-Education Program®
She endorses moderate pruritis and fatigue. She addition of fenofibrate to UDCA in patients with
is not interested in participating in clinical trials. PBC who had an inadequate biochemical response
Which of the following is the next best step in to UDCA resulted in decreased risk of liver decom-
management of this patient’s PBC? pensation, mortality, and liver transplantation.
Studies of methotrexate in PBC have yielded
A. Increase ursodeoxycholic acid dose to conflicting results and thus it is not routinely
18-23 mg/kg/day recommended. Advanced liver disease secondary
B. Discontinue UDCA and start obeticholic to PBC is an indication for liver transplantation,
acid 10 mg per day although this patient does not demonstrate signs
C. Start methotrexate 0.25 mg/kg/week of decompensated cirrhosis at this time.
D. Start fenofibrate 145 mg daily
E. Refer for liver transplantation REFERENCES
Carey EJ, Ali AH, Lindor KD. Primary biliary cir-
CORRECT ANSWER: D rhosis. Lancet. 2015 Oct 17;386(10003):1565-75.
Pares A, Caballeria L, Rodes J. Excellent long-
RATIONALE term survival in patients with primary biliary
This patient has demonstrated an inadequate cirrhosis and biochemical response to ursodeoxy-
response to ursodeoxycholic acid (UDCA), which cholic acid. Gastroenterology 2006;130:715-20.
is generally defined as persistent elevation in Corpechot C, Chazouilleres O, Rousseau A, et
alkaline phosphatase to greater than 1.67 times al. A placebo-controlled trial of bezafibrate in pri-
the upper limit of normal after one year of UDCA mary biliary cholangitis. N Engl J Med. 2018 Jun
therapy. Biochemical response to UDCA predicts 7;378(23):2171-2181.
prognosis in patients with PBC. Approximately Nevens F, Andreone P, Mazzella G, et al. A
30 percent of patients demonstrate suboptimal placebo-controlled trial of obeticholic acid in pri-
response to UDCA and are at risk for more rapid mary biliary cholangitis. N Engl J Med.2016 Aug
progression of disease when compared with pa- 18:375(7):631-43.
tients who normalize alkaline phosphatase after
initiation of UDCA. As a result, adjuvant or alter-
native therapy is recommended in patients who do Question 45
A 31-year-old man is seen for a one-week history
not respond to UDCA.
of epigastric pain and scleral icterus. One month
Obeticholic acid is a farsenoid X receptor
agonist which has been shown to decrease alkaline earlier, he developed diarrhea and fatigue, which
has persisted. He denies any prior medical prob-
phosphatase and total bilirubin levels when used
lems, though he admits he has not seen a doctor in
alone or in combination with UDCA. Worsening
years. He is currently visiting family in the United
pruritus is a common, dose related side effect of
States, but he resides in South Africa. His labora-
obeticholic acid that is more common at 10 mg
tory tests are shown below:
than five mg. Hence, among those without ad-
vanced liver disease, starting obeticholic acid at Total bilirubin 3.5 mg/dL
five mg per day is recommended. Direct bilirubin 2.9 mg/dL
Bezafibrate is an agonist of the peroxisome pro- Alkaline phosphatase 720 U/L
liferator activated receptor which has been shown ALT 105 U/L
to improve biochemical response in patients who AST 117 U/L
had an inadequate response to UDCA. Although Albumin 2.1 g/dL
bezafibrate is not available in the United States, INR 1.2
fenofibrate is approved for management of hy- HIV viral load 450,000 copies/mL
perlipidemia. A retrospective study revealed that CD4 count 25 cells/mm3
116 Digestive Diseases Self-Education Program®
Which of the following is the most likely Total bilirubin 4.9 mg/dL
causative organism? Direct bilirubin 4.1 mg/dL
Alkaline phosphatase 428 U/L
A. Entamoeba histolytica ALT 95 U/L
B. Echinococcus granulosis AST 106 U/L
C. Cryptosporidium parvum
Intravenous antibiotics are initiated. An abdomi-
D. Clonorchius sinensis
nal ultrasound shows dilated intra- and extrahe-
E. Schistosoma japonicum
patic ducts. An ERCP is performed (cholangio-
gram below), and multiple stones are extracted.
CORRECT ANSWER: D
RATIONALE
This patient presents with a history consistent
with recurrent pyogenic cholangitis. This is clas-
sically described in patients of East Asian descent
who present with recurrent episodes of cholangitis
with intra- and extrahepatic ductal dilation and
choledocholithiasis. This disease can lead to liver
abscesses and constitutes a risk for cholangiocar-
The patient is most likely to have which of the
cinoma. While the exact etiology of the disease is
following?
not known, the most quoted theory links biliary
tract infection with parasites. The most com-
A. Brown pigmented stones
monly implicated parasites include Clonorchis
B. Black pigmented stones
sinensis, Opisthorchis species, Fasciola hepatica,
C. Cholesterol stones
and A. Lumbricoides.
D. Primary sclerosing cholangitis
Entamoeba histolyticum is a parasite
E. Gangrenous cholecystitis
which causes infection in the developing world.
Liver abscess is the most common extraintestinal
CORRECT ANSWER: A
manifestation of amebic infection. Echinococcus
can also cause multiseptated, cystic lesions
RATIONALE
in the liver. Cryptosporidium parvum is the
Recurrent pyogenic cholangitis (oriental cholangi-
most frequently implicated organism in AIDS
tis) is characterized by the development of biliary
cholangiopathy. Schistosomiasis typically
pigmented stones and strictures which results in
presents with portal fibrosis and granulomas
recurrent episodes of cholangitis. It is found among
of the liver with evidence of portal hypertension
individuals who have lived in Southeast Asia. The
and eosinophillia.
precise etiology is unknown, although parasitic
and/or bacterial infections have been postulated as
Question 47 contributing factors, particularly since this disorder
A 26-year-old man is admitted to the hospital for is limited to an area where biliary parasites are en-
recurrent fever, rigors and jaundice. He is a recent demic. The presence of bile duct stones is frequent-
immigrant from Vietnam. His physical exam is ly associated with intrahepatic ductal dilation (left
significant for a temperature of 39°C, scleral ic- hepatic ducts are more often involved than those
terus and right-upper quadrant tenderness. on the right) and downstream strictures. Brown
pigment stones are commonly associated with this
His laboratory tests are shown as follows: condition because they form as a result of stasis and
118 Digestive Diseases Self-Education Program®
infection. These stones are soft and radiolucent. D. Transfuse packed red blood cells and fresh
Often, colonies of bacteria can be found growing frozen plasma
on brown pigmented stones. Black pigmented and E. Upper endoscopy
cholesterol stones are generally formed in sterile
bile within the gallbladder. Patients with primary CORRECT ANSWER: B
sclerosing cholangitis have cholangiographic ap-
pearance of “beads on a string”, and while such RATIONALE
patients can develop hepatolithiasis, this would not Hemobilia is a rare cause of gastrointestinal bleeding.
be a typical presentation. The triad of bleeding, jaundice and abdominal pain
has been described with this condition. However, only
REFERENCE a minority (approximately one-third) of patients will
Tsui W, Chan Y, Wong C, et al. Hepatolithiasis and present with all of these features. An important clue
the syndrome of recurrent pyogenic cholangitis: that should raise the index of suspicion of hemobilia is
clinical, radiologic, and pathologic features. Semin a recent history of biliary tract or hepatic parenchymal
Liver Dis. 2011;31:33-48. instrumentation. Trauma can also be a precipitating
factor. Hemobilia secondary to cholelithiasis, choledo-
cholithiasis, or a ruptured hepatic artery aneurysm
Question 48 can occur, but are less common than iatrogenic or
A 63-year-old man is admitted to the hospital for traumatic causes. Angiography can identify the source
abdominal pain and melena. He has a history of of bleeding and achieve hemostasis through emboli-
hepatitis C and is currently enrolled in a hepatitis zation. An ERCP can be employed to clear the biliary
C therapeutic trial. As part of that clinical study, he tree in a subset of patients with ascending cholangitis
underwent a percutaneous liver biopsy and labora- secondary to a thrombus, but it usually does not play
tory tests which showed AST of 105 U/L and ALT a significant role in hemostasis.
of 115 U/L, and the remainder of his liver tests were
normal. Two days later, he developed mild right REFERENCE
upper quadrant pain. Over the next 24 hours, his Chin WM, Enns R. Hemobilia. Curr Gastroenterol
pain intensified and he had recurrent episodes of Rep. 2010;12:121-129.
melena. He subsequently presented to the emer-
gency room and is admitted. Aside from sinus
tachycardia, his vital signs are normal. His labora- Question 49
tory studies on admission are shown below: A 24-year-old woman is referred to your practice
for a second opinion regarding abnormal imaging
Hemoglobin 6.9 g/dL
findings and recurrent low grade fevers. Ultra-
WBC 10.5/L
sound done to evaluate right upper quadrant pain
Total bilirubin 3.4 mg/dL
revealed dilation of the common bile duct. ERCP
Direct bilirubin 2.6 mg/dL
is recommended and revealed multiple dilations of
Alkaline phosphatase 160 U/L
the extra-hepatic biliary tree as shown below.
ALT 190 U/L
AST 185 U/L
A. Colonoscopy with biopsies Which of the following agents has potential effica-
B. Liver biopsy cy to reduce gallstone formation for this patient?
C. Surgical referral for cholecystectomy and
hepaticojejunostomy A. Conjugated estrogens
D. ERCP with biliary brushings and stenting B. Ursodeoxycholic acid
of the common bile duct C. Fenofibrate
E. Referral to liver transplant center D. Simvastatin
E. Cholestyramine
CORRECT ANSWER: C
CORRECT ANSWER: B
RATIONALE
This patient’s imaging findings, with multiple RATIONALE
dilations of the extrahepatic biliary tree, are Risk factors for gallstone formation include
characteristic of type IVB biliary cysts. Due to increased age, female gender, pregnancy, dyslip-
high malignant potential, type IVB biliary cysts idemia, diabetes, obesity and rapid weight loss
require cholecystectomy and cyst excision with – especially after gastric bypass surgery. Medica-
Roux-en-Y hepaticojejunostomy. If intrahe- tions such as hormone replacement therapies/
patic ducts are involved, hepatic lobe resection oral contraceptive agents, fibrates, somatostatin
is also recommended. analogues also increase gallstone risk. Currently,
This patient does not have evidence of pri- there is evidence suggesting potential benefit of
mary sclerosing cholangitis and therefore does prophylactic cholecystectomy during Roux-en-Y
not require screening colonoscopy with biop- gastric bypass, given the potential risk of gallstone
sies to assess for IBD. Due to the malignant formation with rapid weight loss following sur-
potential of her cysts, ERCP with stenting is not gery. However, there is also data from random-
sufficient. ized controlled trials that the use of ursodeoxycho-
lic acid following surgery may help reduce risk of
REFERENCES
gallstone formation for this group of patients.
Ronnekleiv-Kelly SM, Soares KC, Ejaz A, et al.
Management of choledochal cysts. Curr Opin Gas-
REFERENCE
troenterol. 2016 May;32(3):255-31.
Stokes et al. Ursodeoxycholic Acid and Diets
Law R, Topazian M. Diagnosis and treatment
Higher in Fat Prevent Gallbladder Stones Dur-
of choledochoceles. Clin Gastroenterol Hepatol.
ing Weight Loss: A Meta-analysis of Randomized
2014;12:196-203.
Controlled Trials. 2014. Clin Gastroenterol Hepa-
Soares KC, Arnaoutakis DJ, Kamel I, et al.
tol. 2014;12:1090-1100.
Choledochal cysts: presentation, clinical dif-
ferentiation, and management. J Am Coll Surg.
2014;219:1167-1180.
120 Digestive Diseases Self-Education Program®
Answers & critiques
CHAPTER 5
Viral hepatitis
Alfred Sidney Barritt, IV, MD & Andrew S. deLemos, MD
121
122 Digestive Diseases Self-Education Program®
infection, known as co-infection or afterwards, indicated as the patient has decompensated liver
known as superinfection. Hepatitis D is trans- disease. Supportive care is appropriate, although
mitted parenterally and sexually and is of higher there are emerging data for the use of N-acetyl cys-
prevalence in eastern and southern Europe as well teine for cases of non-acetaminophen liver failure,
as parts of Africa and South America. Patients but such supportive care should be in a transplant
with superinfection with hepatitis delta will often center. Hepatitis B immune globulin and vaccina-
have suppressed HBV replication and are at high tion have no role in acute HDV infection.
risk for a fulminant course. This vignette is not
an acute on chronic flare of hepatitis B as there is REFERENCES
no evidence of significant HBV replication. The Hughes SA, Wedemeyer H, Harrison PM. Hepati-
patient is immune to HAV. Since his HBV DNA is tis delta virus. Lancet 2011;378:73-85.
relatively suppressed, it appears that he is taking
the entecavir. Hydrochlorothiazide is exceedingly
unlikely to cause a drug induced liver injury. Question 6
A 28-year-old woman, originally from Taiwan,
REFERENCE presents to labor and delivery late in the third
Hughes SA, Wedemeyer H, Harrison PM. trimester of her pregnancy and is ready to deliver.
Hepatitis delta virus. Lancet 2011;378:73-85. She is found to have chronic hepatitis B and nor-
mal liver tests. She received no prenatal care. She
is asymptomatic from her liver disease and has
Question 5 never had any decompensation events.
Over the following 24 hours, the patient in the
above vignette becomes encephalopathic, the INR What is the appropriate course of action?
rises to 2.9 and the total bilirubin climbs to 11.5
mg/dl. A. Hepatitis B immune globulin for the mother
B. Prophylactic lamivudine for the infant
What is the most appropriate course of action? C. Hepatitis B immune globulin and HBV
vaccination for the infant
A. Switch to tenofovir alafenamide fumarate D. HBV vaccination only for the infant
(TAF) 25mg daily E. No intervention/Supportive care for the infant
B. Start interferon alfa 2b 180mcg weekly
C. Close monitoring in the intensive care unit CORRECT ANSWER: C
with supportive care
D. Emergent transfer to a liver transplant center RATIONALE
E. Administer Hepatitis B immune globulin In order to reduce the possibility of vertically
and Hepatitis B vaccine transmitted HBV, both hepatitis B immune globu-
lin and vaccination should be given to the infant.
CORRECT ANSWER: D Vaccination alone is insufficient as immunity to
vaccination is not immediate. While antiviral
RATIONALE therapy in the third trimester in the pregnant
The patient has become coagulopathic and en- mother may be appropriate in certain cases, there
cephalopathic, from suspected superinfection with is no role for prophylactic antiviral therapy for
hepatitis delta, and should be evaluated for possible infants nor is lamivudine approved for use in this
liver transplantation emergently. Therapies aimed population.
at treating hepatitis B will be ineffective as the HBV
DNA is already suppressed. Interferon is contra-
124 Digestive Diseases Self-Education Program®
Question 7
A 44-year-old African American male has an Question 8
otherwise asymptomatic icteric illness three A 45-year-old African American female with
weeks after consuming raw shellfish. He takes no genotype 1a HCV is considering retreatment of
prescription or over the counter medications. His her HCV with Sofosbuvir/Velpatasvir. She is a
ultrasound shows no ductal dilatation, no stones relapser to prior therapy with peginterferon and
and no pancreatic masses. He is found by his pri- ribavirin. By fibrosure testing and imaging she is
mary care physician to have a positive HAV IgM. suspected to have well compensated cirrhosis, but
has never had a biopsy. Her MELD score is eight.
Which of the following is true regarding
Hepatitis A infection? Which of the following is true ?
A. Most Americans have been exposed to HAV A. If the patient’s HCV RNA is negative at week
by age five four, she is eligible for 8 weeks of therapy
B. HAV can be especially severe in pregnant B. Due to her cirrhosis, she should receive a total
women of 12 weeks of therapy
C. HAV has a human reservoir and thus spreads C. Because of her cirrhosis, her cure rate is
rapidly in overcrowded living situations estimated at less than 90 percent
D. The risk of fulminant liver disease is higher D. Therapy at this point is inappropriate and
with acute HAV infection compared to acute the patient should be evaluated for liver
HBV infection transplantation
E. HAV does not cause chronic liver injury and
is not associated with chronic disease CORRECT ANSWER: B
C. The patient has been vaccinated against HBV HAV total (+)
D. A liver biopsy would show interface hepatitis HCV Ab (-)
and plasma cells Anti nuclear antibody (-)
E. A liver biopsy would show ballooning Anti Actin (smooth muscle)antibody (-)
hepatocytes and steatosis Fasting bile salts normal
intravenous heroin. His last needle use was seven tis. BMJ 2001;322:151-3.
days ago. On physical exam, he has mild sclera
icterus, tender hepatomegaly and needle tracks in
both antecubital fossa. His blood work is below. Question 13
A 19-year-old college student has had a 10 day
Total Bilirubin 5.1 mg/dL history of nausea, vomiting, diarrhea and fatigue.
AST 950 U/L She recently returned from a two week trip to
ALT 1300 U/L Haiti for volunteer work. On exam, she has a low
Alkaline phosphatase 115 IU/L grade fever (380C) tender hepatomegaly and mild
Albumin 3.4 scleral icterus. Two of her classmates have had
HBsAg (-) similar symptoms. Her blood work is below.
HBsAb (+)
HBc IgM (-) Total Bilirubin 4.8 mg/dl
HAV IgM (-) AST 1250 U/L
HCV Ab (-) ALT 1800 U/L
Alkaline phosphatase 99 IU/L
What lab test is most likely to make a diagnosis? INR 0.9
A. Antimitochondrial and anti smooth Which of the following laboratory tests is most
muscle antibodies likely to reveal the diagnosis?
B. HCV RNA
C. HCV RIBA A. HAV IgM
D. HBc IgG B. HAV total
E. HAV total C. CMV stool PCR
D. HBsAb
CORRECT ANSWER: B E. HCV Ab
RATIONALE RATIONALE
This patient has an HBV DNA over 20,000 IU/ Oral nucleoside/tide agents are very effective for
mL but normal LFT’s and thus qulaifies for moni- viral suppression, but have a low rate of HBeAg
toring only. If her ALT was abnormal or if the seroconversion. Interferon given over the course
HBV DNA were several logs higher, a liver biopsy of a year gives the best opportunity for HBeAg
would be a reasonable intervention, although seroconversion and HBsAg loss in this patient
monitoring is still appropriate. She will need HCC who has genotype A infection, high ALT and low
surveillence at age 50 since she is from Asia even viral loads.
in the absence of cirrhosis.
REFERENCE
REFERENCE Yim HJ, Lok AS. Natural history of chronic hepatitis
Terrault NA, Lok SF, McMahon BJ, et al. B virus infection: what we knew in 1981 and what we
Update on preveention, diagnosis, and treatment know in 2005. Hepatology 2006;43:S173-81.
of chronic hepatitis B: AASLD 2018 hepatitis B
guidance. Hepatology 2018;67:1560-1599.
Question 18
A patient with biopsy proven NASH returns to
Question 17 clinic for routine care. She has normal synthetic
A 43-year-old caucasian male with chronic function and is gradually improving her weight
hepatitis B presents to the office. He is and diabetes control. She has never been exposed
asymptommatic from his disease and he has to viral hepatitis.
not had any decompensation events including
jaundice, encephalopathy or variceal bleed- Which of the following viruses cannot currently be
ing. He is naïve to HBV therapy as well. Here prevented by vaccination?
are his labs:
130 Digestive Diseases Self-Education Program®
ment With Tumor Necrosis Factor Antagonists., have HBV ccc DNA within their hepatocytes that
Clin Gastroenterol Hepatol. 2018 Apr 24. is dormant, but under extreme combined B and
pii: S1542-3565(18)30394-X. doi: 10.1016/j. T cell immunosuppression, the patients are at
cgh.2018.04.033. risk for reverse seroconversion where they can
lose HBV surface Ab and manifest HBV surface
antigen and present as an acute HBV infection.
Question 25 Prophylaxis is required during therapy and for at
A 45-year-old man has recently been diagnosed least 12-18 months after therapy due to the long
with leukemia. The chemotherapeutic regimen lasting effects of anti-B cell monoclonal antibod-
will include rituximab and high dose steroids. ies like rituximab. Reactivation of HCV in HCV
He is a former IV drug user but has been sober Ab positive, RNA negative patients has not been
for 20 years. His lab work is below. reported.
RATIONALE Question 27
Prior to initiating therapy for chronic hepatitis C, A 52-year-old male is seen in your clinic for evalu-
the patient needs to be fully evaluated for chronic ation of newly diagnosed cirrhosis in the setting of
hepatitis B infection. The patient has a positive alcohol abuse and hepatitis C genotype 1b infec-
surface antigen and core antibody with a negative tion. His PMH is notable for a hiatal hernia and
surface antibody which is consistent with chronic persistent gastroesophageal reflux despite abstain-
hepatitis B infection. An HBV DNA level is neces- ing from alcohol over last three months. He has
sary to determine whether the patient has chronic mild esophageal dysphagia and is currently taking
active hepatitis B infection or inactive chronic high dose PPI therapy twice a day.
hepatitis B. HBV/HCV coinfected patients can
develop reactivation or a flare of chronic hepatitis The optimal treatment strategy for this patient’s
B during or after treatment with DAA therapy. If hepatitis C infection at this time would be which of
the HBV DNA level is greater than 20,000 IU/mL, the following regimens?
the patient requires antiviral therapy for hepatitis
B concurrently with DAA therapy through SVR12. A. Sofosbuvir/ledipasvir
Patients with low levels of HBV DNA or an unde- B. Sofosbuvir/velpatasvir
tectable level can either receive prophylactic treat- C. Grazoprevir/elbasvir
ment for HBV or be followed at regular intervals D. Sofosbuvir/ribavirin
during DAA therapy for HBV reactivation. Treat-
ment should be initiated if the HBV DNA levels CORRECT ANSWER: C
increase greater than 10-fold or over 1000 IU/mL
from an undetectable level at baseline. There is RATIONALE
insufficient evidence to recommend HBV antiviral The solubility of the NS5A inhibitors, velpatasvir
therapy in patients with a negative surface anti- and ledipasvir decreases as pH increases, so PPI
gen, but positive core antibody. therapy is expected to decrease the concentra-
The other three choices would be acceptable tion of both drugs and can impact the efficacy of
treatment options for this patient’s chronic therapy with those regimens. Coadministration
hepatitis C genotype 1a infection but withholding of velpatasvir with proton pump inhibitors is not
treatment until the hepatitis B is completely recommended.
evaluated is obligatory. Sofosbuvir/ledipasvir can be taken simultane-
ously under fasted conditions with any PPI at a
REFERENCES dose comparable to omeprazole 20 mg or lower.
https://www.hcvguidelines.org/ Antacids should be avoided within four hours of
Chen G, Wang C, Chen J, Ji D, Wang Y, Wu V, sofosbuvir/ledipasviror sofosbuvir/velpatasvir
et al. Hepatitis B reactivation in hepatitis B and C administration. H2 receptor blockers should be
coinfected patients treated with antiviral agents: A taken simultaneously or 12 hours apart.
systematic review and meta-analysis. Hepatology. Since this patient has significant acid reflux
2017;66(1):13-26. symptoms despite high dose PPI therapy, grazo-
Wang C, Ji D, Chen J, Shao Q, Li B, Liu J, et previr/elbasvir, which does not have a PPI interac-
al. Hepatitis due to Reactivation of Hepatitis B tion would be an ideal option for this patient with
Virus in Endemic Areas Among Patients With compensated cirrhosis. NS5A RAV testing would
Hepatitis C Treated With Direct-acting Antiviral be required for genotype 1a patients, but not
Agents. Clinical gastroenterology and hepatol- genotype 1b patients treated with this regimen.
ogy : the official clinical practice journal of the The SVR12 rate was 97 percent in the subgroup of
American Gastroenterological Association. patients with genotype 1b and stage 4 fibrosis in
2017;15(1):132-6. the C-EDGE trial.
Chapter 5 — Viral hepatitis 135
Question 29
Question 28 A 42-year-old woman with pulmonary sarcoidosis
A 45-year-old woman with genotype 1a HCV is hospitalized for shortness of breath and a flare
presents for re-evaluation. She is treatment expe- of her sarcoidosis. She has been started on 60 mg
rienced and did not achieve an SVR after complet- of prednisone. An astute medical resident taking
ing eight weeks of sofosbuvir/ledipasvir therapy care of the patient sees prior hepatitis serologies
12 months ago. A transient elastography in the notable for HBsAg negative, anti-HBc positive.
office today suggests stage 3 fibrosis. The resident sends an HBV DNA level which is un-
detectable. The pulmonologist anticipates at least
What is the best next step? six weeks of prednisone therapy with a dose 20 mg
or more daily. As the consulting gastroenterolo-
A. Sofosbuvir/velpatasvir/voxilaprevir gist, what is the best next step in management?
B. NS5A RAV testing
C. Sofosbuvir/daclatasvir A. Re-check HBV DNA in three months
D. Sofosbuvir/ledipasvir for 24 weeks B. Start tenofovir
C. Observation
CORRECT ANSWER: A D. Hepatitis B vaccination
stopping immunosuppression. Patients who are A. Alpha fetoprotein every six months
HBsAg- negative/anti-HBc+ are likely to have a B. Liver ultrasound and AFP every six months
lower reactivation rate than their counterparts C. No surveillance
with HBsAg+/anti-HBc+ serostatus but given the D. Liver CT every 12 months
paucity of data in this patient group, prophylaxis
is recommended. This approach is preferred over CORRECT ANSWER: B
monitoring except in situations where the patient
“places a higher value on avoiding long- term use RATIONALE
of antiviral therapy and the cost associated with its The annual incidence of hepatocellular carcinoma
use and a lower value on avoiding the small risk of in patients with hepatitis C and cirrhosis is be-
reactivation (particularly in those who are HBsAg tween one to five percent. Hepatocellular carci-
negative) may reasonably select no prophylaxis noma surveillance is recommended for patients
over antiviral prophylaxis.” with hepatitis C and cirrhosis. Although this
Patients who are HBsAg+ and are treated with patient does not have clinical symptoms of cirrho-
moderate (10-20 mg) or high (over 20 mg) of pred- sis, the stage 4 fibrosis as estimated by transient
nisone for four weeks or more are at high risk for elastography in addition to the mildly reduced
reactivation (over 10 percent) and require HBV an- platelet count make HCC screening prudent for
tiviral prophylaxis with continuation of therapy for this patient. The AGA recommends a VCTE cut-
at least six months after discontinuation of therapy. off of 12.5 kPa to detect cirrhosis in patients with
chronic hepatitis C.
REFERENCE The primary modality for screening is liver ul-
Reddy KR, Beavers KL, Hammond SP, et al. trasound at an interval of every six months. Alpha
American Gastroenterological Association Insti- fetoprotein testing alone is not recommended for
tute Guideline on the Prevention and Treatment screening by the AASLD due to lack of sensitiv-
of Hepatitis B Virus Reactivation During Immu- ity and specificity. However, the updated AASLD
nosuppressive Drug Therapy. Gastroenterology HCC guidelines provide a recommendation that
2015;148(1):215-19. u/s with or without the addition of AFP testing
may be used for screening owing to recent data
that AFP may increase the sensitivity of screening
Question 30 when combined with u/s.
A 62-year-old man presents for follow-up of The risk of HCC is considerably reduced in pa-
chronic hepatitis C infection. Prior to initiating tients achieving SVR after DAA therapy. Among
therapy with sofosbuvir/ledipasvir, a transient 22,500 patients treated with DAA (19,518 with
elastography demonstrated stage 4 fibrosis with SVR; 2982 without SVR), without SVR, those
a score of 14.0 kPa. 12 weeks after completing with SVR had a significantly reduced risk of HCC
treatment, he now has an undetectable HCV RNA. (0.90 vs 3.45 HCC/100 person-years; adjusted
He has no symptoms of cirrhosis and continues hazard ratio, 0.28, 95 percent CI=0.22-0.36). The
to work full-time. His physical examination is risk of HCC, though attenuated, does persist in
normal. His ultrasound shows a heterogeneous patients with cirrhosis after their hepatitis C has
appearing liver with top-normal spleen size. been eradicated. Ongoing HCC surveillance in this
Labs are as follows: population with liver ultrasound every six months
is the best answer.
PLT 130,000, bilirubin 1.0 mg/dL, INR 1.0
creatinine 0.9 mg/dL, AST: 35 U/L, ALT: 30 U/L. REFERENCES
Which of the following is the best approach for Van der Meer AJ, Veldt BJ, Feld JJ, et al. Associa-
monitoring this patient in the future? tion between sustained virological response and
Chapter 5 — Viral hepatitis 137
all-cause mortality among patients with chronic the diagnosis of mixed cryoglobulinemia, which
hepatitis C and advanced hepatic fibrosis. JAMA. will typically present with purpura, arthralgias,
2012;308:2584-2593. and weakness along with active sediment on the
Lim JK, Flamm SL, Singh S, Falck-Ytter YT, urinalysis and renal insufficiency. Mixed cryo-
Clinical Guidelines Committee of the American globulinemia can be cured by eradicating hepatitis
Gastroenterological A. American Gastroenterolog- C in most individuals. A minority of patients can
ical Association Institute Guideline on the Role of have persistent mixed cryoglobulinemia despite
Elastography in the Evaluation of Liver Fibrosis. eradication of HCV, due to the development of
Gastroenterology. 2017;152(6):1536-43. monoclonal B-cell populations. In the absence
Kanwal F, Kramer J, Asch SM, Chayanupat- of positive cryoglobulins or findings to support
kul M, Cao Y, El-Serag HB. Risk of Hepatocel- a diagnosis of MPGN, further evaluation with
lular Cancer in HCV Patients Treated With rheumatoid factor and complement testing is not
Direct-Acting Antiviral Agents. Gastroenterology. necessary.
2017;153(4):996-1005 e1. Chronic hepatitis C is independently associated
with the development of chronic kidney disease,
with a 51 percent increase in the risk of protein-
Question 31 uria and 43 percent increase in the incidence of
A 55-year-old woman with treatment naïve CKD. This patient’s renal dysfunction could be at-
chronic hepatitis C genotype 2 infection is seen tributed to chronic hepatitis C infection and war-
for consultation. Her blood pressure is 140/90 rants further evaluation by nephrology. However,
and her physical exam is notable for pre-tibial IV rituximab would be an appropriate option only
edema, but there are no skin findings. Labora- for a patient with severe mixed cryoglobulinemia
tories demonstrate a normal CBC, ALT 50 U/L, with significant end-organ damage, such as acute
AST 47 U/L, total bilirubin 1.0 mg/dL. Creatinine renal failure requiring hemodialysis.
has been stable at 1.5 mg/dL over the last year. A The most appropriate next step in the man-
urinalysis is negative for RBCs, but notable for 2+ agement of this patient is treatment of the
albuminuria. Serum cryoglobulins are negative. hepatitis C. Glecaprevir/pibrentasvir treatment
A transient elastography demonstrates stage 2 for eight weeks would be indicated for genotype 2
fibrosis. and stage 2 fibrosis. Sofosbuvir/ledipasvir is not
indicated for the treatment of genotype 2 chronic
What is the most appropriate next step in the hepatitis C infection.
management of this patient?
REFERENCES
A. Check rheumatoid factor, C3, C4 Dammacco F, Sansonno D. Therapy for hepatitis C
B. Sofosbuvir/ledipasvir treatment for virus-related cryoglobulinemic vasculitis. N Engl J
12 weeks Med. 2013;369(11):1035-45.
C. Referral to nephrology for initiation of Fabrizi F, Verdesca S, Messa P, Martin P.
IV rituximab Hepatitis C virus infection increases the risk
D. Glecaprevir/pibrentasvir treatment of developing chronic kidney disease: a sys-
for eight weeks tematic review and meta-analysis. Dig Dis Sci.
2015;60(12):3801-3813.
CORRECT ANSWER: D Emery JS, Kuczynski M, La D, Almarzooqi
S, Kowgier M, Shah H, et al. Efficacy and Safety
RATIONALE of Direct Acting Antivirals for the Treatment of
The patient has chronic hepatitis C without Mixed Cryoglobulinemia. The American journal of
physical exam or laboratory findings to support gastroenterology. 2017;112(8):1298-308.
138 Digestive Diseases Self-Education Program®
admission to 1.8 mg/dL today in spite of 1 g/kg of treatment in decompensated patients since gleca-
albumin administration over the last 48 hrs. previr is a protease inhibitor. This antiviral drug
Which of the following would be the best manage- class is poorly tolerated in advanced cirrhosis due
ment strategy for this patient at this time? to risk of decompensation.
A. Sofosbuvir/velpatasvir REFERENCES
B. Evaluation for liver transplant Charlton M, Everson GT, Flamm SL, et al. Ledi-
C. Glecaprevir/pibrentasvir pasvir and Sofosbuvir Plus Ribavirin for Treatment
D. Sofosbuvir/ledipasvir and ribavirin of HCV Infection in Patients with Advanced Liver
Disease. Gastroenterology 2015;149(3):649-59.
CORRECT ANSWER: B Curry MP, O'Leary JG, Bzowej N, Muir AJ, Ko-
renblat KM, Fenkel JM, et al. Sofosbuvir and Vel-
RATIONALE patasvir for HCV in Patients with Decompensated
The best management strategy for this patient Cirrhosis. The New England journal of medicine.
would be evaluation for liver transplant. In a 2015;373(27):2618-28.
patient with a MELD score of 28 and concern for Chhatwal J, Samur S, Kues B, Ayer T, Roberts
evolving hepatorenal syndrome with a more than MS, Kanwal F, et al. Optimal timing of hepatitis C
doubling of the creatinine since admission, a liver treatment for patients on the liver transplant wait-
transplant evaluation is indicated. While treat- ing list. Hepatology. 2017;65(3):777-88.
ment with sofosbuvir/velpatasvir or sofosbuvir/
ledipasvir and ribavirin for 12 weeks are approved
therapies for decompensated cirrhosis based on Question 34
data from ASTRAL-4 and SOLAR-1 and SO- A 40-year-old Caucasian female with a history of
LAR-2, immediate hepatitis C treatment for this allogeneic stem cell transplant for AML is referred
patient will likely be insufficient to avoid liver-re- to clinic for evaluation of possible hepatitis B
lated death or the need for liver transplantation. infection. She was recently hospitalized for treat-
The SOLAR-1 and SOLAR-2 trial demonstrated ment of pneumonia. As part of her evaluation in
improvement in MELD scores in the majority of the hospital, she was found to be hypogammaglob-
patients with genotype 1 or 4 HCV and cirrhosis ulinemic, and was treated for four days with 1 g/kg
(Child Pugh B and C) who underwent treatment intravenous immunoglobulin. Prior to discharge,
with 12 or 24 weeks of sofosbuvir/ledipasvir and she inadvertently has hepatitis B serologies
ribavirin. The sustained virologic response rates checked which reveal HBsAg-, HBsAb+, HBcAb+.
in SOLAR-1 and SOLAR-2 for Child-Turcotte- The pt. has not traveled outside the U.S. and
Pugh Class B patients was 85-87 percent after 12 does not have any remote or recent risk factors for
weeks of treatment with sofosbuvir/ledipasvir hepatitis B exposure. Prior to undergoing chemo-
and ribavirin. The ASTRAL-4 trial demonstrated therapy for her stem cell transplant, her hepatitis
the efficacy of sofosbuvir/velpatasvir treatment in B serologies were appropriately checked and at
genotype 1, 2, 3, 4, or 6 and decompensated cir- that time she was HBsAg-, HBsAb+, and HBcAb-.
rhosis with SVR rates of 83-86 percent in the 12 You send an HBV DNA which is undetectable.
or 24-week arms. Treatment of decompensated ALT is 15 U/L. Which of the following is the best
hepatitis C cirrhosis is recommended to be car- treatment recommendation for this patient?
ried out at a liver transplant center and patients
may improve enough to avoid liver transplan- A. Tenofovir
tation, though more data is needed to further B. Entecavir
understand this possibility. C. Hepatitis B vaccination
Glecaprevir/pibrentasvir is not approved for D. Reassurance
140 Digestive Diseases Self-Education Program®
Terrault NA, Lok ASF, McMahon BJ, Chang KM, C. Ledipasvir + sofosbuvir
Hwang JP, Jonas MM, et al. Update on preven- D. Elbasvir/grazoprevir
tion, diagnosis, and treatment of chronic hepatitis E. Glecaprevir/pibrentasvir
B: AASLD 2018 hepatitis B guidance. Hepatology.
2018;67(4):1560-99. CORRECT ANSWER: A
Wang L, Kourtis AP, Ellington S, et al. Safety
of tenofovir during pregnancy for the mother RATIONALE
and fetus: a systematic review. Clin Infect Dis. This patient has evidence of recent decompen-
2013;57(12):1773-81. sated cirrhosis. Treatment of chronic hepatitis C
has been shown to reduce morbidity and mortal-
ity, including in patients with decompensated
Question 36 cirrhosis. Current AASLD guidelines recommend
A 62-year-old male presents to your office for treatment with ledipasvir (90 mg)/sofosbuvir
follow-up of a recent hospitalization for hepatitis (400 mg) with ribavirin (600 mg, increased as
C cirrhosis complicated by new onset ascites. He tolerated) for 12 weeks. This recommendation is
underwent a paracentesis that was negative for based on data from the SOLAR-2 study, which
SBP and had 3L removed. A screening upper was comprised of patients with Child-Pugh class
endoscopy reveals small varices without high risk B and C cirrhosis, hemoglobin over 10.0 g/dL
features. On exam, he has mild muscle wasting, and creatinine clearance over 40 mL/min where
persistent ascites, but no asterixis. His family SVR rates were 87 percent and 86 percent in CTP
members have noted intermittent episodes of class B and CTP class C respectively. Ledipasvir
confusion recently, however. On further ques- and sofosbuvir alone would not be recommended
tioning, he states that he was treated with pe- in this decompensated patient, nor would it be
gylated-interferon and ribavirin many years ago, the correct therapy for a CTP class A, compen-
but his treatment was discontinued by his prior sated cirrhotic treatment experienced patient in
physician because of lack of reduction in his viral whom RBV is also recommended.
load on treatment.
Withholding therapy is not advisable for this re-
Laboratory findings: cently decompensated patient. Protease contain-
Hemoglobin 11.5 g/dL ing regimens: elbasvir/grazoprevir or glecaprevir/
Leukocyte count 6100/uL pibrentasvir are not recommended for patients
Platelets 85,000/uL with decompensated cirrhosis.
Creatinine 1.0 mg/dL
AST 75 U/L REFERENCES
ALT 78 U/L www.hcvguidelines.org/
Alkaline phosphatase 80 U/L Belli LS, Berenguer M, Cortesi PA, Strazzabos-
Total bilirubin 1.4 mg/dL co M, Rockenschaub SR, Martini S, et al. Delisting
INR: 1.3 of liver transplant candidates with chronic hepa-
titis C after viral eradication: A European study.
His HCV RNA is found to be 2,360,000 IU/mL, Journal of hepatology. 2016;65(3):524-31.
with an HCV genotype of 1A. Manns M, Samuel D, Gane EJ, Mutimer D, Mc-
Caughan G, Buti M, et al. Ledipasvir and sofosbuvir
Which is the best treatment option? plus ribavirin in patients with genotype 1 or 4 hepa-
titis C virus infection and advanced liver disease: a
A. Ledipasvir + sofosbuvir + ribavirin multicentre, open-label, randomised, phase 2 trial.
B. No therapy Lancet Infect Dis. 2016;16(6):685-97.
142 Digestive Diseases Self-Education Program®
Cheung MCM, Walker AJ, Hudson BE, Verma S, There is no compelling reason to switch to tenofovir
McLauchlan J, Mutimer DJ, et al. Outcomes after as the patient has excellent viral suppression. In
successful direct-acting antiviral therapy for pa- the absence of a hepatitis flare with increase in
tients with chronic hepatitis C and decompensated ALT, checking for HDV infection is not needed.
cirrhosis. Journal of hepatology. 2016;65(4):741-7.
REFERENCE
Terrault NA, Lok ASF, McMahon BJ, Chang KM,
Question 37 Hwang JP, Jonas MM, et al. Update on preven-
A 48-year-old Vietnamese man who has known tion, diagnosis, and treatment of chronic hepatitis
chronic hepatitis B returns for a follow-up visit. B: AASLD 2018 hepatitis B guidance. Hepatology.
He initiated entecavir therapy 3 years ago and at 2018;67(4):1560-99.
that time was sAg+ and eAg+ with an HBV DNA of
30,000 IU/ml and an ALT of 120 U/L.
Today, he feels well and has a normal physical Question 38
examination. His liver ultrasound shows a mildly A 25-year-old man presents to the emergency room
coarsened liver echotexture without focal liver with two weeks of abdominal pain, nausea, and
lesion and his transient elastography is consistent anorexia. His physical exam is notable for scleral
with stage 4 fibrosis. Laboratory studies today are icterus and mild tenderness to palpation in the right
notable for a normal CBC, ALT 25 U/L, AST 20 upper quadrant. He reports experimenting with IV
U/L, total bilirubin normal, sAg+, eAg-, eAb+, and drugs about one month ago but denies alcohol use.
undetectable HBV DNA. Abdominal ultrasound shows a normal contour of
the liver, no dilation of the bile ducts, and normal
What is the most appropriate treatment strategy? Doppler evaluation of the hepatic and portal vessels.
RATIONALE RATIONALE
This patient is at risk for acute hepatitis C due to This correct answer is C, liver ultrasound. Hepatitis
his recent initiation of IV drug abuse. In patients B is unique as a risk factor for hepatocellular carci-
with exposure less than six months, the hepatitis noma (HCC) in the absence of cirrhosis. The AASLD
C antibody may be negative and hepatitis C viral recommends HCC surveillance for black or Asian
RNA should be obtained. Although acute hepatitis men over 40 years, and Asian women over 50 years
might be due to atypical infections, such as EBV, with chronic hepatitis B regardless of fibrosis status.
the AST and ALT elevation in this patient would A liver ultrasound should be performed every six
be more consistent with acute hepatitis C, whereas months in these populations in addition to those
in acute EBV, one would expect the AST and ALT with cirrhosis secondary to hepatitis B. Alpha-
elevations to be lower and be preceded by a viral fetoprotein every six months is recommended for
prodrome with fever and sore throat. Evaluation patients where ultrasound is not readily available.
for the hepatitis BeAg and eAb is not indicated This patient is an inactive carrier and therefore
with negative surface antigen and core antibody does not require treatment.
testing. A liver biopsy and CT scan are reason-
able tests to investigate liver test abnormalities REFERENCES
but given this patient’s history of recent IV drug Terrault NA, Lok ASF, McMahon BJ, Chang KM,
abuse, acute hepatitis C should be ruled out first. Hwang JP, Jonas MM, et al. Update on preven-
tion, diagnosis, and treatment of chronic hepatitis
REFERENCES B: AASLD 2018 hepatitis B guidance. Hepatology.
Centers for Disease Control and Prevention 2018;67(4):1560-99.
(CDC). Testing for HCV infection: an update of Heimbach JK, Kulik LM, Finn RS, Sirlin CB,
guidance for clinicians and laboratorians. MMWR Abecassis MM, Roberts LR, et al. AASLD guide-
Morb Mortal Wkly Rep. 2013;62(18):362-5. lines for the treatment of hepatocellular carcino-
Blackard JT, Shata MT, Shire NJ, et al. Acute ma. Hepatology. 2018;67(1):358-80.
hepatitis C virus infection: a chronic problem.
Hepatology. 2008 Jan;47(1):321-31.
Question 40
A 40-year-old Chinese male presents to your office
Question 39 for evaluation of chronic hepatitis B. He acquired
A 44-year-old Cambodian male was diagnosed by his HBV from vertical transmission and has never been
primary care provider with chronic hepatitis B. He treated. During his last visit, a transient elastography
is referred for further evaluation and management. was performed which suggested stage 1 fibrosis. He
Examination is normal. Labs demonstrate a normal is asymptomatic, and his physical exam is normal.
complete blood count and aminotransferases. Hepa- He is hepatitis B eAg- and hepatitis B eAb+.
titis B tests are as follows: HBsAg positive, HBeAg The last two sets of laboratories are shown below.
negative, Anti-HBe positive, and HBV DNA 50 IU/ml.
What is the next best step in the management of Lab value 3 months prior Current
this patient? ALT 120 U/L 92 U/L
AST 75 U/L 105 U/L
A. Pegylated Interferon HBV DNA 2050 IU/mL 2725 IU/mL
B. Oral nuceleos(t)ide
C. Liver ultrasound What is the best next step in the management
D. Observation of this patient?
144 Digestive Diseases Self-Education Program®
and B have a more favorable response to PEG-IFN What is the next best step in the management
in contrast to genotypes C and D. of this patient?
REFERENCES A. IV solumedrol
Ahn SH, Marcellin P, Ma X, Caruntu FA, B. IV acyclovir
Tak WY, Elkhashab M, et al. Hepatitis B C. IV N-acetylcysteine
Surface Antigen Loss with Tenofovir Disoproxil D. Ribavirin
Fumarate Plus Peginterferon Alfa-2a: Week
120 Analysis. Digestive diseases and sciences. CORRECT ANSWER: B
2018;63;3487-3497.
Marcellin P, Ahn SH, Ma X, Caruntu FA, Tak RATIONALE
WY, Elkashab M, et al. Combination of Tenofovir The next best step in the management of this
Disoproxil Fumarate and Peginterferon alpha-2a patient is IV acyclovir. The presentation of fe-
Increases Loss of Hepatitis B Surface Antigen in ver, altered mental status, right upper quadrant
Patients With Chronic Hepatitis B. Gastroenterol- pain with hepatomegaly, vital signs consistent
ogy. 2016;150(1):134-44 e10. with systemic inflammatory response syndrome,
leukopenia, high aminotransferases and coagu-
lopathy, collectively should raise concern for
Question 42 herpes hepatitis. A high index of suspicion is
A 22-year-old woman is admitted to the ICU with needed for this condition. In two studies analyz-
fevers, malaise, and mental status changes. Her ing case reports and cases series, the diagnosis
recent history is notable for delivering a healthy of herpes hepatitis was made post-mortem in 57
child two week prior to presentation. She has not percent and 77 percent of the cases, respectively.
taken any acetaminophen or new medications. The case fatality rate exceeds 80 percent in un-
She has no recent travel history. treated patients with progression to acute liver
Laboratory analysis demonstrates a white failure. A recent review of 20 confirmed/prob-
blood cell count of 2,200, platelet count 120,000, able cases by the ALFSG also report 60 percent
ALT 4000 U/L, AST 6500U/L, total bilirubin mortality at 21 days.
5mg/dL, direct bilirubin, 3.9 mg/dL, INR 2.2, and Intravenous acyclovir does improve survival,
creatinine 1.5 mg/dL. Ultrasound demonstrates and therefore, institution of IV acyclovir should
mild hepatomegaly, no features to suggest cirrho- not be delayed in situations where there is clinical
sis, and spleen size of 13 cm. A transjugular liver suspicion for herpes hepatitis. Patients at risk for
biopsy is performed as shown below. this condition are usually immunocompromised
or peripartum. Aminotransferase levels in the
1000s with disproportionately lower bilirubin,
leukopenia, and a septic-like presentation are
typical presenting features. Mucocutaneous or
genital lesions are present in the minority of cases.
The diagnosis can be confirmed with liver biopsy
demonstrating intranuclear nuclear inclusions,
multinucleated giant cells, and coagulative necro-
sis with minimal inflammation.
IV Solu-Medrol is a treatment for severe acute
autoimmune hepatitis. This case does not provide
information about auto-antibodies or gammaglob-
ulin levels, though acute severe presentation of au-
146 Digestive Diseases Self-Education Program®
toimmune hepatitis is frequently accompanied by On exam, he has a low-grade fever, icteric sclera
a lack of autoantibodies and normal gammaglobu- and is mildly tender to palpation in the RUQ. An
lin levels. The liver biopsy shown does not show u/s shows a heterogeneous liver echotexture with-
characteristic features of autoimmune hepatitis. out biliary dilation or gallstones.
Thus, IV solume- drol is incorrect. Laboratories studies are as follows.: hepatitis A
IV N-acetylcysteine is a treatment for acet- IgM negative, hepatitis B DNA negative, hepatitis
aminophen overdose which this patient does not C RNA negative, smooth muscle antibody nega-
have. A recent AGA guideline statement suggests tive, anti-nuclear antibody negative. What is the
that IV N-acetyl cysteine be used in non-acetamin- best treatment for this patient?
ophen ALF only in the context of a clinical trial.
In cases with an indeterminate cause in which A. Observation
acetaminophen could be involved, use of NAC B. Ribavirin
should be strongly considered. C. MRI/MRCP
Ribavirin is the treatment for patients with D. HIDA scan
chronic HEV infection which can occur in
immunosuppressed patients. HEV can cause CORRECT ANSWER: A
ALF in pregnant patients in endemic parts of
the world. RATIONALE
This patient has acute hepatitis E, genotype 3. This
REFERENCES is a zoonotic infection acquired in developed coun-
Norvell JP, Blei AT, Jovanovic BD, et al. Herpes try through exposure to infected animals such pigs,
simplex virus hepatitis: an analysis of the pub- boars, and deer. The presentation is that of an acute
lished literature and institutional cases. Liver viral hepatitis, and the treatment in an immunocom-
Transpl. 2007;13(10)1428-34. petent patient is supportive care. Men over 40 years
Flamm SL, Yang YX, Singh S, Falck-Ytter YT, old seem to be a group at higher risk for infection.
Committee AGAICG. American Gastroenterologi- In immunocompetent individuals, hepatitis E is
cal Association Institute Guidelines for the Di- generally a self-limited condition, but in solid-organ
agnosis and Management of Acute Liver Failure. transplant recipients, chronic infection can ensue.
Gastroenterology. 2017;152(3):644-7. Approximately 60 percent of transplant recipients
Little L, Rule J, Peng L, Gottfried M, Lee WM, who are infected with hepatitis E develop chronic
Acute Liver Failure Study G. Herpes Simplex infections. The best treatment for chronic hepatitis
Virus-Associated Acute Liver Failure Often Goes E in solid organ transplant recipients is ribavirin.
Unrecognized. Hepatology. 2018. In one study, the sustained virologic response rate
was 78 percent after a course of approximately three
Question 43 months of ribavirin.
A 64-year-old man is referred to your GI clinic for HEV RNA can be identified in serum or stool
evaluation of abnormal liver transaminases and for diagnosis of hepatitis E. However, HEV RNA
jaundice. He feels unwell and hasn’t been able to PCR is not readily available outside of research
keep food down for a week. His PMH is notable settings and therefore the CDC states that the
for diet-controlled diabetes and hypertension. He diagnosis can be confirmed only by testing for the
drinks a few beers on Sunday while watching foot- presence of antibody against HEV or HEV RNA.
ball but denies a history of chronic alcohol abuse. Providers must be aware of the possibility of false
He recently returned from a trip to Texas to a positive and negatives for HEV serologies.
gaming ranch where he and his friends hunted Although the pt. has RUQ pain and anorexia,
deer and wild boar. There are no recent antibiotic his u/s does not show any biliary dilation or gall-
exposures. stones. Further work-up for obstructive jaundice
Chapter 5 — Viral hepatitis 147
with MRCP or evaluation for biliary dyskinesia at toms of acute viral hepatitis with negative testing
this time is not warranted. for hepatitis B and hepatitis C and positive hepati-
tis A IgM Ab. There has been a recent increase in
REFERENCES hepatitis A infections. Infections in the U.S. occur
Kamar N, Izopet J, Tripon S, et al. Ribavirin for in patients who have traveled to another country
chronic hepatitis E virus infection in transplant where hepatitis A virus transmission is common
recipients. N Engl J Med. 2014;370(12):1111-20. and in sporadic outbreaks associated with con-
Dao Thi VL, Debing Y, Wu X, et al. Sofos- taminated uncooked foods. An important risk
buvir Inhibits Hepatitis E Virus Replication In group is MSM and injection drug users. A recent
Vitro and Results in an Additive Effect When outbreak in Tennessee has been associated with
Combined With Ribavirin. Gastroenterology. this demographic.
2016;150(1):82-5. Hepatitis A generally resolves over the course
https://www.cdc.gov/hepatitis/hev/hevfaq.htm of weeks and does not evolve into a chronic hepa-
titis. However, in approximately 10 percent of pa-
tients or less, a relapsing course can occur. About
Question 44 one-fifth of these individuals will have more than
A 25-year-old male presents to his local emergency one relapse. The relapse events are associated with
room with malaise, nausea, vomiting, abdominal similar symptoms that were present at the time of
pain, and yellowish discoloration of the skin. The the initial presentation but tend to be milder. The
patient endorses having same sex relationships for hepatitis A IgM remains detectable in those with
money so that he can afford to buy drugs. He has a relapsing course and the hepatitis A virus can be
been intermittently homeless over the last year. detected in the stool indicating the possibility for
Vital signs are within normal limits. On exami- infection transmission. Nearly all patients recover
nation, he has mild jaundice and no findings to completely over a six to 12-month period.
suggest chronic liver disease. Liver and spleen are Autoimmune hepatitis is less likely with this
not enlarged. presentation and the serum IgG is normal with
Lab results are as follows: ALT 850 U/L, AST only weakly positive smooth muscle Ab. Hepati-
700 U/L, total bilirubin 5mg/dL, direct biliru- tis E infection would be associated with zoonotic
bin 3.0 mg/dL, alkaline phosphatase 137 U/L. exposure or travel to an endemic area. While
Hepatitis A IgM positive, HBsAg negative, anti- EBV could be a consideration, the IgM viral
HBc IgM negative, hepatitis C antibody negative, capsid is negative and the pronounced transami-
cytomegalovirus IgM and IgG negative, EBV viral nitis is more than what one would see with EBV
capsid IgM negative, smooth muscle antibody+ hepatitis.
1:40, normal serum IgG.
REFERENCES
Which of the following is the most likely diagnosis? Bornstein JD, Byrd DE, Trotter JF. Relapsing
Hepatitis A: a case report and review of the litera-
A. Autoimmune hepatitis ture. J Clin Gastroenterol. 1999;28(4):355-6.
B. EBV https://www.tn.gov/health/cedep/tennessee-
C. Hepatitis E hepatitis-a-outbreak.html
D. Hepatitis A
Question 45
CORRECT ANSWER: D Which of the following is the most appropriate
management strategy for a healthy 50-year-old
RATIONALE woman whose son recently returned home from
Hepatitis A is responsible for this patient’s symp- Mexico with acute hepatitis A infection?
148 Digestive Diseases Self-Education Program®
RATIONALE
The current guidelines for post exposure pro- Question 46
phylaxis for hepatitis A infection vary by age and A 53-year-old woman status-post kidney trans-
health status. For healthy people over 12 months plant for FSGS 3 months ago is admitted to the
to 40 years old, hepatitis A vaccine is preferred hospital with anorexia and recent bouts of diar-
over IG due to likelihood of response, long-term rhea. She underwent induction with Campath
protection, and equivalent efficacy to IG. IG is and is currently taking tacrolimus and mycophe-
recommended within two weeks of exposure for nolate mofetil. You are asked to see her as the
persons over 40 years old due to lack of knowledge consulting gastroenterologist and arrange for her
on vaccine performance and possibility of more to have an EGD/flex sig and biopsies the following
severe presentation in patients with older age. IG day. CMV serostatus is donor positive and recipi-
is particularly important to give to immunosup- ent positive.
pressed patients exposed to hepatitis A given the
risk of fatal hepatitis A infection and reduced Laboratory findings:
response to vaccination in this population.
Hepatitis A vaccine is currently recommended Hemoglobin 9.0 g/dL
for all children at age one year. Patients with WBC count 2500/uL
chronic liver disease are at increased risk of mor- Platelet count 120,000/uL
bidity and mortality if they acquire acute hepatitis Creatinine 1.3 mg/dL
A infection. Thus, pre-exposure prophylaxis has AST 175 U/L
been recommended for patients with chronic liver ALT 122 U/L
disease who are susceptible to HAV. Alkaline phosphatase 150 U/L
Other high-risk groups include chronically Total bilirubin 0.9 mg/dL
immunosuppressed patients, those who travel to
endemic areas, working in occupations for which You suspect CMV disease with GI and liver
there is a high likelihood for exposure, family involvement. Which of the following treatments
members of infected patients, men who have sex would you recommend?
with men, HIV-positive patients, those with clotting
factor disorders, and those who use injection and A. Oral ganciclovir
non-injection illicit drugs. Hepatitis A is transmit- B. Intravenous ganciclovir
ted via the fecaloral route and parenteral transmis- C. Oral valacyclovir
sion is rare; thus, those on chronic hemodialysis are D. Intravenous acyclovir
not considered an especially high-risk group. E. Oral acyclovir
with concurrent GI involvement. Liver biopsies of In addition to recommending that he abstain from
CMV would be notable for the presence of inclu- alcohol, what is the best treatment option?
sion bodies, resembling an owl’s eye, and a small
foci of mononuclear cell infiltrates and microab- A. Observation
scesses. B. Pegylated Interferon
Intravenous ganciclovir, at 5 mg/kg twice daily, C. Telbivudine
or oral valganciclovir, 900 mg orally twice daily, D. Entecavir
are the treatment of choice for CMV hepatitis and/
or GI CMV. Both valganciclovir and intravenous CORRECT ANSWER: D
ganciclovir have been demonstrated to be effica-
cious for the treatment of mild to moderate CMV RATIONALE
disease in solid organ transplant recipients. Oral The AASLD recommends that adult patients with
ganciclovir has poor oral availability (7 percent) compensated cirrhosis and low-level viremia (less
compared to oral valgancyclovir (70 percent), than 2,000 IU/mL) be treated with antiviral therapy
and therefore is not recommended. Acyclovir and to reduce the risk of decompensation, regardless of
vala- cyclovir have weak activity against CMV ALT level. The preferred treatment regimens are
and should not be used for the treatment of CMV entecavir or tenofovir. Optimal duration of therapy
disease. is not currently known. In a large open label trial
where patients were treated for 240 weeks with te-
REFERENCES nofovir, at the end of the study 71 of the 96 patients
Asberg A, Humar A, Rollag H, et al. Oral valganci- with cirrhosis (74 percent) had regression of cirrho-
clovir is noninferior to intravenous ganciclovir for sis. Therefore, some compensated patients may not
the treatment of cytomegalovirus disease in solid only be protected from flares of active hepatitis but
organ transplant recipients. Am J Transplant. may also experience improvement in their histol-
2007;7:2106–13. ogy and therefore observation is not appropriate.
Kotton CN, Kumar D, Caliendo AM, et al.. Inter- Pegylated interferon should generally be avoided in
national consensus guidelines on the management cirrhosis due to risk of decompensation. Telbivudine
of cytomegalovirus in solid organ transplantation. has an approximately 20 percent risk of resistance at
Transplantation. 2010;89:779–795. two years and therefore is not the best oral agent.
REFERENCES
Question 47 Marcellin P, Gane R, Buti M, et al. Regres-
A 48-year-old male establishes care in your clinic sion of cirrhosis during treatment with teno-
for management of chronic hepatitis B. He is fovir disoproxil fumarate for chronic hepatitis
treatment naïve, only recently obtained insur- B: a 5-year open-label follow-up study. Lancet
ance, but now wants to be sure he has appropriate 2013;381(9865):468-75.
follow-up. He has hypertension and drinks two Terrault NA, Lok ASF, McMahon BJ, Chang
to three beers each night after work. His physical KM, Hwang JP, Jonas MM, et al. Update on
exam is unremarkable. He undergoes a transient prevention, diagnosis, and treatment of chronic
elastography in the office demonstrating stiffness hepatitis B: AASLD 2018 hepatitis B guidance.
values that would be consistent with cirrhosis. Hepatology. 2018;67(4):1560-99.
Notable lab abnormalities include platelet
count 108,000, ALT 40 U/L, AST 32 U/L, T Bili
1.0 mg/dL, creatinine 0.8 mg/dL, HBsAg posi- Question 48
tive, HBeAg negative, Anti-HBe positive, HBV A 35-year-old woman is one month post-op from
DNA 1000 IU/ml. a mastectomy for breast cancer. She received
150 Digestive Diseases Self-Education Program®
neoadjuvant chemotherapy with docetaxel, doxo- A palliative care consult is therefore inappropriate.
rubicin, cyclophosphamide, and trastuzumab. She A liver biopsy with her coagulopathy and obvious
is admitted to her local hospital with new onset of HBV diagnosis is incorrect. HBV antiviral therapy
jaundice, anorexia, and abdominal pain. could be started but is not associated with a mor-
Notable lab abnormalities include ALT 1200 tality benefit in patients with ALF due to HBV.
U/L, AST 1100 U/L, T Bili 25.0 mg/dL, creatinine
1.4 mg/dL, INR: 2.2, HBcIgM positive, HBsAg REFERENCES
positive, HBV DNA over 160,000,000. Karvellas CJ, Cardoso FS, Gottfried M, Reddy
You are the on-call gastroenterologist at a ter- KR, Hanje AJ, Ganger D, et al. HBV-Associated
tiary care facility 60 miles from the local hospital Acute Liver Failure After Immunosuppression
and you are called by the admitting hospitalist for and Risk of Death. Clinical gastroenterology and
your recommendation. He says she is oriented x hepatology : the official clinical practice journal
3, but she seems to be a little bit sleepy. of the American Gastroenterological Association.
2017;15(1):113-22.
What is your advice? Reddy KR, Beavers KL, Hammond SP, Lim
JK, Falck-Ytter YT, American Gastroenterological
A. Palliative care consult Association I. American Gastroenterological As-
B. Liver biopsy sociation Institute guideline on the prevention and
C. Tenofovir treatment of hepatitis B virus reactivation during
D. Transfer for liver transplant evaluation immunosuppressive drug therapy. Gastroenterol-
ogy. 2015;148(1):215-9; quiz e16-7
CORRECT ANSWER: D
RATIONALE Question 49
This patient is presenting with acute reactivation A 65-year-old male with genotype 1a HCV pres-
hepatitis B infection and acute liver failure. She re- ents for evaluation. He is treatment naïve and a
ceived high dose chemotherapy which would have recent transient elastography reveals stage 2 fibro-
placed her at high risk (more than 10 percent) for sis. He had a stroke six months ago in the context
reactivation of hepatitis B infection from previ- of atrial fibrillation, but fortunately has regained
ously undiagnosed chronic hepatitis B. Prior to most of his strength back and is independent with
receiving chemotherapy, this patient should have all ADLs. Medications include amiodarone, lisino-
been screened for occult hepatitis B infection, and pril, and atorvastatin. Which regimen should be
provided immunoprophylaxis if found to be sAg+ used to treat this patient’s hepatitis C?
or even if previously exposed to hepatitis B (sAg-/
cAb+/sAb+). The coagulopathy and signs of grade A. Glecaprevir/pibrentasvir
I hepatic encephalopathy are consistent with acute B. Sofosbuvir/ledipasvir
liver failure. Patients with ALF from HBV infec- C. Sofosbuvir/velpatasvir
tion who are immunosuppressed compared to pa- D. Sofosbuvir/velpatasvir
tients who are not had worse overall survival at 21 E. No treatment since the patient has had a stroke
days, in a retrospective review of 156 cases of HBV
ALF from the ALFSG (43 percent vs. 63 percent). CORRECT ANSWER: A
Patient survival from breast cancer is excellent and
this patient should absolutely be considered for liv- RATIONALE
er transplant even with a recent cancer diagnosis, Glecaprevir/ pibrentasvir should be used for this
while additional data regarding her cancer staging patient due to the interaction between sofosbuvir
and risk of recurrence of malignancy are assessed. containing regimens and amiodarone.
Chapter 5 — Viral hepatitis 151
Question 50
A 48-year-old male with genotype 3 HCV presents
for evaluation. He is treatment naïve. A recent
transient elastography reveals stage 3 fibrosis. He
is a heavy smoker and had an acute myocardial
infarction one year ago with drug-eluting stent
placement. Medications include atenolol, aspirin,
lisinopril, and atorvastatin.
Which regimen should be used to treat this pa-
152 Digestive Diseases Self-Education Program®
Answers & critiques
CHAPTER 6
Metabolic, hereditary,
inflammatory and vascular
diseases of the liver
Kiran Bambha, MD, MSc and Robert C. Lowe, MD
153
154 Digestive Diseases Self-Education Program®
RATIONALE REFERENCES
In a large randomized clinical trial of adults with Adams LA, Lymp JF, St Sauver J, et al. The natural
biopsy-proven nonalcoholic steatohepatitis (NASH), history of nonalcoholic fatty liver disease: a population-
vitamin E demonstrated efficacy in improving ste- based cohort study. Gastroenterology 2005;129:113-21.
atosis and inflammation, but was not associated with Chalasani N, Younossi Z, Lavine JE, et al. The
improvements in hepatic fibrosis. Metformin, ursode- Diagnosis and Management of Nonalcoholic Fatty
oxycholic acid and phlebotomy have all been inves- Liver Disease: Practice Guidance from the Ameri-
tigated as therapeutic agents in NASH and have not can Association for the Study of Liver Disease.
demonstrated efficacy in improving NASH histology. Hepatology 2018;67:328-57.
REFERENCES
Chalasani N, Younossi Z, Lavine JE, et al. The Question 4
Diagnosis and Management of Nonalcoholic Fatty Which of the following conditions is associated
Liver Disease: Practice Guidance from the Ameri- with increased risk for nonalcoholic fatty liver
can Association for the Study of Liver Disease. disease (NAFLD)?
Hepatology 2018;67:328-57.
Sanyal AJ, Chalasani N, Kowdley KV, et al. Pi- A. Alopecia areata
oglitazone, vitamin E, or placebo for nonalcoholic B. Psoriasis
Chapter 6 — Metabolic, hereditary, inflammatory and vascular diseases of the liver 155
phosphatase level of 454 U/L, positive anti-mito- localized within the inner mitochondrial matrix. In
chondrial antibody [AMA] (titer, 1:1280) and el- approximately 95 percent of patients with PBC, AMA
evated serum IgM level comes to see you for initial are directed towards PDC-E2.
consultation and recommendations regarding her
chronic liver disease. Her past medical history is REFERENCE
notable only for hypercholesterolemia with a total Lindor KD, Gershwin E, Poupon R, et al. Primary
cholesterol level of 290. She has no cardiovascular Biliary Cirrhosis. Hepatology 2009;50:291-308.
disease. Her physical exam is normal with a BMI
of 22 kg/m2. Which of the following statements is
true regarding her liver disease? Question 14
You have evaluated a 51-year-old woman with an
A. Her elevated total cholesterol level that is related elevated serum alkaline phosphatase level of 379
to her liver disease places her at increased risk U/L , total bilirubin level of 2.1 mg/dL and positive
for a cardiovascular event despite her young age. anti-mitochondrial antibody (AMA) with a titer of
B. Her first-degree relatives are at the same risk 1:1280. Fibroscan is suggestive of minimal fibro-
for developing PBC as the general population. sis. You have given her a new diagnosis of primary
C. She is at risk for developing varices in the biliary cholangitis (PBC). She presents to your
absence of cirrhosis. clinic with questions about the natural history of
D. The AMA is directed against the bilirubin her PBC and her prognosis. Which of the following
uridine diphosphate glucuronosyl transferase statements is true regarding the natural history and
enzymes found primarily in hepatocytes. epidemiology of primary biliary cholangitis (PBC)?
Question 18 Question 19
Which of the following statements is true regard- A 34-year-old African-American woman presents
ing IgG4-related sclerosing cholangitis? to you for evaluation of persistently elevated liver
enzymes. Her alkaline phosphatase is 256 U/L,
A. It is commonly associated with elevated AST 42 U/L and ALT 24U/L. Total bilirubin level
pANCA levels. is normal. She has no other medical diagnoses.
B. The typical age of onset is 20-25 years old. She does not drink alcohol or smoke tobacco. She
C. Corticosteroids are first-line treatment. complains of fatigue and a chronic dry cough.
D. It affects women more commonly than men. Serologic testing for viral hepatitis, autoimmune
liver disease, iron overload and alpha-one-anti-
CORRECT ANSWER: C trypsin deficiency is negative. Abdominal ultra-
sound reveals an enlarged and heterogeneous-ap-
RATIONALE pearing liver with patent hepatic and portal veins.
IgG4-related sclerosing cholangitis is one manifes- Chest x-ray reveals bilateral hilar adenopathy. You
tation of the disease entity known as IgG4-related recommend a liver biopsy, which reveals multiple
disease (IgG4-RD). IgG4-RD can affect many organs small granulomas. Her serum angiotensin con-
in the body and is characterized by dense lympho- verting enzyme level is elevated to 165 U/L (ULN
plasmacytic infiltrations with a predominance of 8-53 U/L). Which of the following statements is
IgG4-positive plasma cells, often associated with true regarding hepatic sarcoidosis?
some degree of fibrosis. Patients may present with a
mass in the affected organ or diffuse enlargement of A. A diagnosis of hepatic sarcoid mandates
an organ (such as the pancreas). Multiple organs are initiation of corticosteroids.
affected in the majority of patients with IgG4-RD. B. Hepatic sarcoid is characterized by caseating
Serum IgG4 levels may be elevated in up to 66 per- granulomas in the liver.
162 Digestive Diseases Self-Education Program®
REFERENCE REFERENCE
Deutsch-Link S, Fortuna D, Weinberg EM. A Demetris AJ, Batts KP, Dhillon AP, et al. Banff
Comprehensive Review of Hepatic Sarcoid. Schema for Grading Liver Allograft Rejection:
Seminars in Liver Disease 2018;38:284-97. An International Consensus Document. Hepa-
tology 1997;25:658-63.
Question 20
A 63-year-old man underwent deceased donor Question 21
liver transplant eight months ago for alcohol- A 60-year-old man who underwent deceased do-
induced cirrhosis. His liver enzymes normalized nor liver transplant two months ago for hepatitis
soon after transplant and remained normal. His C (HCV) cirrhosis presents for urgent clinical
liver enzymes have been repeatedly elevated evaluation. His HCV was treated and cured
over the past week with AST 180 U/L, ALT 212 pre-transplant. His immediate post liver trans-
U/L, alkaline phosphatase of 188 U/L, normal plant course was uneventful and he received
total bilirubin and normal INR. His tacrolimus four doses of anti-thymocyte globulin (ATG)
level most recently was 2.0 ng/mL (target level induction. His liver enzymes and liver function
5-7 ng/mL). He is feeling reasonably well, but normalized soon post-transplant, as anticipated.
does note some mild fatigue. He takes no other He presents now with a three-day history of
Chapter 6 — Metabolic, hereditary, inflammatory and vascular diseases of the liver 163
RATIONALE RATIONALE
This patient’s presentation is characteristic of This patient meets all the diagnostic criteria for
graft versus host disease (GVHD) post-liver early hepatic artery thrombosis after liver trans-
transplant with fevers, leukopenia, anemia, se- plant, with her deterioration in her clinical status
cretory diarrhea and maculopapular rash. Liver characterized by severe elevations in her AST and
enzymes are normal is the setting of GVHD. ALT with rising INR and acidosis. Her hepatic
Rash, diarrhea and cytopenia are not character- artery could not be visualized on ultrasound with
istics of acute cellular rejection. Additionally, the doppler. Hepatic artery thrombosis early after liver
normal liver enzymes would argue against acute transplant is a surgical emergency. This patient is
cellular rejection. CMV infection is unlikely as unstable with her altered mental status and hypo-
the patient is still on CMV prophylaxis with val- tension requiring dual-pressor support. She needs
ganciclovir. Small for size syndrome is charac- re-transplant and meets criteria for listing as Status
terized by post-operative coagulopathy and liver 1A, connoting highest urgency. CT angiography will
dysfunction due to insufficient functional liver not alter the diagnosis or change management. This
mass, which is not consistent with the patient’s patient is too unstable to undergo interventions
presentation here. on her hepatic artery by Interventional Radiology.
Initiation of IV heparin would be dangerous in the
REFERENCE setting of this patient’s immediate post-transplant
Murali AR, Chandra S, Stewart Z, et al. Graft status and, in the context of her acutely failing
versus Host Disease After Liver Transplantation allograft due to her hepatic artery thrombosis, anti-
in Adults: A Case series, Review of Literature, coagulation should be avoided.
164 Digestive Diseases Self-Education Program®
B. Homogeneous portal perfusion of the liver on 1.6. RUQ ultrasound revealed a normal GB with-
contrast-enhanced CT scan. out gallstones, pericholecystic fluid, and GB wall
C. Hepatic imaging demonstrating hepatic thickening. CBD was three millimeters, liver was
caudate lobe hypertrophy. enlarged, and no liver masses were seen. Doppler
D. Serum-Ascites Albumin Gradient (SAAG) less than study showed normal flow in the hepatic and portal
1.1, and ascites total protein greater than 3 mg/dL. veins. Spleen was mildly enlarged. Upon question-
ing, his family reports that he drinks one to two
CORRECT ANSWER: C pints of hard liquor daily. Which of the following
treatments is the best option for this patient?
RATIONALE
Caudate lobe hypertrophy is found in a majority of A. Corticosteroids
patients with Budd Chiari syndrome, and is due to B. Ursodeoxycholic acid
the separate venous drainage of the caudate into C. Pentoxifylline
the inferior vena cava allowing for sparing of the D. Infliximab
outflow, resulting in compensatory hypertrophy. E. Azathioprine
The characteristic pattern of parenchymal perfu-
sion of a Budd Chiari liver on contrast-enhanced CORRECT ANSWER: A
cross sectional imaging includes early homogeneous
central enhancement with delayed patchy enhance- RATIONALE
ment of the peripheral liver and prolonged retention In a patient with a history of heavy alcohol use, the
of contrast in the hepatic periphery, resulting in diagnosis of acute alcoholic hepatitis (AH) is sug-
inhomogeneous portal perfusion. A characteristic gested by the onset or sudden worsening of jaundice
feature of Budd Chiari syndrome on hepatic imaging with or without other signs of hepatic dysfunction.
is its association with intrahepatic or subcapsular Physical exam in AH may reveal stigmata of liver
hepatic venous collaterals, which are identified in up cirrhosis, as well as SIRS criteria, due to the signifi-
to 80 percent of patients. The SAAG in Budd Chiari cant inflammatory response in the liver. Laboratory
is characteristically elevated 1.1 or more, along with findings include an elevated bilirubin, and an AST/
an elevated ascites total protein level. ALT ratio greater than 1.5 with a total AST or ALT
less than 400 IU/L. Biopsy is not typically per-
REFERENCE formed, but will show macrovesicular steatosis, a
DeLeve L, Valla DC, Garcia-Tsao G. Vascular Dis- neutrophilic lobulitis, the presence of Mallory-Denk
orders of the Liver. Hepatology 2009;49:1729-64. bodies, and pericellular fibrosis. Unfortunately, these
findings are also seen in NASH, so an alcohol history
is a crucial part of the workup. Current guidelines
Question 26 define “Definite AH” by the biopsy findings, but allow
A 57-year-old man presents to the ED with a a diagnosis of “Probable AH” if the following criteria
complaint of RUQ, fever, chills and jaundice for are met: 1) Heavy alcohol use for greater than five
five days. He reports no dysphagia or odynophagia, years; 2) active alcohol use until four weeks prior
no diarrhea or constipation, His VS are as fol- to presentation; 3) sudden onset or worsening of
lows: Pulse 100 bpm and regular, BP 145/70 mm jaundice; 4) AST/ALT greater than 1.5 with levels
Hg, RR 14 per min, O2 saturation 98 percent on less than 400 IU/L; and 5) absence of other causes
room air. His exam is remarkable for moderately of liver disease. In patients with AH, prognosis can
tender hepatomegaly. His CBC: WBC 16.1K cells/ be estimated using the MELD score or the Maddrey
mm3, Hgb 12.1 mg/dL, PLT 147K cells/mm3. Liver Discriminant Function; a DF greater than 32 pre-
function tests: AST 143 U/L, ALT 88 U/L, ALP 110 dicts a 20-50 percent 30-day mortality and patients
U/L, T. bilirubin 9.1 mg/dL, Albumin 3.0 g/L, INR meeting this threshold should be treated. Current US
166 Digestive Diseases Self-Education Program®
guidelines call for the use of corticosteroids (specifi- suggested by the onset or sudden worsening of
cally prednisolone) in severe AH. Response to ste- jaundice with or without other signs of hepatic dys-
roid therapy is assessed at day seven using the Lille function. Physical exam in AH may reveal stigmata
score, with therapy continued for a total of 28 days in of liver cirrhosis, as well as SIRS criteria, due to
initial responders. Pentoxifylline had shown possible the significant inflammatory response in the liver.
benefit in earlier studies, but more recent data calls Laboratory findings include an elevated bilirubin,
this into question. Infliximab had been studied in AH and an AST/ALT ratio greater than 1.5 with a total
and was found to have severe infectious complica- AST or ALT less than 400 IU/L. Biopsy is not
tions, precluding its use. Azathioprine is used in the typically performed, but will show macrovesicular
treatment of autoimmune hepatitis but has no role steatosis, a neutrophilic lobulitis, the presence of
in AH and ursodeoxycholic acid is utilized in Primary Mallory-Denk bodies, and pericellular fibrosis. Un-
Biliary Cholangitis. fortunately, these findings are also seen in NASH,
so an alcohol history is a crucial part of the workup.
Question 27
A 57-year-old man with a history of alcohol abuse Question 28
presents to the ED with a complaint of RUQ, f/c, A 50-year-old man presents to clinic with a com-
and jaundice for five days. He reports no dysphagia plaint of fatigue. He drinks approximately one pint
or odynophagia, no diarrhea or constipation. His of whiskey daily, and has done so for more than 20
VS are as follows: Pulse 92 bpm and regular, BP years. His exam is remarkable for hepatomegaly,
145/70 mm Hg, RR 14, O2 saturation 98 percent on palmar erythema, and spider angiomata of his neck
room air. His exam is remarkable for moderately and chest, but no asterixis or altered mental func-
tender hepatomegaly. His CBC: WBC 16.1K cells/ tioning. Liver function tests: AST 110 U/L, ALT 60
mm3, Hgb 12.1 mg/dL, PLT 147K cells/mm3. Liver U/L, ALP 110 U/L, T. bilirubin 1.8 mg/dL, Albu-
function tests: AST 143 U/L, ALT 88 U/L, ALP 110 min 3.0 g/L, INR 1.2. RUQ ultrasound revealed a
U/L, T. bilirubin 11.2 mg/dL, Albumin 3.0 g/L, INR normal GB without gallstones, pericholecystic fluid,
1.6. RUQ ultrasound revealed a normal GB without and GB wall thickening. CBD was three millime-
gallstones, pericholecystic fluid, or GB wall thicken- ters, liver was small and nodular, no liver masses
ing. CBD was three millimeters, liver was enlarged, were seen, and Doppler study showed normal flow
and no liver masses were seen. Doppler study in the hepatic and portal veins. Spleen was mildly
showed normal flow in the hepatic and portal veins. enlarged. Which of the following will lead to the
Spleen was mildly enlarged. A liver biopsy would greatest improvement in his survival?
most likely show which of the following?
A. Corticosteroids
A. PAS positive, diastase resistant globules in B. Ursodeoxycholic acid
hepatocytes C. Pentoxifylline
B. Ballooning degeneration and Mallory’s hyaline D. Infliximab
C. Lymphoplasmacytic infiltration with rosettes E. Abstinence
D. Extensive fibrosis with minimal inflammation
E. Granulomatous inflammation CORRECT ANSWER: E
A. Deficient alpha-1 AT leads to neutrophilic trypsin Deficiency: Current Approaches and Future
infiltration of the liver and hepatocyte necrosis Directions. Curr Pathobiol Rep 2017;5:243-252.
B. Excess misfolded alpha-1 AT accumulates in Townsend SA, Edgar RG et al. Systematic
the liver, leading to oxidative damage to review: the natural history of alpha-1 antitrypsin
hepatocytes. deficiency, and associated liver disease. Aliment
C. Excess misfolded alpha-1 AT accumulates in Pharmacol Ther 2018;47:877-885.
the liver, leading to lymphocytic infiltration of
the liver and subsequent fibrosis.
D. Deficient alpha-1 AT leads to hepatocyte Question 31
damage by enzymes that would normally be A 32-year-old woman with a history of hypothy-
deactivated by this enzyme. roidism presents to the ED with a complaint of
fatigue and jaundice. Her exam is remarkable
CORRECT ANSWER: B for moderate RUQ tenderness on palpation. Her
CBC: WBC 11.1K cells/mm3, Hgb 11.1 mg/dL,
RATIONALE PLT 100K cells/mm3. Liver function tests: AST
Alpha-1 antitrypsin deficiency is a common meta- 131 U/L, ALT 155 U/L, ALP 109 U/L, T. bilirubin
bolic cause of liver disease in children and adults, 4.1 mg/dL, Albumin 3.0 g/L, TP 8.9 g/L, INR
affecting approximately 1:3500 live births. Produc- 1.2. RUQ ultrasound revealed a normal GB with
tion of a misfolded A1 AT protein leads to low A1AT no gallstones, pericholecystic fluid, or GB wall
levels in the serum (less than 15 percent of normal), thickening. CBD was three millimeters, liver was
which leads to lung damage via the unopposed ac- of small size with a nodular contour, and no liver
tion of pulmonary neutrophil elastase. In the liver, masses were seen. Doppler study showed normal
however, damage is caused by accumulation of the flow in the hepatic and portal veins. Moder-
abnormal protein in the endoplasmic reticulum of ate splenomegaly was present. ANA was 1:2560.
hepatocytes, causing oxidative damage and eventu- Which of the following therapies would be a com-
al fibrosis and cirrhosis. On biopsy, the misfolded ponent of the initial treatment of this patient?
protein appears as PAS positive globules within
hepatocyte that are resistant to digestion with A. Azathioprine
diastase. A1 AT should be suspected in any patient B. Cyclosporin
with unexplained cirrhosis or unexplained mild C. Ursodeoxycholic acid
elevations of AST and ALT. A clinical clue to A1 AT D. Tenofovir alefenamide
deficiency is a personal or family history of early E. Cyclophosphamide
COPD (before age 50). Although a low A1 AT level
in the serum can suggest the diagnosis, this enzyme CORRECT ANSWER: A
is an acute phase reactant and the A1AT level may
be falsely elevated in the setting of inflammation. RATIONALE
The gold standard of diagnosis is A1 AT phenotyp- Autoimmune hepatitis (AIH) is a chronic inflamma-
ing using isoelectric focusing. This testing allows tory disorder of the liver characterized by elevated
identification of the abnormally folded protein (the transaminases, positive autoimmune serologies,
S or Z forms) instead of the normal protein (the M and elevated immunoglobulin levels, with vary-
form). Patients with the PiZZ phenotype express ing degrees of liver damage. Clues to the diagnosis
10-20 percent of the normal A1AT and tend to have include personal or family history of autoimmune
the most severe disease. conditions, and female gender, though the sex ratio
is only 3.6:1. The presentation can vary from asymp-
REFERENCES tomatic disease with abnormal liver enzymes, to
Mitchell EL, Khan Z. Liver Disease in Alpha-1 Anti- decompensated cirrhosis, to an acute hepatitis with
Chapter 6 — Metabolic, hereditary, inflammatory and vascular diseases of the liver 169
data is not conclusive, and it is most often used in inflammation. Progression to cirrhosis and ESLD is
children with significant cholestatic liver disease. rare, but patients with GD have an increased risk of
HCC (as well as hematologic malignancies). Treat-
REFERENCE ment of GD with drugs such as eliglustat will lead to
Sathe MN, Freeman AJ. Gastrointestinal, a marked decrease in liver and spleen size.
Pancreatic, and Hepatobiliary Manifesta-
tions of Cystic Fibrosis. Pediatr Clin North Am REFERENCE
2016;63:679-698. Nagral A. Gaucher Disease. J Clin Exp Hepatol
2014;4:37-50.
Question 35
A 13-year-old boy of Eastern European descent, Question 36
with a history of humeral and femoral fractures in A four-month-old child is evaluated for lethargy
the past, is found to have significant hepatospleno- and seizures that occur between feedings. Exam is
megaly on physical exam. His CBC: WBC 9.1K remarkable for hepatomegaly and skin xanthomas.
cells/mm3, Hgb 9.1 mg/dL, PLT 95K cells/mm3. He is found to be profoundly hypoglycemic between
Liver function tests: AST 31 U/L, ALT 33 U/L, ALP feedings, and responds to frequent feedings. Even
95 U/L, T. bilirubin 0.8 mg/dL, Albumin 4.0 g/L, with control of his metabolic derangement, which
INR 0.9. RUQ ultrasound revealed a normal GB, long-term sequela may be expected in this child?
CBD three millimeters, markedly enlarged liver and
spleen with no liver masses noted. Doppler study A. Cirrhosis
showed normal flow in the hepatic and portal veins. B. Hepatocellular carcinoma
Which of the following enzyme deficiencies may be C. Insulinoma
responsible for this clinical syndrome? D. Episodic hyperbilirubinemia
E. Movement disorders
A. Glycogen phosphorylase
B. Glucose 6-phosphate dehydrogenase CORRECT ANSWER: B
C. Glucocerebrosidase
D. Hexosaminidase A RATIONALE
E. Sphingomyelinase Glycogen storage disease type I (von Gierke’s Dis-
ease) is an inborn error of metabolism occurring in
CORRECT ANSWER: C 1/100,000 births. A defect in glucose-6-phosphatase
makes it impossible for the child to break down gly-
RATIONALE cogen into glucose. The glycogen-rich liver becomes
Gaucher’s Disease (GD) is a lysosomal storage enlarged, while the lack of glucose production leads
disease caused by mutations in the beta-glucocer- to profound hypoglycemia and lactic acidosis be-
ebrosidase gene, which leads to accumulation of tween feedings, manifesting as lethargy and seizures.
glucocerebroside in macrophages, with the liver, Patients also have significant hyperlipidemia and may
spleen, and bone marrow being the major sites of develop xanthomata. Dietary management (frequent
accumulation. This is an autosomal recessive disor- feedings either orally or via gastrostomy tube) can
der that has a high prevalence in Ashkenazi Jew- manage the hypoglycemia, but patients remain at risk
ish populations. Clinical manifestations typically for the development of hepatic adenomas, which may
include hepatosplenomegaly, and liver enzymes undergo malignant transformation in up to 10 percent
may be normal or only mildly elevated. On liver of cases. Liver transplantation may be considered
biopsy, Gaucher’s cells (lipid-laden macrophages) when HCC is identified, and leads to correction of
are prominent, with variable amounts of associated much of the metabolic defect in this condition.
172 Digestive Diseases Self-Education Program®
REFERENCE REFERENCE
Rake JP, Visser G et al. Glycogen storage disease Rake JP, Visser G et al. Glycogen storage disease
type I: diagnosis, management, clinical course type I: diagnosis, management, clinical course
and outcome. Results of the European Study on and outcome. Results of the European Study on
Glycogen Storage Disease Type I (ESGSD I). Eur J Glycogen Storage Disease Type I (ESGSD I). Eur J
Pediatrics 2002;161 Supp1:S20. Pediatrics 2002;161 Supp1:S20.
Question 37 Question 38
A 14-week-old child presents to his pediatrician A 72-year-old man with a history of CAD and PVD
with lethargy and seizures that occur between feed- is day five post-op from aorto-bifemoral bypass
ings. Exam is remarkable for hepatomegaly and surgery, and he has developed jaundice. His VS
skin xanthomas on the buttocks. He is found to be are as follows: Pulse 100 bpm and regular, BP
profoundly hypoglycemic between feedings, with 125/70 mm Hg, RR 14, O2 saturation 98 percent
evidence of lactic acidosis on lab evaluation. Liver on room air. His exam is remarkable for jaundice
biopsy will show which of the following findings? and mild RUQ tenderness on palpation. His CBC:
WBC 9.1K cells/mm3, Hgb 12.5 mg/dL, PLT 320K
A. PAS positive/diastase resistant granules in cells/mm3. Liver function tests: AST 1920 U/L,
hepatocytes ALT 2130 U/L, ALP 103 U/L, T. bilirubin 6.1 mg/
B. Microvesicular steatosis within hepatocytes dL, albumin 3.9 g/L, INR 1.0. Serum LDH was
C. Lymphocytic infiltrate with piecemeal necrosis 2600 U/L. RUQ ultrasound revealed a normal
of hepatocytes GB with no gallstones, CBD was three millime-
D. Glycogenosis of hepatocytes ters, liver was of normal size and contour, no liver
E. Normal Histology masses were seen, Doppler study showed normal
flow in the hepatic and portal veins. Which of the
CORRECT ANSWER: D following treatments should be administered?
and an elevated serum LDH. Alkaline phospha- so there should be a high index of suspicion in
tase levels are only mildly elevated. Transaminases patients with a less obvious decrease in hepatic
usually decrease rapidly and normalize within two perfusion. Patients may experience symptoms
weeks, but elevations in bilirubin may persist after of nausea, RUQ pain, and malaise, and the liver
the transaminases improve. Treatment consists of enzyme pattern is characterized by markedly
supportive care and restoration of cardiac output elevated transaminases and an elevated serum
and liver perfusion – no specific drug therapy is LDH. Alkaline phosphatase levels are only mildly
recommended in ischemic hepatitis. elevated. Transaminases usually decrease rapidly
and normalize within two weeks, but elevations
REFERENCES in bilirubin may persist after the transaminases
Lightsey JM and Rockey DC. Current Concepts improve. Liver biopsy typically reveals hepatocyte
in Ischemic Hepatitis. Curr Opin Gastroenterol necrosis in Zone 3 of the acinus with few inflam-
2017;33:158-163. matory cells. Treatment consists of supportive
Tapper EB, Sengupta N, Bonder A. The In- care and restoration of cardiac output and liver
cidence and Outcomes of Ischemic Hepatitis: A perfusion – no specific drug therapy is recom-
Systematic Review with Meta-analysis. Am J Med mended in ischemic hepatitis.
2015;128:1314-1321.
REFERENCES
Lightsey JM and Rockey DC. Current Concepts
Question 39 in Ischemic Hepatitis. Curr Opin Gastroenterol
A 64-year-old man with a history of CAD and PVD 2017;33:158-163.
is day five post-op from a three-vessel CABG. His Tapper EB, Sengupta N, Bonder A. The In-
VS are as follows: Pulse 100 bpm and regular, BP cidence and Outcomes of Ischemic Hepatitis: A
125/70 mm Hg, RR 14, O2 saturation 98 percent Systematic Review with Meta-analysis. Am J Med
on room air. His exam is remarkable for scleral ic- 2015;128:1314-1321.
terus and mild RUQ tenderness on palpation. His
liver function tests: AST 1538 U/L, ALT 1684 U/L,
ALP 112 U/L, T. bilirubin 4.1 mg/dL, albumin 3.9 Question 40
g/L, INR 1.0. Serum LDH is 2000 U/L. Which of Which of the following is a risk factor for ischemic
the following pathologic lesions would be seen on cholangiopathy?
liver biopsy?
A. Primary biliary cholangitis
A. Periportal lymphocytic infiltrates B. Liver transplantation
B. Steatohepatitis C. Budd-Chiari Syndrome
C. Sinusoidal dilation D. Autoimmune hepatitis
D. Zone 3 hepatocyte necrosis E. Acetaminophen toxicity
E. PAS positive granules within hepatocytes
CORRECT ANSWER: B
CORRECT ANSWER: D
RATIONALE
RATIONALE Ischemic cholangiopathy is a rare condition of
Ischemic hepatitis, due to an acute lowering of bile duct injury due to impaired blood supply. It
blood flow to the liver, is an uncommon condi- is most commonly associated with liver trans-
tion, accounting for one to three percent of ICU plantation and with the administration of hepatic
admissions. It is often called “shock liver,” but intra-arterial chemotherapy, but it is also seen in
only 50 percent of patients have overt shock, AIDS cholangiopathy, polyarteritis nodosa, and
174 Digestive Diseases Self-Education Program®
RATIONALE A. 10 percent
Portal vein thrombosis (PVT) typically presents B. 25 percent
with abdominal pain, and may be accompanied by C. 50 percent
fever or other inflammatory markers. If the clot is D. 75 percent
confined to the portal vein, symptoms are mild-to- E. 90 percent
moderate, but if there is extension into the SMV,
intestinal ischemia or infarction may occur. Dop- CORRECT ANSWER: C
pler ultrasound is used to make the diagnosis of
PVT, with an absence or decreased flow in the PV RATIONALE
lumen. If Doppler flow is seen within the obstruct- Acute portal vein thrombosis in patients without
ing thrombus, or if there is evidence of pulsatile ar- cirrhosis is an uncommon condition, typically as-
terial flow in the thrombus, the possibility of tumor sociated with a hypercoagulable state. Myelopro-
thrombus must be entertained. CT will confirm the liferative disorders are the most common etiology
diagnosis, showing a luminal filling defect without of PVT, seen in 30-40 percent of cases. Other
enhancement. In the absence of cirrhosis, a num- common causes include Protein C and S deficien-
ber of hypercoagulable states may result in PVT, cy, antithrombin deficiency, and Factor V Leiden
with myeloproliferative disorders present in 30-40 mutation. Anticoagulation therapy is indicated in
176 Digestive Diseases Self-Education Program®
REFERENCE
Question 47 Chinsky JM, Singh R et al. Diagnosis and Treat-
A two-month-old boy of French-Canadian descent ment of Tyrosinemia Type 1: A US and Canadian
presents with hypoglycemia, jaundice, and failure Consensus Group review and Recommendations.
to thrive. He is found to have significant hepato- Genet Med 20178;19:1380.
megaly on physical exam. His liver function tests:
AST 71 U/L, ALT 90 U/L, ALP 105 U/L, T. bilirubin
4.6 mg/dL, Albumin 3.7 g/L, INR 0.9. His AFP is Question 48
markedly elevated. He also has significant amino- A 27-year-old man with a history of bipolar disor-
aciduria and renal tubular acidosis. der presents to the ED with confusion and jaun-
dice. His VS are as follows: Pulse 100 bpm and
Which of the following tests will be abnormal in regular, BP 125/70 mm Hg, RR 14, O2 saturation
this child? 98 percent on room air. His exam is remarkable
Chapter 6 — Metabolic, hereditary, inflammatory and vascular diseases of the liver 179
Question 50
A 19-year-old male with elevated liver enzymes
is found to have a serum ceruloplasmin of three
mg/L, and a urine copper excretion of 210 mcg/24
hrs. He is started on trientine to treat his ill-
ness. What level of urine copper is the goal of this
therapy?
CHAPTER 7
Cirrhosis and
liver transplantation
Catherine Lucero, MD and Jamile’ Wakim-Fleming, MD
181
182 Digestive Diseases Self-Education Program®
Question 3
A 53-year-old woman with longstanding alcohol Question 4
use presents to your office for management of new What is the most likely mechanism responsible for
diagnosis of cirrhosis. She continues to drink two beta-blockers decreasing risk of variceal bleed?
beers daily. She has no complaints of abdominal
pain and has no other significant medical history. A. Splanchnic vasodilatation
She is taking thiamine and folic acid. Her vital B. Reduction in portal pressure
signs are stable and she does not have ascites but C. Rise in cardiac output
has palmar erythema. Her labs include AST 300 D. Enhancement of bacterial translocation
U/L, ALT 150 U/L, total bilirubin of 1.2 mg/dL,
albumin of 3.0 g/dL, INR 1.2, and a platelet count CORRECT ANSWER: B
of 125,000/µL. She had a recent ultrasound that
was negative for liver cancer. What is the next best RATIONALE
step in management? Non-selective beta-blockers (propranolol, nadolol)
reduce portal pressure and are recommended for
A. Observation primary or secondary prophylaxis of variceal hem-
B. Beta-blocker orrhage. They likely improve survival in cirrhosis
Chapter 7 — Cirrhosis and liver transplantation 183
echocardiogram that did not reveal any evidence gram but returned to drinking and currently drinks
of heart failure or pulmonary hypertension.What three beers daily. He has no other medical history.
is the next best step in management to prevent Vitals include T 98.7o F, BP of 122/64 mmHg, HR
further bleeding episodes and improve survival? 86 beats per minute. His abdomen is distended
with large ascites and he has 2+ pitting edema up
A. Balloon tamponade to his knees bilaterally. Labs include AST 220 U/L,
B. Sclerotherapy ALT 100 U/L, total bilirubin of 1.5 mg/dL, albumin
C. Transjugular intrahepatic portosystemic shunt of 3.7 g/dL, sodium 142mEq/L, creatinine of 0.5
(TIPS) placement mg/dL, and a platelet count of 159,000/µL. He had
D. Proton pump inhibitor a diagnostic paracentesis, which revealed ascites
E. Beta blocker polymorphonuclear count of 100/mm3, albumin
2.0 g/dL, total protein 1.6 g/dL. In addition to
CORRECT ANSWER: C alcohol cessation, what is the next best step in
management?
RATIONALE
Candidates for preemptive “early” TIPS are those A. Start furosemide and spironolactone
patients with variceal hemorrhage who are Child C B. Peritoneovenous shunt
(score 10-13) or Child Class B with active hemor- C. Placement of peritoneal catheter
rhage at endoscopy. The patient is Child Class C D. TIPS placement
(score 11). Data from a randomized clinical trial E. Referral for liver transplantation
demonstrate that early TIPS placed within 72
hours of diagnostic endoscopy following initial CORRECT ANSWER: A
vasoactive/endoscopic therapy is associated with
improved outcomes including survival rates. This RATIONALE
patient does not have contraindications to TIPS Ascites is the most common decompensating
such as advanced age, hepatocellular carcinoma, event in cirrhosis. The initial approach to treat-
heart failure, or significant encephalopathy. ing ascites involves dietary sodium restriction and
oral diuretics. Fluid restriction is not required
REFERENCES unless there is hyponatremia (sodium less 130
Monescillo A, et al. Influence of portal hyperten- mEq/L). Spironolactone is more effective than
sion and its early decompression by TIPS place- loop diuretics and should be started at a dose of
ment on the outcome of variceal bleeding. Hepa- 50–100 mg daily. Side effects of diuretics include
tology 2004; 40:793-801. encephalopathy, electrolyte abnormalities, and
Garcia-Pagan JC, et al. Early TIPS (Transjugu- painful gynecomastia. TIPS placement would be
lar Intrahepatic Portosystemic Shunt) Cooperative indicated if the patient had diuretic intolerant or
Study Group. Early use of TIPS in patients with refractory ascites. Liver transplant can be consid-
cirrhosis and variceal bleeding. N Engl J Med ered, however would not be the first choice in a
2010; 362:2370-2379. patient with alcoholic liver disease who recently
relapsed. A peritoneal catheter is not the first line
treatment and is mainly used in the palliative set-
Question 9 ting. A peritoneovenous shunt is a subcutaneously
A 50-year-old man with new onset ascites presents placed silicone tube that transfers ascites from the
to the emergency room with abdominal pain and peritoneal cavity to the systemic circulation. It can
discomfort. He has a history of heavy alcohol use be performed in patients who are not candidates
for thirty years. He was abstinent for two years for TIPS or transplant, but it carries risk of occlu-
after completing an alcohol relapse prevention pro- sion, infection, and bleeding.
186 Digestive Diseases Self-Education Program®
Question 10
A 42-year-old man with hepatitis B cirrhosis Question 11
with ascites was started on furosemide 40 mg and You are called while consulting on the liver service
spironolactone 100mg after recent discharge from for a newly diagnosed patient with hepatitis C
the hospital and was stable for two weeks. He now cirrhosis who was admitted for lower extremity
returns to the emergency room with acute dys- swelling. The team started oral diuretics and is
pnea. He is visibly uncomfortable. Vital signs in- asking for an outpatient appointment. He has
clude heart rate 110 beats per minute, blood pres- no complaints and feels better with initiation of
sure 110/68, and oxygen saturation is 94 percent diuretics. His abdomen is distended with large as-
on room air. His abdomen remains distended cites and he has 1+ pitting edema up to his knees
but his ascites and edema has improved. He has bilaterally. Labs include AST 50 U/L, ALT 22 U/L,
decreased breath sounds on the right side. Labs total bilirubin of 2.5 mg/dL, albumin of 3.4 g/
include AST 150 U/L, ALT 68 U/L, total bilirubin dL, sodium 144mEq/L, potassium of 3.2 mEq/L
of 1.5 mg/dL, sodium 142 mEq/L, creatinine of creatinine of 0.5 mg/dL, INR of 1.5, and a platelet
0.6 mg/dL, potassium of 4.1 mEq/L, and a platelet count of 159,000/µL. His ultrasound reveals large
count of 159,000/µL. His imaging shows a new ascites but does not show hepatocellular carci-
pleural effusion halfway up the right lung field. noma or new thrombosis.
In addition to consideration of TIPS placement, What is the next best step in management prior to
which of the following should be involved in this discharge?
patient’s management?
A. Endoscopy for variceal screening
A. Start beta-blockers for tachycardia B. Diagnostic paracentesis
B. Hold diuretics C. Liver transplant evaluation
C. Chest tube placement D. MRI abdomen
D. Liver transplant evaluation E. Prophylactic antibiotics
RATIONALE RATIONALE
Management of hepatic hydrothorax should not be All patients with cirrhosis and ascites who are hos-
treated with a chest tube. The use of oral diuret- pitalized should undergo a diagnostic paracentesis
ics and repeated thoracentesis is only transiently to exclude spontaneous bacterial peritonitis (SBP).
effective for patients with refractory hydrothorax, Diagnosis of SBP is an ascites neutrophil count
and in these cases, TIPS should be considered. (not total WBC count) greater than 250/mm3. The
Chapter 7 — Cirrhosis and liver transplantation 187
most frequent manifestations are abdominal pain, is greater than 2.5, the ascites is related to post-
fever, and elevated white blood cell count, but up sinusoidal hypertension. The etiologies are cardiac
to one third of patients with SBP may be entirely ascites, Budd-Chiari and veno-occlusive disease.
asymptomatic. This patient does not have Budd-Chiari based on
imaging and no history of hematologic diseases,
REFERENCE which would put him at higher risk for veno-
Moore KP, et al. The management of ascites in occlusive disease. Given history of uncontrolled
cirrhosis: report on the consensus conference of hypertension and alcohol use this patient likely
the International Ascites Club. Hepatology 2003; has heart failure and resultant cardiac ascites from
38:258–266. uncontrolled disease. If this were related to alco-
holic cirrhosis, the patient would give a high SAAG
and low protein ascites. Patients with tuberculous
Question 12 peritonitis and malignant causes usually have a
A 57-year-old man from China presents to the low SAAG and high protein ascites.
emergency room with new onset ascites. He has
had decreased appetite and lost ten pounds in the REFERENCES
last two months. He has a history of heavy alcohol Runyon BA, Montano AA, Akriviadis EA, Antillon
use and hypertension. He has not seen a physi- MR, Irving MA, McHutchison JG. The serum-as-
cian in over twenty years. Vitals include T 98.8 F, cites albumin gradient is superior to the exudate-
BP of 90/60 mmHg, HR 89 beats per minute. His transudate concept in the differential diagnosis of
abdomen is distended with large ascites and he has ascites. Ann Intern Med 1992; 117: 215-220.
2+ pitting edema up to his knees bilaterally. Labs Runyon BA. Cardiac ascites: a characterization.
include AST 60 U/L, ALT 30 U/L, total bilirubin J Clin Gastroenterol. 1988; 10: 410.
of 1.2 mg/dL, albumin of 3.2 g/dL, sodium 138
mEq/L, creatinine of 0.8 mg/dL, and a platelet
count of 162,000/µL. He had an ultrasound, which Question 13
was limited, but did not show any masses or throm- A 57-year-old woman with hepatitis B cirrhosis
boses. He had a paracentesis, which revealed fluid is admitted with melena and was found to have
polymorphonuclear count of 100/mm3, albumin moderate portal hypertensive gastropathy with ac-
2.0 g/dL, and total protein 3.6 g/dL. What is the tive oozing. She underwent a diagnostic paracen-
most likely etiology of his ascites? tesis, which revealed ascites polymorphonuclear
count of 120/mm3, albumin 1.5 g/dL, total protein
A. Alcoholic cirrhosis 1.6 g/dL. Labs are AST 45 U/L, ALT 40 U/L,
B. Budd Chiari syndrome total bilirubin of 2.5 mg/dL, albumin of 3.7 g/dL,
C. Cardiac ascites sodium 142mEq/L, and a creatinine of 1.1 mg/
D. Tuberculous peritonitis dL. Which of the following is an indication to start
E. Malignant ascites prophylaxis in this patient?
RATIONALE A. Infection
The only clear indications for antibiotic prophy- B. Constipation
laxis in patients with cirrhosis are those admitted C. Narcotic use
with GI hemorrhage and patients who have recov- D. Volume overload
ered from an episode of SBP. Although ascites flu- E. Gastrointestinal bleeding
id total protein less than one g/dl increases risk of
development of SBP, long-term prophylaxis in this CORRECT ANSWER: D
patient population is not generally recommended
as the risk for developing antibiotic resistance RATIONALE
and C. difficile colitis outweighs the small effect Episodic hepatic encephalopathy (HE) is usually
in preventing infection. Norfloxacin prophylaxis precipitant-induced in over 80 percent of cases
has been shown to decrease risk of SBP in patients and include dehydration, infections, over diuresis,
with the following parameters: Child–Pugh score gastrointestinal bleeding, constipation, and the
greater than nine points with serum bilirubin level use of narcotics and sedatives. Key is to identify
more than three mg/dl, creatinine level greater and treat the precipitant. A diagnostic workup to
than 1.2 mg/dl, blood urea nitrogen level greater rule out other disorders that can alter brain func-
than 25 mg/ dl, or sodium level less than 130 tion and mimic HE should also be performed.
mEq/L. The patient in this case does not have any
of the above laboratory parameters. REFERENCE
Viltstrup H, et al. Hepatic encephalopathy in
REFERENCES chronic liver disease: 2014 Practice Guideline by
Fernandez J, et al. Antibiotic prophylaxis in the American Association for the Study of Liver
cirrhosis: Good and bad. Hepatology. 2016; 63: Diseases and the European Association for the
2019-2031. Study of the Liver. Hepatology. 2014; 60(2):715-35.
Fernandez J, Navasa M, Planas R et al. Pri-
mary prophylaxis of spontaneous bacterial perito-
nitis delays hepatorenal syndrome and improves Question 15
survival in cirrhosis. Gastroenterology 2007; 133: A 64-year-old woman with alcoholic cirrhosis is
818-824. admitted to the hospital with recurrent episode of
hepatic encephalopathy. She was recently started
on lactulose and has been having two to three soft,
Question 14 formed bowel movements daily. Vital signs include
A 62-year-old man with hepatitis C cirrhosis is ad- heart rate 80 BPM, blood pressure 122/67 mm Hg,
mitted with altered mental status. He had a recent and oxygen saturation is 100 percent on room air.
dental procedure and was given pain medication She is alert and oriented but has asterixis. Her abdo-
and a short course of antibiotics. He is only taking men is soft and nondistended, and she has no evi-
spironolactone 50mg for small ascites. Patient is dence of edema on exam. Labs include AST 100 U/L,
alert but not oriented to place and time. He has evi- ALT 50 U/L, total bilirubin of 2.5 mg/dL, albumin of
dence of asterixis. His mucous membranes are dry 3.2 mg/dL, and a platelet count of 122,000/µL
and he has no evidence of ascites on exam. His labs
include WBC 4.7K/mm3, AST 45 U/L, ALT 40 U/L, In addition to working up reversible causes, what
total bilirubin of 2.5 mg/dL, albumin of 3.7 g/dL, is the next best step in management?
sodium 142mEq/L, and a creatinine of 0.5 mg/dL.
A. Addition of rifaximin
What is the LEAST likely etiology of his B. Protein restriction
encephalopathy? C. Salt restriction
Chapter 7 — Cirrhosis and liver transplantation 189
Question 17 Question 18
A 62-year-old woman with cirrhosis second- A 56-year-old man with recently diagnosed
ary to primary biliary cholangitis is admitted to hepatitis B is in your office for further evalua-
the hospital with first variceal bleed. Vital signs tion for new diagnosis of liver cancer. He has
include heart rate 88 beats per minute, blood no other past medical history and is a former
pressure 90/52 mmHg, and oxygen saturation is smoker. He recently started tenofovir. His exam
98 percent on room air. She is alert and orient- is notable for palpable hepatomegaly without
ed without evidence of asterixis. Her abdomen is evidence of jaundice, spider angiomata, or pal-
mildly distended with ascites. Labs include AST mar erythema. His current blood work includes
100 U/L, ALT 150 U/L, total bilirubin of 1.4 mg/ AST 120 U/L, ALT 150 U/L, sodium of 137
dL, albumin of 3.5 g/dL, creatinine of 0.6 mg/ mEq/L, total bilirubin of 1.8 mg/dL, albumin of
dL, INR of 1.1 and a platelet count of 100,000/ 3.7 g/dL, creatinine of one mg/dL, INR of one
µL. She underwent successful band ligation and a platelet count of 150,000/µL. You review
and her diagnostic paracentesis was negative his most recent MRI, which reveals a three
for spontaneous bacterial peritonitis. She had a centimeter segment 3 lesion and a two centime-
normal ultrasound without masses or thrombo- ter segment 7 lesion. Both lesions demonstrate
ses and is started on diuretics and a nonselective arterial enhancement, venous washout, and a
beta-blocker with good response. She is ready pseudocapsule. There is no evidence of vascu-
for discharge. lar invasion or extrahepatic disease.
What is the next step in management? What is the next best step in management?
RATIONALE RATIONALE
Management of a patient with cirrhosis should Management of hepatocellular carcinoma (HCC)
include referral to a transplant center at their includes liver transplant evaluation regardless
first decompensation regardless of MELD score of decompensation. Liver transplantation is an
given median survival in decompensated liver effective option for patients with up to three
disease is as low as 1.5 years. tumors within Milan criteria: three lesions each
measuring less than three centimeters, or a
REFERENCE single lesion less than five centimeters without
Martin P, DiMartini A, Feng S, Brown R Jr, Fal- evidence of vascular invasion or extrahepatic
lon M. Evaluation for liver transplantation in disease. Surgical resection should be considered
adults: 2013 practice guideline by the American in patients with a single mass (usually less than
Association for the Study of Liver Diseases and five centimeters) in the setting of good liver
the American Society of Transplantation. Hepa- function and absence of clinically significant
tology. 2014; 59(3):1144-65. portal hypertension. Ablation with or without
transarterial chemoembolization (TACE) has
been used as a bridge to liver transplantation for
Chapter 7 — Cirrhosis and liver transplantation 191
controlling tumor burden. TACE is recommend- such as hereditary hypercholesterolemia and hy-
ed for nonsurgical candidates who have reason- peroxaluria. There are a few special conditions
able liver function and multinodular tumors that whereby patients can receive MELD exception
are beyond Milan criteria. Sorafenib is a multi- points, including HCC, hepatopulmonary syn-
kinase inhibitor with antiproliferative properties drome, portopulmonary hypertension, familial
that is reserved for those patients with advanced amyloid polyneuropathy, polycystic liver disease,
HCC who are not candidates for locoregional and primary hyperoxaluria. There are no MELD
treatment and have good liver function. exception points granted for refractory ascites.
REFERENCES REFERENCE
Mazzaferro V, Regalia E, Doci R et al. Liver Freeman RB Jr, Gish RG, Harper A, Davis GL,
transplantation for the treatment of small hepa- Vierling J, Lieblein L, et al. Model for end-stage
tocellular carcinomas in patients with cirrhosis. liver disease (MELD) exception guidelines:
N Engl J Med 1996;334(11):693-699. results and recommendations from the MELD
Llovet JM, Ricci S, Mazzaferro V et al. Exception Study Group and Conference (MES-
Sorafenib in advanced hepatocellular carcinoma. SAGE) for the approval of patients who need
N Engl J Med 2008;359(4):378-390. liver transplantation with diseases not con-
sidered by the standard MELD formula. Liver
Transpl 2006;12(12 Suppl 3):S128-S136.
Question 19
A 25-year-old man who has known primary
hyperoxaluria is seeing you for transplant evalu- Question 20
ation. He is now dialysis dependent. His current A 64-year-old man with alcoholic cirrhosis is ad-
blood work includes AST 20 U/L, ALT 25 U/L, mitted for dyspnea on exertion. He takes furose-
sodium of 137 mEq/L, total bilirubin of one mg/ mide and spironolactone for ascites. Vital signs
dL, albumin of 3.7 g/dL, creatinine of 2.6 mg/ include heart rate 90 beats per minute, blood
dL, INR of one and a platelet count of 200,000/ pressure 94/62 mmHg, and oxygen saturation is
µL. He recognizes that his Model for End Stage 88 percent on room air. He is alert and oriented
Liver Disease (MELD) score is low and he is without evidence of asterixis. His abdomen is
worried he will not get transplanted. He asks nondistended. He has evidence of clubbing in
about his access to organ transplantation. his extremities. Labs include AST 100 U/L, ALT
50 U/L, total bilirubin of 1.6 mg/dL, albumin of
Which of the following conditions do NOT 3.2 g/dL, creatinine of 0.5 mg/dL, INR of 1.1 and
qualify for MELD exception points? a platelet count of 90,000/µL. His echocardio-
gram is notable for late bubbles and he had an
A. Familial amyloid polyneuropathy arterial blood gas of 7.38/36/58/24.
B. Primary hyperoxaluria
C. Polycystic liver disease What aspect of the patient’s hepatopulmonary
D. Hepatopulmonary syndrome syndrome will qualify him for MELD exception
E. Refractory ascites points?
Question 22 REFERENCE
A 42-year-old woman who is two months post- Lucey MR, et al. Long-term management of the
transplant for cirrhosis secondary to autoimmune successful adult liver transplant: 2012 practice
hepatitis is seeing you in the office. She has no com- guideline by the American Association for the
plaints and is feeling well. She is taking tacrolimus Study of Liver Diseases and the American Society
two mg twice daily, Mycophenolate Mofetil 1000mg of Transplantation. Liver Transplantation. 2013
twice daily, and is on a prednisone taper. Her exam Jan;19(1):3-26.
is normal aside from a healing surgical incision.
Her labs include AST 250 U/L, ALT 300 U/L, total Question 23
bilirubin of 1.6 mg/dL, alkaline phosphatase 300 A 56-year-old man who was transplanted for
U/L, GGT 250 U/L. Tacrolimus level is 4.6 ng/mL. hepatitis C cirrhosis three months ago is noted to
Abdominal ultrasound with doppler shows patent have abnormal liver tests. He was viremic at the
hepatic portal vein and hepatic artery. There is no time of transplantation. He was recently treated
evidence of fluid collection or biliary dilatation. for rejection with pulse steroids in the setting of
Which of the following is the next best step in lowered immunosuppression. His exam is normal
management? aside from a healing surgical incision. His labs
include AST 250 U/L, ALT 300 U/L, total biliru-
A. Takeback to the operating room for surgical bin of 1.6 mg/dL, alkaline phosphatase 300 U/L,
repair of bile leak GGT 250 U/L, and tacrolimus level eight ng/mL.
B. Liver biopsy In addition to obtaining imaging, what is the next
C. Hepatic venogram best step in management?
D. Endoscopic cholangiopancreatography
E. Relist for liver transplantation A. Liver biopsy
B. Bolus steroids
CORRECT ANSWER: B C. Send for endoscopic retrograde
cholangiography
RATIONALE D. Start anticoagulation
Acute cellular rejection is diagnosed on liver E. Lower immunosuppression
biopsy alone. The most common early post-
transplant complications are acute cellular CORRECT ANSWER: A
rejection, hepatic artery thrombosis and biliary
stricture. Her imaging findings argue against a RATIONALE
hepatic vein stenosis, hepatic artery thrombosis, Recurrent hepatitis C is universal for individuals
bile leak, or biliary stricture. Given the timing of transplanted with a detectable viral load prior to
her transplant and young age her goal tacrolimus transplantation. Clinical course is variable but the
level should be closer to 10 ng/mL. Acute cellular progression of liver damage and fibrosis appears
rejection occurs in 20–30 percent of transplant to be accelerated. Treatment post-transplant can
recipients. There are three major histologic fea- be curative in as high as 96-98 percent of pa-
tures associated with acute cellular rejection: a tients on immunosuppression. Prior to bolusing
mixed inflammatory infiltrate in the portal triad steroids, it is important to differentiate between
(predominantly lymphocytic), destructive or non- recurrent hepatitis C and acute cellular rejection,
destructive nonsuppurative cholangitis involving which can only be assessed on liver biopsy.
interlobular bile duct epithelium (ductulitis), and
endotheliitis. First-line treatment is with high- REFERENCES
dose corticosteroids, to which greater than 90 Berenguer M. Natural history of recurrent hepati-
percent of patients respond. tis C. Liver Transpl. 2002 Oct;8(10 Suppl 1):S14-8.
194 Digestive Diseases Self-Education Program®
RATIONALE Question 27
Ascites denotes the presence of portal hyperten- A 56-year-old woman has autoimmune hepatitis-
sion and decompensated cirrhosis. Patients at related cirrhosis, history of hepatic encephalopa-
this stage are at risk for gastrointestinal bleed- thy and ascites. She is brought to the emergency
ing. In this case scenario, a bleeding source from department by her daughter for being “sluggish”
anywhere in the GI tract is plausible. An upper for the past few days. Her daughter increased
endoscopy was performed but did not reveal an lactulose frequency but the patient did not im-
upper GI source. A diagnosis should be firmly prove despite having had 10 bowel movements
established in order to apply the proper treatment a day for the past two days. Her medications
plan. Thus option D is the best next option. Lower include spironolactone, furosemide and lactulose.
endoscopy may reveal diverticular bleed, rectal On exam her vital signs are stable and she is alert
variceal bleed, colon polyps, colon cancer or hem- and oriented to place and person with no asterixis.
orrhoidal bleed. Each of these possibilities dictate
a different plan of action. Thus it is important to What is the next best step in the management of
determine the bleeding source. this patient?
Hemorrhoidal bleed is not an indication for liver
transplantation and is not indicative of portal A. Admit her for more testing to rule out
hypertension. Treatment of bleeding hemorrhoids electrolyte abnormalities and infection
may require surgical ligation and local conserva- B. Add rifaximin and zinc
tive therapy. The bleeding stopped and the INR is C. Obtain an MRI of the brain
1.8, therefore transfusion of FFP is not necessary. D. Obtain blood ammonia level
Even though the bleeding stopped, discharging a E. Refer her for psychometric testing
196 Digestive Diseases Self-Education Program®
Question 36 Question 37
A 45-year-old man with chronic alcohol and HCV A 45-year-old man with chronic alcohol and HCV
well compensated cirrhosis presents with a liver well compensated cirrhosis presents with a liver
ultrasound report in hand that notes a 1.6 cm liver ultrasound report in hand that notes a 1.6 cm liver
lesion in the left lobe of the liver. lesion in the left lobe of the liver. The patient now
returns to discuss the result of a triphasic liver CT
Which of the following is the next appropriate step? that you ordered. Now the lesion measures 2.7 cm
and enhances on arterial phase and washes out
A. Repeat a liver ultrasound in 6 months to on delayed images and a rim of capsule is visible
assess for growth around the tumor. Platelet count is noted to be
B. Refer for liver transplantation 85K/mm3.
C. Obtain triphasic CT liver
D. Obtain a liver biopsy What is the next best step management?
E. Refer for surgical resection
A. Measure serum AFP
CORRECT ANSWER: C B. Refer for tumor biopsy
C. Refer for liver transplantation
RATIONALE D. Refer for surgical resection
The diagnosis of hepatocellular carcinoma HCC E. Refer for ablative therapy and liver
should be suspected in patients with cirrhosis who transplant evaluation
present with a new liver lesion on imaging study.
However other benign lesions may occur and should CORRECT ANSWER: E
be ruled out. The diagnosis of HCC is typically made
by dynamic radiologic imaging, CT scan or MRI RATIONALE
with contrast. The AASLD suggests several options The patient in question has a growing hepatocellu-
in patients with cirrhosis and an indeterminate lar carcinoma and is now within the Milan criteria
nodule. These include follow-up imaging, imaging T2. The radiographic characteristics meet LIRADs
with an alternative modality or alternative contrast five criteria. The recommendation is bridging
agent, or biopsy, but cannot recommend one option therapy to decrease the progression of the tumor
over the other. Biopsy may be required in selected and subsequent drop out of the transplant wait-
cases, but its routine use is not suggested. Biopsy ing list. There is no recommendation favoring one
Chapter 7 — Cirrhosis and liver transplantation 201
Question 44 Question 45
A patient with well compensated cirrhosis is found A 65-year-old woman with primary sclerosing chol-
to have large esophageal varices on first screen- angitis and ulcerative colitis with cirrhosis presents
ing endoscopy. You decide to start a non-selective with a new onset ascites and abdominal pain. You
beta blocker (NSBB). The patient is concerned that perform a diagnostic and therapeutic paracentesis.
he may not tolerate a NSBB because his friend had There is no evidence of SBP. Serum sodium and
problems with these medications, and he is asking renal function are noted to be normal.
you to discuss the necessity of this drug. Which of the
following is correct concerning NSBB What is the next best step in the management of
in cirrhosis? this patient?
A. Both nadolol and propranolol are safe and A. Start diuretics and pursue vascular imaging of
without any side effects the liver
B. NSBB prevents variceal bleeding via β1 and β2 B. Refer for liver transplantation
effects C. Restrict water intake
C. NSBB reduces the risk of variceal bleeding by D. Administer NSAIDs for pain control
80 percent E. Start diuretics alone
Chapter 7 — Cirrhosis and liver transplantation 205
REFERENCE REFERENCE
Runyon BA, AASLD. Introduction to the revised Klebanoff MJ, Corey KE, Chhatwal J, Kaplan LM,
American Association for the Study of Liver Chung RT, Hur C. Bariatric surgery for nonalco-
Diseases Practice Guideline management of adult holic steatohepatitis: A clinical and cost-effective-
patients with ascites due to cirrhosis: update 2012. ness analysis. Hepatology. 2017 Apr;65(4):1156-
Hepatology. 2013;57: 1651. 1164.
Question 46 Question 47
A 48-year-old man has a BMI of 52 kg/m2 and A 35-year-old woman presents to clinic for advice
features of the metabolic syndrome. He takes regarding pregnancy. She has end-stage liver
blood pressure medication, insulin and a statin. disease secondary to autoimmune hepatitis. She
He is mostly sedentary. He has knee pain. He has is listed for liver transplantation. She desires to
1+ edema in the lower extremities. Platelets are have a baby before it is “too late” and wants to dis-
150,000/µL, hemoglobin A1c 6.0 percent, ALT 87 cuss risks of pregnancy after liver transplantation.
U/L and bilirubin 0.9 mg/dl. He wears a CPAP at Which of the following regarding pregnancy after
night. HVPG is eight mm Hg, and the liver pathol- transplantation is true?
ogy confirms NASH and stage 3 fibrosis. He has
been unable to lose weight despite the urging of A. She will be at increased risk for death
his PCP and work with a nutritionist. B. She is unlikely to get pregnant after transplant
due to high infertility rates
Which of the following is the next best step in C. She will be at increased risk for hypertensive
management? disorder
206 Digestive Diseases Self-Education Program®
Which of the following assessments would confirm B. Portpulmonary hypertension PoPHT with
this diagnosis? mean pulmonary arterial pressure 32 mm Hg
C. HIV
A. Left heart cardiac catheterization D. Hepatocellular carcinoma with macroscopic
B. Repeat cardiac echo invasion in the right hepatic vein
C. Cardiac echo with bubble study showing
left sided bubbles after the fourth beat CORRECT ANSWER: D
D. Cardiac echo with bubble study showing left
sided bubbles at the second beat RATIONALE
E. Arterial blood gas showing an alveolar arterial Of the above options, advanced hepatocellular
oxygen gradient less than15 mmHg carcinoma with invasion of the hepatic vein is a
contraindication to transplant. Room air PO2 less
CORRECT ANSWER: C than 60 mmHg with normal CXR and spirom-
etry in HPS is indication for liver transplant and
RATIONALE may qualify for MELD exception points. A mean
The hepatopulmonary syndrome (HPS) is a pul- PoPHT greater than 35 mmHg is considered a
monary complication of cirrhosis and/or portal contraindication for transplant unless it can be
hypertension whereby patients develop hypoxemia decreased with medications.
as a result of alterations in pulmonary microvas- AIDS, not HIV infection, is a contraindication
cular tone and architecture. HPS occurs in up to for liver transplantation.
30 percent of patients with cirrhosis. Patients
with HPS have a higher mortality than do patients REFERENCES
with cirrhosis without the disorder. The diagnosis Porres-Aguilar M, Zuckerman MJ, Figueroa-Casas
is made by excluding major intrinsic cardiopul- JB, Krowka MJ. Portopulmonary hypertension:
monary disease, and finding a PA02 less than 80 state of the art. Ann Hepatol. 2008;7:321–330.
mmHg, or an alveolar arterial oxygen gradient Koch DG, Fallon MB. Hepatopulmonary syn-
greater than 15 mmHg on arterial blood gases, drome. Clin Liver Dis. 2014 May;18(2):407-20.
along with intrapulmonary vasodilatation shown
by contrast echocardiography and/or perfusion
lung scanning. Bubbles showing at the second
beat reflects the presence of intracardiac shunt.
Bubbles crossing to the left ventricle by the fourth
or fifth beat are typical of intrapulmonary shunt-
ing as is present in HPS.
REFERENCE
Koch DG, Fallon MB. Hepatopulmonary syn-
drome. Clin Liver Dis. 2014 May;18(2):407-20.
Question 50
Which of the following is a contraindication to
liver transplantation?
CHAPTER 8
Gastrointestinal and
liver disease in pregnancy
Sushila R Dalal, MD, Christina Ha, MD FACG AGAF, Moira Hilscher, MD,
John Eaton, MD and Christina Lindenmeyer, MD
209
210 Digestive Diseases Self-Education Program®
Question 3 Question 4
A 30-year-old female with ulcerative colitis with A 26-year-old female with ulcerative colitis is 29
active disease just started a new treatment regi- weeks pregnant and recently has had more frequent
men with infliximab and would like to become bowel movements and abdominal cramping. She is
pregnant. She is concerned that her ulcerative not sure if this is due to a flare of her UC or not.
colitis may make this impossible.
Which testing should be chosen to assess her
Which of the following statements is true? disease activity:
Question 5 REFERENCE
A 31-year-old female with ileal Crohn’s in stable Mahadevan U, McConnnell R, Chambers C. Drug
remission on adalimumab and methotrexate Safety and Risk of Adverse Outcomes for Pregnant
comes for preconception counseling. She recently Patients with Inflammatory Bowel Disease. Gas-
had MR enterography that confirmed stable troenterology. 2017; 152: 451-462.
remission and is feeling well. She would like to
know if any changes should be made to her medi-
cation regimen. Question 6
A 32-year-old G1P0 woman who is 22 weeks
Which of the following statements is correct? pregnant presents to the ED by ambulance after
an episode of hematemesis at home. While in
A. She should make no changes to her current the ED, she has another episode of hematemesis,
medication regimen since she is well and her hemoglobin has decreased by 2gm/dL
B. She should stop methotrexate and continue from the last recorded value 1 week prior. She is
adalimumab to maintain remission tachycardic and hypotensive. After resuscitation
C. She should stop methotrexate and switch and stabilization, the next intervention is:
adalimumab to certolizumab pegol
D. She should stop all medications immediately to A. Medical management alone.
minimize risk B. Upper endoscopy, but sedation cannot be
E. She should stop methotrexate now and stop used.
adalimumab after the first trimester. C. Sedated upper endoscopy with patient supine.
D. Sedated upper endoscopy with patient in left
CORRECT ANSWER: B lateral decubitus position.
E. Sedated upper endoscopy with monopolar
RATIONALE cautery if needed.
Maintenance of remission is important to prevent
212 Digestive Diseases Self-Education Program®
Question 11 REFERENCES
A 26-year-old woman presents to the office with Gerson LB. Proton pump inhibitors and
a chief complaint of progressive heartburn and safety during pregnancy. Gastroenterology.
indigestion. She is currently 14 weeks pregnant, 2011;141:389-91.
and denies any dysphagia or odynophagia, denies Mahadevan and Kane. American Gastroen-
weight loss, and has been gaining weight ap- terological Association Institute Medical Position
propriately during her pregnancy. Her labs are Statement on the Use of Gastrointestinal Medica-
within normal limits, no anemia, normal meta- tions in Pregnancy. 2006;131:278-82.
bolic profile. She has tried lifestyle modifications,
including dietary modifications, staying upright
after eating and elevation the head of her bed with Question 12
persistent symptoms. A 29-year-old G1P0 woman presents to your office
to discuss her worsening constipation. She reports
Which of the following is the next appropriate step a lifelong history of constipation, at baseline has a
in her care? bowel movement every two to three days with the
use of stool softeners and psyllium. She is cur-
A. Calcium carbonate rently eight weeks pregnant and reports worsened
B. Upper GI series constipation described as Bristol scale one or two
C. Upper Endoscopy stools every four to five days with associated bloat-
D. Rantidine ing and abdominal discomfort despite daily fiber
E. Omeprazole and stool softener use. She denies rectal bleeding,
Chapter 8 — Gastrointestinal and liver disease in pregnancy 215
fevers, abdominal pain, vomiting or weight loss. formed, non-bloody bowel movements daily with-
She has no history of diabetes, thyroid disorders out urgency, pain or nocturnal bowel movements.
and is not taking any opiates. Which of the follow- She denies extra-intestinal manifestations of IBD.
ing agents is the most appropriate next step in her Sigmoidoscopy six months ago demonstrated
treatment? mucosal healing. She and her husband would like
to start trying to become pregnant now. Which
A. Magnesium citrate of the following statements about her ulcerative
B. Sodium phosphate colitis are correct?
C. Bisacodyl
D. Polyethylene glycol A. She will likely have difficulty conceiving due to
E. Sennosides decreased fecundity
B. She should switch her medications to another
CORRECT ANSWER: D agent prior to conception
C. She has a greater risk of having a miscarriage
RATIONALE or spontaneous abortion
Constipation is prevalent during pregnancy, af- D. She should not breastfeed while on 5-amino-
fecting 35-40 percent of women during the first salicylates
and second trimesters. Factors associated with E. She is at greater risk of requiring a Caesarian
worsened constipation during pregnancy include section
increased progesterone which can decrease intes-
tinal motility, increased iron intake due to anemia, CORRECT ANSWER: B
and the growing uterus may impact colonic tran-
sit. Osmotic laxatives are considered safe to use RATIONALE
throughout pregnancy, and polyethylene glycol is Women with IBD should have pre-conception
the first line treatment of choice. Saline hyperos- counseling with an updated disease activity as-
molar laxatives, magnesium citrate and stimulant sessment as sustained remission, defined as inac-
laxatives are not recommended for long-term or tive disease for at least three or more months, is
continued use during pregnancy. recommended prior to pregnancy as disease activ-
ity can be associated with persistent or worsened
REFERENCES symptoms throughout pregnancy and breastfeed-
Mahadevan and Kane. American Gastroenterolog- ing. While there are higher rates of adverse preg-
ical Association Institute Medical Position State- nancy outcomes among women with IBD com-
ment on the Use of Gastrointestinal Medications pared to the general population, active ulcerative
in Pregnancy. 2006;131:278-82. colitis is associated with a four-fold increased risk
Gomes et al. Gastrointestinal disease during of spontaneous abortion or miscarriage compared
pregnancy:what does the gastroenterologist need to patients in remission. The decision regarding
to know? Ann Gastroenterol. 2018;31(4):385-94. mode of delivery should be based on factors at the
time of labor, maternal factors and not the diag-
nosis of Ulcerative colitis. Caesarian section is
Question 13 primarily recommended for persons who have had
A 34-year-old woman presents to your office to a colectomy for their UC with an ileal pouch anal
discuss family planning. She has a one-year his- anastomosis to decrease risk of sphincter injury
tory of extensive ulcerative colitis, initially requir- and for women with perianal Crohn’s disease.
ing a course of corticosteroids, now maintained Breastfeeding is considered safe while on 5-ASAs,
on mesalamine with dibutyl phthalate 2.4 grams steroids, thiopurines or biologics. Because of the
twice daily. She is feeling very well, one to two dibutyl phthalate (DBP) present on the coating of
216 Digestive Diseases Self-Education Program®
taneous preterm labor, and fetal distress. The A. Avoidance of invasive fetal scalp monitoring
risk of adverse fetal outcomes correlates with B. Start antiviral therapy in the third trimester
maternal serum bile acid levels, with increased for prevention of vertical transmission given
risk described with serum bile acid levels above HCV RNA more than 1 million IU/L
40 µmol/L. In women presenting with serum C. Screen for HIV
bile acid levels over 40 µmol/L, delivery at 36- D. Mode of delivery should occur based on
37 weeks gestation has been proposed to reduce routine obstetrical indications
perinatal mortality risk compared with expect- E. HCV RNA level should be re-evaluated after
ant management. delivery to assess for spontaneous clearance
Ursodeoxycholic acid (UDCA) is commonly
used in the management of ICP. UDCA im- CORRECT ANSWER: B
proves pruritis and liver test abnormalities in
ICP and may improve fetal outcomes. Chole- RATIONALE
styramine can be used in refractory cases of The safety of direct-acting antivirals in preg-
pruritus. Obeticholic acid has not been studied nancy and lactation is unknown. As a result,
in ICP. HCV treatment is not recommended in pregnant
women or in women who are breastfeeding, ir-
REFERENCES respective of viral load. Spontaneous clearance
Puljic A, Kim E, Page J, et al. The risk of infant of HCV can occur in the postpartum period.
and fetal death by each additional week of ex-
pectant management in intrahepatic cholestasis REFERENCE
of pregnancy by gestationaly age. Am J Obstet AASLD-IDSA. Recommendations for testing,
Gynecol. 2015 May;212(5).667.e1-5. managing, and treating hepatitis C. http://www.
Bacq Y, Sentilhes L, Reyes HB, et al. Efficacy hcvguidelines.org
of ursodeoxycholic acid in treating intrahepatic
cholestasis of pregnancy: a meta-analysis. Gas-
troenterology. 2012 Dec;143(6):1492-501. Question 18
Dixon PH, Williamson C. The pathophysiology A 26-year-old female with a past medical of
of intrahepatic cholestasis of pregnancy. Clin Res chronic hepatitis C virus infection, naive to
Hepatol Gastroenterol. 2016 Apr;40(2):141-53. therapy, presents to establish prenatal care at 25
weeks gestation. She complains of severe pruri-
tus, in addition to epigastric pain and fatigue.
Question 17
A 32-year-old G1P0 woman presents for her What is the next most appropriate step in her
first routine obstetrical visit. Her past medical evaluation?
history is notable for a history of intravenous
drug use as a teenager and a history of hypothy- A. Measure HCV viral load and HCV genotype.
roidism, for which she takes levothyroxine. As B. Measure serum AST, ALT and serum bile
part of routine prenatal screening, lab work is acids.
significant for positive anti-HCV IgG, HCV RNA C. Initiate empiric therapy for HCV with
1,655,000 IU/L, HCV genotype 1a, negative HBV glecaprevir/pibrentasvir.
surface antigen, positive anti-HBV Core IgG and D. Screen for fatty acid oxidation defects.
positive anti-HBV Surface IgG. E. Measure serum AST, ALT, bilirubin and
prothrombin time.
Which of the following is not recommended as
part of her routine pre-and perinatal care? CORRECT ANSWER: B
Chapter 8 — Gastrointestinal and liver disease in pregnancy 219
A. HBV DNA more than 20,000 IU/L Liver biopsy pathology, if obtained in this case
B. AST more than 30 IU/L would be most likely to demonstrate:
C. Absence of current antiviral therapy
D. HBV e antigen positivity A. Fibrin deposition, periportal hemorrhage,
E. Anti-HBV surface antigen IgG negativity hepatocyte necrosis/infarction and microve-
sicular fat infiltration.
CORRECT ANSWER: D B. Microvesicular fat accumulation in the peri-
central zone with perioportal sparing.
RATIONALE C. Centrilobular cholestasis, little to no inflamma-
Maternal risk factors that increase the likelihood tion and bile plugs in hepatocytes.
of vertical transmission are a high level of hepa- D. Hepatocyte necrosis, bile plugs, cholestasis and
titis B viremia (HBV DNA more than 200,000 steatosis.
IU/L, transmission rate of 70-90 percent) and the E. Lymphoplasmacytic portal inflammatory infil-
presence of hepatitis B e antigen positivity (trans- trate, interface hepatitis, bridging necrosis.
mission rate of 10-40 percent). The American As-
sociation for the Study of Liver Disease (AASLD) CORRECT ANSWER: A
guidance on hepatitis B recommends institution of
antiviral therapy with lamivudine, telbivudine or RATIONALE
tenofovir to mothers with HBV DNA of more than Preeclampsia is characterized by the presence
200,000 IU/L. Tenofovir is the preferred antiviral of de-novo hypertension and proteinuria occur-
agent due to its potency and due to increased rates ring after 20 weeks gestation. Organ dysfunction,
of drug resistance seen with use of lamivudine and including liver involvement, renal dysfunction,
telbivudine. and hematologic abnormalities can be seen.
Patients may present with epigastric and/or right
REFERENCE upper quadrant abdominal pain, nausea and/or
Terrault NA, Lok ASF, McMahon BJ, Chang vomiting. Biochemical changes include the pres-
KM, Hwang JP, Jonas MM, Brown RS, Bzowej ence of liver enzyme abnormalities, including
NH, Wong JB. Update on prevention, diagnosis, 10- to 20-fold elevations in aminotransferases,
and treatment of chronic hepatitis B: AASLD elevations in alkaline phosphatase that are above
2018 hepatitis B guidance. Hepatology 2018 what is typically observed in pregnancy, and
Apr;67(4):1560-1599. mild elevations in bilirubin up to 5 mg/dL. Liver
histology in preeclampsia includes the presence
of fibrin deposition, periportal hemorrhage, he-
Question 21 patocyte necrosis/infarction, and microvesicular
A 39-year-old G1P0 female with a past medical fat infiltration. Severe hepatic complications of
history of systemic lupus erythematous presents preeclampsia/eclampsia include development of
for obstetrical follow-up at 30 weeks gestation. a hematoma below Glisson’s capsule and hepatic
She complains of epigastric pain, nausea and fre- rupture.
quent vomiting. Vital signs are as follows: Blood
pressure 150/95 mmHg, heart rate 72 bpm, tem- REFERENCE
perature 98.2°F, oxygen saturation 94 percent on American College of Obstetricians and Gynecolo-
room air. Laboratory studies are notable for: AST gists, Task Force on Hypertension in Pregnancy.
525 U/L, ALT 450 U/L, total bilirubin 4 mg/dL, Hypertension in pregnancy. Report of the Ameri-
hemoglobin 14 g/dL, platelets 140 k/µL, serum can College of Obstetricians and Gynecologists’
creatinine 1.45 mg/dL. Task Force on Hypertension in Pregnancy. Obstet
Gynecol 2013 Nov;122(5):1122-1131.
Chapter 8 — Gastrointestinal and liver disease in pregnancy 221
Question 22 Question 23
A 26-year-old G1P0 female with a past medical A 34-year-old female with a past medical history
history of anorexia nervosa at 33 weeks gestation of autoimmune hepatitis-related cirrhosis pres-
presents to the emergency room for evaluation of ents to establish care at eight weeks gestation. She
right upper quadrant abdominal pain, pruritus, has maintained clinical and histologic remission
nausea, vomiting, anorexia and fatigue. She is di- on azathioprine 50mg daily and prednisone 5mg
agnosed with acute fatty liver of pregnancy based daily. Which of the following would you recom-
on the Swansea criteria. Which of the following mend as part of her ongoing liver related care?
should be recommended?
A. Screen for esophageal varices before the end of
A. Supportive care followed by delivery at 34 the first trimester
weeks B. Continue current immune suppressive regimen
B. Post-partum non-invasive assessment of he- C. Stop azathioprine and increase prednisone
patic fibrosis dose
C. Infant should be monitored closely for hepatic D. Resumption of quarterly lab monitoring after
failure, cardiomyopathy, nonketotic hypogly- delivery
cemia, myopathy and neuropathy E. Recommend early delivery to minimize liver
D. Intravenous magnesium sulfate, intravenous related risks
dexamethasone and delivery after 34 weeks
E. Percutaneous liver biopsy CORRECT ANSWER: B
REFERENCES REFERENCE
Fesenmeier MF, Coppage KH, Lambers DS, Bar- Schramm C, Herkel J, Beuers U, Kanzler S, Galle
ton JR, Sibai BM. Acute fatty liver of pregnancy in PR, Lohse AW. Pregnancy in autoimmune hepati-
3 tertiary care centers. Am J Obstet Gynecol 2005 tis: outcome and risk factors. Am J Gastroenterol
May;192(5):1416-1419. 2006;101:556-560.
Goel A, Ramakrishan B, Zachariah U, Ram-
achandran J, Eapen CE, Kurian G, Chandry S.
How accurate are the Swansea criteria to diagnose Question 24
acute fatty liver of pregnancy in predicting hepatic A 28-year-old G2P1 female at 21 weeks gestation
microvesicular steatosis? Gut Jan;60(1):138-139. presents with new-onset right upper quadrant
222 Digestive Diseases Self-Education Program®
pain, nausea, vomiting, jaundice and pruritus. and resultant cirrhosis, which was diagnosed at
Laboratory studies are notable for: AST 600 U/L, the age of 12 as part of an evaluation for neurocog-
ALT 725 U/L, total bilirubin 6 mg/dL, alkaline nitive delay. Her cirrhosis has been complicated
phosphatase 320 U/L, platelets 150 k/µL, serum by esophageal varices that have bled in the past,
creatinine 1.30 mg/dL, prothrombin time 18 sec. for which she has undergone previous esophageal
Which of the following is the most likely etiology variceal band ligation; she is on daily propranolol
for the patient’s presentation? for secondary bleeding prophylaxis. She is cur-
rently maintained on daily penicillamine chelation
A. Hyperemesis gravidarum therapy. All of the following should be recom-
B. Intrahepatic cholestasis of pregnancy mended as part of her antepartum care:
C. Acute hepatitis A infection
D. Pre-eclampsia A. Stop penicillamine and switch to daily
E. Choledocholithiasis trientene with a 25-50 percent dose
reduction in the third trimester; stop
CORRECT ANSWER: C propranolol and survey esophageal varices
in the second trimester.
RATIONALE B. Continue penicillamine with a 25-50 percent
Acute viral hepatitis is the most common cause of dose reduction in the third trimester; stop
jaundice in pregnancy. Hyperemesis gravidarum propranolol and survey esophageal varices in
typically occurs in the first trimester of pregnancy; the second trimester.
mild elevations in aminotransferases are most C. Stop penicillamine and switch to daily zinc;
commonly seen, although AST and ALT levels continue propranolol and survey esophageal
have been observed to rise to as high as 700 to varices in the second trimester.
1000 U/L in some patients. Pre-eclampsia is typi- D. Continue penicillamine with a 25-50 percent
cally seen in the second trimester of pregnancy; dose reduction in the third trimester; continue
biochemical changes include the presence of liver propranolol and survey esophageal varices in
enzyme abnormalities, including 10- to 20-fold the second trimester.
elevations in aminotransferases, elevations in al- E. Stop penicillamine and switch to daily tri-
kaline phosphatase that are above what is typically entene with a 25-50 percent dose reduction
observed in pregnancy, and mild elevations in in the third trimester; continue propranolol
bilirubin up to 5 mg/dL. The hallmark symptom of and survey esophageal varices in the second
intrahepatic cholestasis of pregnancy is pruritus; trimester.
jaundice occurs in about 10 percent to 25 percent
of patients and usually does not occur until after CORRECT ANSWER: D
about two to four weeks after the onset of pruritus.
RATIONALE
REFERENCE Published guidelines on Wilson’s disease have
Licata A et al. Clinical course and management of recommended that penicillamine and trientine
acute and chronic viral hepatitis during pregnan- should be continued during pregnancy, with 25
cy. J Viral Hepat 2015;22:515-523. percent to 50 percent dose reductions occurring
during the third trimester in order to facilitate
wound healing in the event of cesarean section.
Question 25 Nadolol has a long half-life with low protein bind-
A 22-year-old nulliparous female at 8 weeks gesta- ing and low rate of excretion and thus is less fa-
tion presents to establish routine obstetrical care. vored compared to propranolol, which is thought
She has a past medical history of Wilson’s disease to be safe in pregnancy.
Chapter 8 — Gastrointestinal and liver disease in pregnancy 223
REFERENCES REFERENCE
Roberts EA, Schilsky ML; American Association Fitzpatrick KE, Hinshaw K, Kurinczuk JJ, Knight
for Study of Liver Diseases (AASLD). Diagno- M. Risk factors, management, and outcomes
sis and treatment of Wilson disease: an update. of hemolysis, elevated liver enzymes, and low
Hepatology 2008 Jun;47(6):2089-111. platelets syndrome and elevated liver enzymes,
Sandhu BS, Sanyal AJ. Pregnancy and liver low platelets syndrome. Obstet Gynecol 2014
disease. Gastroenterol Clin North Am 2003 Mar;123(3):618-627.
Mar;32(1):407-436.
Question 27
Question 26 A 34-year-old female presents for routine antena-
A 29-year-old woman at 37 weeks gestation tal care at 33 weeks gestation. She complains of
presents to the emergency room with right upper several weeks of diffuse pruritus, and has recently
quadrant pain, nausea, vomiting. She is diag- noted yellowing of her sclera. Laboratory results
nosed with preeclampsia. She is treated with in- are significant for: AST 650 U/L, ALT 525 U/L,
travenous magnesium, antihypertensive therapy, total bilirubin 3.9 mg/dL, GGT 45 U/L, hgb 12.2
and labor is induced. Prior to delivery, laboratory g/dL, platelets 190,000 k/µL, INR 1.3, serum bile
values were as follows: AST 240 U/L, ALT 220 acids 35 μmol/L. Which of the following is the best
U/L, total bilirubin 1.8 mg/dL, hemoglobin 10.1 next step in management?
g/dL, platelets 110,000 k/µL. Forty-eight hours
following delivery, she complains of worsening A. Intravenous dexamethasone and delivery at
right upper quadrant pain and headache. Repeat 34 weeks
laboratory values 48 hours post-partum were as B. Start ursodeoxycholic acid (UDCA) at a dose
follows: AST 410 U/L, ALT 390 U/L, total bili- of 10 mg/kg to 15 mg/kg
rubin 5.1 mg/dL, hemoglobin 7.9 g/dL, platelets C. Test for deficiency of 3-hydroxyacyl-CoA dehy-
75,000 k/µL. Which of the following is the most drogenase (LCHAD)
likely diagnosis? D. Right upper quadrant ultrasound
E. Percutaneous liver biopsy
A. Sepsis with hemolytic-uremic syndrome
(HUS) and thrombotic thrombocytopenic CORRECT ANSWER: B
purpura (TTP)
B. Acute fatty liver of pregnancy RATIONALE
C. Flare of autoimmune hepatitis This clinical scenario is consistent with intrahepatic
D. Hemolysis, elevated liver enzymes, low platelet cholestasis of pregnancy. The treatment of choice
count (HELLP) syndrome in ICP is ursodeoxycholic acid (UDCA) at a dose of
E. Acute hepatitis A infection 10 mg/kg to 15 mg/kg. The mechanism of UDCA in
the management of ICP is not fully understood but
CORRECT ANSWER: D is thought to involve increases in the expression of
bile salt export pumps and increases in placenta bile
RATIONALE transporters. UDCA has been shown to significantly
HELLP syndrome is a multisystemic disorder that reduce pruritus, normalize or decrease ALT levels
is characterized by the development of hemolytic and reduce bile acid levels. Early delivery of the
anemia, elevated liver enzymes, and low platelets. fetus at 37 weeks is advised since the risk of intra-
Most cases occur between 28 and 36 weeks of uterine death is thought to be more common during
gestation, but it can also develop up to one week the last month of pregnancy. LCHAD deficiency is
postpartum in 30 percent of cases. associated with acute fatty liver of pregnancy.
224 Digestive Diseases Self-Education Program®
prematurity, and intrauterine fetal death. Verti- vudine, telbivudine, or tenofovir to mothers with
cal transmission of HEV has also been reported. HBV DNA of more than 200,000 IU/L. Tenofovir
Treatment is supportive. Although ribavirin has is the preferred antiviral agent due to its potency
been used in cases of HEV infection in non-preg- and due to drug resistance seen with use of lami-
nant patients, its use is precluded in pregnancy vudine and telbivudine.
due to teratogenicity.
REFERENCE
REFERENCES Terrault NA, Lok ASF, McMahon BJ, Chang
Devarbhavi H, Kremers WK, Dierkhising R, KM, Hwang JP, Jonas MM, Brown RS, Bzowej
Padmanabhan L. Pregnancy-associated acute NH, Wong JB. Update on prevention, diagno-
liver disease and acute viral hepatitis: differen- sis, and treatment of chronic hepatitis B: AASLD
tiation, course and outcome. J Hepatol 2008 2018 hepatitis B guidance. Hepatology 2018
Dec;49(6):930-5. Apr;67(4):1560-1599.
Khuroo MS, Kamili S, Khuroo MS. Clinical
course and duration of viremia in vertically trans-
mitted hepatitis E virus (HEV) infection in babies Question 31
born to HEV-infected mothers. J Viral Hepat. All of the following are indicated as part of the
2009 Jul;16(7):519-523. postnatal management of infants born to moth-
ers with chronic hepatitis B virus (HBV) infection
EXCEPT:
Question 30
A 32-year-old female G1P0 at 32 weeks gestation A. Breastfeeding is contra-indicated if the
presents for routine antenatal follow-up. She has mother is on anti-viral therapy.
a history of vertically-acquired chronic hepatitis B. Administer hepatitis B immune globulin
B virus (HBV) infection and has never required (HBIG) and hepatitis B vaccination to infants
antiviral therapy. Her current laboratory studies within first 12 hours after birth.
are notable for the following: AST 25 U/L, ALT 20 C. Administer hepatitis B vaccination to infants
U/L, ALP 210 U/L, HBV surface antigen positive, at one to two months of age.
anti-HBV surface IgG negative, anti-HBV core IgG D. Administer hepatitis B vaccination to infants
positive, anti-HBV e antigen IgG negative, HBV e at six months of age.
antigen positive, HBV DNA 280,000 IU/L. E. Confirm response to hepatitis B vaccination
Which of the following is the best next step in with anti-HBV surface IgG between nine to 18
management? months of age.
REFERENCE
Terrault NA, Lok ASF, McMahon BJ, Chang
KM, Hwang JP, Jonas MM, Brown RS, Bzowej
NH, Wong JB. Update on prevention, diagno-
sis, and treatment of chronic hepatitis B: AASLD
2018 hepatitis B guidance. Hepatology 2018
Apr;67(4):1560-1599.
Answers & critiques
CHAPTER 9
227
228 Digestive Diseases Self-Education Program®
irritable bowel syndrome with constipation. Am J rologies in patients with suspected IBS-diarrhea or
Gastroenterol. 2012 Nov;107(11):1714-24. IBS-mixed, especially in potential high-risk groups
Chey WD, Kurlander J, Eswaran S. Irritable (for example, patients with Down’s syndrome, type
bowel syndrome: a clinical review. JAMA. 2015: 1 diabetes, Turner syndrome). The recommended
313(9): 949-958. initial screening test for celiac disease is anti-tissue
transglutaminase (TTG) immunoglobulin A (IgA),
as well as a total IgA to make sure the patient is not
Question 2 IgA deficient. Colonoscopy with random biop-
A 34-year-old woman with type 1 diabetes pres- sies is recommended if the clinical suspicion for
ents for evaluation of diarrhea, abdominal cramp- microscopic colitis is high based on history (typi-
ing improved with a bowel movement and bloat- cally a middle-aged female with watery diarrhea).
ing. She has no rectal bleeding, weight loss or Colonoscopy should also be performed if alarm
nocturnal stools. She has not noticed symptom features are present to evaluate for malignancy and
association with food. She takes no medications, inflammatory bowel disease (IBD), or in those over
does not smoke cigarettes and drinks four glasses the age of 50 for colorectal cancer screening. For
of wine on the weekends. Her brother was diag- patients who meet criteria for IBS without alarm
nosed with celiac disease in his 20s by serology features, the combination of a normal C-reactive
and duodenal biopsy. Her physical exam is unre- protein and fecal calprotectin can be used to rule-
markable. A complete blood count is normal. out IBD without doing a colonoscopy. If a patient
is consuming significant lactose or there is a clear
Which one of the following diagnostic approaches link between lactose ingestion and symptoms by
is most appropriate for this patient? history or review of a food diary, a trial of lactose
exclusion or a lactose hydrogen breath test can be
A. Anti-tissue transglutaminase immunoglobulin considered, though a lactose breath test is time
A and total IgA consuming. The likelihood of thyroid abnormali-
B. Colonoscopy with random biopsies ties is no greater in patients with IBS as compared
C. Trial of lactose exclusion and reintroduction to normal controls, and therefore checking routine
D. Thyroid stimulating hormone thyroid stimulating hormone (TSH) is not recom-
E. Stool culture mended. Similarly, the yield of checking stool
cultures in patients with suspected IBS is low and
CORRECT ANSWER: A is not routinely recommended.
RATIONALE REFERENCES
The diagnosis of irritable bowel syndrome (IBS) is Brandt L J, Chey W D, Foxx-Orenstein A E. et
based on symptom-based criteria and the appro- al. An evidence-based position statement on the
priate evaluation of other causes. The Rome IV management of irritable bowel syndrome. Am J
criteria for the diagnosis of IBS includes the asso- Gastroenterol. 2009;104(Suppl 1):S1–S35.
ciation of abdominal discomfort with altered bowel Chey WD, Kurlander J, Eswaran S. Irritable
habits (diarrhea, constipation, or mixed) and/ bowel syndrome: a clinical review. JAMA. 2015:
or defecation. Patients who fulfill criteria for IBS 313(9): 949-958.
and who have no alarm features (nocturnal stools,
rectal bleeding, weight loss, unexplained iron
deficiency anemia, or family history of colorectal Question 3
cancer or inflammatory bowel disease) require A 34-year-old man has a history of IBS-D. He has
little formal testing to exclude other causes for loose stools up to four times per day associated
symptoms. It is recommended to check celiac se- with abdominal cramping, which is relieved by
Chapter 9 — Diarrhea and and constipation 229
a bowel movement. He takes loperamide daily ritable bowel syndrome: a systematic review. Gut.
which improves his diarrhea, however, he contin- 2010;59(3):325.
ues to have significant abdominal cramping. He
has tried bentyl in the past with little benefit.
Question 4
What of the following treatments should be tried A 42-year-old man presents for initial evaluation
next for this patient? of diarrhea. He has associated abdominal cramp-
ing which is relieved by a bowel movement and
A. Selective norepinephrine-reuptake inhibitors bloating. His symptoms are worse after eating,
(SNRI) though he does not identify any specific food
B. Probiotics triggers. His symptoms are worse with job stress.
C. Selective serotonin-reuptake inhibitors (SSRI) Celiac serologies are negative, thyroid stimulating
D. Tricyclic antidepressant (TCA) hormone is normal. His hematocrit is 33 percent
E. Diphenoxylate/atropine (normal range: 38 to 50 percent) and his iron
studies are low.
CORRECT ANSWER: D
What is the next best step?
RATIONALE
Tricyclic antidepressants (TCAs) and perhaps A. Colonoscopy
SNRIs can be used to treat patients with IBS due to B. Fiber supplementation
their effects on pain perception and motility. Side C. Loperamide
effects of TCAs include dose-dependent constipa- D. CT scan of the abdomen and pelvis
tion, dry mouth and eyes, drowsiness, weight gain, E. Low FODMAP diet
and QT interval prolongation. Because TCAs can
cause constipation, they should be preferentially CORRECT ANSWER: A
(but not exclusively) used in patients with IBS-D.
Side effects of SSRIs, on the other hand, include RATIONALE
diarrhea, sexual dysfunction, agitation, nausea and IBS should be suspected in patients with ab-
drowsiness. There is no data to support SSRI use dominal pain and constipation and/or diarrhea.
in IBS. There are little data addressing the efficacy Specifically, the Rome IV criteria define IBS as
of SNRIs for the treatment of IBS. Probiotics have abdominal pain at least one day per week in the
been associated with an improvement in symptoms last three months, associated with two or more
in patients with IBS, however, the magnitude of of the following: related to defecation, associated
benefit and the most effective species and strain are with a change in stool frequency or associated
uncertain. Diphenoxylate/atropine (lomotil) can be with a change in stool form. Generally, in patients
used to improve diarrhea, however, it has no effect with IBS-D, a CBC, inflammatory markers, TTG
on improving abdominal pain. IgA and IgA should be obtained. Further testing
should be guided by the clinical presentation or
REFERENCES the presence of “red flag” symptoms (onset after
Dekel R, Drossman DA, Sperber AD. The age 50 years, rectal bleeding or melena, noctur-
use of psychotropic drugs in irritable bowel nal diarrhea, unexplained weight loss, laboratory
syndrome. Expert Opin Investig Drugs. abnormalities such as anemia or elevated inflam-
2013;22(3):329-339. matory markers, or a family history of IBD or
Moayyedi P, Ford AC, Talley NJ, Cremonini colorectal cancer). Due to his anemia, a colonos-
F, Foxx-Orenstein AE, Brandt LJ, Quigley EM. copy would be the best next step to evaluate for
The efficacy of probiotics in the treatment of ir- malignancy or IBD.
230 Digestive Diseases Self-Education Program®
Question 8
An 18-year-old man has a history of diarrhea,
abdominal pain and weight loss. A thyroid stimu-
lating hormone was 1.5 mIU/L (normal range: .4
to 4.0 mIU/L) and inflammatory markers were
What is the best next step in evaluation? normal. A hematocrit was 33 percent (normal
range: 40 to 54 percent). His physical exam was
A. Colonoscopy with random biopsies normal. His tissue transglutaminase IgA was 15
B. Review her medications (normal less than 19) and his total IgA was 20 mg/
C. Abdominal/pelvis CT scan dL (normal range: 80 to 350 mg/dL).
D. Start loperamide
E. Start probiotics What is the best next step in evaluation?
232 Digestive Diseases Self-Education Program®
A. HLA DQ2 and DQ8 showed villous blunting with increased intraepi-
B. Deaminated gliadin peptide IgG thelial lymphocytes.
C. Anti-endomysial IgA
D. Upper endoscopy with biopsy What is the preferred treatment for this patient?
E. Deamidated gliadin peptide IgA
A. Gluten free diet
CORRECT ANSWER: B B. Ceftriaxone IV followed by Bactrim PO
C. Low FODMAP diet
RATIONALE D. Tetracycline and folic acid
IgA deficiency is more common in celiac disease E. Rifaxmin
than in the general population. When testing for
celiac disease, total IgA levels should be mea- CORRECT ANSWER: D
sured and if low, IgA-based celiac serology testing
should not be used. Instead, IgG-based testing is RATIONALE
recommended, including DGPs IgG and/or TTG This patient has tropical sprue based on her travel
IgG. HLA DQ2 and DQ8 should be used to rule to an endemic country, negative celiac serologies,
out celiac disease in clinical situations where a pa- laboratories revealing a macrocytic anemia and
tient is already gluten-free, as patients with celiac low albumin and characteristic histology (villous
disease need to possess HLA DQ2 or DQ8. An up- blunting, increased intraepithelial lymphocytes).
per endoscopy should be obtained after a positive Treatment is with tetracycline and folate. Diag-
serology, not before. nosis of tropical sprue is ultimately confirmed by
a response to treatment. A gluten-free diet is not
REFERENCES appropriate, as the patient does not have celiac
Villalta D, Alessio MG, Tampoia M et al. Testing disease, confirmed by normal celiac serologies.
for IgG class antibodies in celiac disease patients Ceftriaxone IV followed by Bactrim PO is the cor-
with selective IgA deficiency. A comparison of the rect treatment for Whipple’s disease. A diet low
diagnostic accuracy of 9 IgG anti-tissue transglu- in fermentable oligo-, di-, and monosaccharides
taminase, 1 IgG anti-gliadin and 1 IgG anti-deam- and polyols (FODMAPs) is beneficial treatment
inated gliadin peptide antibody assays. Clin Chim in some patients with IBS with abdominal bloat-
Acta 2007;382:95–99. ing or pain. Rifaximin is the correct treatment for
Villalta D, Tonutti E, Prause C et al. IgG small intestine bacterial overgrowth or IBS-D.
antibodies against deamidated gliadin peptides
for diagnosis of celiac disease in patients with IgA REFERENCES
deficiency. Clin Chem 2010;56:464–468. Brown IS, Bettington A, Bettington M, Rosty C.
Tropical sprue: revisiting an underrecognized
disease. Am J Surg Pathol 2014; 38:666.
Question 9 Shah VH, Rotterdam H, Kotler DP, et al. All
An 18-year-old woman presents for evalua- that scallops is not celiac disease. Gastrointest
tion of chronic diarrhea, fatigue and abdominal Endosc 2000; 51:717.
cramping. She was recently in Puerto Rico for six
months visiting family and returned a few weeks
ago. Her labs are significant for a hemoglobin of 11 Question 10
with an MCV of 109. Her albumin is 3.6. She had A 46-year-old male with a history of chronic
stool studies which ruled out infection, including arthritis is referred for evaluation of diarrhea and
parasites. TtG IgA and total IgA were within nor- a 10-pound weight loss over the past five months.
mal limits. EGD with multiple duodenal biopsies His wife also notes that he has been more forgetful
Chapter 9 — Diarrhea and and constipation 233
RATIONALE RATIONALE
Due to this patient’s history of cervical cancer Potential complications of SIBO range include mini-
with radiation, she likely has radiation ileitis mal vitamin deficiencies including malabsorption of
causing bile acid (BA) diarrhea. Typically, BA B12, fat soluble vitamins (vitamin A, D, E and K) and
are reabsorbed in the ileum and returned in the iron. The nutritional consequences of SIBO result
enterohepatic circulation. In states of normal from maldigestion and malabsorption of nutrients
health, only a small amount of BA are not reab- in the intestine, due to microscopic damage to the
sorbed in the ileum and reach the colon. In states microvilli in the small intestine. B12 deficiency
of diseased ileum (Crohn’s disease, radiation results from competitive uptake of B12 from bacte-
ileitis after gynecologic cancers for example), BA ria. Folate can also be elevated in some patients due
are not effectively reabsorbed and reach the co- to increased production of folate by bacteria in the
lon, where bacteria deconjugate and dehydroxyl- small bowel. SIBO does not predispose patients to
ate them, producing secondary BA, which stimu- developing other medical conditions.
late water secretion in the colon and diarrhea.
The best treatment for BA diarrhea is colestipol, a REFERENCE
bile acid sequestrant. These medications improve Dukowicz AC, Lacy BE, Levine GM. Small In-
diarrhea, but have side effects of abdominal dis- testinal Bacterial Overgrowth: A Comprehen-
comfort and bloating. sive Review. Gastroenterology & Hepatology.
2007;3(2):112-122.Adike A, DiBaise JK. Small
REFERENCES Intestinal Bacterial Overgrowth: Nutritional Im-
Camilleri M. Bile Acid Diarrhea: Prevalence, plications, Diagnosis, and Management. Gastroen-
Pathogenesis, and Therapy. Gut and Liver. terol Clin North Am. 2018 Mar;47(1):193-208
2015;9(3):332-339.
Andreyev J. Gastrointestinal symptoms after
pelvic radiotherapy: a new understanding to Question 16
improve management of symptomatic patients. An 82-year-old woman reports abdominal pain,
Lancet Oncol. 2007 Nov;8(11):1007-17. bloating and constipation for the past three weeks.
Her stools were previously regular and soft. She
now has a bowel movement every three to four days
Question 15 and her stools are hard but contain no blood. Her
A 28-year-old woman has a diagnosis of small medical history includes depression, as her hus-
intestine bacterial overgrowth (SIBO). She is won- band died one month ago. Her medications include
dering if SIBO put her at risk for developing other amitriptyline 50mg nightly since her husband’s
medical conditions, or if there are any potential death, aspirin 81mg daily and a multivitamin with
complications. iron. Her rectal exam reveals normal tone and solid
stool in the vault. Laboratory testing reveals normal
Which of the following do you counsel her on? chemistries and complete blood count.
CORRECT ANSWER: A three times per week. She has significant strain-
ing with a bowel movement and feelings of in-
RATIONALE complete evacuation. She tried fiber supplemen-
Risk factors for chronic constipation in older tation, senna and polyethylene glycol 17grams
adults include female gender, physical inactiv- daily without improvement. She has no blood in
ity, low income/education, and depression. her stool and no weight loss.
The new onset constipation is likely caused by
amitriptyline, a tricyclic antidepressant with What is the best next step in evaluation?
a known side effect of constipation. She may
benefit from changing classes of antidepressants. A. MR defecography
Lubiprostone can be used to treat constipation, B. Colonoscopy
but the first step should be to stop any causative C. Digital rectal examination
medications if possible, rather than adding a D. Anorectal manometry with balloon expulsion
new medication, especially in an elderly patient. E. Check thyroid stimulating hormone
In addition, lubiprostone would generally not
be used before trying other laxatives such as CORRECT ANSWER: C
senna and polyethylene glycol, as they are less
expensive and available over the counter. Iron RATIONALE
supplementation is known to cause constipation, Defecatory disorders should be suspected based
abdominal discomfort, nausea and bloating, but on history and may include a report of significant
the patient has been chronically on iron; while straining with defecation, the need for perianal
the tricyclic antidepressant was newly started. or vaginal pressure to aid in defecation, feeling
There are no data to support the role of imaging of incomplete evacuation, and failure to respond
in the evaluation of constipation in older adults. to various laxatives. A digital rectal examination
Colonoscopy may be appropriate for the evalu- is the first step in evaluation of defecatory disor-
ation of constipation in older adults if there are ders, however a normal exam does not exclude
also alarm symptoms, such as rectal bleeding, this diagnosis. After a careful history and physi-
weight loss, iron deficiency anemia. cal exam, further workup should be obtained
based on history and symptoms. In the absence
REFERENCES of other clinical features, metabolic tests such
American Gastroenterological Association., as TSH are not recommended for evaluation of
Bharucha AE, Dorn SD, Lembo A, Pressman A. chronic constipation. A colonoscopy should not
American Gastroenterological Association medi- be performed in patients without alarm features
cal position statement on constipation. Gastro- such as blood in stools, anemia or weight loss,
enterology. 2013 Jan;144(1):211-7. unless age-appropriate colon cancer screening
Wald A. Constipation: Advances in Di- has not been performed. Anorectal manometry
agnosis and Treatment. JAMA. 2016 Jan with balloon expulsion should be performed in
12;315(2):185-91. patients who fail to respond to laxatives, but after
Stern T, Davis AM. Evaluation and Treat- a digital rectal exam. An MR defecography should
ment of Patients With Constipation. JAMA. 2016 be performed after anorectal manometry and
Jan 12;315(2):192-3. balloon expulsion test, especially when results are
inconclusive.
Question 17 REFERENCE
A 47-year-old woman presents for evaluation of American Gastroenterological Association,
chronic constipation. She has a bowel movement Bharucha AE, Dorn SD, Lembo A, Pressman A.
Chapter 9 — Diarrhea and and constipation 237
American Gastroenterological Association medi- not the correct therapy for dyssynergia. In the
cal position statement on constipation. Gastro- absence of other clinical features, metabolic tests
enterology. 2013 Jan;144(1):211-7. such as TSH are not recommended for evalua-
tion of chronic constipation
Question 18 REFERENCE
A 39-year-old woman has a history of dyssyner- American Gastroenterological Association,
gic defecation documented by anorectal manom- Bharucha AE, Dorn SD, Lembo A, Pressman A.
etry and balloon expulsion. Her initial symptoms American Gastroenterological Association medi-
included infrequent bowel movements, signifi- cal position statement on constipation. Gastro-
cant straining, the need to use perianal pressure enterology. 2013 Jan;144(1):211-7.
to defecate, and failure to respond to multiple
laxatives. She was referred for biofeedback
therapy and has had multiple sessions, but she Question 19
has not noticed any improvement. A 50-year-old woman presents with diarrhea
and fecal urgency, especially while eating out at
What is the best next step in evaluation? restaurants. She provides a stool sample after
one of these episodes and testing reveals the
A. MR defecography following:
B. Colonoscopy
C. Start linaclotide Stool osmolality: 290 mOsm/kg
D. Repeat balloon expulsion Stool Na+: 15mEq/L
E. Check thyroid stimulating hormone Stool K+: 10 mEq/L
pH: 4.5
CORRECT ANSWER: D
Based on this sample, which of the following is the
RATIONALE best recommendation to improve her symptoms?
Pelvic floor retraining by biofeedback therapy
rather than laxatives is recommended for defeca- A. Bile acid sequestrant
tory disorders. Biofeedback can be used to train B. Vancomycin
patients to relax their pelvic floor muscles during C. Proton pump inhibitor
straining and to correlate relaxation and pushing D. Eliminate lactose from diet
to achieve defecation. Normal pelvic floor coor- E. Budesonide
dination can be restored. Biofeedback therapy
improves symptoms in more than 70 percemt of CORRECT ANSWER: D
patients with defecatory disorders. If biofeed-
back is unsuccessful, it is recommended to re- RATIONALE
peat balloon expulsion. If a repeat balloon expul- The stool osmotic gap may be used to differenti-
sion is abnormal, the next recommended step is ate whether diarrhea is from an osmotic cause
to obtain an MR defecography to assess for any or a secretory cause. The stool osmotic gap is
significant structural abnormalities. Colonosco- calculated using the equation 290 − 2(stool Na
py is not indicated in the workup of constipation + stool K), where 290 is the value of the stool os-
unless alarm features (blood in stools, anemia or molality. A stool osmotic gap between 50 and
weight loss) are present or age-appropriate colon 100 mosm/kg, a stool osmotic gap of greater
cancer screening has not been performed. Start- than 100 mosm/kg implies an osmotic cause for
ing linaclotide is not appropriate, as laxatives are diarrhea and a stool osmotic gap of less than 50
238 Digestive Diseases Self-Education Program®
mosm/kg implies a secretory cause for diarrhea. tion of the guanylate cyclase-C receptor leads to
This patient has a high gap, indicating an osmotic cyclic GMP release, which inhibits nociceptors,
cause for her diarrhea. Lactose intolerance is the and improves other symptoms of IBS including ab-
only option in the answer choices that has a high dominal pain. This medication therefore would be a
stool osmotic gap, indicating osmotic diarrhea. good option for this patient. Lubiprostone is another
prosecretory agent that activates the chloride chan-
REFERENCE nels on the small intestine enterocytes, leading to
Shiau YF, Feldman GM, Resnick MA, Coff PM water secretion into the lumen, however, it has no
(June 1985). “Stool electrolyte and osmolality effects on improvement in abdominal pain. Senna is
measurements in the evaluation of diarrheal dis- a stimulant laxative, but has no effect on improving
orders”. Ann. Intern. Med. 102 (6): 773–5. abdominal pain and can have side effects of pain and
cramping. Eluxadoline is a mu and kappa-opioid re-
ceptor agonist and is a delta opioid receptor antago-
Question 20 nist in the enteric nervous system. It is approved for
A 21-year-old woman has a history of IBS-C. She the management of IBS-D, not IBS-C.
has been taking polyethylene glycol (PEG) 34
grams a day for the past few months with good REFERENCES
effect, but she wishes to switch to another medica- Shah ED, Kim HM, Schoenfeld P. Efficacy and
tion as she doesn’t like the taste of PEG. She has Tolerability of Guanylate Cyclase-C Agonists for
bothersome abdominal cramping and bloating Irritable Bowel Syndrome with Constipation and
which did not improve with stopping PEG. Chronic Idiopathic Constipation: A Systematic
Review and Meta-Analysis. Am J Gastroenterol.
Which of the following medications do you recom- 2018 Mar;113(3):329-338.
mend? Ford AC, Moayyedi P, Chey WD, Harris LA,
Lacy BE, Saito YA, Quigley EMM; ACG Task Force
A. Insoluble fiber on Management of Irritable Bowel Syndrome.
B. Linaclotide American College of Gastroenterology Monograph
C. Lubiprostone on Management of Irritable Bowel Syndrome. Am
D. Senna J Gastroenterol. 2018 Jun;113(Suppl 2):1-18.
E. Eluxadoline
C. Stool culture positive for Clostridium difficile C. Glucose hydrogen breath test
D. Stool osmotic gap greater than100 D. Stop eluxadoline
E. Duodenal biopsies showing increased E. Low FODMAP diet
epithelial lymphocytes and villous blunting
CORRECT ANSWER: D
CORRECT ANSWER: D
RATIONALE
RATIONALE Eluxadoline is a mu and kappa-opioid receptor
This patient has osmotic diarrhea from drinking agonist and is a delta opioid receptor antagonist
significant quantities of soda containing fructose. in the enteric nervous system. It is approved by
Fructose is a natural sugar and a sweetener in so- the FDA for the management of IBS-D. The use of
das. Some healthy patients (up to 50 percent) have eluxadoline is contraindicated in in patients who
incomplete fructose absorption, as the capacity of have had a prior cholecystectomy, with a known
the intestine to transport fructose across the intes- or suspected history of biliary duct obstruction,
tinal epithelium is exceeded and therefore malab- sphincter of Oddi dysfunction, a history of pan-
sorption occurs. This non-absorbed sugar thereby creatitis or structural diseases of the pancreas,
causes an osmotic diarrhea. Removing soda from significant alcohol use (more than three drinks
the diet resolves the diarrhea. per day) or abuse, hepatic impairment. From May
2015 (when eluxadoline was first approved by the
REFERENCES FDA) through February 2017, the FDA received 120
Choi YK, Johlin FC Jr, Summers RW, Jackson reports of serious cases of pancreatitis or death.
M, Rao SS. Fructose intolerance: an under- Among 68 patients who reported their gallbladder
recognized problem. Am J Gastroenterol. 2003 status, 56 of them did not have a gallbladder and
Jun;98(6):1348-53. received the currently recommended dosage of
Gibson PR, Newnham E, Barrett JS, Shepherd eluxadoline. Given her history of cholecystectomy,
SJ, Muir JG. Review article: fructose malabsorp- she is at risk for pancreatitis with use of eluxadoline
tion and the bigger picture. Aliment Pharmacol due to sphincter of Oddi spasm. The other options
Ther. 2007 Feb 15; 25(4):349-63. are not appropriate as she is not at the age to start
colorectal cancer screening now and her symptoms
are well controlled on medication; therefore further
Question 22 workup and evaluation is unnecessary.
A 40-year-old woman presents to your office to es-
tablish care for her IBS-D. She has a history of hy- REFERENCE
pertension and a cholecystectomy for gallstones. VIBERZI (eluxadoline) [prescribing information].
She takes lisinopril and eluxadoline. She has no Irvine, CA: Allergan USA, Inc.; 2018.
family history of IBD, colon cancer or polyps. She https://www.fda.gov/Drugs/DrugSafety/
appears well and has no complaints. She is hav- ucm546154.htm (Accessed on January 8, 2019).
ing one formed bowel movement daily or every
other day, which is improved since she was first
diagnosed with IBS-D. She has minimal abdomi- Question 23
nal pain and bloating when she eats certain food, A 45-year-old woman presents for evaluation of
which she attributes to lactose-containing prod- constipation. She has a bowel movement every
ucts. What do you recommend to her? three days and her stools are hard. About one year
ago, she had a soft stool daily or every other day
A. Colonoscopy for screening now without straining or blood. She has a history of
B. Fecal calprotectin recently diagnosed hypertension and hyperlip-
240 Digestive Diseases Self-Education Program®
idemia. Her medications include verapamil and reveal lymphocytic colitis. Which of the following
atorvastatin. She has never tried any laxatives for medications may have predisposed her to develop-
her constipation. What is the best next step in her ing microscopic colitis?
evaluation?
A. Olmesartan
A. Start soluble fiber daily B. Sertraline
B. Stop verapamil C. Lisinopril
C. Sitz marker study D. Lorazepam
D. Anorectal manometry E. Metoprolol
E. MRI defecography
CORRECT ANSWER: B
CORRECT ANSWER: B
RATIONALE
RATIONALE Certain drugs have been identified as risk factors for
Verapamil is a calcium-channel blocker with a developing microscopic colitis. Multiple drugs have
known side effect of constipation. The best first been implicated, including aspirin, NSAIDS, selec-
step is to try to stop the medication and switch to tive serotonin reuptake inhibitors (SSRIs) and pro-
a different agent without side effects of constipa- ton pump inhibitors (PPIs). Smoking has also been
tion. Starting fiber would be an appropriate medi- identified as an environmental risk factor. Patients
cation choice for constipation however, the best should be counseled to discontinue any potential of-
first step is to stop any constipating medications. fending medications after the diagnosis is confirmed.
A sitz marker study, anorectal manometry or MRI
defecography could all be obtained for the workup REFERENCE
of constipation, however, offending medications Keszthelyi D, Jansen SV, Schouten GA, de Kort S,
should be stopped first if possible. Scholtes B, Engels LG, Masclee AA. Proton pump
inhibitor use is associated with an increased risk
REFERENCE for microscopic colitis: a case-control study. Ali-
Locke GR III, Pemberton JH, Phillips SF. AGA ment Pharmacol Ther. 2010 Nov;32(9):1124-8.
technical review on constipation. American Gas- Fernández-Bañares F, Esteve M, Espinós JC,
troenterological Association. Gastroenterology. Rosinach M, Forné M, Salas A, Viver JM. Drug
2000;119(6):1766–1778. consumption and the risk of microscopic colitis.
Bassotti G, Calcara C, Annese V, Fiorella S, Am J Gastroenterol. 2007 Feb;102(2):324-30.
Roselli P, Morelli A. Nifedipine and verapamil
inhibit the sigmoid colon myoelectric response to
eating in healthy volunteers. Dis Colon Rectum. Question 25
1998 Mar;41(3):377-80.80. An 18-year-old woman has a one-year history of
diarrhea associated with food intake. She is healthy,
but has iron deficiency anemia and Grave’s disease.
Question 24 She has no family history of inflammatory bowel
A 65-year-old woman presents for a surveillance disease, celiac disease or other gastrointestinal dis-
colonoscopy due to a personal history of adeno- eases. She notes that she has maintained a “100 per-
mas. She mentions in the procedure room that she cent%” gluten-free diet for the past four months and
is also having three to four loose bowel move- the consistency of her stools have improved, though
ments per day, sometimes waking her from sleep. she continues to have intermittent diarrhea, espe-
You perform the colonoscopy and find no polyps. cially when drinking lactose containing products.
You take random biopsies of the colon which What is the best recommendation for this patient?
Chapter 9 — Diarrhea and and constipation 241
A. Gluten-free diet and lactose free diet indefinitely A. Measurement of serum IgA tissue
B. Gluten challenge and upper endoscopy transglutaminase (tTG) with serum IgA
C. Colonoscopy with random biopsies B. Colonoscopy with biopsies
D. SIBO breath testing C. Glucose hydrogen breath test
E. Thyroid stimulating hormone D. Upper endoscopy with duodenal biopsies
RATIONALE RATIONALE
Based on her history of diarrhea, iron deficiency There is evidence that celiac disease is more
anemia and a diagnosis of an endocrine autoim- common in patients with Type I DM compared
mune condition, the diagnosis of Celiac Disease to the general Caucasian population. A variety of
should be highly suspected. It is also likely that she serologic assays are available to aid in the pri-
has secondary lactose intolerance, which is com- mary diagnosis of celiac disease (CD). Currently,
mon in Celiac Disease. Because she is gluten-free, serum IgA with IgA tissue transglutaminase is
the best next step to confirm the diagnosis is a the preferred single test for the detection of CD.
gluten challenge followed by upper endoscopy with A colonoscopy may be indicated if the diagnostic
duodenal biopsies. Celiac disease should be differ- evaluation for CD is negative however, this would
entiated from nonceliac gluten sensitivity because not be the best next step in the evaluation of this
patients with celiac disease are at risk for nutrition- patient’s symptoms. A glucose breath test would
al deficiencies, osteoporosis, small bowel tumors, aid in the diagnostic evaluation of small intestinal
and require a pneumococcal vaccine and DEXA bacterial overgrowth, but CD remains the most
scan. The diagnosis of celiac disease may also have likely diagnosis. According to current published
important implications for family members. guidelines, an upper endoscopy with duodenal
biopsies would be indicated in those patients in
REFERENCES whom the suspicion for CD is high despite nega-
Rubio-Tapia A, Hill ID, Kelly CP, et al. ACG clini- tive serologies, however, serologic evaluation
cal guidelines: Diagnosis and management of should be pursued first.
celiac disease. Am J Gastroenterol 2013; 108:656.
Lebwohl B, Rubio-Tapia A, Assiri A et al. Di- REFERENCE
agnosis of celiac disease. Gastrointest Endosc Clin Rubio-Tapia A, Hill ID, Kelly CP, et al. ACG clini-
N Am 2012;22:661–677. cal guidelines: diagnosis and management of
celiac disease. Am J Gastroenterol 2013; 108:656
Question 26
A 19-year-old woman with Type I diabetes mel- Question 27
litus (DM) is referred to you for new onset symp- Which of the following statements regarding
toms of intermittent loose stools and bloating. vitamin deficiencies or excess in small intestinal
She denies weight loss or GI bleeding. There is no bacterial overgrowth is true?
family history of inflammatory bowel disease or
gastrointestinal cancer. Physical exam is normal. A. Enteric bacteria compete with the host for
Laboratory evaluation reveals a low Vitamin D vitamin B12, resulting in vitamin B12 deficiency
level and iron deficiency. B. Enteric bacteria compete with the host for
vitamin K, resulting in vitamin K deficiency
What is the best next step in the evaluation of this C. Thiamine is synthesized by enteric bacteria,
patient’s symptoms? leading to thiamine excess
242 Digestive Diseases Self-Education Program®
D. Enteric bacteria compete with the host for with a prolonged taper and pulsed vancomycin
folate, resulting in folate deficiency regimen versus fidaxomicin. The most recent ACG
guideline for the treatment of CDI recommends
CORRECT ANSWER: A treatment of the first recurrence with the same
regimen as the initial episode.
RATIONALE
Enteric bacteria compete with the host for vitamin REFERENCES
B12, resulting in a vitamin B12 deficiency. Fo- L Clifford McDonald, Dale N Gerding, Stuart
late and vitamin K levels are typically normal or Johnson, et al. ; Clinical Practice Guidelines
increased in SIBO due to synthesis by the enteric for Clostridium difficile Infection in Adults and
bacteria. Thiamine levels are not significantly af- Children: 2017 Update by the Infectious Diseases
fected in SIBO. Society of America (IDSA) and Society for Health-
care Epidemiology of America (SHEA), Clinical
REFERENCES Infectious Diseases, Volume 66, Issue 7, 19 March
Su J, Smith MB, Rerknimitr R, Morrow D. Small 2018, Pages e1–e48.
intestine bacterial overgrowth presenting as Surawicz CM, Brandt LJ, Binion DG, et al.
protein-losing enteropathy. Dig Dis Sci. 1998 Guidelines for diagnosis, treatment, and preven-
Mar;43(3):679-81. tion of Clostridium difficile infections. Am J Gas-
DiBiase, JK. Nutritional Consequences of troenterol 2013; 108:478.
Small Intestinal Bacterial Overgrowth. Practical
Gastroenterology. 2008.
Question 29
A 66-year-old woman presents for an evaluation
Question 28 of an eight month history of diarrhea. She denies
A 66-year-old man with a past medical history of weight loss, GI bleeding, or family history of IBD
uncomplicated C. difficile infection (CDI), which or colorectal neoplasia. Physical examination is
was treated with vancomycin six weeks ago, pres- normal. Colonoscopy with biopsies reveal greater
ents for evaluation of recurrent diarrhea for the than 20 intraepithelial lymphocytes per 100 sur-
past two weeks. Stool testing reveals a positive face epithelial cells with focal cryptitis.
glutamate dehydrogenase and positive toxin A/B.
What is the next best step in the management of
What is the best next step in the management of this patient?
this patient?
A. Start oral budesonide therapy
A. Start intravenous immunoglobulins (IVIG) B. Refer for video capsule endoscopy (VCE)
B. Metronidazole C. Start oral mesalamine therapy
C. Vancomycin D. Refer for bone marrow biopsy
D. Fecal microbiota transplant (FMT) E. Start rifaximin therapy
RATIONALE RATIONALE
According to the most recently published Infec- This patient has lymphocytic colitis. The man-
tious Diseases Society of America (IDSA) guide- agement of microscopic colitis can encompass
lines, the first recurrence of CDI in a patient previ- a variety of medications from over-the-counter
ously treated with vancomycin should be treated anti-diarrheal therapies to systemic steroids. Of
Chapter 9 — Diarrhea and and constipation 243
those listed here, budesonide therapy is the next gastritis. SIBO can be diagnosed by a glucose
best step in the management of this condition. A hydrogen breath test, and thus it is the best
video capsule endoscopy would aid in the evalu- next step in the management of this patient’s
ation of possible inflammatory bowel disease or symptoms given the history of atrophic gastritis.
for the diagnostic evaluation of suspected small Chromogranin A and video capsule endoscopy
bowel bleeding, but would not be indicated in the are used in the diagnostic evaluation of sus-
clinical scenario above. Mesalamine and rifaxi- pected carcinoid syndrome and inflammatory
min are not indicated for the first-line treatment bowel disease, respectively and may be indicated
of microscopic colitis. The histopathologic find- in the subsequent evaluation of this patient’s
ings above reflect lymphocytic colitis and a bone symptoms. In addition, both of these diagnoses
marrow biopsy is not indicated. are unlikely to cause intermittent diarrhea. Elux-
adoline is an agent that combines a mu-opioid
REFERENCE receptor agonist and a delta-opioid receptor
American Gastroenterological Association Insti- antagonist and is indicated for the treatment of
tute Guideline on the Medical Management of irritable bowel syndrome – diarrhea predomi-
Microscopic Colitis. Geoffrey C. Nguyen, Walter E. nant (IBS-D). The diagnosis of IBS requires the
Smalley, Santhi Swaroop Vege, Alonso Carrasco- presence of abdominal pain, and is unlikely in an
Labra, and the Clinical Guidelines Committee. elderly patient with new onset of symptoms, thus
Gastroenterology 2016;150:242–246. this is not the diagnosis in this patient’s case.
REFERENCE
Question 30 Jan Bures, Jiri Cyrany, Darina Kohoutova, Miro-
A 66-year-old woman with history of atrophic slav Förstl, Stanislav Rejchrt, Jaroslav Kvetina,
gastritis presents for evaluation of intermittent Viktor Vorisek, and Marcela Kopacova. Small in-
diarrhea. She denies abdominal pain, weight testinal bacterial overgrowth syndrome. World J
loss, GI bleeding or a family history of colorectal Gastroenterol. 2010 Jun 28; 16(24): 2978–2990.
neoplasia or IBD. Physical exam is normal. Labs
including thyroid function testing, celiac screen
and CRP are normal. A colonoscopy with random Question 31
colon biopsies is normal. A 56-year-old woman with no significant past
medical history is admitted with diarrhea which
What is the best next step in the management of began after a course of antibiotics prescribed for
this patient? bronchitis. C. difficile toxin A/B is positive, along
with a positive glutamate dehydrogenase. Her
A. Measure chromogranin A WBC count is 16,000 cells/mm3. She is started
B. Start eluxadoline on oral vancomycin therapy. Over the next 24
C. Video capsule endoscopy hours, she develops fever to 38.8°C, decreased
D. Glucose hydrogen breath test bowel movements and abdominal distension with
decreased bowel sounds. What is the recommend-
CORRECT ANSWER: D ed treatment for this patient?
Labs ordered by his primary care doctor reveal Labra, and the Clinical Guidelines Committee.
normal CBC, thyroid function tests, celiac screen Gastroenterology 2016;150:242–246.
and CRP. Pardi DS, Kelly CP. Microscopic colitis. Gas-
troenterology. 2011 Apr;140(4):1155-65.
What is best next step in the management of
this patient?
Question 36
A. Repeat colonoscopy with biopsies from the A 22-year-old woman is referred by her primary
right and left colon care physician for endoscopy. She has a long-
B. Start rifaximin standing history of “brain fog,” abdominal bloat-
C. Refer for lactose breath testing ing and diarrhea. One of her friends has celiac
D. Start nortriptyline disease and suggested she try a gluten-free diet
(GFD). She initiated a gluten-free diet six months
CORRECT ANSWER: A ago and reports a significant improvement in
her symptoms. Despite what she considers strict
RATIONALE adherence to a gluten-free diet, she has noted
This patient is at increased risk of microscopic a recurrence of symptoms recently. Diagnostic
colitis given his medication list. Statins and proton evaluation by her primary care doctor last week
pump inhibitors have both been associated with revealed a normal serum IgA and tissue transglu-
microscopic colitis. Endoscopic findings are typi- taminase (tTG) IgA 26 U/mL (normal less than
cally normal, but can include subtle macroscopic 19 U/mL). You perform an upper endoscopy with
findings such as edema and erythema. Biopsies adequate biopsies of the duodenal bulb and sec-
should be obtained from both the right and left ond portion of the duodenum, which are normal.
colon. Histologic findings can be patchy, with the What is the best next step?
highest diagnostic yield in the transverse colon
and right colon and least in the rectosigmoid. A. Repeat endoscopy with additional duodenal
Rifaximin and TCAs are indicated for the man- biopsies
agement of irritable bowel syndrome – diarrhea B. Check anti-gliadin antibody
predominant (IBS-D) however, the absence of C. Initiate a gluten challenge followed by repeat
significant abdominal pain makes IBS unlikely in endoscopy with biopsies
this patient. Carbohydrate malabsorption does D. Refer the patient to nutrition for non-celiac
not appear to be playing a significant role in this gluten sensitivity
patient’s symptoms by history. E. Perform colonoscopy with biopsies
REFERENCE REFERENCES
Rubio-Tapia A, Hill ID, Kelly CP, et al. ACG Fine KD, Santa Ana CA, Fordtran JS Diagnosis of
clinical guidelines: diagnosis and management of magnesium-induced diarrhea. N Engl J Med. 1991
celiac disease. Am J Gastroenterol 2013; 108:656. Apr 11;324(15):1012-7.
Wald, A. Factitious diarrhea: Clinical manifes-
tations, diagnosis, and management. UpToDate.
Question 37
A 37-year-old nurse is self-referred for evaluation
of diarrhea. She has seen several previous gastro- Question 38
enterologists and has undergone extensive diag- A 28-year-old man with no significant medical
nostic evaluation including EGD with biopsies, CT history presents for evaluation of abdominal pain
enterography, normal hydrogen breath testing, associated with diarrhea of six months dura-
and multiple colonoscopies with biopsies, which tion. The abdominal pain improves with defeca-
have all been normal. There is no anemia, no fam- tion. He denies weight loss or overt GI bleeding.
ily history of IBD or colon cancer. Which of the Family history is negative for colorectal cancer,
following results most supports your diagnosis? but his brother has inflammatory bowel disease.
Stool studies for infections are negative. Routine
A. Stool magnesium 107 mEq/L laboratory testing reveals a normal CBC, normal
B. Stool phosphate 20 mmol/L thyroid function tests, normal serum IgA, normal
C. Stool osmolality 290 mmol/L IgA tissue transglutaminase, and a CRP 15mg/L
D. Laxative screen showing mineral oil (normal less than 3mg/L). Physical examination
is normal.
CORRECT ANSWER: A
What is the next best step in the evaluation of this
RATIONALE patient?
There are several clues in this patient’s presen-
tation pointing to possible laxative abuse and A. Glucose breath testing
factitious diarrhea. More than 90 percent of B. HLA-DQ2/DQ8
patients with factitious diarrhea are women and C. Colonoscopy
have a history of work in the healthcare field. In D. Upper endoscopy
addition, they typically present with a history of E. Video capsule endoscopy
extensive healthcare utilization. Stool sodium,
potassium, magnesium, and phosphate should be CORRECT ANSWER: C
measured in patients suspected of having facti-
tious diarrhea. Stool magnesium greater than 90 RATIONALE
mEq/L suggests magnesium-induced diarrhea. A This patient meets Rome Criteria for irritable
stool phosphate greater than 33mmol/L suggests bowel syndrome. Because of the low pretest prob-
a phosphate-induced diarrhea. An osmotic gap ability of inflammatory bowel disease and colonic
greater than 75 can be due to laxatives contain- neoplasia, routine colonoscopy is not recom-
ing magnesium, sorbitol, lactose, lactulose or mended in patients younger than 50 years of age
polyethylene glycol while an osmotic gap less with typical IBS symptoms and no alarm features.
than 75 can be seen with secretory laxatives or in Alarm features include rectal bleeding, weight
an osmotic diarrhea from a sodium containing loss, iron deficiency anemia, nocturnal symptoms,
laxative. A normal stool osmolality is estimated and a family history of selected organic diseases
to be 290 mmol/L. There is currently no laxative including colorectal cancer, inflammatory bowel
screen for mineral oil. disease (IBD), and celiac sprue. Colonoscopy
248 Digestive Diseases Self-Education Program®
should be performed in IBS patients younger than serologies can be obtained, but a negative result
50 years of age if concomitant alarm features does not preclude a diagnosis of CD. The recom-
are present, as in the case of this patient (family mended serology for diagnosis of celiac disease is
history of IBD and elevated CRP). A colonoscopy IgA tissue transglutaminase Ab. The diagnosis of
is also recommended in those over the age of 50 CD in those patients already on a GFD will require
years for the purpose of colorectal cancer screen- a gluten challenge with subsequent serology and
ing if not already done. duodenal biopsies as indicated. HLA DQ2/DQ8
genotyping should be undertaken to exclude CD
REFERENCE prior to initiating a formal gluten challenge. This
Brandt LJ, Chey WD, Foxx-Orenstein AE, Schiller test has a very good negative predictive value (99
LR, Schoenfeld PS, Spiegel BM, Talley NJ, Quigley percent), but the positive predictive value is only
EM. An evidence-based position statement on 12 percent%, due to the high prevalence of these
the management of irritable bowel syndrome. Am haplotypes in the general population. Therefore,
J Gastroenterol. 2009;104 Suppl 1:S1. a negative HLA DQ2/DQ8 rules out celiac disease,
and precludes the need for gluten challenge and
endoscopy. If HLA DQ2/DQ8 is positive, the next
Question 39 step would be a gluten challenge and endoscopy
A 22-year-old woman is referred to you for evalua- with biopsies to confirm the diagnosis of CD.
tion of chronic post-prandial bloating and inter-
mittent loose stools. She reports that her first REFERENCE
cousin had similar symptoms and was recently Rubio-Tapia A, Hill ID, Kelly CP, et al. ACG
diagnosed with celiac disease (CD). In anticipa- clinical guidelines: diagnosis and management of
tion of her visit with you, she decided to trial a celiac disease. Am J Gastroenterol 2013; 108:656.
gluten-free diet and reports significant improve-
ment in her symptoms over the past six weeks,
since she has been adhering to the diet. She states Question 40
that she will never eat gluten again, as she feels so A 26-year-old man with no significant history
much better off of it. presents for evaluation of six-month history of
abdominal bloating associated with non-bloody
Which of the following is the best next step in diarrhea. Physical exam is normal. There is no
evaluating this patient for celiac disease? contributory family history. Stool testing for
infections, CBC, celiac screen and thyroid func-
A. Refer for video capsule endoscopy tion testing are normal. A colonoscopy revealed
B. Schedule an upper endoscopy with duodenal normal findings with random colon biopsies
biopsies showing normal colonic mucosa. Stool electrolyte
C. Check anti-endomysial antibodies testing reveals:
D. Check HLA-DQ2/DQ8
E. Refer for D-xylose testing Stool osmolality (estimated): 290 mOsm/kg
Stool sodium: 44 mEq/L
CORRECT ANSWER: D Stool potassium: 31 mEq/L
Question 41
A 76-year-old woman with a past medical his- Question 42
tory of hypertension and gastroesophageal reflux A 65-year-old man with a past medical history of
disease is referred for evaluation of intermittent short-segment Barrett’s esophagus on once daily
diarrhea over the past six months. Medications proton pump inhibitor (PPI) and an episode of
include verapamil and lansoprazole. Colonoscopy non-severe C. difficile infection (CDI) three months
revealed normal appearing mucosa with biopsies ago after a course of antibiotics given for bronchitis
showing mild cryptitis with greater than 100 in- is due to undergo routine dental work for which his
traepithelial lymphocytes. dentist has prescribed prophylactic antibiotics.
What is the best next step in the management of Which of the following is recommended for the
this patient? prevention of CDI?
Rubio-Tapia A, Rahim MW, See JA, et al. Mu- Bharucha AE, Pemberton JH, Locke GR 3rd.
cosal recovery and mortality in adults with celiac American Gastroenterological Association techni-
disease after treatment with a gluten-free diet. Am cal review on constipation. Gastroenterology 2013;
J Gastroenterol 2010;105:1412–1420. 144:218.
Question 44 Question 45
A 37-year-old woman with no significant past medi- A 36-year-old man on chronic methadone main-
cal history presents for further evaluation of chronic tenance therapy reports worsening constipation
constipation with straining. She denies significant characterized by significant straining, less than
abdominal pain, gastrointestinal bleeding or weight three spontaneous bowel movements per week,
loss. There is no family history of colorectal neo- and sensation of incomplete evacuation after re-
plasia or inflammatory bowel disease. She has not cent methadone dose escalation.
responded to many laxatives and undergoes anorec-
tal manometry with balloon expulsion testing. Which of the following is the best next step in the
management of this patient?
Which of the following results suggests a diagnosis
of dyssynergic defecation? A. Lubiprostone
B. Naloxegol
A. Absent rectoanal inhibitory reflex (RAIR) C. Osmotic laxatives
B. Expulsion of water-filled balloon in three D. Methylnaltrexone
minutes
C. Defecation index of 2.0 CORRECT ANSWER: C
D. Decreased anal sphincter pressure during
simulated defecation RATIONALE
This patient meets Rome IV criteria for opioid
CORRECT ANSWER: B induced constipation (OIC). According to the
most recent published guidelines for the medical
RATIONALE management of opioid induced constipation, first-
The balloon expulsion test is highly suggestive of line agents should be traditional laxatives. There
dyssynergia. A balloon expulsion time of greater is insufficient evidence to recommend the use of
than two minutes is abnormal. An absent RAIR lubiprostone for OIC. If traditional laxatives are
can be seen in Hirschsprung’s disease or megarec- ineffective, current guidelines recommend the
tum. Defecation index = maximum rectal pres- use of Peripherally-Acting Mu-Opioid Receptor
sure during attempted defecation/minimum anal Antagonists (PAMORAs) such as naloxegol and
residual pressure during attempted defecation. A methylnaltrexone over no treatment.
normal defecation index is greater than 1.5. De-
creased anal sphincter pressure during simulated REFERENCES
defecation is normal and therefore not consistent Spierings ELH, Drossman DA, Cryer B, Mazen
with dyssynergic defecation. Jamal M, Losch-Beridon T, Mareya SM, Wang M.
Efficacy and Safety of Lubiprostone in Patients
REFERENCES with Opioid-Induced Constipation: Phase 3 Study
Wald A, Bharucha AE, Cosman BC, Whitehead Results and Pooled Analysis of the Effect of Con-
WE. ACG clinical guideline: management of comitant Methadone Use on Clinical Outcomes.
benign anorectal disorders. Am J Gastroenterol. Pain Med. 2018 Jun 1;19(6):1184-1194.
2014 Aug;109(8):1141-57. Seth D. Crockett, Katarina B. Greer, Joel J.
252 Digestive Diseases Self-Education Program®
Heidelbaugh, et al. on behalf of American Gastro- rience is limited. Pelvic floor physical therapy
enterological Association Institute Clinical Guide- would be appropriate if defecography confirms
lines Committee. American Gastroenterological impaired rectal evacuation. A colon transit study
Association Institute Guideline on the Medical is the appropriate next step once defecation disor-
Management of Opioid-Induced Constipation ders are ruled out or if symptoms persist despite
Gastroenterology 2018. normalization of defecation disorders. A RAIR is
present, thus a rectal suction biopsy to evaluate
for Hirschsprung’s disease is not indicated.
Question 46
A 35-year-old woman with no significant history REFERENCES
presents for worsening chronic constipation. She Wald A, Bharucha AE, Cosman BC, Whitehead
endorses a sense of incomplete evacuation, signifi- WE. ACG clinical guideline: management of
cant straining with bowel movements, and occa- benign anorectal disorders. Am J Gastroenterol.
sional need to apply perineal pressure. Abdominal 2014 Aug;109(8):1141-57
pain and bloating are occasionally present but are Bharucha AE, Pemberton JH, Locke GR 3rd.
not the predominant symptoms. She denies rectal American Gastroenterological Association techni-
bleeding, unintentional weight loss and there is cal review on constipation. Gastroenterology 2013;
no family history of colonic neoplasia or IBD. She 144:218.
had previously been maintained on daily PEG but Maria G, Cadeddu F, Brandara F, Marniga
has recently found it to be inconsistently effective. G, Brisinda G. Experience with type A botulinum
On rectal exam, soft stool is noted in the rectal toxin for treatment of outlet-type constipation.
vault along with a high resting sphincter pressure Am J Gastroenterol. 2006;101(11):2570. Epub
and suggestion of incomplete anal contraction 2006 Oct 4.
with simulated defecation. You refer her for ano-
rectal manometry, which confirms a dyssynergic
pattern with adequate rectal propulsive forces and Question 47
paradoxical anal contraction. A rectoanal inhibi- A 34-year-old woman with no significant past
tory reflex (RAIR) is present. She is able to pass medical history presents with an eight month his-
a water-filled balloon in 30 seconds. What is the tory of abdominal pain/bloating associated with
next best step in the management of this patient? altered bowel habits, namely worsening consti-
pation characterized by straining and sense of
A. Anal sphincter botulinum toxin injection incomplete evacuation. Daily polyethylene glycol
B. Defecography (PEG) therapy has been ineffective. She denies
C. Pelvic floor physical therapy rectal bleeding or family history of colorectal
D. Colon transit study neoplasia or IBD. Labs including CBC and thyroid
E. Rectal suction biopsy function panel are normal. Rectal examination
reveals no masses, soft stool in the rectal vault,
CORRECT ANSWER: B no overt paradoxical anal contraction, but limited
perineal descent with simulated defecation.
RATIONALE
In situations when the anorectal manometry Which of the following is the most appropriate
and balloon expulsion testing are discordant or next step in the management of this patient?
inconclusive, a defecography is the next step in
the evaluation of patients. While there have been A. Colonic transit testing
small studies evaluating the utility of botulinum B. Anorectal manometry with balloon
toxin injection in obstructive defecation, expe- expulsion testing
Chapter 9 — Diarrhea and and constipation 253
A. Clindamycin
B. Sucralfate
C. Sertraline
D. Amitriptyline
E. Sumatriptan
CORRECT ANSWER: C
256 Digestive Diseases Self-Education Program®
Answers & critiques
CHAPTER 10
Gastrointestinal motility
Walter W. Chan, MD, MPH, Ronnie Fass, MD, FACG, Sushila R. Dalal, MD
and Christina Ha, MD, FACG, AGAF
257
258 Digestive Diseases Self-Education Program®
RATIONALE RATIONALE
Failed balloon expulsion test (answer c) has been This patient’s esophageal manometry showed an
found to correlate with an evacuation disorder increase in ineffective swallows (40 percent weak
and predict biofeedback therapy response. Anal [DCI 100 to 450 mmHg*s*cm] and 20 percent
sphincter tone (answer a) and squeeze pres- failed [DCI greater than 100 mmHg*s*cm] swal-
sure (answer e) are most useful in the evalua- lows) with otherwise normal findings. This pat-
tion of fecal incontinence, but play less of a role tern is suggestive of ineffective esophageal motility
in assessment of obstructive defecation. Rectal (IEM) per Chicago classification v3.0. IEM is most
sensation testing (answer d) may be abnormal in commonly associated with gastroesophageal reflux
constipation, with rectal hyposensitivity causing a (answer e). Diffuse esophageal spasm (answer
decrease in urge for defecation. However, it alone a) is characterized by increased spastic contrac-
is not associated with obstructive defecation. tions, defined by a short DL (greater than 4.5 sec).
Recto-anal inhibitory reflex (RAIR) (answer b) is Esophagogastric junction outflow obstruction,
used as an assessment of normal colonic inner- defined by a high IRP, is often associated with
vation, and would be abnormal in patients with tumor at the esophagogastric junction (answer
Hirschsprung’s disease. b) and chronic opioid use (answer c). Jackham-
mer esophagus (answer d) is characterized by an
REFERENCE increase in hypercontractile swallows (DCI greater
Carrington EV et al. Expert consensus document: than 8000 mmHg*s*cm).
Advances in the evaluation of anorectal func-
tion. Nat Rev Gastroenterol Hepatol 2018 May; REFERENCE
15(5):309-323. Kahrilas PJ et al. The Chicago Classification of
Esophageal Motility Disorders v3.0. Neurogastro-
enteral Motil 2015;27(2):160-174.
Question 4
A 34-year-old otherwise healthy man presents for
pre-operative evaluation for fundoplication. His Question 5
esophageal manometry revealed a basal lower A 28-year-old woman underwent anorectal
esophageal pressure of 20 mmHg and an inte- manometry for chronic constipation refractory
grated relaxation pressure (IRP) of 5 mmHg. The to multiple laxatives. The anorectal manometry
distal latency (DL) was noted to be greater than showed an increase in resting anal sphincter pres-
4.5 sec in all 10 test swallows. The distal contrac- sure (83 mmHg), normal squeeze pressure (180
tile integral (DCI) ranged 500-1000 mmHg*s*cm mmHg), normal rectal contraction pressure (50
in four swallows, 200-300 mmHg*s*cm in two mmHg), increased residual pressure during simu-
swallows, and 30-60 mmHg*s*cm in four swal- lated defecation (85 mmHg), and no relaxation of
lows. The peristaltic break was less than 5 cm in the anal sphincter during balloon distention. The
all swallows. Which of the following is the most sensation testing showed a mildly elevated first
likely underlying cause of these findings? sensation pressure (50 mmHg). She could not
expel the 50 mL balloon in two minutes.
A. Diffuse esophageal spasm
B. Tumor at the esophagogastric junction What should be the next step in management?
C. Chronic opioid use
D. Jackhammer esophagus A. Referral for biofeedback therapy.
E. Gastroesophageal reflux disease B. Botox injection to anal sphincter.
260 Digestive Diseases Self-Education Program®
Question 6 REFERENCE
A 45-year-old woman with a history of cholecystec- Ford, A.C., et al., American College of Gastro-
tomy presents for evaluation of chronic abdominal enterology monograph on the management of
pain, bloating, and constipation since her teenage irritable bowel syndrome and chronic idiopathic
years. She reports bowel movements every three constipation. Am J Gastroenterol, 2014;109 Suppl
to five days, with no hematochezia or melena. Her 1: p. S2-26; quiz S27.
abdominal pain and bloating are worse when she is
constipated and decrease with bowel movements.
She reports some nausea daily, but no vomiting. Question 7
She denies any fever, nocturnal symptoms, greasy A 42-year-old woman was hospitalized for nau-
Chapter 10 — Gastrointestinal motility 261
sea, vomiting, abdominal distention, and inability obstruciton in children and adults: diagnosis and
to tolerate her oral diet. Abdominal CT revealed therapeutic options. Neurogastroenterol Motil
diffuse small intestinal dilatation without a focal 2017 Jan;29(1).
point of obstruction. She has had several simi-
lar presentations and CT findings in the past.
Evaluation with an antroduodenal manometry
Question 8
revealed fasting phase III migrating motor com-
A 62-year-old male with dysphagia presents for
plex (MMC) with a frequency of ~12/minute and
esophageal manometry for evaluation of his swal-
average contraction amplitude of 3-5 mmHg in
lowing symptoms. He
the duodenum.
previously completed an
upper endoscopy and
Which of the following is the most likely etiology
a barium esophagram
of her clinical presentation and study findings?
for assessment of his
symptoms. The upper
A. Diabetes mellitus.
endoscopy was noted to
B. Systemic sclerosis.
be normal. The barium
C. Carcinoid tumor.
esophagram revealed the
D. Neurofibromatosis.
following findings:
E. Multiple sclerosis.
of swallows (greater than 20 percent) with a What would be the most appropriately treatment
decreased distal latency (greater than 4.5 sec). for her condition?
Increased IRP with pan-esophageal pressuriza-
tion (answer a) is the characteristic finding of A. Pneumatic dilation to 30 mm.
type II achalasia, which would often demonstrate B. Heller myotomy with Dor fundoplication
a dilated esophagus with “bird’s beak” appear- C. POEM
ance on esophagram. A normal IRP with failed D. Botulinum toxin injection into the lower
contractions in all swallows (answer b) suggests esophageal sphincter
absent contractility per Chicago classification E. Heller myotomy with Nissen fundoplication.
v3.0, while an elevated IRP with normal DCI and
distal latency (answer d) indicates esophagogastric CORRECT ANSWER: D
junction outflow obstruction (EGJOO). An in-
crease in weak swallows (DCI between 100 to 450 RATIONALE
mmHg*s*cm) would suggest ineffective esopha- The patient’s presentation and esophageal ma-
geal motility (answer e). nometry are consistent with type II achalasia.
While Heller myotomy, per oral esophageal my-
REFERENCE otomy (POEM), and pneumatic dilation are effec-
Kahrilas PJ et al. The Chicago Classification of tive first-line therapies for achalasia, this patient’s
Esophageal Motility Disorders v3.0. Neurogastro- significant medical comorbidities (coronary artery
enteral Motil 2015;27(2):160-174. disease, congestive heart failure, and advanced
lung disease requiring home oxygen) would make
her a poor candidate. As pneumatic dilation is
Question 9 associated with a perforation risk of around five
A 68-year-old woman with a history of coronary percent, it would not be an appropriate treatment
artery disease, congestive heart failure, and COPD for patients who are poor surgical candidates. In-
on three liters of oxygen at home presents with jection of botulinum toxin to the lower esophageal
progressive dysphagia over the last five years. She sphincter is relatively safe and has a high short-
describes difficulty swallowing both solids and term response rate. However, its treatment effect
liquids, often needing to regurgitate them back deteriorates within six to 12 months of therapy, of-
up. She underwent an upper endoscopy that was ten requiring repeat injection. Therefore, it is gen-
unremarkable, including biopsies obtained. An erally reserved for patients who are not candidates
esophageal manometry performed revealed the for the more durable invasive options (myotomy
following findings shown below: or pneumatic dilation). The first-step of deter-
Chapter 10 — Gastrointestinal motility 263
mining therapy for achalasia should, therefore, heartburn and regurgitation. The esophageal ma-
depend on the patients’ overall health and surgi- nometry revealed the following findings for all the
cal risk. Myotomy, surgical or endoscopic, and test swallows shown below:
pneumatic dilation should be reserved for low-risk
patients, while high-risk candidates should receive What is the most appropriate next step in the
the safer options, including botulinum toxin injec- management of this patient?
tion and pharmacotherapy.
A. Referral for Nissen fundoplication.
REFERENCE B. Schedule botulinum toxin injection to the
Vaezi MF et al. ACG clinical guideline: diagnosis lower esophageal sphincter.
and management of achalasia. Aliment Am J Gas- C. Schedule endoscopic pneumatic dilation.
troenterol. 2013 Aug;108(8):1238-49. D. Lifestyle modification.
E. Start calcium channel blockers daily.
also be performed to assess for signs of major manometry would most likely explain her physical
motility disorders that may be contraindication examination findings?
to anti-reflux surgery or pose high risk for post-
operative dysphagia. Her esophageal manometry A. Squeeze pressure of 70 mmHg.
showed evidence of a hypotensive lower esopha- B. Rectal contraction pressure of 50 mmHg
geal sphincter with no evidence of esophagogastric during strain maneuver.
outflow obstruction, as well as absent esophageal C. Resting pressure of 110 mmHg.
body contraction. These findings are consistent D. Expulsion of 50 mL balloon at 60 seconds.
with absent contractility per Chicago classification E. Relaxation of the internal anal sphincter with
v3.0, and may be suggestive of underlying con- inflation of balloon to 30 mL in the rectum.
nective tissue or smooth muscle disorders such as
scleroderma. Anti-reflux surgery, particularly a CORRECT ANSWER: C
full Nissen fundoplication (answer a), would pose
a high risk of post-operative achalasia in patients RATIONALE
with absent contractility or scleroderma and This patient’s symptoms and physical examination
should generally be avoided. Botulinum toxin in- findings suggest an anal fissure. Chronic anal fis-
jection to the lower esophageal sphincter (answer sures are believed to be caused by decreased per-
b) and pneumatic dilation (answer c) are therapies fusion and relative ischemia to the posterior anal
for achalasia, and they would likely worsen her sphincter. Internal anal sphincter hypertonia is
reflux symptoms. Smooth muscle relaxants (an- thought to contribute to the reduced perfusion to
swer e) are used for treatment of hypercontractile the sphincter, as increased pressure on the vessels
disorders and would further worsen the hypocon- passing perpendicularly through the internal anal
tractility in this patient. Therefore, conservative sphincter muscle may compromise flow. Topical
management with lifestyle modification for reflux smooth muscle relaxants or sphincterotomy aim-
(answer d) would be the most appropriate choice ing at reducing anal sphincter tone are, therefore,
for treatment of this patient. the major modalities of treatment for chronic anal
fissure. Internal anal sphincter tone is measured on
REFERENCES anorectal manometry by the resting anal sphincter
Kahrilas PJ et al. The Chicago Classification of pressure (answer c), which would be the most likely
Esophageal Motility Disorders v3.0. Neurogastro- finding in this patient with anal fissure. The squeeze
enteral Motil 2015;27(2):160-174. pressure is a measure of the contractility of the
Katz PO et al. Diagnosis and management of external anal sphincter (answer a). While defecatory
gastroesophageal reflux disease. Am J Gastroen- function may also be impaired (answers b and d) in
terol 2013;108:308-28. some patients with anal fissure, it is not the primary
underlying pathophysiology of the injury. Patients
with Hirschsprung’s disease (answer e) may also
Question 11 develop anal fissures, but it would not be the most
A 55-year-old female with a history of recurrent common finding expected in these patients.
hematochezia, rectal pain, and chronic constipa-
tion presents for anorectal manometry. A recent REFERENCES
colonoscopy was unremarkable. Perianal examina- Van Koughnett JA et al. Anorectal physiology
tion prior to the start of the manometry revealed and testing. Gastroenterol Clin North Am 2013
an ulcer-like longitudinal tear at the posterior Dec;42(4):713-28.
midline region of her anal canal. Zaghiyan KN et al. Anal Fissure. Clin Colon
Rectal Surg 2011;24(1):22-30.
Which of the following results on her anorectal
Chapter 10 — Gastrointestinal motility 265
of the study (four in ascending, five in trans- slow transit constipation, upper gastrointestinal
verse, six in descending, and four in rectosig- motility assessment (e.g. gastric emptying scin-
moid). She inquired about surgical options for tigraphy, fluoroscopic upper GI series, wireless
treatment of her chronic constipation. motility capsule) and evaluation of defecatory
function (anorectal manometry) should be per-
Which of the following would be a negative pre- formed prior to surgery.
dictor of surgical outcome?
REFERENCES
A. Decreased anal sphincter squeeze pressure Kim CR et al. How to interpret a functional or a
on anorectal manometry. motility test-colon transit study. J Neurogastro-
B. Increased small intestinal transit time on enterol Motil 2012;18:94-9.
wireless motility capsule study. Bharucha AE et al. American Gastroentero-
C. Expulsion of 50-mL balloon in 30 seconds. logical Association Medical Position Statement
D. Retained radiopaque markers in the on Chronic Constipation. Gastroenterology
ascending and transverse colon. 2013;144:211–217.
E. Melanosis coli on colonoscopy.
prandial pain, early satiety, fullness, as well as A. Hypercontractile swallows (DCI greater
retained food in stomach after prolonged fast than 8000 mmHg*s*cm) in 40 percent of
and delayed emptying on scintigraphy. Pain- test swallows.
predominant symptoms are predictors for B. Decreased distal latency in 60 percent of
more suboptimal response in gastric electrical test swallows.
stimulation (answer a), as is idiopathic gastro- C. Increased integrated residual pressure (IRP)
paresis, compared to diabetic and post-surgical and no esophageal body contraction in all
gastroparesis. While erythromycin (answer b) swallows.
is effective as a pro-motility agent, chronic use D. Failed swallows (DCI greater than 100
of an antibiotic would not be appropriate in this mmHg*s*cm) in 30 percent and weak
patient with a history of recurrent C difficile swallows (DCI 250 to 350 mmHg*s*cm) in
infection. Development of tardive dyskinesia is 40 percent of swallows.
an important potential adverse effect for the use E. Increased integrated residual pressure and
of metoclopramide (answer d), and the risk is pan-esophageal pressurization greater than
higher in patients with a pre-existing movement 30 mmHg in all swallows.
disorder, such as Parkinson’s. Dietary modifica-
tion is an important component of gastropare- CORRECT ANSWER: D
sis management, especially as part of first-line
therapy. Small meals low in fiber and fat are the RATIONALE
mainstay of diet for gastroparesis, as fiber and In patients presenting with non-cardiac chest
fat would slow gastric emptying. pain, the most common etiologies are gastro-
esophageal reflux disease (29 to 60 percent)
REFERENCES and functional chest pain (32 to 60 percent). A
Camilleri M et al. Clinical guideline: Manage- major primary esophageal motor disorder only
ment of gastroparesis. AM J Gastroenterol accounts for a minority of cases of non-cardiac
2013;108:18-37. chest pain (10 to 18 percent). In this over-
Rao SS et al. Metoclopramide and tardive dys- weight patient with a hiatal hernia, underlying
kinesia Aliment Pharmacol Ther 2010;31:11-19. gastroesophageal reflux disease would be the
most likely etiology of his chest pain. Ineffective
esophageal motility (answer d) is the esophageal
Question 15 manometry diagnosis most commonly associat-
A 65-year-old overweight man presents for eval- ed with gastroesophageal reflux disease. Jack-
uation of intermittent chest pain and tightness. hammer esophagus (answer a), diffuse esopha-
His symptoms are most frequent after eating, geal spam (answer b), type I achalasia (answer
although they may occur at any time of the day. c), and type II achalasia (answer e) are major
Cardiac evaluations including a stress test per- primary esophageal motor disorders that are less
formed were unremarkable. He completed an common causes of chest pain than gastroesopha-
upper endoscopy with esophageal biopsies that geal reflux disease.
were normal. A barium esophagram performed
revealed a normal caliber esophagus and medi- REFERENCES
um-sized hiatal hernia, but no other abnormali- Fass R et al. Non-cardiac chest pain: an update.
ties. He was then referred for further evaluation Neurogastroenterol Motil 2006 Jun;18(6):4
with esophageal function testing. 08-17.
Barret M et al. Diagnostic yield of 24-hour
Which of the following would be the most likely esophageal manometry in non-cardiac chest
finding on esophageal manometry? pain 2016 Aug;28(8):1186-93.
268 Digestive Diseases Self-Education Program®
Question 16 Question 17
Which of the following statements regarding A 67-year-old man with peptic ulcer disease
normal defecation and continence is correct? status post recent antrectomy with Billroth I
presents with post-prandial fullness, nausea and
A. Rectal distention results in relaxation of the vomiting. His symptoms usually occur about 30
external anal sphincter. minutes after meals and may last for an hour.
B. Normal defecation requires an increase in He also reports associated abdominal cramps
intraabdominal pressure and contraction of and diarrhea, as well as sweating and palpitation
the puborectalis muscle. during these episodes. He underwent an upper
C. During evacuation, the pelvic floor descends endoscopy that revealed a healthy-appearing
and the anorectal angle decreases. anastomosis.
D. Straining or “bearing down” during defecation
results in relaxation of the internal anal Which of the following is the most appropriate
sphincter. next step in management?
E. Phasic contractions of the internal anal
sphincter during cough maintain normal A. Octreotide LAR 20 mg monthly.
continence of feces. B. Initiation of a low fiber, low residual diet.
C. Ensure adequate liquid intake during meals.
CORRECT ANSWER: D D. Increase simple sugars and avoid complex
carbohydrates intake.
RATIONALE E. Domperidone 15 mg before meals.
When straining during normal defecation, the in-
ternal anal sphincter relaxes to allow opening of the CORRECT ANSWER: A
anal canal for passage of stools (answer d). When
fecal contents reach the rectal vault, distention of the RATIONALE
rectum results in the relaxation of the internal anal Dumping syndrome is a potential complication
sphincter (rectoanal inhibitory reflux, or RAIR), not of gastric surgery resulting from rapid emptying
external anal sphincter (answer a), which actually of hyperosmolar stomach content into the small
contracts to help maintain continence. Normal def- bowel. Early dumping symptoms are character-
ecation requires an increase in intraabdominal pres- ized by post-prandial nausea, vomiting, fullness,
sure with simultaneous relaxation, not contraction abdominal cramps, and diarrhea, with associ-
(answer b), of the puborectalis muscle, which forms ated diaphoresis, lightheadedness, palpitation,
a posterior “sling” around the rectum. The descent and flushing. Treatment of dumping syndrome
of the pelvic floor and relaxation of the puborectalis involves mainly dietary changes, including eat-
muscle during evacuation result in an increase, not ing multiple, small meals, avoiding simple sugars,
decrease (answer c), of the anorectal angle. This increasing dietary fat and fiber, and separating
leads to straightening of the rectum to allow passage ingestion of liquids from solids during meals (i.e.
of stools. During cough or other maneuvers that no liquids 30 minutes before and after meals).
increase intraabdominal pressure, phasic contrac- Answers b, c, and d are, therefore, the opposite
tions of the external, not internal (answer e), anal of the recommended diet. Domperidone (answer
sphincter helps maintain continence. e) is a promotility agent used for gastroparesis.
Octreotide, either administered in short-acting
REFERENCE form before meals or in long-acting form monthly
Rao, S.S.C., et al., Anorectal Disorders. Gastroen- (answer a), has been shown to reduce dumping
terology, 2016. 150(6): p. 1430-1442.e4. symptoms.
Chapter 10 — Gastrointestinal motility 269
(including achalasia and EGJOO), major disorders having the poorest outcome. Graded pneumatic
of peristalsis, and minor disorders of peristalsis. dilations and surgical myotomy have been found
Major disorders (including diffuse esophageal to have similar treatment response at five year
spasm [answer c], Jackhammer esophagus [an- follow-up in the European Achalasia Trial, the
swers a and e], absent contractility [answer b]) are largest randomized controlled trial for treatment
nearly universally symptomatic and not found in of achalasia to-date. In the study, ~25 percent of
normal, asymptomatic subjects. Minor disorders patients in the pneumatic dilation arm required
(including ineffective esophageal motility and frag- more than one dilation. Intrasphincteric injec-
mented peristalsis [answer d]) may sometimes be tion of botulinum toxin to the lower esophageal
associated with symptoms, but they are also found sphincter has good initial response that decreases
incidentally in healthy, asymptomatic individuals. with time, with ~40 percent recurrence within
one year. However, the treatment efficacy has
REFERENCE been found to decrease with subsequent injec-
Kahrilas PJ et al. The Chicago Classification of tions (answer e). Therefore, for patients with low
Esophageal Motility Disorders v3.0. Neurogastro- surgical risk, pneumatic dilation and myotomy
enteral Motil 2015;27(2):160-174. should be first-line options. Botulinum toxin
injection should be reserved for those who are at
high surgical risk and are thus poor candidates
Question 20 for myotomy or pneumatic dilation. Pharma-
Which of the following statements regarding treat- cotherapy has limited efficacy and is inferior to
ment for achalasia is true? endoscopic or surgical therapies. It should be
reserved for patients who are not candidates for
A. Patients with type I achalasia have a higher other treatments.
rate of response to all treatments than
patients with type II achalasia. REFERENCES
B. Heller myotomy has been shown to have a Moonen A et al. Long-term results of the Euro-
higher sustained response rate at five year pean achalasia trial: a multicentre randomized
follow-up than graded pneumatic dilations in controlled trial comparing pneumatic dilation
randomized clinical trial. versus laparoscopic Heller myotomy. Gut. 2016
C. Intrasphincteric injection of botulinum toxin May;65(5):732-9.
to the lower esophageal sphincter should be Kahrilas PJ et al. Expert consensus document:
used as a first-line therapy prior to consid- Advances in the management of oesophageal mo-
eration for myotomy or dilation for all tility disorders in the era of high-resolution ma-
achalasia patients. nometry: a focus on achalasia syndromes. Nat Rev
D. Pharmacotherapy with calcium channel Gastroenterol Hepatol. 2017 Nov;14(11):677-688.
blockers or nitrates has similar initial efficacy
as endoscopic therapies.
E. The efficacy of intrasphincteric botulinum Question 21
toxin injection decreases with subsequent A 28-year-old woman presents for evaluation of
repeat injections. post-prandial fullness, early satiety, and epi-
gastric pain. These symptoms occur 15 to 30
CORRECT ANSWER: E minutes after initiation of a meal. She denies
any heartburn, reflux, regurgitation, chest pain,
RATIONALE or vomiting. She underwent evaluations includ-
Treatment efficacy for achalasia overall is high- ing an upper endoscopy that was unremarkable.
est with type II followed by type I, with type III Gastric biopsies obtained were also normal.
Chapter 10 — Gastrointestinal motility 271
sified as having functional heartburn. Antireflux D. Decreased anal sphincter resting pressure.
surgeries (answers a and b) would not be appro- E. Increased anal sphincter pressure during
priate for treatment of functional heartburn. In strain or bear-down maneuver.
addition, IEM is currently an exclusion criterion
for the use of magnetic sphincter augmentation CORRECT ANSWER: A
(LINX). Baclofen (answer c) has been shown to
increase basal lower esophageal sphincter pres- RATIONALE
sure and decrease transient lower esophageal Anal sphincter squeeze pressure measures the vol-
sphincter relaxation, and may play a role as an untary contraction of the external anal sphincter
anti-reflux agent. However, it would likely have (EAS), while the anal sphincter resting pressure
no effect in functional heartburn. Sucralfate reflects the tonically active internal anal sphinc-
(answer e) has not been found to provide signifi- ter (IAS). The EAS reflexively contracts during
cant benefits in the treatment of gastroesophageal actions associated with increased intraabdominal
reflux disease (erosive or nonerosive) or functional pressure (e.g. coughing, laughing) to maintain
heartburn. Tricyclic antidepressants such as nor- continence. It is also under voluntary control to
triptyline (answer d) have been shown to provide “hold” when an urge for defecation is felt. The
benefits for functional gastrointestinal disorders, patient in question suffers from symptoms sug-
including functional heartburn. gestive of urge incontinence, which is likely due
to weakness in the EAS. Therefore, her anorectal
REFERENCES manometry would likely show a decrease in anal
Gyawali CP et al. Modern diagnosis of GERD: the sphincter squeeze pressure (answer a). Weak-
Lyon Consensus. Gut 2018 Jul;67(7):1351-1362. ness in the IAS or resting tone, characterized by
Hachem C et al. Diagnosis and management decreased anal sphincter resting pressure (answer
of functional heartburn. Am J Gastroenterol 2016 d), would result in passive stool leakage without
Jan: 111(1):53-61. sensation. Hyposensitivity of the rectum (answer
b), Hirschsprung’s disease (answer c), and dys-
synergic defecation (answer e) are associated with
Question 23 stool impaction and overflow incontinence, also
A 54-year-old woman presents for evaluation of fecal characterized by silent, passive leakage of stools.
incontinence. She describes a sudden urge for bowel
movements, but difficulty holding long enough to REFERENCE
get to the bathroom, resulting in leakage of stools. Carrington EV et al. Expert consensus document:
She also reports leakage of a small amount of fecal Advances in the evaluation of anorectal function. Nat
material when she coughs and laughs. She denies Rev Gastroenterol Hepatol 2018 May;15(5):309-323.
any silent seepage or soiling of underwear. A flexible
sigmoidoscopy performed did not reveal any ab-
normalities within the rectum. She then underwent Question 24
anorectal manometry for further evaluation. Which of the following statements regarding gas-
tric and small intestinal motility is true?
Which of the following is the most likely finding
on anorectal manometry for this patient? A. Segmentation occurs during fed state
following a meal with propagated contractions
A. Decreased anal sphincter squeeze pressure. of the intestine aborally.
B. Increased rectal sensation volume. B. The fasting state is characterized by the
C. No change in anal sphincter pressure with migrating motor complex (MMC) that cycles
distention of balloon in rectum. every 90-120 minutes.
Chapter 10 — Gastrointestinal motility 273
C. Peristalsis of the small intestine involves of the following medications are appropriate for
activation of acetylcholine-containing motor management of his constipation, EXCEPT:
neurons distal to the bolus of chyme.
D. Activation of nitric oxide-containing motor A. Naloxegol (selective mu-opioid receptor
neurons proximal to the bolus of chyme helps antagonist)
propulsion of the bolus during peristalsis. B. Methylnaltrexone (peripheral mu-opioid
E. Carbohydrate content in the ileum activates receptor antagonist and delta-opioid receptor
the chemoreceptors and the “ileal brake” antagonist)
which delays gastric emptying. C. Eluxadoline (mu- and kappa-opioid receptor
agonist and delta-opioid receptor antagonist)
CORRECT ANSWER: B D. Lubiprostone (ClC-2 chloride channel agonist)
E. Naldemedine (peripherally selective mu-
RATIONALE opioid receptor antagonist)
The fasting motility pattern of the small intestinal
involves migrating motor complex (MMC) trav- CORRECT ANSWER: C
eling from the proximal to distal small bowel in
90-120-minute cycles (answer b). The fed state is RATIONALE
characterized by two processes: (1) segmentation, Pharmacotherapy for opioid-induced constipation
which is a localized contraction at multiple levels involves the use of traditional laxatives as first-line
simultaneously, with no net forward movement of agents, including lubiprostone, a ClC-2 agonist.
chyme to allow chopping and mixing with diges- If traditional laxatives are ineffective, further
tive enzymes, and (2) peristalsis, which is a slow, management would include several peripherally-
coordinated contraction that results in forward acting mu-opioid receptor antagonists (naldeme-
propulsion of chyme along the intestine. Peristal- dine, naloxegol, and methylnatrexone). These
sis involves activation of excitatory motor neurons agents selectively inhibit the mu-opioid receptors
(acetylcholine or substance P) proximal to the in the gastrointestinal tract, thereby restoring
bolus of chyme and inhibitory motor neurons (ni- function of the enteric nervous system and gut
tric oxide or VIP) distal to the bolus. This allows motility. They do not enter the central nervous
propulsion of the bolus aborally. The “ileal brake” system, thereby preserving the analgesic effects of
is triggered by the presence of fat content in the the opioid. Eluxadoline (answer c) is a mu-opioid
ileum, delaying gastric emptying. receptor agonist, and would therefore decrease gut
motility. It is primarily approved for treatment of
REFERENCE diarrhea-predominant irritable bowel syndrome,
Schemann M. Control of gastrointestinal motility and would not be appropriate for management of
by the “gut brain” – the enteric nervous system. constipation.
J Pediatr Gastroenterol Nutr 2005 Sep;41 Suppl
1:S4-6. REFERENCE
Crokett SD et al. American Gastroenterological
Association Institute Guideline on the Medical
Question 25 Management of Opioid-Induced Constipation.
A 65-year-old man with chronic pancreatitis on Gastroenterology 2018 Epub ahead of print.
daily opioids presents for management of chronic
constipation. His symptoms have been worsening
over the past year, now with bowel movements Question 26
once per week that require significant straining. A 45-year-old female with a three-year history of
He describes his stools as hard and lumpy. All type 2 diabetes mellitus was seen in GI clinic re-
274 Digestive Diseases Self-Education Program®
porting post prandial fullness, early satiety, nausea A. Improve symptoms severity and gastric
and epigastric discomfort. Her blood sugar levels emptying in diabetic gastroparesis.
were erratic and recent HgA1c was 10.1. Physical B. Improve symptoms in post-surgical gastroparesis.
examination was consistent with peripheral neu- C. Improve symptoms and gastric emptying in
ropathy. An upper endoscopy was unremarkable idiopathic gastroparesis
except mild gastritis. A 4-hour gastric emptying D. Improve gastric emptying in patients with
study revealed 26 percent meal retention. opiate induced gastroparesis
Question 29 Question 30
A 45-year-old male was referred to you after a A patient was seen for primarily GERD related
comprehensive workup for chest pain that was un- symptoms that awaken her from sleep during the
revealing. The patient was evaluated by a cardiolo- night. Which of the following statements is correct
gist, underwent an upper endoscopy with biopsies, regarding nighttime reflux?
wireless pH capsule and a PPI trial without an
identifiable cause or improvement in symptoms. A A. There are less reflux events during sleep time.
physical examination in your office was unremark- B. Short arousals due to reflux commonly trigger
able. You referred the patient for high resolution symptoms
esophageal manometry. C. Patients with daytime symptoms have similar
risk for Barrett’s esophagus and adenocarci-
What would likely be the finding in this clinical noma of the esophagus as those with
scenario? nighttime symptoms.
D. Lower esophageal sphincter pressure is lower
A. Distal esophageal spasm during sleep as compared to awake period
B. Normal esophageal motility
C. Jackhammer esophagus CORRECT ANSWER: A
D. Achalasia
CORRECT ANSWER: B
276 Digestive Diseases Self-Education Program®
RATIONALE
Question 31 Polymyositis affects swallowing by involving the
What is the cause of dysphagia lusoria? striated muscle of the pharynx and esophagus in
up to 50-70 percent of the patients. Diabetes and
A. Enlargement of the left atrium SLE may result in non-specific motor abnormali-
B. Aneurismal thoracic aorta ties of the esophagus. Scleroderma, solely affects
C. Aberrant right subclavian artery the distal 2/3 smooth muscle of the esophagus.
D. Aberrant thoracic duct
REFERENCE
CORRECT ANSWER: C Gasiorowska A et al. Current approach to dyspha-
gia Gastroenterol Hepatol 2009;5:269-79.
RATIONALE
A right aortic arch giving rise to aberrant right
subclavian artery. Patients commonly complain of Question 33
dysphagia for solids associated with regurgitation A 34-year-old male was seen in GI clinic for 10-
of unchewed food, post prandial bloating, chest year history of GERD – related symptoms. The
pain and symptoms that may change with position. patient has received a standard dose of PPI that
fully controlled his symptoms but was unable to
REFERENCE be compliant with daily dosing. A recent upper
Gasiorowska A et al. Current approach to dyspha- endoscopy off treatment was unremarkable. The
gia Gastroenterol Hepatol 2009;5:269-79. patient inquires about anti-reflux surgery as a
long-term solution for his symptoms. Which of
the following patient’s characteristics is a positive
Question 32 predictive factor for successful outcome after anti-
A 51-year-old female was seen for dysphagia for reflux surgery?
both solids and liquids that occurred immediately
after swallowing. The patient reports coughing A. Young age
and choking during meals with occasional nasal B. Being a male
Chapter 10 — Gastrointestinal motility 277
What would be the first therapeutic approach in Which of the following test results prior surgery
this patient? was needed to further understand patient’s cur-
rent clinical status?
A. Pneumatic dilation
B. Nifedipine 10 mg tid A. High resolution esophageal manometry
C. Botulinum toxin injection into the lower B. Wireless pH capsule of PPI treatment
esophageal sphincter C. Impedance +pH test on PPI treatment
D. PPI once daily D. Chest CT
RATIONALE RATIONALE
Using the updated Chicago classification this All patients who are candidates for surgical
NCCP patient has normal high resolution esopha- fundoplication, require an upper endoscopy and
geal manometry. However, consistently through- esophageal manometry prior to surgery. The latter
out the 10 swallows the patient demonstrates a is to exclude major esophageal motor disorders,
hypertensive esophagus (Distal contractile inte- especially achalasia. In addition, if endoscopy off
gral greater than5000mmHg.cm.sec), what used treatment is negative, a pH test, preferably the
to be called nutcracker esophagus. Studies have wireless pH capsule, should be performed off PPI
demonstrated that chest pain associated with treatment to ensure the diagnosis of gastroesopha-
278 Digestive Diseases Self-Education Program®
RATIONALE Question 43
Anorectal manometry may help in diagnos- A 60-year-old female with history of idiopathic
ing Hirschsprung disease, especially in patients gastroparesis was seen in GI clinic, reporting
with ultrashort segment. The presence of normal symptoms of epigastric discomfort, post prandial
recto-anal inhibitory reflex (RAIR) excludes the fullness and nausea despite lifestyle modifications,
disease and lack of it suggests the presence of metoclopramide 10 mg TID and ondansetron PRN
Hirschsprung disease (positive predictive value of for the last two years. During her clinic visit the
up to 95 percent). physician noted that the patient repeatedly licking
and smacking her lips.
REFERENCE
Heuckeroth RO Hirschsprung disease – integrat- Which of the following is true about this
ing basic science and clinical Medicine to im- phenomenon?
prove outcomes. Nat Rev Gastroenterol Hepatol
2018;15:152-167. A. Drug holidays may decrease the prevalence of
this side effect
B. Cessation of metoclopramide will result in
Question 42 resolution of this phenomenon
Which of the following statements about gastropa- C. It is more common in younger patients with
resis is accurate? greater than three months consumption
D. Adding a PPI to metoclopramide increases the
A. Diabetes is currently the most common cause. risk by two-fold for this adverse event
B. Overall, severity of symptoms has been
correlated with degree of gastric emptying. CORRECT ANSWER: A
C. Early satiety and postprandial fullness.
correlate better with gastric emptying than RATIONALE
abdominal pain and bloating. The patient demonstrates tardive dyskinesia due
D. Scintigraphic gastric emptying of solids to long term use of metoclopramide. The risk of
should not be done in a diabetic patient with developing tardive dyskinesia increases with the
blood glucose of 190 mg/dl. duration of treatment and the total cumulative
Chapter 10 — Gastrointestinal motility 281
dose. Cessation of metoclopramide may not always Harvey MA et al. Obstetrical Anal Sphincter Inju-
result in the resolution of tardive dyskinesia. Con- ries (OASIS): Prevention, Recognition, and Repair
comitant use of PPI’s does not increase the risk of J Obstet Gynaecol Can 2015;37(12):1131–1148.
metoclopramide induced tardive dyskinesia.
REFERENCE Question 45
Rao SS et al. Metoclopramide and tardive dyskine- You were called to see a 74-year-old male, status
sia Aliment Pharmacol Ther 2010;31:11-19. post hip replacement, who developed abdominal
distension. The patient is alert and oriented but
reports some abdominal discomfort. On physical
Question 44 examination the abdomen is distended, tympanic
A 54-year-old female with history of mixed con- but positive bowel sounds. Abdominal CT revealed
nective tissue disorder (lupus and scleroderma) proximal colonic dilation (11cm) with an intermedi-
was seen because of fecal incontinence for the last ate transitional zone adjacent to the splenic flexure.
eight months. The patient reports finding smears
and sometimes fecal particles on her underwear, What would be your first therapeutic intervention?
especially during nighttime. The patient had two
normal deliveries that required episiotomies. An A. Immediately administer neostigmine 2 mg by
anorectal manometry was ordered. slow intravenous injection
B. Surgical cecostomy
Which of the following manometric abnormalities C. Nasogastric tube attached to suction
do you expect to find in this patient? D. Colonoscopic decompression
A. Short bowel syndrome the small bowel and the colon. In addition, the
B. Candy cane Roux syndrome frequency changes from one section of the small
C. Dumping syndrome bowel or colon to the other.
D. Stomal ulcer
REFERENCE
CORRECT ANSWER: C Blair PJ et al. The significance of interstitial cells
in Neurogastroenterology. J Neurogastroenterol
RATIONALE Motil 2014;20:294-317.
Early dumping syndrome is a complication seen
in up to 50 percent of post-gastric bypass pa-
tients especially after consuming food that are Question 48
rich with high levels of simple carbohydrates. The A 54-year-old male reported dysphagia for both
syndrome is the result of rapid emptying of this solids and liquids for the last three years that
type of food into the small bowel. The hyper- progressively worsened. The patient also reports,
osmolar content of the small bowel results in a regurgitation of food particles, heartburn, epi-
rapid shift of fluids from the plasma and into the sodes of severe chest discomfort and the need to
small bowel lumen leading to hypotension and sleep in a chair. An upper endoscopy revealed
sympathetic response. puckered esophagogastric junction with resistance
when traversing to the stomach. High resolution
REFERENCE esophageal manometry revealed an integrated
Neilsen JB et al. Prevalence, severity and predic- relaxation pressure of 34 mmHg, aperistalsis with
tors of symptoms of dumping and hypoglycemia 40 percent simultaneous contractions (see figure
after Roux-eny gastric bypass. Surg Obes Relat Dis on the following page).
2016;12:1562-68.
Which of the following interventions would you
recommend to this patient?
Question 47
Which of the following statements is true about A. Pneumatic dilation
slow waves or basic electrical rhythm (BER)? B. Botulinum toxin injection
C. Peroral endoscopic myotomy (POEM)
A. The source is a pacemaker on the greater D. Sildenafil 50 mg
curvature of the stomach
B. Their frequency throughout the GI tract is CORRECT ANSWER: C
three per minute
C. They are generated by the interstitial cells RATIONALE
of Cajal (ICC) The high resolution esophageal manometry is
D. They are absent in the colon consistent with achalasia type 3. Treatment of this
type of achalasia with pneumatic dilation, botu-
CORRECT ANSWER: C linum toxin or medications have been relatively
disappointing. In the last few years, POEM has
RATIONALE emerged as the leading and most efficacious thera-
The source of the slow wave or basic electrical peutic approach to achalasia type 3.
rhythm (BER) is the interstitial cells of Cajal,
which are spread throughout the gastrointestinal REFERENCE
tract from the esophagus down the rectum. The Patel DA et al. An overview of achalasia and its sub-
frequency of BER changes from the stomach to types. Gastroenterol Hepatol 2017;13:411-421.
Chapter 10 — Gastrointestinal motility 283
Question 49 REFERENCE
A 54-year-old female with known scleroderma Crowell MD et al. Esophageal motor abnormalities
was seen in GI clinic for severe GERD-related in patients with scleroderma: Heterogeneity, risk
symptoms and dysphagia for solids and liquids. factors and effects on quality of life. Clin Gastro-
Her GERD required double dose PPI for full enterol Hepatol 2017;15:207-213.
symptom control, but the dysphagia remained
unchanged. An upper endoscopy on PPI treatment
was unremarkable except a three cm hiatal hernia. Question 50
The patient underwent high resolution esophageal A 42-year-old female with history of documented
manometry revealing integrated relaxation pres- Grade C erosive esophagitis on an upper endosco-
sure of 5 mmHg, 90 percent failed swallows and py, was recently seen because of continued heart-
10 percent weak swallows. What is the diagnosis? burn despite double dose PPI. A recent repeat
upper endoscopy with biopsies on PPI treatment
A. Absent peristalsis was unremarkable. The patient reports almost
B. Ineffective esophageal motility daily symptoms, primarily during daytime. She
C. Achalasia type 1 has tried antacids and over the counter H2 block-
D. Fragmented peristalsis ers with no improvement. An impedance +pH test
on PPI treatment revealed normal esophageal acid
CORRECT ANSWER: B exposure and normal nonacidic reflux. Symptom
index and symptom association probability were
RATIONALE 50 percent and 98 percent for weakly acidic reflux
The patient’s high resolution esophageal manometry and heartburn, respectively. High resolution
is consistent with ineffective esophageal motil- esophageal manometry was unremarkable. What
ity. Scleroderma patients will demonstrate various is your diagnosis?
esophageal motor abnormalities which may progress
overtime and become more severe. The question A. Nonerosive reflux disease (NERD)
also demonstrates the wide range of esophageal mo- B. Reflux hypersensitivity overlapping with
tor changes that fall under the category of ineffective GERD
esophageal motility (from 50 percent weak swallows C. Functional heartburn overlapping with GERD
to 90 percent failed swallows). D. Hypersensitive esophagus
284 Digestive Diseases Self-Education Program®
REFERENCE
Question 51 Stanghellini et al. Gastroduodenal disorders. Gas-
A 28-year-old woman presents for consultation re- troenterology. 2016;150:1380-92.
garding a longstanding history of abdominal pain,
described as epigastric in location, with associ-
ated symptoms of bloating, belching and nausea, Question 52
oftentimes precipitated by foods but present even A 35-year-old woman is referred to gastroenterol-
when fasting, and often severe enough for her to ogy clinic due to longstanding constipation. She
miss multiple days of work. She denies vomiting, describes her stools as lumpy to hard, with fre-
fevers, heartburn, NSAID use, relationship with quent straining, and reports “there is always more
bowel movements, or pregnancy. Her extensive stool that {she} can’t push out.” She feels that
evaluation including bloodwork, pancreaticobiliary she has a blockage in her colon as she can rarely
imaging, upper endoscopy and small bowel imaging have a bowel movement without manually disim-
have been normal. She has taken multiple courses pacting herself, she denies significant bloating or
of antacids, antibiotics, and antispasmodics with- abdominal pain. Her rectal exam reveals normal
out relief. Based on the Rome IV criteria, which of sphincter tone, no palpable masses, but hard stool
the following is the most likely diagnosis? is felt on the tip of the finger. Laboratory evalua-
tion and colonoscopy have been unremarkable.
A. Epigastric pain syndrome
B. Postprandial distress syndrome Which of the following diagnoses is most consis-
C. Rumination syndrome tent with her symptoms?
D. Excessive gastric belching
E. Chronic nausea and vomiting syndrome A. Hirschprung’s disease
B. Colonic pseudo-obstruction
CORRECT ANSWER: A C. Functional constipation
D. Irritable bowel syndrome
RATIONALE E. Anorectal stricture
The Rome IV criteria have subcategorized func-
tional dyspepsia into two conditions – Epigas- CORRECT ANSWER: C
Chapter 10 — Gastrointestinal motility 285
REFERENCE
Rao SS, Seaton K, Miller M, et al. Randomized
controlled trial of biofeedback, sham feedback,
and standard therapy for dyssynergic defecation.
Clin Gastroenterol Hepatol. 2007; 5: 331-338.
Question 56
A 63-year-old female presents with a recent
change in bowel habits. She previously had one
formed bowel movement a day, but now has diar-
rhea three to four times a day with incontinence.
She had prior normal colonoscopy three years ago.
A. CT scan
B. Colonoscopy to evaluate for inflammation
Answers & critiques
CHAPTER 11
GI infections of the
small intestine and colon
Tauseef Ali, MD, FACP, FACG, AGAF and John C. Sun, MD
287
288 Digestive Diseases Self-Education Program®
improperly frozen raw salmon. Anisakis species reports a history of travel to Southeast Asia five
are parasitic nematodes found in different types of months ago, where he participated in volunteer
seafood, including salmon, herring, cod, mackerel missionary work in the countryside. He returned to
and squid. About 95 percent of cases of anisakiasis the United States four months ago, and developed
involve the stomach, while the remainder mainly a cough associated with mild shortness of breath
includes intestinal disease. Gastric anisakiasis which he attributed to a viral respiratory infection,
may result in epigastric pain, nausea and vomiting as it resolved shortly thereafter. He reports no
within a few hours after ingestion of the parasite. other medical conditions, and is on no other medi-
Intestinal involvement may present with colicky cations. His physical examination is significant for
abdominal pain, nausea, vomiting or obstructive moderate periumbilical tenderness. His laboratory
symptoms, usually within five days. Diagnosis of results are significant for an eosinophil count of 20
gastric anisakiasis is confirmed by visualization of percent. A CT of the abdomen and pelvis showed
the parasite during EGD, and is treated by worm small bowel dilation proximal to a whirlpool-like
extraction during upper endoscopy. The FDA mass in the mid-jejunum. What is the most likely
recommends freezing fish at a temperature below diagnosis?
-35°C for 15 hours or under 20℃ for seven days
prior to raw or semi-raw consumption. Although A. Clonorchis sinensis
rare, this condition is increasing in frequency in B. Ascaris lumbricoides
the Western hemisphere. Scromboid poisoning is C. Trichuris trichiura
due to buildup of histamine in improperly refriger- D. Cystoisospora belli
ated fish, and is associated with development of E. Taenia saginata
rashes and flushing. Ciguatera poisoning is caused
by accumulation of dinoflagellate toxins in large CORRECT ANSWER: B
predatory fish. It is associated with disturbances
of temperature, taste and paresthesias, which are RATIONALE
not present in this case. D. latum infection may Ascaris lumbricoides is prevalent in Asia, Africa,
occur with freshwater fish consumption, and typi- South America, and the Southeastern United
cally manifests as vitamin B12 deficiency. Inges- States, and transmitted via food or water contami-
tion of preformed toxin may result in nausea and nated by A. lumbricoides or A. suum eggs. In the
vomiting a few hours after exposure, but the other early stage, which occurs one to two weeks after
features of the case favor anisakiasis. infection, the larvae pass through the lungs, and re-
sult in Loeffler’s syndrome, characterized by cough-
REFERENCE ing, wheezing and shortness of breath; hemoptysis
Shimamura Y, Muwanwella N, Chandran S, et. al. or pleural effusions may also rarely occur. Gas-
Common Symptoms from an Uncommon Infection: trointestinal symptoms begin roughly two months
Gastrointestinal Anisakiasis. Canadian Journal of after infection, and may be characterized by biliary
Gastroenterology and Hepatology. Volume 2016, manifestations such as pancreatitis, acute cholecys-
Article ID 5176502, 7 pages. titis, acute cholangitis, or luminal symptoms such
as abdominal pain, nausea, vomiting, intestinal
obstruction, and acute abdomen. Diagnosis of as-
Question 7 cariasis relies on identification of the worm during
A 45-year-old man presents to the emergency endoscopy or ERCP, eggs passed in feces, or larvae
department with nausea, vomiting and abdomi- on pulmonary specimens. Treatment of ascariasis
nal pain. He states that the symptoms are worse is with albendazole 400 mg orally for one dose,
with eating, and that his abdominal pain has been mebendazole 500 mg orally for one dose, or 100 mg
increasing over the past three to four months. He twice daily for three days.
292 Digestive Diseases Self-Education Program®
The gastrointestinal and cardiovascular symptoms B. Vancomycin 125 mg mg orally four times daily
start within one to 12 hours after exposure, and for 10 days
tend to abate within a few days, but the neurologic C. Fecal microbiota transplantation
symptoms may last for weeks. Treatment is largely D. Vancomycin 500 mg orally four times daily for
supportive, and prevention is aimed at avoiding 10 days
ingestion of reef fish. E. Fidaxomicin 200 mg orally twice daily for
Anisakiasis is caused by a parasitic nematode seven days
present in under-frozen fish. Scromboid fish
poisoning tends to resolve within 24 hours. C. CORRECT ANSWER: B
perfringens toxin ingestion causes mainly diarrhea
and abdominal pain. S. aureus preformed toxin
ingestion causes nausea, vomiting and abdominal Question 12
pain. None of the above four conditions result in The patient in the previous question received the
significant neurologic symptoms. appropriate therapy and did well. However, two
months later, he returns with similar complaints.
REFERENCE He reports five episodes of watery stools a day,
Dickey RW, Plakas SM. Ciguatera: A public health without fevers, or abdominal pain. He remains
perspective. Toxicon 2010; 56:123-136 MMWR afebrile, with a normal CBC and comprehensive
February 1, 2013. 62(04):61-65. metabolic panel. A C. difficile PCR is again posi-
tive. Which of the following is appropriate treat-
ment at this time?
Question 11
A 55-year-old man presents with diarrhea. He A. Vancomycin 125 mg orally four times daily
reports four to five loose to watery stools a day for 10 days
that began about three days ago, with mild lower B. Fidaxomicin 200 mg orally twice daily for
abdominal cramping. He denies any nausea or 30 days
vomiting, gastrointestinal bleeding, fevers, chills or C. Vancomycin 125 mg orally four times daily for
abdominal distension. He reports an initial episode 10 days, followed by rifaximin 400 mg orally
of C. difficile infection six weeks ago, presumably three times daily for 20 days
from antibiotics he received for a strep throat infec- D. Initiation of Saccharomyces boulardii therapy
tion. The initial episode was successfully treated E. Metronidazole 500 mg orally three times daily
with a 10 day course of metronidazole 500 mg for 10 days
orally three times daily, and he has been well until
recently. His past medical history is significant for CORRECT ANSWER: C
hypercholesterolemia for which he takes simv-
astatin. On examination, he is afebrile, in sinus
rhythm and normotensive. His minimally tender Question 13
in the lower abdomen. His CBC, and comprehen- The patient in the previous two questions is treated
sive metabolic panel are within normal limits. A for his recurrent CDI successfully. He returns to
C. difficile PCR sent by his primary care provider is clinic in three months for a routine follow up visit.
positive, consistent with C. difficile infection (CDI). He reports having one formed bowel movement
daily, without abdominal pain. He denies any
What is the next appropriate step in therapy? fevers, chills, nausea, vomiting, or other systemic or
gastrointestinal symptoms. He has not had any ad-
A. Metronidazole 500 mg orally three times daily ditional antimicrobial use. What is the appropriate
for 10 days management for this patient?
Chapter 11 — GI infections of the small intestine and colon 295
A. Repeat C. difficile PCR testing for confirmation In the first question, the patient presented with
of cure. his first recurrence after successful metronidazole
B. Proceed with fecal microbiota transplantation therapy. The appropriate choice is to treat his re-
to reduce risk of relapse. currence with 10 days of vancomycin 125 mg orally
C. Initiate long-term suppressive therapy with four times daily for 10 days over retreatment with
vancomycin. metronidazole. FMT is not appropriate for an ini-
D. Inform the patient that he should be empirically tial recurrence. Fidaxomicin 200 mg orally twice
treated with vancomycin 125 mg orally four daily for 10 days (not seven days as in the ques-
times daily should he require any antibiotic tion) would be appropriate if the patient received
therapy in the future. vancomycin therapy for his initial episode.
E. Monitor patient conservatively for CDI For the second question, C would be appro-
recurrence. priate. Retreatment with a standard course of
vancomycin at 125 mg orally four times a day for
CORRECT ANSWER: E 10 days, treatment with metronidazole, treatment
with a prolonged course of fidaxomicin, or Saccha-
RATIONALE romyces boulardii, would not be indicated.
Clostridium difficile infection (CDI) has a recurrence For the third question, repeat C. difficile test-
rate of up to 25 percent after the initial episode, and ing for confirmation of cure is not recommended
after the first recurrence, additional recurrence rates in either the 2013 ACG or 2017 IDSA / SHEA
range from 40-65 percent. The 2017 IDSA / SHEA guidelines. Only patients with diarrhea (2017
guidelines address treatment of recurrent CDI. IDSA / SHEA guidelines recommend three or
more unformed stools per day) should be tested
First recurrence: for C. difficile (an exception is in patients with
If metronidazole was used for the initial episode, ileus). In addition, C. difficile testing may remain
treat the first recurrence (second episode) with van- positive for a period of time, and these positive
comycin 125 mg orally four times daily for 10 days. tests in otherwise asymptomatic patients may
This is preferred over treating with a second lead to unnecessary additional CDI therapy. FMT
course of metronidazole. is recommended for the third or greater number
If vancomycin was used for the initial episode, of recurrences that do not respond to antibiotic
treat the first recurrence (second episode) with: therapy. As this patient is now asymptomatic,
Fidaxomicin 200 mg orally twice daily for 10 days, or there is no indication for FMT. There is no de-
prolonged vancomycin tapered and pulsed regimen. fined role for either long-term suppressive therapy
The IDSA / SHEA guidelines suggest vancomycin or empiric treatment for CDI in patients receiving
125 mg orally four times daily for 10-14 days, then antibiotic therapy for other indications, as there
twice daily for one week, then daily for one week, are no large scale randomized controlled trials
then every two to three days for two to eight weeks. addressing these issues. The decision to admin-
ister secondary prophylaxis for recurrent CDI in
Second or subsequent recurrence: patients receiving antibiotics should be individual-
Prolonged vancomycin tapered and pulsed regi- ized. The 2017 IDSA / SHEA guidelines suggest
men, or vancomycin 125 mg orally four times daily daily vancomycin or fidaxomicin for cases where
for 10 days, followed by rifaximin 400 mg orally secondary CDI prophylaxis may be indicated.
three times daily for 20 days, or fidaxomicin 200
mg orally twice daily for 10 days, or fecal micro- REFERENCES
biota transplantation (FMT) if the patient has McDonald LC, Gerding DN, Johnson S, et. al.
had appropriate antibiotic therapy for at least two Clinical Practice Guidelines for Clostridium diffi-
recurrences (three episodes). cile Infection in Adults and Children: 2017 Update
296 Digestive Diseases Self-Education Program®
Management of STEC infection is supportive; and normal differential count; her B12 and folate
aggressive rehydration and monitoring of elec- levels are both low. Stool testing is negative for
trolytes, renal function, and blood counts are the routine culture, C. difficile, giardia and cryptospo-
cornerstones of therapy. Use of antibiotics, anti- ridium. Fecal fat is 12 grams in 24 hours. EGD
motility and narcotic agents have been associated with duodenal biopsies show villous blunting.
with an increased risk of HUS development, and
anti-motility or narcotic agents have been associ- What is the most likely diagnosis?
ated with increased risk of neurologic complica-
tions. There is concern that use of NSAIDs result A. Celiac disease
in decreased renal perfusion so these agents B. Diphyllobothrium latum infection
should be avoided. C. Tropical sprue
The other organisms listed in the first ques- D. Necator americanus infection
tion, listeria, campylobacter, aeromonas and E. Cystoisospora belli infection
salmonella would not cause HUS as noted in this
patient. CORRECT ANSWER: C
REFERENCES RATIONALE
Tarr PI, Gordon CA, Chandler WL. Shiga- Tropical sprue causes chronic diarrhea in patients
toxin producing Escherichia coli and haemo- either from or having traveled to endemic areas
lytic uraemic syndrome. The Lancet 2005; near the equator, such as Puerto Rico, Haiti,
365(9464):1073-1086. Cuba, Southeast Asia and India for at least two
Shane AL, Mody RK, Crump JA, et. al. 2017 weeks to a month. Although no clear etiology
Infectious Diseases Society of America Clinical has been found, data suggest a likely infectious
Practice Guidelines for the Diagnosis and Manage- etiology. Clinically, patients present with mal-
ment of Infection Diarrhea. CID 2017; 65(12):e absorption, steatorrhea, weight loss, and fatigue.
45-e80. Laboratory testing shows anemia, B12 and folate
Riddle MS, Herbert LD, Conner BA. ACG deficiency, as well as increased fecal fat. Biopsies
Clinical Guideline: Diagnosis, Treatment, and of the small bowel during upper endoscopy show
Prevention of Acute Diarrhea Infections in Adults. villous blunting with negative celiac serologies.
Am J Gastroenterol 2016; 111:602-622. Treatment is a three to six month course of tetra-
cycline 250 mg orally four times daily with folate
5 mg orally daily.
Question 19
A 22-year-old caucasian woman presents with REFERENCES
four month history of diarrhea and 20 pound Ghoshal UC, Srivastava D, Verma A, et. al. Tropi-
weight loss upon returning from a three month cal sprue in 2014: the new face of an old disease.
summer trip to Puerto Rico. She has experienced Curr Gastroenterol Rep 2014; 16(6):391-393.
increasing fatigue and shortness of breath during Batheja MJ, Leighton J, Azueta A, et. al. The
this time period, and notes increased flatulence Face of Tropical Sprue in 2010. Case Rep Gastro-
and very loose stools that seem to float. She has enterol 2010; 4:168-172.
no other significant past medical history, tobacco, Jansson-Knodell CL, Hujoel IA, Rubio-Tapia
alcohol or drug use. She states that her mother A, et. al. Not All That Flattens Villi is Celiac Dis-
has hypothyroidism, and a cousin has celiac dis- ease: A Review of Enteropathies. Mayo Clin Proc
ease. On examination, she has pale conjunctiva 2018; 93(4):509-517.
and hyperactive bowel sounds. Her CBC is signifi-
cant for a hemoglobin of 8.1 g/dL, MCV of 112.2 fl
300 Digestive Diseases Self-Education Program®
RATIONALE
Question 22 This patient most likely has Strongyloides sterco-
A 54-year-old man from Kentucky with a history of ralis hyperinfection. S. stercoralis is a nematode
rheumatoid arthritis (RA) is admitted for hypoten- that can complete its entire life cycle within the
sion and E. coli sepsis. He reports that about four human host. The parasite may be acquired orally
months ago, he had an RA flare, and prednisone or percutaneously through the soil. In the latter
was started in addition to his methotrexate regi- case, it burrows through the skin where it leaves
men; more recently, a tumor necrosis factor inhibi- a pruritic rash (larva currens), and travels to the
tor has been added to his treatment regimen. He pulmonary capillaries. From there, the larvae
reports anal pruritis in the recent past associated migrate to the larynx and then to the small intes-
302 Digestive Diseases Self-Education Program®
tine. The pulmonary phase may cause coughing, mission trip to Southeast Asia, and developed five
wheezing, hemoptysis, and the intestinal phase to six watery, non-bloody bowel movements daily,
may be asymptomatic or result in abdominal pain, associated with stool urgency, lower abdominal
diarrhea, nausea or vomiting. Laboratory testing cramping, fatigue, anorexia and subjective fevers.
may reveal eosinophilia. The female worms lodge Due to these symptoms, she has missed work and
in the duodenal or jejunal wall and produce eggs, often could not leave her home. She has no other
which hatch into rhabditiform larvae; these can significant past medical history, is on no medica-
subsequently mature into infectious filariform tions, reports no recent antibiotic use, and has no
larvae within the bowel, and auto-infect the host family history of any gastrointestinal diseases. On
through penetration of the colonic wall or through examination, her temperature is 100.9 F and she
the perianal skin. In immunosuppressed hosts, has moderate lower abdominal tenderness. She
this cycle of autoinfection may result in hyperin- wants to know if there’s something that can be
fection syndrome, leading to a substantial parasite done so she may return to work and resume her
burden, disseminated disease, sepsis, gastrointes- regular activities. What is the most appropriate
tinal bleeding, and potentially death. Suppression management at this time?
or compromise of cell-mediated immunity such
as with HIV, corticosteroids, and use of tumor A. Metronidazole
necrosis factor-alpha inhibitors are risk factors B. Levofloxacin
for hyperinfection syndrome. An ELISA test C. Supportive care and oral rehydration
may be used for diagnosis. Stool testing may be therapy only
obtained, but it has a lower sensitivity. Endoscopy D. Azithromycin
with biopsies, skin biopsies, and bronchoalveolar E. Stool culture and sensitivity testing
lavage may also be performed. Treatment is with
ivermectin. CORRECT ANSWER: D
Cyclospora and giardia would not cause this
constellation of symptoms including pulmonary RATIONALE
findings and eosinophilia. Ancylostoma duodenale This patient has moderately severe traveler’s
may cause eosinophilia and pulmonary symptoms, diarrhea (TD), of which Enterotoxigenic E. coli is
however, Necator americanus is the type of hook- the most common etiologic agent; other causes
worm predominantly found in the United States. may include campylobacter and salmonella. Mild
This patient has E. coli sepsis as a result of S. ster- symptoms are defined as not causing change in
coralis infection, but not STEC infection per se. activities. Patients with moderate symptoms
can function, but have to curtail activities. Se-
REFERENCES vere symptoms are incapacitating and result in
Corti M. Strongyloides stercoralis in Immunosup- complete cessation of activities, or are associated
pressed Patients. Arch Clin Infect Dis 2016; 11(1): with bloody diarrhea. Studies have shown that
e27510. antibiotic therapy shortens the clinical course of
Jourdan PM, Lamberton PH, Fenwick A, et. al. TD to roughly one day, and guidelines recom-
Soil-transmitted helminth infections. Lancet 2018; mend treatment for patients with moderate or
391:252-65. severe symptoms. Although fluoroquinolones
MMWR 2013; 62(42): 843. are used for empiric therapy of TD, data suggest
that there is increased campylobacter resistance
to fluoroquinolones in Southeast Asia and the
Question 24 preferred therapy for travelers to that region is
A 32-year-old woman presents with complaints of azithromycin 1000 mg orally for one dose, or 500
diarrhea. She has just returned from a volunteer mg orally daily for three days. Metronidazole is
Chapter 11 — GI infections of the small intestine and colon 303
Ciguatera fish poisoning is associated with been identified. Patients may present with mal-
sensory symptoms of paresthesias and changes absorption, steatorrhea, weight loss, and fatigue.
in temperature perception from ingestion of Laboratory testing shows anemia, B12 and folate
ciguatoxins produced by dinoflagellates, and not deficiency, as well as increased fecal fat. Biopsies
ascending motor weakness. Yersinia, vibrio and of the small bowel during upper endoscopy show
STEC infections are not associated with GBS. villous blunting with negative celiac serologies.
Treatment is a three to six month course of tetra-
REFERENCE cycline 250 mg orally four times daily with folate
Nachamkin I, Allos BM, Ho T. Campylobacter 5 mg orally daily. The macrocytic anemia, normal
Species and Guillian-Barre Syndrome. Clinical iron studies, and low vitamin B12 and folate levels
Microbiology Reviews 1998; 11(3):555-567. argue against celiac disease, so this patient is un-
likely to respond to a gluten-free diet.
Question 27 REFERENCES
A 36-year-old caucasian woman returned from a Ghoshal UC, Srivastava D, Verma A, et. al. Tropi-
three month missionary trip to India, and sub- cal sprue in 2014: the new face of an old disease.
sequently developed diarrhea, and a 20 pound Curr Gastroenterol Rep 2014; 16(6):391-393.
weight loss in the recent past. She reports in- Batheja MJ, Leighton J, Azueta A, et. al. The
creased abdominal bloating but denies reports fa- Face of Tropical Sprue in 2010. Case Rep Gastro-
tigue, but denies any symptoms of gastrointestinal enterol 2010; 4:168-172.
bleeding. Her CBC reveals a macrocytic anemia, Jansson-Knodell CL, Hujoel IA, Rubio-Tapia
normal iron studies, and low vitamin B12 and fo- A, et. al. Not All That Flattens Villi is Celiac Dis-
late levels. Her stool tests are negative for routine ease: A Review of Enteropathies. Mayo Clin Proc
bacterial pathogens, giardia, ova and parasites. 2018; 93(4):509-517.
Her duodenal biopsies show villous blunting.
A. Initiate empiric therapy with ciprofloxacin. iting and non-bloody diarrhea that lasted for three
B. Supportive care with hydration and loperamide days with spontaneous resolution. She reports that
as needed. her symptoms began shortly her two-year-old son
C. Probiotic therapy has been shown in large ran- had a similar but more severe illness, with nausea,
domized controlled clinical trials to shorten her vomiting, profuse diarrhea, fevers and seizures.
disease course. She states that he was hospitalized for two nights
D. Schedule an EGD and colonoscopy with biopsies. due to dehydration, but his stool cultures did
E. Treat the patient and her two friends with em- not show any bacterial infection. The patient is
piric antibiotics to prevent spread of disease. otherwise healthy, and reports no drug allergies or
medication use. She believes in holistic medicine,
CORRECT ANSWER: B and has not pursued any vaccinations for her son.
Question 30 REFERENCES
An 18-year-old man presents to the Emergency Hall AJ, Wikswo ME, Pringle K, et al. Vital Signs:
Department with nausea, vomiting and non- Foodborne Norovirus Outbreaks – United States,
bloody diarrhea for two days, with associated 2009-2012. MMWR 2014; 63(22): 491-495.
symptoms of fatigue and generalized headache. Hall AJ, Lopman BA, Payne DC, et al. Norovi-
He was in his usual state of health until three rus Disease in the United States. Emerg Infect Dis
days ago, when he went to a high school gradu- 2013; 19(8):1198-1205.
ation party where he ate a catered lunch buffet. Scallan E, Hoekstra RM, Angulo FJ, et al. Food-
He states that he has felt somewhat better over borne Illness Acquired in the United States - Major
the last 12 hours or so. A few of his friends that Pathogens. Emerg Infect Dis 2011; 17(1): 7-15.
attended the same function developed fever and
diarrhea early the following morning, but did not
seek medical attention. Question 31
A 49-year-old female pig farmer from Maine
What is the most likely diagnosis? presents with intermittent cough and wheezing for
which she was unsuccessfully treated with albuterol
A. Enteric adenovirus infection inhalers. She also reports epigastric discomfort for
B. Shiga toxin producing E. coli (STEC) six months which did not respond to an omeprazole
infection trial. She reports passing a large round worm in
C. Norovirus infection her stools, which she has brought in for examina-
D. Rotavirus infection tion. Testing confirms Ascaris lumbricoides.
E. B. cereus toxin ingestion
What is the most appropriate treatment for this
CORRECT ANSWER: C patient?
Question 32 Question 33
A 43-year-old man from rural Alabama with no A 45-year-old man presents to the clinic with
significant past medical history presents with worsening right lower quadrant pain and diarrhea
severe fatigue. His symptoms have been gradually for the last two days. His past medical history is
worsening for the last year, and he now reports significant for hemochromatosis and he undergoes
significant dyspnea on exertion. He denies any regular therapeutic phlebotomies. He admits to
chest pain but has diarrhea three to four times a dining out in a newly-opened restaurant in his town
day. Examination reveals pale conjunctiva and a four days ago. He describes having five non-bloody
2/6 systolic murmur. His laboratory evaluation is watery stools and also has been experiencing sore
significant for severe anemia, with a hemoglobin throat for the last two days. His physical examina-
of 6.1 g/dL, eosinophilia, and an iron saturation of tion is unremarkable except some mild abdominal
three percent. EGD and colonoscopy was sig- tenderness at right lower quadrant. There was no
nificant for a worm in the cecum, consistent with rebound tenderness. Laboratory data shows mild
Necator americanus infection. leukocytosis. Infectious diarrhea secondary to Yer-
sinia enterocolitica is strongly suspected.
What is the appropriate therapy for this patient?
What is the most likely mechanism through which
A. Ivermectin 200 mg / kg orally as a single dose this infection causes diarrhea?
B. Praziquantel 25 mg / kg orally three times daily
for one day A. Release of enterotoxins that stimulate secretion
C. Albendazole 400 mg orally as a single dose and/or impair absorption
D. Metronidazole 500 mg orally three times a day B. Penetration into the submucosa and prolifera-
for 10 days tion in the lymph nodes
E. Nitazoxanide 500 mg orally every 12 hours for C. Production of the toxins in contaminated food
seven days D. Adherence to the intestinal mucosa and efface-
ment of microvilli
CORRECT ANSWER: C E. Mucosal damage with erosions and ulcers
308 Digestive Diseases Self-Education Program®
Question 35
Question 34 A 56-year-old man presents to the emergency
A 23-year-old HIV positive Hispanic man presents department with severe abdominal pain and
to the clinic with history of diarrhea for the last one bloody diarrhea six hours after having a dinner at
week. He recently returned from a month-long trip a local seafood restaurant. His friends mentioned
to Mexico. He describes sudden onset of watery di- that he ordered raw oysters with his meal. He has
arrhea associated with nausea and headache while history of alcohol abuse and drinks half a pint of
still in Mexico. He denies any blood in the stool hard liquor every day. On physical examination
but does describes his stools to be malodorous. his blood pressure was 80/40 mmHg, pulse 120/
His physical examination is unremarkable except min and temperature 38℃. Skin examination
for dry skin and dry oral mucus membranes, and revealed bullous lesions. Abdominal examination
Chapter 11 — GI infections of the small intestine and colon 309
revealed mild tenderness in the periumbilical area nal of medicine. Jan 1 2004;350(1):38-47.
and hepatomegaly on palpation. Bowel sounds Bross MH, Soch K, Morales R, Mitchell RB. Vib-
were present. rio vulnificus infection: Diagnosis and treatment.
Am Fam Physician. 2007 Aug 15;76(4):539-44.
There was no focal neurological deficit.
Which of the following infections is the most likely
cause of the septic shock? Question 36
A 45-year-old Korean man presents to the clinic
A. Ciguatara toxin with worsening right upper quadrant abdominal
B. Scromboid pain and generalized malaise. He also reports los-
C. Shellfish food poisoning ing 10 pounds over the last six months. He visited
D. Vibrio vulnificus his family in South Korea about eight months ago.
He also reports having non-bloody diarrhea for
CORRECT ANSWER: D the last one month. On physical examination, his
vital signs are stable. He has mild tenderness in
RATIONALE the right upper quadrant. His laboratory investiga-
Vibrio vulnificus is a gram-negative bacterium that tions revealed hemoglobin 10 g/dL, white blood
can cause serious wound infections, septicemia, cells 4.8x109/L with 1500/μL eosinophils. His ALT
and diarrhea. It is the leading cause of shellfish- was 35 U/L, AST 35 U/L, Alkaline phosphatase 192
associated deaths in the United States. A pre- U/L and total bilirubin 1.2 mg/dL. Ultrasound of
sumptive diagnosis of V. vulnificus septicemia the abdomen reveals a distended gallbladder and
should be made in any person with fever, hypoten- dilated common bile duct with non-shadowing
sion, or symptoms suggestive of septic shock and echogenic foci. Liver fluke infection is suspected to
characteristic bullous skin lesions. Patients with be the cause of his illness. Stool specimen revealed
liver cirrhosis and alcoholism are at increased oval shaped by formalin ethyl-acetate concentra-
risk of severe infection. Scombroid poisoning is a tion technique.
common seafood-associated disease throughout
the world. The most common findings consist Which of the following is the treatment of choice
of a rapid onset of flushing of the face and neck, for this infection?
erythematous and urticarial rash, diarrhea, and
headache occurring soon after consumption of A. Ciprofloxacin
contaminated fish or cheese. Paralytic shellfish B. Ivermectin
poisoning is caused by ingestion of bivalve mol- C. Levamisole
lusks, such as mussels, clams, scallops, and oys- D. Praziquantel
ters, as well as crabs and snails. Physical findings
occur within hours of ingestion and include neu- CORRECT ANSWER: D
rologic symptoms ranging from perioral tingling,
ataxia, difficulty swallowing, dizziness, paresthe- RATIONALE
sias, weakness, paralysis, brainstem dysfunction, This case represents an example of Clonorchis-
and respiratory failure. The rapid onset of weak- sinensis (or Opisthorchissinensis) infection, which
ness and paralysis distinguishes paralytic shellfish is also known as the Chinese liver fluke. Symptoms
poisoning from ciguatera fish poisoning. can include fatigue, abdominal discomfort, an-
orexia, weight loss, dyspepsia, and diarrhea. The
REFERENCES gallbladder is often nonfunctional and enlarged.
Thielman NM, Guerrant RL. Clinical practice. The diagnosis can be established by identifying
Acute infectious diarrhea. The New England jour- eggs in stool, duodenal aspirates, or bile specimens.
310 Digestive Diseases Self-Education Program®
Eggs may be visible in the stool approximately mon findings consist of a rapid onset of flushing
four weeks following infection. The “gold stan- of the face and neck, erythematous and urticarial
dard” detection method for infection is formalin rash, diarrhea, and headache occurring soon after
ethyl-acetate concentration technique (FECT). The consumption of contaminated fish. Scombroid poi-
treatment of choice is praziquantel. Alternatives to soning usually resolves within 12 to 48 hours if un-
praziquantel include albendazole or mebendazole. treated and has no long-term sequelae. Symptoms
Ivermectin is used in the treatment of Strongyloi- of Ciguatera fish poisoning typically begins three
diasis. Levimasole is used in ascariasis treatment. to six hours after eating contaminated fish, but can
be delayed up to 30 hours. Clinical findings include
REFERENCES peri-oral paresthesias, pruritus without urticaria
Qian MB, Utzinger J, Keiser J, Zhou XN. Clonor- or erythema and temperature-related dysesthe-
chiasis. Lancet 2016; 387:800. sias (cold stimuli perceived as hot or producing an
Lai DH, Hong XK, Su BX, et al. Current status abnormal, unpleasant sensation). Ingestion of pre-
of Clonorchis sinensis and clonorchiasis in China. formed botulinum toxin, typically found in home
Trans R Soc Trop Med Hyg 2016; 110:21. canned foods, causes vomiting, diarrhea, abdomi-
nal pain, cranial nerve dysfunction, and descending
paralysis beginning hours to days after consump-
Question 37 tion. Paralytic shellfish poisoning is caused by in-
A 33-year-old man presents to the emergency gestion of bivalve mollusks, such as mussels, clams,
department with abdominal pain, nausea, vom- scallops, and oysters, as well as crabs and snails.
iting and diarrhea. His symptoms started soon Physical findings occur within hours of ingestion
after eating rare tuna steaks at a local restaurant. and include neurologic symptoms ranging from
He has no other significant medical history. On perioral tingling, ataxia, difficulty swallowing, dizzi-
physical examination, his blood pressure is 110/70 ness, paresthesias, weakness, paralysis, brainstem
mmHg, pulse is 115/min and respiratory rate is dysfunction, and respiratory failure. Staphylococ-
17/min. He is afebrile. He has noticeable erythem- cus aureus enterotoxin may cause abrupt onset of
atous rash on face and upper trunk. Mild tender- nausea, vomiting, and abdominal cramps approxi-
ness in the epigastric area is also appreciated. mately one hour after eating contaminated food.
Laboratory investigations are unremarkable. He is Patients with staphylococcal food poisoning may
resuscitated with intravenous fluids and adminis- develop a fever, which is not seen in patients with
tration of oral antihistamine with rapid resolution scombroid poisoning.
of symptoms.
REFERENCES
Which is the most likely cause of this illness? Sobel J, Painter J. Illnesses caused by marine tox-
ins. Clinical infectious diseases : an official publica-
A. Botulism tion of the Infectious Diseases Society of America.
B. Ciguatera fish poisoning Nov 1 2005;41(9):1290-1296.
C. Paralytic shellfish poisoning Scombroid fish poisoning associated with tuna
D. Scombroid fish poisoning steaks--Louisiana and Tennessee, 2006. MMWR.
E. Staphylococcal food poisoning Morbidity and mortality weekly report. Aug 17
2007;56(32):817-819.
CORRECT ANSWER: D
RATIONALE Question 38
Scombroid poisoning is a common seafood-associ- A 36-year-old man was admitted to the hospital
ated disease throughout the world. The most com- with worsening abdominal pain, vomiting and diar-
Chapter 11 — GI infections of the small intestine and colon 311
rhea. His symptoms started few hours after eating hours of ingestion and include neurologic symp-
at a local hotel buffet. A few hours later, he experi- toms ranging from perioral tingling, ataxia, diffi-
enced numbness of his extremities and change in culty swallowing, dizziness, paresthesias, weakness,
his taste sensation prompting him to seek medical paralysis, brainstem dysfunction, and respiratory
attention. He explained that cold objects felt hot failure. The rapid onset of weakness and paralysis
and his teeth also felt numb and loose. He devel- distinguishes paralytic shellfish poisoning from
oped bradycardia and hypotension in the emergen- ciguatera fish poisoning. Signs and symptoms of
cy department requiring treatment with intrave- scombroid toxicity usually begin within an hour of
nous atropine. Physical examination revealed mild eating contaminated fish. The symptoms resemble
periumbilical abdominal tenderness. His laboratory an IgE-mediated allergic reaction. The patient
investigations were unremarkable. He was resusci- may suddenly experience flushing, a sensation of
tated with intravenous fluids and was subsequently warmth, an erythematous rash, palpitations, and
hospitalized. Few days later, he was discharged to significant tachycardia. Ingestion of preformed
home with complete resolution of his symptoms. botulinum toxin, typically found in home canned
foods, causes vomiting, diarrhea, abdominal pain,
What is the most likely cause of his illness? cranial nerve dysfunction, and descending paralysis
beginning hours to days after consumption.
A. Botulism
B. Ciguatera fish poisoning REFERENCES
C. Paralytic shellfish poisoning Pennotti R, Scallan E, Backer L, et al. Ciguatera
D. Scombroid fish poising and scombroid fish poisoning in the United States.
E. Staphylococcal food poisoning Foodborne Pathog Dis 2013; 10:1059.
Sobel J, Painter J. Illnesses caused by marine
CORRECT ANSWER: B toxins. Clinical infectious diseases : an official
publication of the Infectious Diseases Society of
RATIONALE America. Nov 1 2005;41(9):1290-1296.
Ciguatera fish poisoning accounts for approxi-
mately 20 percent of the fish-related foodborne
outbreaks in the United States. Gastroenteritis, Question 39
including vomiting, diarrhea, and abdominal A 62-year-old man with no significant past medical
cramping, typically begins three to six hours after history is admitted to the hospital after develop-
eating contaminated fish, but can be delayed up to ing bloody diarrhea and severe abdominal cramps
30 hours. Neurologic abnormalities usually appear for several days. He denies any fever. There is no
three to 72 hours after the meal. Clinical findings history of any recent travel or contact with sick
include perioral paresthesias, pruritus without person. On physical examination, his vital signs are
urticaria or erythema, a metallic taste in the mouth, stable. Laboratory investigations reveals leukocyte
painful dentition, a feeling that the teeth are loose, count to be mildly elevated at 13.0x109/L with
painful urination, blurred vision, and temperature- normal differential. His metabolic panel and liver
related dysesthesias (cold stimuli perceived as hot function tests are also unremarkable. He has only
or producing an abnormal, unpleasant sensation). rare leukocytes found in the stool by microscopy.
Cardiovascular signs, including bradycardia, heart A stool sample sent to the laboratory, is negative
block, and hypotension, can occur within hours for Clostridium difficile toxin A. Stool cultures at
of consumption. Paralytic shellfish poisoning is 48 hours are negative for Shigella, Salmonella, and
caused by ingestion of bivalve mollusks, such as Campylobacter. Colonoscopy with biopsy reveals
mussels, clams, scallops, and oysters, as well as submucosal edema, hemorrhage and ulcerations
crabs and snails. Physical findings occur within consistent with ischemic colitis.
312 Digestive Diseases Self-Education Program®
Which of the following stool tests would most likely nausea and vomiting. He denied any recent
be useful in this clinical situation? travelling but recalled consuming watermelons a
day before the onset of his symptoms. On physi-
A. Enzyme immunoassay for C. difficile toxin B cal examination, his blood pressure was 90/60
B. Enzyme immunoassay for microsporidia mmHg, pulse 120/min, respiratory rate 16/min. He
C. Enzyme immunoassay for rotavirus was afebrile. Moderate periumbilical tenderness
D. Enzyme immunoassay for Shiga toxin on abdominal examination was noted. Rectal exam
E. Stool smear for amoebae was unremarkable and negative for occult blood.
He was aggressively resuscitated with intravenous
CORRECT ANSWER: D fluids and was admitted to the hospital with the
diagnosis of cholera and circulatory shock.
RATIONALE
The clinical features of bloody diarrhea with Which of the following is the treatment of choice
severe abdominal pain, lack of fever and rare leu- for her infection?
kocytes in stool all suggest Shiga toxin-producing
E. coli. Although the usual cause in the USA is A. Ampicillin
0157:H7, other serotypes have been implicated. B. Azithromycin
The enzyme immunoassay test can detect Shiga C. Metronidazole
toxin produced by any of the serotypes. The acute D. No antibiotic is indicated
onset and bloody diarrhea are unusual for micro- E. Trimethoprim-sulfamethoxazole
sporidiosis. C. difficile can cause bloody diarrhea.
However, disease related to toxin B without toxin CORRECT ANSWER: B
A is uncommon. The presentation is not typical
for viral enteric infection such as rotavirus. While RATIONALE
majority of the case of the amoebiasis are asymp- Vibrio cholera is a bacterial pathogen linked to
tomatic, severe dysentery can occur. Risk factors epidemics of diarrhea. Infection with V. cholerae
for acute severe dysentery include malnutrition, results in a spectrum of disease, ranging from
steroid use, pregnancy, alcoholism, immunosup- asymptomatic intestinal colonization to severe
pression and malignancy which are absent in this diarrhea. While mild cases of V. cholerae infec-
case. In contrast to bacterial dysentery, which tion may be clinically indistinguishable from other
typically begins abruptly, amebic colitis begins causes of watery diarrheal illness, the profound
gradually over one to several weeks. and rapid loss of fluid and electrolytes mark
severe cholera as a clinical entity distinct from
REFERENCES other causes. Aggressive volume repletion is the
Thielman NM, Guerrant RL. Clinical practice. mainstay of treatment for cholera. Antibiotics
Acute infectious diarrhea. N Engl J Med 2004; can shorten the duration of diarrhea, reduce the
350:38. volume of stool losses, and lessen the duration
Page AV, Liles WC. Enterohemorrhagic Esch- of V. cholera shedding. The antibiotic options for
erichia coli Infections and the Hemolytic-Uremic cholera include macrolides, fluoroquinolones, and
Syndrome. Med Clin North Am 2013; 97:681. tetracyclines.
Most V. cholerae strains are resistant to
trimethoprim-sulfamethoxazole. Ampicillin and
Question 40 metronidazole are not indicated in cholera.
A 42-year-old woman presented to the emergency
department with a sudden development of profuse REFERENCES
watery diarrhea associated with abdominal cramps, Harris JB, LaRocque RC, Qadri F, et al. Cholera.
Chapter 11 — GI infections of the small intestine and colon 313
REFERENCE REFERENCES
Hotez PJ, Brooker S, Bethony JM, et al. Hookworm Cangemi JR. Food poisoning and diarrhea: small
infection. N Engl J Med. 2004;351(8):799-807. intestine effects. Curr Gastroenterol Rep. 2011
Oct;13(5):442-8.
Bresee JS, Marcus R, Venezia RA, et al. The eti-
Question 43 ology of severe acute gastroenteritis among adults
A 44-year-old woman presents to the afterhours visiting emergency departments in the United
urgent care clinic with worsening nausea, vomiting States. J Infect Dis. 2012 May 1;205(9):1374-81.
and abdominal cramps. She felt well earlier in the
day, and played in a softball tournament followed
by a potluck lunch with the other players. She de- Question 44
nies any fever and has no diarrhea. On physical ex- A 27-year-old woman presented to the clinic with
amination, her vital signs are stable. Her abdominal a history of diarrhea for the last 10 days. She re-
examination is notable for mild diffuse tenderness. ported 10 to 12 loose stools with passage of blood.
Bowel sounds are normal and rectal examination is Diarrhea was associated with nausea and ab-
unremarkable. Laboratory investigations including dominal cramps. She denied any contact with sick
complete blood count, electrolytes and liver func- person or recent traveling. She recently moved
tion tests are unremarkable. to a farmhouse with her husband. On physi-
cal examination, her blood pressure was 110/60
Which of the following is the most likely pathogenic mmHg, pulse100/min, respiratory rate 13/min
mechanism for her illness? and temperature 100.5℉. Abdominal examination
Chapter 11 — GI infections of the small intestine and colon 315
Question 46 Question 47
A 36-year-old man presents to the clinic with a A 36-year-old man presents to the clinic with
history of diarrhea and significant fatigue for the a history of diarrhea and significant fatigue for
last two months. He has no significant past medi- the last two months. He has no significant past
cal history and works as a chef in a local sushi bar. medical history and works as a chef in a local
He complains of six to seven watery stools daily sushi bar. He complains of six to seven watery
with nocturnal symptoms. Diarrhea is associated stools daily with nocturnal symptoms. Diarrhea is
with abdominal cramps and he denies any passage associated with abdominal cramps and he denies
of blood. His physical examination, including any passage of blood. His physical examination,
vital signs, is unremarkable. Laboratory investiga- including vital signs, is unremarkable. Labora-
tion reveals 9.8 g/dL hemoglobin, with a mean tory investigations reveals 9.8 g/dL hemoglobin,
corpuscular volume 110 fL. Peripheral eosino- with a mean corpuscular volume 110 fL. Periph-
philia is also noted. A stool sample is sent to the eral eosinophilia is also noted. A stool sample is
lab and is pending. sent to the lab and is pending.
Which of the following is the most likely cause Which of the following medications is most likely
of this illness? to help?
Chapter 11 — GI infections of the small intestine and colon 317
A. Albendazole A. Ampicillin
B. Mebendazole B. Azithromycin
C. Paromomycin C. Ciprofloxacin
D. Praziquantel D. Metronidazole
E. Pyrantel pamoate E. Trimethoprim-sulfamethoxazole
RATIONALE RATIONALE
This is likely a tapeworm infection with Diphyllo- Travelers’ diarrhea is an infectious illness, caused
bothrium latum. D. latum infection can be ac- by a variety of bacterial, viral, and parasitic
quired from ingesting certain forms of freshwater organisms, although bacterial pathogens are the
fish, and those who consume raw fish, including most frequent cause in acute cases. More than 90
sushi, are at increased risk. The classical manifes- percent of illnesses in most geographic areas are
tation of infection with D. latum is megaloblastic caused by bacteria; the most common organism is
anemia due to vitamin B12 deficiency. Praziquan- enterotoxigenic Escherichia coli (ETEC). The clas-
tel is the treatment of choice for all of the tape- sic “turista” due to ETEC generally produces mal-
worm infections discussed above. All the other aise, anorexia, and abdominal cramps followed by
agents listed would not be used for treatment of the sudden onset of watery diarrhea. Antibiotics
tapeworm infection. are warranted to treat diarrhea in those who de-
velop moderate to severe diarrhea, characterized
REFERENCE by more than four unformed stools daily, fever,
Webb C, Cabada MM. Intestinal cestodes. Curr or blood, pus, or mucus in the stool. Fluoroqui-
Opin Infect Dis. 2017 Oct;30(5):504-510. nolones have well-established efficacy for travel-
ers’ diarrhea and are generally well-tolerated for
the short duration. Azithromycin is the preferred
Question 48 agent for children and pregnant women. Wide-
A 36-year-old pregnant woman presented to the spread antibiotic resistance among many routine
clinic with diarrhea after a one-week vacation in enteric pathogens to drugs such as ampicillin and
Thailand. Her symptoms started as generalized trimethoprim-sulfamethoxazole has made these
malaise and loss of appetite followed by sudden on- agents less useful for the treatment of travelers’
set of diarrhea. She reported passing three to four diarrhea in much of the world.
non-bloody stools in a day associated with nausea
and abdominal pain. Physical examination, includ- REFERENCE
ing vital signs, was unremarkable except palpation Steffen R, Hill DR, DuPont HL. Traveler’s diarrhea:
of gravid uterus. A clinical diagnosis of traveler’s a clinical review. JAMA. 2015 Jan;313(1):71-80.
diarrhea was made and the patient was instructed
to increase her hydration and watch for any alarm
symptoms. She called back after three days report- Question 49
ing worsening of diarrhea with passage of some A 27-year-old man presents to the clinic with
mucus in the stool. She denies any fever or worsen- worsening abdominal pain and diarrhea for the last
ing of abdominal pain. three weeks. For the last two days, he also noticed
blood in his stools. He recently returned from a
Which of following antibiotics will be the most ap- business trip from Taiwan where he stayed in a
propriate treatment in this case? hotel and only drank bottled water. He admits to
being sexually active with several partners of the
318 Digestive Diseases Self-Education Program®
streaks on her legs and buttocks. She occasion- abdominal pain and weight loss of four weeks dura-
ally has dry cough and shortness of breath. She tion. He is HIV positive and last his CD4 count is
has history of rheumatoid arthritis and has been 50 cells/microL. He denies any recent travelling
on prednisone and methotrexate for several years. or sick contacts. He admits being sexually active
Physical examination, including vital signs are with multiple male partners. Physical examination,
unremarkable. Laboratory tests are significant for including vital signs, is unremarkable. Ultrasound
a microcytic anemia and peripheral eosinophilia. of the abdomen shows a thickened gallbladder
EGD shows congested duodenal mucosa and ery- and mildly dilated common bile duct without any
thematous spots. Stools studies are negative for any stones. Stool examination is negative for leukocytes
ova and parasite. With a high suspicion for parasitic but microscopy reveals small pink colored oocytes
infection, intestinal aspiration is performed and with modified acid-fast stain.
reveals filariform larvae.
Laboratory studies:
Which of the following is the treatment of choice
for this parasitic infection? Hemoglobin 14 g/dL (140 g/L)
Leukocyte count 6000/µL (6x109/L)
A. Ivermectin Alanine aminotransferase 35 U/L
B. Paromomycin Aspartate aminotransferase 25 U/L
C. Praziquantel Alkaline phosphatase 110 U/L
D. Pyrantel pamoate Bilirubin, total 1.0 mg/dL (17.1
E. Thiabendazole µmol/L)
Serum albumin 3.1 g/dL (31 g/L)
CORRECT ANSWER: A
In addition to starting antiretroviral therapy, which
RATIONALE of the following is the most appropriate treatment
The patient has strongyloidiasis caused by Stron- for this infection?
gyloides stercoralis. In the past, thiabendazole
was administered for uncomplicated infection. A. Albendazole
However, thiabendazole is now rarely used due B. Metronidazole
to the availability of more efficacious and better- C. Nitazoxanide
tolerated medications. The treatment of choice D. Trimethoprim-sulfamethoxazole
for strongyloidiasis at present is ivermectin, with
albendazole as an alternative option. Standard CORRECT ANSWER: C
dosing of ivermectin consists of two single 200
mcg/kg doses of ivermectin administered on two RATIONALE
consecutive days or administered two weeks apart. The patient is infected with Cryptosporidium..
Cryptosporidium can cause an asymptomatic infec-
REFERENCE tion but in immunocompromised hosts (particularly
Greiner K, Bettencourt J, Semolic C. Strongyloidia- those with T-cell immunodeficiency), the illness is
sis: a review and update by case example. Clin Lab more frequently protracted and severe particularly
Sci 2008; 21:82-8. when the CD4 count is less than 100 cells/microL.
For HIV-infected patients, the most important part
of therapy is initiating highly-active antiretroviral
Question 52 therapy in order to reconstitute immunity. Ni-
A 43-year-old man presents to the clinic with tazoxanide has a limited effect on clinical symptoms
history of watery diarrhea, right upper quadrant in immunocompromised hosts and can be consid-
320 Digestive Diseases Self-Education Program®
REFERENCE
Wumba R, Longo-Mbenza B, Menotti J, et al. Epidemiol-
ogy, clinical, immune, and molecular profiles of microspo-
ridiosis and cryptosporidiosis among HIV/AIDS patients.
International journal of general medicine. 2012;5:603-611.
Answers & critiques
CHAPTER 12
Gastrointestinal bleeding
Daniel Wild, MD and
Brandon Sprung, MD
Question 1 REFERENCE
A 78-year-old man presents to the Emergency Lau JY, Leung WK, Wu JC, et al. Omeprazole
Department with hematemesis. He is hemody- before endoscopy in patients with gastroin-
namically stable and his initial hemoglobin is 9.8 testinal bleeding. N Engl J Med. 2007 Apr
g/dL. His BUN is 28 mg/dL and his Cr is 1.1 mg/ 19;356(16):1631-40.
dL. He has been on aspirin and clopidogrel since
having a stroke two months ago. The patient is Question 2
started on a continuous infusion of pantoprazole What is the minimum volume of blood loss from
while the GI team is mobilized for an urgent up- an upper GI source that is required for a patient
per endoscopy. to pass melena?
321
322 Digestive Diseases Self-Education Program®
Question 9
A 72-year-old man with alcohol-related cirrhosis is
admitted with hematemesis. His heart rate is 115
bpm and blood pressure is 88/60 mmHg. Admis-
sion labs are notable for a hemoglobin of 8.3 g/dL,
Which of the following is the most effective treat- platelets of 110K and an INR of 1.4. He is not on
ment option for this condition: blood thinners or anti-platelet agents. He is intu-
bated and admitted to the ICU where you perform
A. Transjugular Intrahepatic Portosystemic urgent upper endoscopy which reveals fresh red
Shunt (TIPS) blood in the stomach and a lesion evident in the
B. Non-selective beta blocker gastric cardia on retroflexion. While examining the
C. High dose sucralfate elixir lesion, it starts to bleed briskly. (See two figures on
D. Argon Plasma Coagulation (APC) the following page.)
E. Bipolar probe cautery
CORRECT ANSWER: D
326 Digestive Diseases Self-Education Program®
Question 10
A 57-year-old American man presents with three
days of melena and a few episodes of blood-tinged
emesis. His hemoglobin is noted to be three
grams down from his baseline. He is hemodynam-
ically stable. He does not take any medications of
any kind and had no symptoms until his melena
started three days ago.
What is the most appropriate next step in this What is the most likely source for this patient’s
patient’s management? upper GI bleeding?
RATIONALE RATIONALE
This patient is bleeding from a gastric varix and will The most common etiologies for upper GI bleed-
require immediate therapy. A Hemoclip is effec- ing in the US are: Peptic Ulcer Disease (20-50
tive therapy for bleeding related to peptic ulcers or percent); Gastroduodenal erosions (eight to 15
small exposed vessels but would be the best treat- percent); Esophagitis (five to 15 percent); Mallory-
ment for a gastric varix. A Sengstaken – Blakemore Weiss Tear (eight to 15 percent); Esophageal
tube provides effective tamponade to temporarily Varices (five to 20 percent) and vascular malfor-
arrest bleeding from esophageal varices but would mations (five percent).
be less effective in managing a bleeding gastric
varix. Epinephrine injection into the varix would REFERENCE
not provide definitive therapy and electrocautery Hwang JH, Fisher DA, Ben-Menachem T, et al.
could worsen the bleeding. Injecting Cyanoacrylate The role of endoscopy in the management of acute
into the bleeding varix has emerged as an effective non-variceal upper GI bleeding. Gastrointest En-
and definitive therapy for gastric varices. dosc. 2012 Jun;75(6):1132-8.
REFERENCES
Ríos Castellanos E, Seron P, Gisbert JP, Bonfill Question 11
Cosp X. Endoscopic injection of cyanoacrylate You perform an upper endoscopy on a 70-year-
glue versus other endoscopic procedures for acute old patient with melena and discover a seven mm
bleeding gastric varices in people with portal cratered ulcer in her gastric antrum. The ulcer
hypertension. Cochrane Database Syst Rev. 2015 is not actively bleeding, but there is an adherent
May 12;(5):CD010180. clot tethered to the ulcer base. You elect not to
Hwang JH, Shergill AK, Acosta RD, et al. disturb the clot or perform any therapy. When
The role of endoscopy in the management of discussing these findings with the patient’s son,
variceal hemorrhage. Gastrointest Endosc. 2014 he asks you what her risk of rebleeding is over the
Aug;80(2):221-7. next few days.
Chapter 12 — Gastrointestinal bleeding 327
A. 1 percent RATIONALE
B. 5 percent Diverticulosis is very common, with prevalence rates
C. 25 percent of up to 50 percent in the United States (though this
D. 50 percent is hard to ascertain as most patients are asymptom-
E. 80 percent atic). Acute lower GI bleeding (LGIB) is commonly
encountered by practicing gastroenterologists with
CORRECT ANSWER: C an annual incidence rate of 0.03 percent. Diver-
ticulosis remains the most common source of LGIB,
RATIONALE but still only occurs in three to 15 percent of pa-
Peptic ulcers are classified by stigmata. It is im- tients with underlying diverticulosis. Its incidence
portant to know the risk of re-bleeding depending increases significantly with age and the mean age
on the appearance of the ulcer. Without therapy, for first presentation is 63-77 years. The mortality
a clean-based ulcer has a less than three percent remains low with an estimated rate between one and
chance of re-bleeding, while an ulcer with an ac- four percent. It remains a common indication for
tive arterial bleed will almost always continue to hospitalization with approximately 36 per 100,000
bleed or re-bleed. In between these stigmata is U.S. adults admitted for it annually.
a range including: ulcer with a visible vessel (50
percent re-bleeding rate without treatment); ulcer REFERENCES
with adherent clot (eight to 35 percent re-bleeding Fox JM, Stollman NH.. (2015). Diverticular
rate without treatment); oozing ulcer without ves- Disease of the Colon. In Sleisinger and Fordtran’s
sel or other stigmata (10-27 percent re-bleeding Gastrointestinal and Liver Disease. (pp. 2073-
rate without treatment) and ulcer with a flat pig- 2086). Philadelphia: Saunders/Elsevier.
mented spot (less than eight percent re-bleeding Longstreth GF. Epidemiology and outcome of
rate without therapy). patients hospitalized with acute lower gastrointes-
tinal hemorrhage: a population-based study. Am J
REFERENCE Gastroenterol. 1997;92:419-24.
Hwang JH, Fisher DA, Ben-Menachem T, et al. Bounds BC, Kelsey PB. Lower gastrointes-
The role of endoscopy in the management of acute tinal bleeding. Gastrointest Endosc Clin N Am
non-variceal upper GI bleeding. Gastrointest En- 2007;17:273-88,vi.
dosc. 2012 Jun;75(6):1132-8.
Question 13
Question 12 An 83-year-old man is admitted for the third time
Which of the following statements regarding di- in two months for anemia and transiently melenic
verticular bleeding is the most accurate? stool. His presenting Hgb has been as low as 5 g/
dL and he has had associated shortness of breath.
A. Clinically significant diverticular bleeding occurs On his first admission, an Upper Endoscopy and
in 25 percent of patients with diverticulosis Colonoscopy were performed and neither revealed
B. Clinically significant diverticular bleeding occurs a bleeding source. On his second admission, push
in 90 percent of patients with diverticulosis enteroscopy to the proximal jejunum was also
C. Clinically significant diverticular bleeding occurs unremarkable. You decide now to perform video
in 5 percent of patients with diverticulosis capsule endoscopy (VCE). Which of the following
D. Clinically significant diverticular bleeding occurs statements regarding the diagnostic yield of VCE
in 50 percent of patients with diverticulosis in this patient is most accurate?
328 Digestive Diseases Self-Education Program®
A. The diagnostic yield of his VCE will be higher is 32 mg/dL with a Cr of 1.2 mg/dL and his initial
if the study is performed electively after he is hemoglobin is 9.3 g/dL with no known baseline.
discharged from the hospital. You suspect that he has a bleeding ulcer and make
B. His study will have a lower diagnostic yield preparations to perform urgent upper endoscopy.
because of his advanced age. You also start him on an intravenous infusion of
C. The degree of his anemia will not influence the proton pump inhibitor (PPI). What is the mecha-
diagnostic yield of his VCE. nism of benefit of PPI in this acute setting?
D. His recent melenic stool increases the diag-
nostic yield of his VCE. A. Facilitate mucosal healing at the ulcer site
E. Studies in patients like him show a diagnostic B. Optimize platelet aggregation by increasing
yield for VCE of approximately 35 percent. gastric pH
C. Cytoprotection to prevent further mucosal
CORRECT ANSWER: D damage
D. Decrease the risk of intra-procedural aspira-
RATIONALE tion
Performing VCE is a reasonable next step in the E. Competitive Cyclooxygenase Enzyme inhibi-
patient. Studies show that the diagnostic yield of tion decreases NSAID toxicity
VCE is higher in patient’s who are older than 60,
have more severe anemia with a Hgb less than 10 CORRECT ANSWER: B
g/ dL, have been bleeding for longer (greater than
six months), are male and who have overt rather RATIONALE
than obscure bleeding. It has also been shown In the acute setting, the primary mechanism of
that the yield of VCE increases the more proximate benefit of PPI’s is acute elevation of the gastric pH
it is performed to the timing of the episode of which optimizes platelet aggregation and clot for-
overt bleeding. The diagnostic yield of most large mation. The other choices are either not actions
studies that evaluated the ability of VCE to detect performed by PPI’s or are not mechanisms by
a potential bleeding source is approximately 60 which PPI’s are beneficial in an acute setting.
percent.
REFERENCE
REFERENCES Green FW Jr, Kaplan MM, Curtis LE, Levine PH.
Gurudu SR, Bruining D, Acosta RD, Eloubeidi MA, Effect of acid and pepsin on blood coagulation
et al. The role of endoscopy in the management and platelet aggregation. A possible contributor
of suspected small-bowel bleeding. Gastrointest prolonged gastroduodenal mucosal hemorrhage.
Endosc. 2017 Jan;85(1):22-31. Gastroenterology. 1978;74(1):38.
Liao Z, Gao R, Xu C, Li ZS. Indications and de-
tection, completion, and retention rates of small-
bowel capsule endoscopy: a systematic review. Question 15
GastrointesEndosc. 2010 Feb;71(2):280-6. An 83-year-old woman presents to see you in clin-
ic to discuss a several year history of intermittent
melena and iron deficiency anemia in the setting
Question 14 of moderate aortic valve stenosis. Over the past
A 24-year-old man is admitted with bright red several years, she has required pack red blood cell
hematemesis. He has no medical or surgical transfusions three times despite being maintained
history but takes ibuprofen regularly for exercise- on intravenous iron infusions. She has undergone
related arthralgias. His initial heart rate is 118 and multiple evaluations including Upper Endoscopy,
blood pressure is 97 / 64 mmHg. His initial BUN Colonoscopy, two video capsule endoscopies, push
Chapter 12 — Gastrointestinal bleeding 329
years ago who takes daily aspirin monotherapy aspirin immediately is too aggressive as if this
is admitted to the ICU with melena and a sig- strategy were employed, the risk of re-bleeding
nificant hemoglobin drop. An upper endoscopy would exceed that of cardiac complications.
reveals an antral ulcer with a small clot. This Changing treatment from aspirin to clopidogrel
is removed and a pigmented spot is seen on the monotherapy is not advisable as data suggest
ulcer base prompting the placement of three that aspirin + PPI has lower bleeding risk than
hemoclips. H. pylori testing is negative. with clopidogrel alone.
CORRECT ANSWER: E
Question 18
RATIONALE An otherwise healthy 64-year-old woman is
Balancing the risk of re-bleeding with the risk admitted with red hematemesis. She is hemody-
of thrombogenic complications is a common namically stable. Her hemoglobin has dropped
clinical challenge. Though aspirin avoidance is by three gm/dL from her baseline two months
commonly recommended after ulcer detection ago. She has never had hematemesis before
because aspirin has been associated with a two- but she has had intermittent melena every few
fold increase in the risk of recurrent bleeding, months and has been maintained on oral iron for
data suggest that stopping aspirin results in in- mild anemia for the past two years. A CT scan
creased mortality. The increased risk of throm- on admission was unremarkable other than not-
bogenic complications typically starts seven to ing a medium-sized hiatus hernia. You perform
10 days after aspirin has been held as inhibited urgent upper endoscopy which reveals the fol-
platelets tend to remain in the circulation for this lowing finding in the gastric cardia, just distal to
long. This patient’s risk of rebleeding is rela- the GE-junction:
tively low in light of the absence of visible vessel,
the endoscopic treatment with hemoclips and
the use of aggressive PPI therapy. Given these
considerations, since it is needed for cardiac pro-
tection, his aspirin should be resumed as quickly
as possible. Holding it until ulcer healing is
confirmed could result in a several month aspirin
abstinence which is too long. Even one month of
aspirin cessation is likely too long in the setting
of concomitant twice daily PPI use. Resuming
Chapter 12 — Gastrointestinal bleeding 331
Which of the following is most likely to result in more likely to have recurrent ulcer hemor-
definitive therapy for this lesion? rhage than controls
C. The dose of NSAIDs does not influence the
A. Treat the ulcer with endoclips bleeding risk associated with their use
B. Apply argon plasma coagulation to the ulcer D. Less than 60 percent of ulcer bleeding will
C. Add sucralfate to her daily proton pump inhibitor resolve without treatment
D. Inject the ulcer base with diluted epinephrine E. Bleeding is more likely to occur after being
E. Refer to surgery for hiatus hernia repair on NSAIDs for more than six months than
during the initial treatment period
CORRECT ANSWER: E
CORRECT ANSWER: B
RATIONALE
This patient’s linear ulcer wwithin the hia- RATIONALE
tus hernia sac is classic for a Cameron lesion. Peptic ulcers remain the most common cause of
Though rarely the cause of acute hematemesis, it upper GI hemorrhage. Patient age, prior pep-
can occur. It is also the most likely cause of her tic ulcer disease bleeding and medications are
previous intermittent melena. Since the lesion important risk factors. Corticosteroids alone
is clean-based and not actively bleeding, acute are not associated with an increased risk of ulcer
endoscopic therapy with injection, APC or clips bleeding but when taken with NSAIDs they
is not clearly necessary and would not provide double the risk of complications compared to
definitive therapy. Though PPI may help, there NSAIDs alone. NSAIDs increase the risk of ulcer
is no clear role for adding sucralfate and the complications like bleeding in a dose dependent
lesion is likely to persist because of repeated mu- fashion and they are more likely to cause compli-
cosal prolapse across the hiatus hernia. Surgical cations in their first month of use. Ulcer bleed-
repair of the hiatus hernia would result in defini- ing will resolve spontaneously in 75-80 percent
tive treatment. of patients. Patients who have bled from ulcers
are 10-20 times as likely to develop recurrent
REFERENCES ulcer bleeding than controls.
Kapadia S, Jagroop S, Kumar A. Cameron
ulcers: an atypical source for a massive upper REFERENCE
gastrointestinal bleed. World J Gastroenterol. Laine L, Jensen DM. Management of patients
2012 Sep 21;18(35):4959-61. with ulcer bleeding. Am J Gastroenterol.
Zullo A, Manta R, De Francesco V et al. Cam- 2012;107: 345-60.
eron lesions: A still overlooked diagnosis. Case
report and systematic review of literature. Clin Res
Hepatol Gastroenterol. 2018 Dec;42(6):604-609. Question 20
A 45-year-old man presents with hematemesis and
melena. He is healthy but takes ibuprofen regular-
Question 19 ly for arthralgias. He is tachycardic to 105 bpm but
Which of the following statements regarding his blood pressure is 130/72 mm Hg. His hemo-
peptic ulcers and bleeding is most accurate? globin is noted to be down three grams from when
it was checked during a routine health screening
A. Corticosteroid use alone is associated with seven months ago. You perform urgent upper
an increased risk of ulcer bleeding endoscopy and find a six mm ulcer in his duodenal
B. Even with documented healing, patients bulb. You remove adherent clot and discover a
who have bled from ulcers are 10 – 20 times non-bleeding visible vessel at the ulcer’s base.
332 Digestive Diseases Self-Education Program®
Which of the following statements regarding endo- bright red rectal bleeding. He sees the blood pri-
scopic therapy in this patient is most accurate? marily coating the surface of his stool. He does
not have abdominal pain nor diarrhea and he has
A. Endoclips should be avoided as they can in- not lost weight. His hemoglobin has remained
hibit ulcer healing stable. You perform colonoscopy which reveals
B. Since there is no active bleeding, epinephrine the following finding:
injection alone is appropriate
C. If multipolar coagulation is used, short burst
of high energy should be employed
D. Endoclips are more likely to provide definitive
hemostasis than injection alone
E. Thermal therapy is more likely to provide
definitive hemostasis than Endoclips
CORRECT ANSWER: D
RATIONALE
Given the presence of an ulcer with high-risk stig- This patient most likely has a history of which of
mata, (a visible vessel), endoscopic therapy should the following:
be employed. Endoclips have proven safe and
there is no evidence that they cause tissue injury A. Cirrhosis
or inflammation or that they impair ulcer heal- B. Systemic Sclerosis (Scleroderma)
ing. When the three main hemostatic modalities, C. Prostate Cancer
injection therapy, thermal therapy and mechanical D. Melanoma
therapy have been compared, injection alone has E. Heart Failure
been shown to be less effective than dual modality
therapy and to either mechanical or thermal thera- CORRECT ANSWER: C
py alone but no studies have shown thermal therapy
alone to be superior to mechanical therapy alone. RATIONALE
If multipolar coagulation is used, it should be used This patient’s rectal findings are consistent with
with low energy applied over a long duration. Radiation Proctitis which most commonly occurs
in male patients following treatment for prostate
REFERENCES cancer. It is important to be able to make this
Kovacs TOG, Jensen DM. Endoscopic treatment diagnosis visually, as biopsying this mucosa can
of peptic ulcer bleeding. Curr Treat Opt Gastroen- lead to fistulization and is therefore relatively
terol. 2007;10:143-148. contraindicated. Systemic sclerosis is associ-
Vergara M, Bennett C, Calvet X, Gisbert JP. ated with GAVE but not rectal telangiectasias
Epinephrine injection versus epinephrine injec- like these. Melanoma frequently metastasizes to
tion and a second endoscopic method in high risk the GI tract but it does not have this appearance.
bleeding ulcers. Cochrane Database Syst Rev 2014, Heart failure may be linked to GI tract angioecta-
Oct 13: CD005584. sias but not in a confluent pattern in the rectum
like this.
Question 21 REFERENCE
An otherwise healthy 82-year-old man presents Tabaja L, Sidani SM. Management of Radiation
with a several month history of intermittent Proctitis. Dig Dis Sci. 2018 Sep;63(9):2180-2188.
Chapter 12 — Gastrointestinal bleeding 333
Question 23
A 77-year-old man with CAD, COPD, prostate
You irrigate the site and see no evidence of un- cancer with prior radiation therapy and an aortic
derlying mass, mucosal ulceration or associated aneurysm repair presents with hematemesis fol-
angioectasia but the bleeding persists. lowed by hematochezia. He is tachycardic to the
Which of the following statements regarding this 110’s with an initial blood pressure of 84/60 mm
lesion is most accurate? Hg that improves to 95/73 mm Hg with one liter of
intravenous fluid. He is admitted to the ICU and
A. The bleeding is from a normal mucosal capil- you perform urgent Upper Endoscopy. His esopha-
lary gus and stomach appear normal but when you push
B. Thermal therapy should not be used because the scope to the third segment of the duodenum,
of a high perforation risk you see a large amount of fresh red blood and clot
C. The bleeding is from a dilated aberrant sub- that cannot be cleared adequately with lavage.
mucosal artery
D. This lesion more commonly occurs more dis- Which is the following is the best next step in this
tally in the GI tract patient’s management?
E. Since there is no ulcer, this lesion is unrelated
to his NSAID use A. Perform colonoscopy because he also present-
ed with hematochezia
CORRECT ANSWER: C B. Send him for red blood cell scintigraphy
C. Send him for mesenteric angiography
RATIONALE D. Place him on a proton pump inhibitor drip
This is a Dieulafoy lesion: a dilated aberrant sub- and repeat upper endoscopy in a few hours
mucosal artery that erodes the overlying epithelium E. Send him for a stat CT scan of his abdomen
in the absence of an associated ulcer. These lesions
occur most commonly in the proximal stomach as CORRECT ANSWER: E
334 Digestive Diseases Self-Education Program®
This primary concern in this case is that this pa- With voluminous hematemesis, especially from a
tient is bleeding from an aorto-enteric fistula. His proximal source like the esophagus, airway pro-
presentation, history of a AAA repair and finding tection is crucial so this patient should be intu-
of copious blood in the third portion of the duode- bated. Patients like this are at high risk to develop
num all suggest this diagnosis. Once considered, infected ascites so IV antibiotics should be given.
it is paramount to rule this diagnosis out quickly Antibiotics have been shown to decrease mortal-
as it has a very high mortality if not corrected im- ity in cirrhotic patients admitted with GI bleeding.
mediately. The most effective way to assess for an Somatostatin analogs decrease portal inflow by
A-E fistula is with an abdominal CT scan which causing splanchnic vasoconstriction and have been
can be obtained easily and performed quickly. proven to achieve hemostasis and decrease the risk
None of the other choices would be as an appropri- of rebleeding. One has to be cautious with resus-
ate next step in this patient’s management. citation efforts, as excessive resuscitation can lead
to accelerated bleeding due to increased portal
REFERENCE pressures. However, this patient’s Hemoglobin
Raman SP, Kamaya A, Federle M, Fishman EK. concentration is well below the threshold that war-
Aortoenteric fistulas: spectrum of CT findings. rants transfusion, so giving him PRBCs is appro-
Abdom Imaging. 2013 Apr;38(2):367-75. priate. In the acute setting of an upper GI bleed,
proton pump inhibitors work to help optimize
platelet function by increasing gastric pH. Since
Question 24 the source here is varices in the more pH neutral
A 52-year-old man with NASH-cirrhosis is admit- esophageal environment, intravenous PPI likely
ted to the ICU with red hematemesis and hemody- has little effect in the acute setting. However after
namic instability. For the past few months he has band ligation is performed, it may help decrease
been maintained on diuretics but has still required the risk of forming post-banding ulcers. Since this
frequent paracenteses for ascites management. An patient’s banding was performed a month ago, this
Upper Endoscopy four weeks ago revealed only episode of bleeding is more likely to be from recur-
large esophageal varices which were incompletely rent varices than from a post-banding ulcer.
eradicated with banding but the patient did not
show up for his scheduled repeat Upper Endos- REFERENCES
copy last week. His initial hemoglobin is 5.8 g/dL. Garcia-Tsao G, Sanyal AJ, Grace ND, et al. Preven-
His INR is 1.8 and his platelet count is 94K. tion and management of gastroesophageal varices
and variceal hemorrhage in cirrhosis. Hepatology
Which of the following treatment options is 2007; 46:922.
LEAST likely to benefit this patient? Tripathi D, Stanley AJ, Hayes PC, et al. U.K.
guidelines on the management of variceal haemor-
A. Intravenous proton pump inhibitor drip rhage in cirrhotic patients. Gut 2015; 64:1680.
B. Intravenous somatostatin analog
C. Endotracheal intubation
D. Intravenous antibiotics Question 25
E. Packed red blood cell transfusion A 27-year-old woman presents to the Emergency
Department with a history of melena. Her hemo-
CORRECT ANSWER: A globin is noted to be 11.2 g/dL with no prior values
for comparison. Her BUN is not elevated. She de-
RATIONALE nies abdominal pain or weight loss. She has been
It is important to understand the initial manage- training for a marathon and so has been taking
ment of patients with bleeding esophageal varices. ibuprofen several days per week. She is hemody-
Chapter 12 — Gastrointestinal bleeding 335
namically stable. You perform Upper Endoscopy associated with nonsteroidal antiinflammatory
which reveals a five mm clean-based ulcer in her drugs. Acid Suppression Trial: Ranitidine versus
gastric antrum and so she is discharged from the Omeprazole for NSAID-associated Ulcer Treat-
Emergency Department. Biopsies from the ulcer ment (ASTRONAUT) Study Group. N Engl J Med
margins show inflammatory changes only and ex- 1998; 338:719.
tensive biopsies from the rest of the stomach show ASGE Standards of Practice Committee,
no evidence of H. pylori. Banerjee S, Cash BD, et al. The role of endoscopy
in the management of patients with peptic ulcer
Which of the following is the most appropriate disease. Gastrointest Endosc 2010; 71:663.
management strategy for this patient?
ondary to H. pylori infection are typically man- include hemoglobin 8.2 g/dL and BUN 28 mg/
aged with antibiotic treatment of the infection in dL, creatinine 1.4 mg/dL. Two large bore IVs are
addition to PPI therapy, according to accepted placed, resuscitation with IV crystalloid solution is
management standards for H. pylori (covered initiated, along with an IV pantoprazole drip. The
elsewhere). An important issue arises when con- patient then undergoes an upper endoscopy. Find-
sidering the need for continued PPI therapy after ings reveal a scant amount of coffee ground materi-
treating the infection. Data supports the notion al within the stomach and duodenal bulb. Multiple
that eradication of H. pylori infection is superior small lesions are then identified in the duodenum,
to short-term and long-term anti-secretory ther- with a representative image shown.
apy in reducing recurrent ulcer bleeding risk and
that the overall rebleeding risk is low (1.5 percent)
once infection has been eradicated. In general,
once H. pylori eradication is documented and
ulcer disease healed, it is unnecessary to continue
anti-secretory therapy unless there is an addition-
al compelling indication for continued therapy.
Therefore, in this otherwise healthy asymptomatic
low risk patient, it would be best to discontinue
PPI therapy. If this patient had other indications
to continue PPI therapy, such as ongoing NSAID
requirement or had an idiopathic bleeding peptic Image Courtesy of Brandon Sprung, MD
ulcer, then maintenance PPI therapy could be
recommended. There is no clinical role for H2- Which of the following is the best endoscopic man-
blockers in this scenario. agement strategy for this finding?
Question 27 RATIONALE
A 64-year-old female presents to the Emergency The endoscopic findings in this scenario represent
Department with 24 hours of melena. Her history a clean-based duodenal bulb ulcer without high
includes atrial fibrillation managed on warfarin and risk stigmata of recent hemorrhage. This lesion is
rheumatoid arthritis on naproxen. On presentation, of a low rebleeding risk (less than 5 percent), and
heart rate 86 bpm, blood pressure 108/62 mmHg, therefore no endoscopic intervention is necessary.
oxygen saturation 99 percent on room air. Perti- Therefore, answer A is correct.
nent exam findings reveal irregular heart rate and The other answer choices would be reserved
a soft, non-tender, non-distended abdomen with for lesions with a high-risk of rebleeding, such as a
normoactive bowel sounds. Relevant lab studies non-bleeding visible vessel or active bleeding.
Chapter 12 — Gastrointestinal bleeding 337
hemorrhage (active bleeding, non-bleeding visible demic for H. pylori infection. When peptic ulcers
vessel or adherent clot) at initial endoscopy (OR are found at the time of endoscopy, they should be
= 0.67) and need to undergo endoscopic therapy classified according to their risk of rebleeding, of-
at that initial endoscopy (OR = 0.68). In this ten using the Forrest classification system. Specific
analysis, there were no significant differences in stigmata include active arterial spurting, active
mortality, re-bleeding or surgery between PPI and oozing, non-bleeding visible vessel, adherent clot,
control. Therefore, answer C is correct. flat pigmented spot, and clean base. The lesions
at highest risk for rebleeding, including active
REFERENCE bleeding and non-bleeding visible vessel, require
Sreedharan A, Martin J, Leontiadis GI et al. definite endoscopic therapy to achieve and main-
Proton pump inhibitor treatment initiated prior tain hemostasis. However, lesions at low risk for
to endoscopic diagnosis in upper gastrointestinal re-bleeding, including those with a flat pigmented
bleeding. Cochrane Database Syst Rev 2010 (7): spot (seven percent re-bleeding risk) or clean-base
CD005415. (three percent re-bleeding risk) are unlikely to
cause significant hemorrhage and re-bleeding and
do not require endoscopic therapy. In this case, an
Question 30 ulcer with a flat pigmented spot was found at the
A 52-year-old male originally from China pres- time of endoscopy, and therefore answer D is cor-
ents to the Emergency Department with fatigue rect. The other answers might be a good choice for
and epigastric abdominal pain for eight days. He lesions with higher risk stigmata of hemorrhage.
denies prior medical history and reports no history
of NSAID use or anti-coagulation. Vital signs are REFERENCE
normal. Exam reveals mildly tender epigastrium Laine L, Jensen, DM. Management of patients
without rebound or guarding. Hemoglobin is with ulcer bleeding. Am J Gastroenterol. 2012;107:
7.8 g/dL, BUN 18 mg/dL, creatinine 0.9 mg/dL. 345-60.
Intravenous PPI is initiated and he undergoes an
upper endoscopy which reveals a one cm ulcer in
the gastric antrum with a flat pigmented spot. Question 31
Which of the following is the appropriate endo- A 24-year-old male presents to the Emergency
scopic management for this patient’s ulcer? Department with two days of aching epigastric pain
and an episode of black stool one day ago. He has
A. Endoscopic hemoclip placement across the no significant past medical history other than a
ulcer edges recent sprained ankle for which he has been taking
B. Thermal probe therapy to the base of the ulcer ibuprofen four times daily for the past two weeks.
C. Epinephrine injection into the base of the On presentation, relevant vital signs are heart rate
ulcer 86 bpm, blood pressure 126/82 mmHg, SaO2 99
D. No endoscopic therapy is indicated percent on room air. He is alert and oriented and
E. Epinephrine injection plus endoscopic hemo- abdominal exam reveals mild tenderness over the
clip placement across the ulcer edges epigastrium without rebound or guarding, and nor-
moactive bowel sounds. Rectal exam reveals brown
CORRECT ANSWER: D stool. Hemoglobin is 13.9 g/dL. An upper endos-
copy is performed revealing a one cm clean-based
RATIONALE gastric antral ulcer with no high-risk stigmata of
This patient presents with concern for an upper bleeding. Stool antigen H pylori testing is negative.
GI source of anemia given his epigastric pain and Which of the following is the next best step in
tenderness and origin from a region highly en- management of this patient?
Chapter 12 — Gastrointestinal bleeding 339
A. Admit for 24 hour observation with intrave- ing or mortality, such as those with hemodynamic
nous PPI therapy changes, advanced age, significant co-morbidities,
B. Admit to the medical floor for monitoring and or high-risk stigmata of hemorrhage on endos-
intravenous PPI therapy copy are candidates for hospital admission.
C. Admit to the medical floor for monitoring and
oral PPI therapy REFERENCES
D. Admit to the intensive care unit for continuous Bethea ED, Travis AC, Saltzman JR. Initial assess-
intravenous PPI therapy ment and management of patients with nonvari-
E. Discharge home with oral PPI therapy ceal upper gastrointestinal bleeding. J Clin Gastro-
enterol. 2014 Nov-Dec;48(10):823-9.
CORRECT ANSWER: E Cipolletta L, Bianco MA, Rotondano G, et al.
Outpatient management for low-risk nonvariceal
RATIONALE upper GI bleeding: a randomized controlled trial.
This patient presents with a low risk (Forrest III) Gastrointest Endosc. 2002; 55:1–5.
clean-based gastric ulcer which is presumed to be Laine L, Jensen, DM. Management of patients
secondary to recent heavy NSAID use. Guidelines with ulcer bleeding. Am J Gastroenterol. 2012;107:
suggest performing a risk assessment on patients 345-60.
presenting with upper GI bleeding in order to Monteiro S, Gonçalves TC, Magalhães J, Cot-
assist management decisions such as timing of ter J. Upper gastrointestinal bleeding risk scores:
endoscopy, level or care and disposition. Who, when and why? World Journal of Gastroin-
There are several risk-assessment scores testinal Pathophysiology. 2016;7(1):86-96.
available both pre- and post-endoscopy to de-
termine a patient’s risk for rebleeding or mortal-
ity, which can directly affect the management. Question 32
This patient presents with normal vital signs, A 46-year-old male with no past medical his-
no co-morbidities and is of relatively young age, tory presents to the Emergency Department with
with normal blood work and no active melena. 24 hours of severe, massive hematochezia. Vital
In addition, no high-risk bleeding stigmata were signs are: heart rate 118 bpm, blood pressure
found on endoscopy. Therefore, for example, the 86/42 mmHg. Examination reveals a soft non-
patient’s complete Glasgow-Blatchford Score tender abdomen without distension. There are no
is 0 and AIM65 Score is 0, so he would be con- stigmata of liver disease on examination. Intrave-
sidered low risk. Since the patient is at low risk nous crystalloid and PPI therapy are initiated. An
for re-bleeding and significant morbidity or urgent endoscopy is planned to rule-out an upper
mortality from the gastric ulcer, it would be safe GI bleeding source.
to discharge home with oral PPI therapy. Ideal
candidates for discharge right after endoscopy are Which of the following is the most common cause
those with normal hemodynamics with a stable of severe upper GI hemorrhage?
blood count, no significant co-morbidities, and
who are reliable for follow-up. Evidence supports A. Peptic ulcer disease
that early discharge is safe in low risk peptic ulcer B. Esophageal varices
patients, with a very low rate of re-bleeding and C. Gastroduodenal erosive disease
a significantly lower cost compared to those who D. Mallory-Weiss tear
are hospitalized. Admission to the hospital and E. Angioectasia (AVM)
intravenous PPI is not necessary for such low risk
patients, therefore the other answers are incor- CORRECT ANSWER: A
rect. Patients who are at higher risk for re-bleed-
340 Digestive Diseases Self-Education Program®
epigastric tenderness on exam without rebound or fore, the best management strategy for this case is
guarding. Hemoglobin is 8.4 g/dL, BUN 38 mg/ repeat EGD for second attempt at ulcer hemostasis
dL, creatinine 0.9 mg/dL. Intravenous PPI infusion to attempt to reduce the need for surgery.
is initiated and EGD is performed, revealing fresh Answer A is not appropriate for this patient
blood in the stomach and duodenum with a large with recurrent active GI hemorrhage which re-
two cm duodenal bulb ulcer with an oozing bleed- quires intervention. Answer B is incorrect because
ing vessel. This was treated with epinephrine and there is no need to delay treatment in order to
hemoclip placement over the vessel. The patient did localize the bleeding location in this patient with
well initially, until 48 hours later when she devel- a known high-risk peptic ulcer. Answers D and
oped recurrent melena with transfusion requiring E are incorrect because surgical or angiographic
anemia and worsening hemoglobin to 5.6 g/dL. intervention for peptic ulcer bleeding should be
reserved for failure of a second endoscopic attempt
Which of the following is the most appropriate at treatment, or for other ulcer complications.
management strategy for this patient?
REFERENCE Question 38
Strate LL and Gralnek IM. ACG Clinical Guideline: A 68-year-old female with heart failure with re-
Management of Patients With Acute Lower Gas- duced ejection fraction, chronic renal insufficiency,
trointestinal Bleeding. Am J Gastroenterol 2016; and irritable bowel syndrome with constipation
111:459–474. presents with acute abdominal pain and hema-
tochezia. The patient describes developing sud-
den left-sided abdominal cramping followed by a
Question 37 brown bowel movement and six hours later devel-
A 64-year-old male with a history of prostate can- oped bright red blood per rectum. Vital signs are
cer treated with radiation therapy presents to your stable and lab testing reveals C-reactive protein 15
office with chronic painless bright red blood per mg/dL (normal 0-10 mg/dL) with normal lactate,
rectum. The patient is otherwise well. Hemoglo- CBC and blood chemistry. Abdominal exam re-
bin is 11.2 g/dL. A colonoscopy is performed after veals mild tenderness to palpation in the left lower
appropriate PEG bowel lavage. On colonoscopy, quadrant. A CT scan of the abdomen is obtained
the rectum is notable for scant fresh blood which and reveals colonic wall thickening and mild peri-
is washed to reveal multiple small actively oozing colonic stranding along the splenic flexure and
angioectasias. What is the next best step in endo- proximal descending colon. What is the most likely
scopic management of this patient? etiology for this patient’s presentation?
ischemia from low flow states (i.e. SMA and IMA B. Repeat upper endoscopy in 24 hours
watershed area). Early colonoscopy with biopsy C. Nuclear tagged RBC scan
within 48 hours is the next suggested diagnostic D. CT angiography
test to confirm the diagnosis, but is not absolutely E. Video Capsule Endoscopy
necessary in the right setting with supportive clini-
cal and radiographic information. Importantly, the CORRECT ANSWER: E
finding of isolated right colon ischemia on imaging
or colonoscopy is high-risk for poor clinical out- RATIONALE
comes including severe bleeding, need for surgical This patient presents with worsening symptom-
intervention and increased mortality and should atic anemia with suspected occult GI blood loss
prompt evaluation of the mesenteric vasculature. and a negative upper endoscopy and colonoscopy,
The other answer choices are less likely given the consistent with small bowel bleeding. The patient’s
patient’s acute classic symptoms, risk factors, and risk factors for GI bleeding primarily include
image findings. LVAD placement and anti-coagulation. Patients
with LVADs are known to be at high risk for devel-
REFERENCE opment of GI bleeding from angioectasias. Often
Brandt LJ, Feuerstadt P, Longstreth GF et al. ACG these lesions are located within the small bowel
Clinical Guideline: Epidemiology, Risk Factors, and present with occult bleeding and anemia, but
Patterns of Presentation, Diagnosis, and Manage- can also present with overt GI bleeding. Given this
ment of Colon Ischemia (CI). Am J Gastroenterol patient’s negative endoscopy and colonoscopy, a
2015; 110:18–44. small bowel source of bleeding should be sought.
Guidelines suggest that a video capsule endoscopy
(VCE) should be considered first-line to evaluate
Question 39 the small bowel for a source of bleeding after up-
A 54-year-old male with non-ischemic cardiomy- per and lower GI sources have been excluded with
opathy, status-post left ventricular assist device standard endoscopic evaluations. If a small bowel
(LVAD) placement six months ago, presents to the source of bleeding is identified on VCE which can
Emergency Department with shortness of breath be associated with anemia, endoscopic therapy is
and fatigue. Medications include warfarin and recommended.
lisinopril. On presentation, he is hemodynamically Answer A is incorrect because a small bowel
stable and awake and alert. Examination reveals source of anemia should be sought in this case and
an LVAD hum over the precordium and a soft it is not appropriate to correct INR in an LVAD
obese non-tender abdomen. Pertinent labs re- patient who is otherwise stable, due to risk for clot
veal hemoglobin 6.9 g/dL (baseline 9.8 g/dL four formation. Oral iron and clinical monitoring are
weeks prior), and INR 2.8. The patient denies me- an option if no source of bleeding is found after a
lena, hematochezia, or hematemesis. Stool is posi- thorough small bowel evaluation and the patient
tive for occult blood. The patient undergoes both remains stable. Answer B is incorrect because
an upper endoscopy to the distal duodenum and there is no clinical suggestion of recurrent overt
a colonoscopy the following day which are both upper GI bleeding to warrant a second look endos-
normal, without evidence for a cause of anemia. copy, and the yield is therefore low. Second look
endoscopy or push enteroscopy can be consid-
What is the next best step in management of this ered for recurrent hematemesis, melena or if the
patient? initial exam was incomplete. Answer C is incorrect
because tagged RBC scan is not likely to pick up on
A. Correct INR, start oral iron, and monitor for the very slow blood loss rate in this patient, and is
active bleeding or persistent anemia not the first line test to evaluate for suspected oc-
Chapter 12 — Gastrointestinal bleeding 345
cult small bowel bleeding. Tagged RBC scan can be CORRECT ANSWER: D
considered for active overt GI bleeding if angiog-
raphy is being considered or for occult small bowel RATIONALE
bleeding if VCE is not available or after a negative This patient presents with clinical concern for a
VCE and CT enterography. Answer D is incorrect small bowel source of anemia and GI blood loss.
because CT angiography is generally reserved for Standard evaluations with EGD, colonoscopy and
active overt brisk GI bleeding prior to considering VCE are negative. Guidelines suggest that the next
angiography. A CT enterography can also be con- best diagnostic test for this patient would be a
sidered for occult small bowel bleeding if the VCE CT enterography (CTE). CTE is suggested as the
is negative or if there is suspicion for small bowel next test to evaluate for small bowel bleeding after
obstruction, to rule-out small-bowel lesions. a negative VCE, due to its ability to detect and
locate small bowel masses and other enhancing
REFERENCE mucosal lesions.
Gerson LB, Fidler JL, Cave DR, et al. ACG Clini- Answer A is incorrect because a second look
cal Guideline: Diagnosis and Management of colonoscopy should be considered if there is
Small Bowel Bleeding Am J Gastroenterol 2015; recurrent hematochezia or if the first exam was
110:1265–1287. incomplete or poorly prepped. The yield is low
in this patient without overt bleeding and after
a recent well prepped complete colonoscopy.
Question 40 Answers B and C are incorrect because nuclear
A 78-year-old male smoker presents to your office tagged RBC scan and CTA are generally consid-
for evaluation of iron deficiency anemia and occult ered in patients with brisk or massive overt active
positive stool. He does not take any medications bleeding, which is not the case in this patient.
and is otherwise healthy. Vital signs and exam are Answer E is incorrect because barium studies are
normal. Pertinent labs reveal hemoglobin 9.8 g/ not recommended for the standard evaluation of
dL (baseline 14.6 g/dL), MCV 76 fL and low fer- small bowel bleeding.
ritin and iron levels. Celiac disease blood screen
is negative. The patient undergoes both an upper REFERENCE
endoscopy to second portion of the duodenum Gerson LB, Fidle JL, Cave DR, et al. ACG Clini-
and a well-prepped colonoscopy to the terminal cal Guideline: Diagnosis and Management of
ileum which are both normal, without evidence Small Bowel Bleeding Am J Gastroenterol 2015;
for a cause of anemia. A follow-up video capsule 110:1265–1287.
endoscopy is also normal, without any mucosal
lesions or blood identified. The patient returns for
a follow-up visit without any clinical change and Question 41
continues to be anemic. A 74-year-old female with a history of recur-
rent DVT on therapeutic warfarin presents to the
What is the next best step in management of Emergency Department with one hour of large vol-
this patient? ume bright red blood per rectum. Vital signs are:
heart rate 110 bpm, blood pressure 72/48 mmHg.
A. Repeat colonoscopy Examination reveals a pale, confused female in no
B. Nuclear tagged RBC scan acute distress, tachycardia, and a soft non-tender
C. CT angiography (CTA) abdomen without distension and no stigmata of
D. CT Enterography (CTE) liver disease. Labs reveal INR 2.0, hemoglobin 6.4
E. Barium upper GI series with small g/dL, platelets 180,000/uL. Intravenous access is
bowel follow through established and crystalloid resuscitation is initi-
346 Digestive Diseases Self-Education Program®
angiography is a recommended first-line diagnos- bleeding, passing bright red blood with clots in
tic study, and therefore answer D is correct. CTA the toilet every 30 minutes. She denies abdominal
is superior to EGD in diagnosing changes associ- pain, nausea, vomiting, melena, hematemesis,
ated with aortoenteric fistula (79 percent vs 25 diarrhea, or prior episodes of GI bleeding. She
percent). has never had a colonoscopy. On exam, vital signs
Patients at risk for aortoenteric fistulas include are normal and abdomen is soft and non-tender.
those with a known abdominal aortic aneurysm Hemoglobin is 11.6 g/dL (baseline 13.4 g/dL).
and/or prior aortic graft placement (surgical or en-
dovascular), history of aortic instrumentation and Which of the following is the most likely etiology
rarely patients with aortitis. These patients may of hematochezia in this patient?
present with an initial low-grade self-limited GI
bleed (“herald bleed”) which can be later followed A. Internal hemorrhoids
by a massive life threating hemorrhage. Other find- B. Colorectal cancer
ings can include abdominal pain and palpable ab- C. Angioectasia (AVM)
dominal mass. In one study, 55 percent of patients D. Ischemic colitis
presented with a “herald bleed” with a median time E. Diverticular disease
to diagnosis of 14 days after first bleeding. There-
fore, a high index of clinical suspicion is required CORRECT ANSWER: E
for this uncommon diagnosis, and it must be
promptly recognized and diagnosed. In general, the RATIONALE
clinician should have a high suspicion for aortoen- This patient presents with painless hematoche-
teric fistula in any patient with prior aortic surgery zia. While the differential diagnosis is broad, the
presenting with significant GI bleeding. most common cause of acute severe hematochezia
Answer A is incorrect as there was blood noted remains diverticular disease. The other answer
in the duodenum on second endoscopy, making choices can also cause painless hematochezia,
lower GI bleeding unlikely. Answers B, C and E are however they are not as common as diverticular
all options to investigate for a source of GI bleed- bleeding. Hemorrhoidal bleeding uncommonly
ing in various scenarios, however this patient has causes enough blood loss to lead to anemia.
a high clinical suspicion for an aortoenteric fistula, Colorectal cancer and angioectasias more com-
so these tests would not be the next best step in monly present with sub-acute or chronic blood
diagnosis for this patient. loss anemia, and less commonly severe hemato-
chezia. Ischemic colitis is an often encountered
REFERENCE cause of hematochezia in the elderly, however it is
Deijen CL, Smulders YM, Covelier, HME et al. still less common than diverticular bleeding, and
The Importance of Early Diagnosis and Treatment usually presents with acute onset of abdominal
of Patients with Aortoenteric Fistulas Presenting pain. Therefore, answer E is the best answer.
with Herald Bleeds. Annals of Vascular Surgery
2016:28-34. REFERENCES
Jensen DM, Machicado GA, Jutabha R et al. Ur-
gent colonoscopy for diagnosis and treatment
Question 43 of severe diverticular hemorrhage. N Engl J Med
A 72-year-old female with chronic constipation 2000;342:78-82.
presents with to the Emergency Department with Strate LL and Gralnek IM. ACG Clinical
four hours of rectal bleeding. She described being Guideline: Management of Patients With Acute
in her usual state of health until this morning, Lower Gastrointestinal Bleeding. Am J Gastroen-
when she developed acute onset of heavy rectal terol 2016; 111:459–474.
348 Digestive Diseases Self-Education Program®
ing his first screening colonoscopy seven days ago high-risk endoscopic stigmata of bleeding, includ-
and was told that there was a large polyp re- ing active bleeding, non-bleeding visible vessel
moved. He resumed warfarin that evening. Today or possibly adherent clot. Options for endoscopic
he woke up and passed three large bloody bowel hemostasis of post-polypectomy bleeding from
movement with clots. At presentation, heart rate high-risk ulcers include epinephrine, mechani-
112 bpm, blood pressure 108/62 mm Hg, exam cal clips, electrocoagulation and band ligation.
reveals a diaphoretic male with a soft non-tender Epinephrine should not be used alone, and should
abdomen. Labs are notable for hemoglobin 10.6 instead be used to control initial active bleeding,
g/dL (baseline 14.2 g/dL), BUN 10 mg/dL, cre- reduce bleeding risk from application of mechani-
atinine 0.8 mg/dL, INR 2.2. Intravenous access cal/thermal therapy and to improve visualiza-
is obtained and crystalloid resuscitation is initi- tion. The use of mechanic or thermal therapy for
ated. The patient undergoes a rapid bowel prep post-polypectomy bleeding is recommended, with
and colonoscopy, which reveals a 14 mm ulcer in or without epinephrine. However, mechanical clip
the ascending colon with a non-bleeding visible placement is often preferred over thermal contact
vessel. therapy to limit thermal tissue injury to the colon
wall (especially in the setting of ongoing use of
Which of the following is the most appropriate anticoagulation). Therefore, answer D is the most
next step in management for this patient? appropriate next step.
CORRECT ANSWER: D
Question 46
RATIONALE A 62-year-old female presents for evaluation of he-
This patient’s clinical scenario is consistent with matemesis. She has a history of recent myocardial
post-polypectomy bleeding after recent large pol- infarction and placement of bare metal coronary
ypectomy. Patients with large polyps, particularly stent, maintained on aspirin and clopidogrel. On
in the right colon and those on anti-thrombotic presentation, heart rate is 120 bpm, blood pres-
therapy, are at particularly high risk for post- sure 98/62 mmHg. Examination reveals tachy-
polypectomy bleeding. Bleeding can occur up to cardia with mildly tender epigastrium without re-
30 days after polypectomy, but in general is seen bound or guarding. Hemoglobin is 6.2 g/dL, BUN
five to seven days later. The causative mechanism 48 mg/dL, creatinine 1.3 mg/dL. Two large-bore
is likely related to sloughing of eschar over a vessel IVs are placed, fluid and blood product resuscita-
and necrosis and ulceration into the submucosa tion and intravenous continuous PPI is initiated.
after thermal polypectomy. After initial resuscita- An urgent upper endoscopy reveals old blood in
tion, a bowel prep and colonoscopy is indicated, the stomach and a 15 mm duodenal bulb ulcer.
provided the patient is stable. Similar to upper GI Which of the following endoscopic stigmata of
bleeding, endoscopic therapy is recommended for peptic ulcer disease at time of endoscopy requires
patients with lower GI bleeding from lesions with definitive endoscopic therapy?
350 Digestive Diseases Self-Education Program®
A. Clean-base Question 47
B. Flat pigmented spot A 74-year-old male presents for evaluation of
C. Firm mature adherent clot melena and coffee ground emesis for 24 hours. He
D. Non-bleeding visible vessel has a history of end-stage renal disease on hemo-
dialysis and takes aspirin. On presentation, heart
CORRECT ANSWER: D rate is 86 bpm, blood pressure 106/68 mmHg.
Examination is unrevealing. Hemoglobin is 7.6 g/
RATIONALE dL (baseline 10.2 g/dL). Two large-bore IVs are
Stigmata of bleeding seen during endoscopy for placed, fluid resuscitation and intravenous contin-
peptic ulcer disease are important predictors of uous PPI are initiated. An urgent upper endoscopy
rebleeding, surgery and mortality and are used to reveals several gastric ulcers from eight to 12 mm
guide the decision for and choice of endoscopic in size, including an antral ulcer with an overlying
therapy. Lesions at highest risk for rebleeding, blood clot.
including those with active arterial spurting and
non-bleeding visible vessel, require endoscopic Which of the following is the best next step in
therapy to achieve hemostasis and reduce the risk endoscopic management of this ulcer?
for rebleeding. Lesions with an adherent clot are
at intermediate risk for rebleeding and therefore A. No endoscopic therapy indicated for overlying
can be considered for endoscopic therapy depend- blood clots
ing on the clinical scenario. However endoscopic B. Epinephrine injection into clot base and pe-
therapy is not required for lesions with an adher- riphery alone
ent clot. Data is inconsistent as to the benefit of C. Hemoclip placement over the clot followed by
endoscopic therapy with lesions with adherent attempted clot removal with snare
clots, and much of the therapeutic indication D. Epinephrine injection followed by irrigation
depends on the presence or absence of an under- and polypectomy snare clot removal
lying high-risk lesion, the clinical scenario and E. Thermal heater probe therapy applied with
patient specific risk factors. Doppler endoscopic gentle pressure through the clot to ulcer bed
probe may be used to evaluate for underlying arte-
rial flow, and if it is present, endoscopic therapy CORRECT ANSWER: D
should be performed to decrease risk of rebleed-
ing. Low risk lesions, specifically those with a flat RATIONALE
pigmented spot or a clean-base, do not require Peptic ulcers with adherent clots are at inter-
endoscopic therapy. mediate risk for re-bleeding without endoscopic
therapy (20-30 percent). Clots tend to be soft
REFERENCES and acute, or mature and firm. The indication
de Groot, N.L et. al. Reassessment of the predic- and ability to treat these lesions relies on several
tive value of the Forrest classification for peptic factors, including clinical stability of the patient,
ulcer rebleeding and mortality: can classification patient specific risk factors for rebleeding, and
be simplified? Endoscopy 2014; 46(01): 46-52. adequate endoscopic positioning and access to the
Laine L, McQuaid KR. Endoscopic therapy for ulcer. While data is variable, some studies sug-
bleeding ulcers: an evidence-based approach based gest a rebleeding benefit to endoscopic treatment
on meta-analyses of randomized controlled trials. of adherent clots versus medical therapy alone.
Clin Gastroenterol Hepatol 2009; 7: 33 – 47. Therefore, guidelines suggest that consideration
Laine L, Jensen, DM. Management of patients be made to treat these lesions with endoscopic
with ulcer bleeding. Am J Gastroenterol. 2012;107: therapy when feasible. The recommended ap-
345-60. proach to managing these lesions includes vigor-
Chapter 12 — Gastrointestinal bleeding 351
ous washing to attempt to dislodge the overlying management strategy given this patient’s hemato-
clot, followed by attempt to remove the clot with logic abnormalities?
cold polypectomy snare or a grasping device such
as forceps. This can be preceded by epinephrine A. Transfuse packed red blood cells to goal he-
injection around the ulcer base to assist with moglobin greater than 10 g/dL
hemostasis. Following adequate clot removal, the B. Transfuse packed red blood cells if hemoglo-
clinician can make an assessment of any underly- bin drops less than 7 g/dL
ing stigmata of hemorrhage and determine the C. Transfuse platelets for goal greater than
need for definitive endoscopic therapy. Answer 100,000/uL
A is incorrect, as the endoscopist should further D. Transfuse fresh frozen plasma and administer
assess the clot and underlying ulcer before deter- oral vitamin K to achieve INR less than 1.5
mining that there is no need for additional thera-
py. Answer B is incorrect as epinephrine alone is CORRECT ANSWER: B
not a preferred treatment modality for peptic ul-
cers with stigmata of hemorrhage. Answers C and RATIONALE
E are incorrect, as mechanical or thermal therapy It is important for gastroenterologists to be able
of the adherent clot itself is not going to treat the to assist in determination of transfusion support
underlying lesion and cause for the clot. goals of patients with GI hemorrhage. Guidelines
and evidence are available to support specific
REFERENCES transfusion strategies in patients with acute GI
Kahi CJ, Jensen DM, Sung JJ, et al. Endoscopic bleeding. In a landmark study of patients with up-
therapy versus medical therapy for bleeding peptic per GI bleeding, a restrictive transfusion strategy
ulcer with adherent clot: a meta-analysis. Gastro- with a threshold hemoglobin less than seven g/dL
enterology. 2005; 129:855-62. improved outcomes (survival, rebleeding, adverse
Laine L, Jensen, DM. Management of patients events) compared with a threshold of nine g/dL
with ulcer bleeding. Am J Gastroenterol. 2012;107: in severe acute upper GI bleeding. Therefore, a re-
345-60. strictive transfusion goal set at hemoglobin great-
er than seven g/dL is generally preferred over a
liberal strategy, unless there is significant active
Question 48 cardiovascular disease which would support a
A 48-year-old female with history of DVT on hemoglobin goal of nine to 10 g/dL. Patients with
warfarin and acute myelogenous leukemia status- a hemodynamically significant active massive GI
post stem cell transplant presents with melena and hemorrhage may be transfused blood products
coffee ground emesis for 24 hours. Vital signs are without a specific hemoglobin goal in the acute
normal. Labs reveal: hemoglobin 8.6 g/dL, plate- setting, to assist tissue perfusion and volume
lets 80,000/uL and INR 2.1. Intravenous access is support. In addition, patients with symptomatic
obtained, IV fluids and PPI are initiated and she anemia or peripheral or cerebrovascular disease
undergoes an upper endoscopy which reveals a 12 may require different transfusion thresholds.
mm ulcer in the duodenal bulb with a non-bleed- In general, expert opinion recommends to main-
ing visible vessel, which is treated with combina- tain a platelet count of greater than or equal to
tion epinephrine and thermal contact therapy with 50,000/uL in patients with significant bleed-
excellent effect. Gastric biopsies are obtained to ing. There is no recommendation for platelet
evaluate for H. pylori. The patient is admitted to count greater than 100,000/uL in the setting of
the medical floors and the medical team asks if the GI bleeding, and therefore this patient does not
patient requires any blood product transfusions. require a platelet transfusion. Evidence supports
Which of the following is the best blood product the notion that there is no increased rebleeding
352 Digestive Diseases Self-Education Program®
REFERENCE
Gerson LB, Fidler JL, Cave DR, Leighton JA. ACG
Clinical Guideline: Diagnosis and Management of
Small Bowel Bleeding Am J Gastroenterol 2015;
110:1265–1287.
Answers & critiques
CHAPTER 13
355
356 Digestive Diseases Self-Education Program®
group were more likely to achieve clinical remission tent courses of budesonide for several years until
(p=0.1) and endoscopic remission (p=0.43). she eventually proceeded with ileocecal resection
Few small studies, including three RCTs investigat- six months ago. Immunomodulator and biologic
ed the role cannabis in IBD patients, and revealed therapies were discussed prior to her resection,
better quality of life and general health perception, but she elected to proceed with surgery due to
but no impact on rates of remission, disease activity concerns of potential adverse effects related
scores or inflammatory markers. to the therapies. Recent blood tests revealed
VSL#3 is the most-studied probiotic in IBD. normal white cell count, hemoglobin and MCV.
Its role is more established in patients with ileal Both C-reactive protein and fecal calprotectin are
pouch-anal anastomosis (IPAA), where VSL#3 was within normal limits. Her six-month postopera-
found to be superior to placebo in reducing risk of tive ileocolonoscopy revealed focal ulceration at
first episode of pouchitis as well as reducing rates of the anastomosis with normal colonic and neo-
recurrence after an episode of pouchitis. In mild to terminal ileal mucosa [see figure]. Which of the
moderate ulcerative colitis patients, a recent meta- following is the most appropriate next step in
analysis concluded that the use of VSL#3 as an management?
add on to conventional therapies is safe and more
effective than conventional therapy alone in achiev-
ing clinical response and remission. In a small
pediatric cohort of ulcerative colitis patients, the
addition of VSL#3 to 5-ASA was superior to 5-ASA
alone in maintaining remission. In Crohn’s dis-
ease patients, the evidence is more limited; based
on two recent meta-analyses, VSL#3 has no clear
role in inducing remission or preventing relapse in
patients with Crohn’s disease
LDN has been primarily evaluated in patients
Ileoanal anastomosis
with Crohn’s disease and the evidence was summa-
rized in a recent Cochrane review, which concluded
that the current evidence is insufficient to allow
any firm conclusion regarding efficacy and safety of
LDN in patients with active Crohn’s disease.
REFERENCE
Lin, S.C. and A.S. Cheifetz, The Use of Comple-
mentary and Alternative Medicine in Patients
With Inflammatory Bowel Disease. Gastroenterol
Hepatol (N Y), 2018. 14(7): p. 415-425.
Neoterminal ileum
A. Intermittent use of ciprofloxacin for perianal trimester, has been linked to cleft lip/palate and
fistula symptoms premature rupture of membranes. This is another
B. Continue both adalimumab and azathioprine reason to continue medical therapy during preg-
during pregnancy nancy in an effort to reduce the chance of disease
C. Stop azathioprine and continue adalimumab relapse that could require systemic corticosteroid
monotherapy therapy.
D. Stop adalimumab and continue azathioprine A few studies have reported worsening peri-
during pregnancy anal disease after vaginal delivery. Furthermore,
E. Vaginal delivery is preferred over C-section in active perianal disease has been associated with
this patient increased risk of fourth degree laceration with
vaginal deliveries; therefore, in patients with ac-
CORRECT ANSWER: B tive perianal disease, C-section delivery is recom-
mended.
RATIONALE
In women with IBD, ongoing disease activity car- REFERENCE
ries the highest risk to pregnancy outcome. Stud- Nguyen, G.C., et al., The Toronto Consensus
ies have shown an increased risk of preterm birth, Statements for the Management of Inflammatory
low birth weight, stillbirth and miscarriages or Bowel Disease in Pregnancy. Gastroenterology,
abortions in patients with active disease at con- 2016. 150(3): p. 734-757.e1.
ception and during pregnancy. Therefore, keep-
ing the disease under control during pregnancy
is very important. In this high-risk patient who Question 6
failed infliximab and has perianal disease, discon- A 24-year-old medical student with a history of
tinuing her medical therapy increases her risk of extensive ulcerative colitis is currently maintained
disease relapse/flare, which could have a more on adalimumab 40 mg every two weeks. He is
adverse impact on her pregnancy than continuing preparing to go on a mission trip to Uganda and
her medications. Hence, continuing combination presents today to discuss appropriate preventive
therapy during pregnancy is the most appropriate measures prior to his trip.
medical option and the correct answer for this
question. The majority of available IBD therapies Which of the following preventive measures does
have been shown to be safe during pregnancy the patient need to avoid?
with no significant impact on the mother or the
fetus. This includes TNF-inhibitors, thiopurines, A. Hepatitis A vaccine
and most of the 5-ASA compounds. Vedolizumab, B. Malaria prophylaxis
ustekinumab and tofacitinib also appear to be safe C. Typhoid VI capsular polysaccharide vaccine
during pregnancy, although the available safety D. Yellow fever vaccine
data for these medications are more limited. On E. Influenza vaccine
the other hand, methotrexate is teratogenic and
should be avoided at conception and during preg- CORRECT ANSWER: D
nancy. Metronidazole may be associated with cleft
lip and palate and should be avoided during the RATIONALE
first trimester of pregnancy. Ciprofloxacin should Live-attenuated vaccines are contraindicated in
also be avoided during pregnancy due to the patients receiving immunosuppressive therapies.
potential risk of skeletal deformities in the fetus. These include measles, mumps, rubella, varicella,
Systemic corticosteroids can be used to control herpes zoster live-attenuated vaccine (ZVL), and
flares; however, their use, especially in the first the yellow fever vaccine. The yellow fever virus
360 Digestive Diseases Self-Education Program®
is transmitted to people by the bite of an infected evaluation revealed a mildly elevated C-reactive
mosquito. While the majority of infected pa- protein, WBC of 3.6 thousand/uL, hemoglobin of
tients have no symptoms or report mild/transient 8 g/dL, MCV of 76 fl, platelet count 430,000/mL,
symptoms of viral illness (fatigue, headache, fever, AST 67 U/L, ALT 90 U/L, and normal alkaline
etc.), some have a severe form of the disease with phosphatase and total bilirubin. Azathioprine
multiorgan failure and death. Therefore, patients metabolites included 6-TGN 300 pmol/8x108 RBC
who are unable to receive the yellow fever vac- ( normal 230-450 pmol/8x108 RBC and 6-MMP
cine should be discouraged from traveling to an 6200 pmol/8x108 RBC ( normal less than 5700
endemic region. They should also consult with pmol/8x108 RBC)
an infectious disease specialist or traveler’s clinic
to discuss potential risks of exposure to yellow Which of the following is the most appropriate
fever virus without immunization and review next step in this patient’s management?
other protective measures (use of insect repel-
lent, wearing protective gear, and avoiding travel A. Stop azathioprine and start infliximab 5mg/
during the rainy season). Influenza, Hepatitis A kg at weeks 0, two and six, then every eight
and typhoid VI capsular polysaccharide vaccines weeks
are non-live vaccines and can be administered to B. Continue azathioprine at the current dose and
this patient. None of the available medical options start oral mesalamine therapy
for malaria prophylaxis (doxycycline, mefloquine, C. Increase the dose of azathioprine to 2.5 mg/
primaquine, chloroquine and hydroxychloroquine, kg/day
and atovaquone-proguanil) is contraindicated in D. Reduce the azathioprine dose by half and
patients with IBD or those receiving TNF-inhibi- repeat liver tests in one week
tors therapies. E. No changes in therapy are warranted now, to
allow more time for the azathioprine to work
REFERENCE
Farraye, F.A., et al., ACG Clinical Guideline: Pre- CORRECT ANSWER: A
ventive Care in Inflammatory Bowel Disease. Am J
Gastroenterol, 2017. 112(2): p. 241-258. RATIONALE
The patient has clinical and biomarker evidence
of ongoing disease activity despite a therapeutic
Question 7 6-TGN level (230-450 pmol/8x108 RBC); this
A 36-year-old man with a recent diagnosis of small suggests failure of thiopurine therapy. Fur-
bowel Crohn’s disease presents for evaluation. thermore, the patient has an elevated 6-MMP
He initially presented with non-bloody diarrhea level (more than 5700 pmol/8x108 RBC) in
and weight loss. Ileocolonoscopy showed normal association with laboratory changes concern-
colonic mucosa and diffuse inflammatory changes ing for possible hepatotoxicity (elevated liver
with ulcers in the terminal ileum. Video capsule enzymes). The most appropriate therapy choice
endoscopy (VCE) showed active inflammation at this time is to change therapeutic class, such
involving the jejunum and more proximal ileum as as an anti-TNF agent. Both monotherapy with
well. Therapy with budesonide and azathioprine 2 an anti-TNF agent or combination therapy using
mg/kg/day was initiated. He completed a three- a lower dose of thiopurine would be reasonable
month budesonide taper and on follow up one options.
month later, reports ongoing non-bloody diarrhea The role of mesalamine in CD was recently
and abdominal cramps despite 12 weeks of aza- assessed in two Cochrane reviews that showed
thioprine therapy. He tested negative for Clostrid- no significant benefit over placebo in induction
ium difficile toxins in the stool. Further laboratory or maintenance of remission in CD patients.
Chapter 13 — Inflammatory bowel disease 361
Increasing the azathioprine dose is unlikely to active colitis with focal areas indefinite for dys-
provide any benefit and will increase risk of toxic- plasia in the sigmoid colon biopsies. The changes
ity. His WBC is already low, so he would be at were confirmed by a second expert pathologist.
increased risk for worsening leukopenia, as well
as worsening hepatotoxicity, with an increase in
the azathioprine dose. Reducing the azathioprine
without adding a second therapeutic agent would
not address the patient’s disease activity and
hence option D is also incorrect. Finally, although
thiopurines have a slow onset of action and may
take up to six to 12 weeks to achieve full clinical
response, it is unlikely any additional benefit will
be observed after 12 weeks, especially in patients
with a therapeutic 6-TGN level.
Which of the following is the most appropriate
REFERENCES intervention?
Vande Casteele, N., et al., American Gastroentero-
logical Association Institute Technical Review on A. Initiate combination oral and rectal 5-ASA
the Role of Therapeutic Drug Monitoring in the therapy and schedule repeat surveillance colo-
Management of Inflammatory Bowel Diseases. noscopy in three months
Gastroenterology, 2017. 153(3): p. 835-857.e6. B. Refer the patient for total colectomy with ileal
Dubinsky, M.C., et al., Pharmacogenomics and pouch anal anastomosis (IPAA)
metabolite measurement for 6-mercaptopurine C. Refer the patient for segmental resection of
therapy in inflammatory bowel disease. Gastroen- the sigmoid colon
terology, 2000. 118(4): p. 705-13. D. Repeat colonoscopy with chromoendoscopy now
E. Initiate combination therapy with infliximab
and azathioprine and schedule repeat colonos-
Question 8 copy in three to six months
A 45-year-old man with a history of extensive
ulcerative colitis diagnosed 10 years ago presents CORRECT ANSWER: A
to clinic. He was treated initially with oral mesala-
mine with good clinical response, but after a year RATIONALE
of therapy was lost to follow up and eventually Indefinite dysplasia usually represents atypia with
self-discontinued the mesalamine. He did well off confounding background inflammation, mak-
therapy until one month ago when he experienced ing it difficult for the pathologist to confirm if the
recurrent diarrhea with three to four loose bowel changes represent a secondary effect of inflamma-
movements per day with urgency and intermittent tion or true dysplasia. This finding on histology
rectal bleeding. Stool studies were negative for warrants close follow up after optimizing medical
any infectious process and his laboratory evalu- therapy. Therefore, option A is the most appropri-
ation revealed a normal CRP, Hgb of 12.1 g/dl, ate answer for this case.
MCV of 80 fL and normal liver enzymes. He is a Options B and C are incorrect as it is too early
nonsmoker and has no family history of colon can- to recommend colectomy. However, if invis-
cer. Colonoscopy using high-definition white light ible dysplasia is confirmed after controlling the
endoscopy (HD-WLE) showed mild to moderate inflammation, then colectomy should be consid-
pancolitis (see picture). Random biopsies obtained ered, especially in cases with multifocal dysplasia
from each segment of the colon showed chronic or high-grade dysplasia. Total colectomy is the
362 Digestive Diseases Self-Education Program®
recommended approach given the high risk of Virtual Chromoendoscopy for Detection of Co-
both synchronous and metachronous lesions. lonic Neoplastic Lesions During IBD Surveillance
Segmental colectomy can be considered in certain Colonoscopy. Am J Gastroenterol, 2018. 113(2): p.
situations on a case by case basis, for example in 225-234.
those who are not felt to be candidates for more
extensive surgery or those who are unable to
care for an ostomy, etc. Chromoendoscopy is a Question 9
technique that uses dye to enhance visualization You are asked to consult on a 27-year-old man
and dysplasia detection during surveillance. The hospitalized with severe bloody diarrhea, up to 15
Surveillance for Colorectal Endoscopic Neoplasia bowel movements per day, abdominal pain, nau-
Detection and Management in IBD patients: inter- sea and low-grade fevers for the last two weeks.
national Consensus (SCENIC) group recommend- He was diagnosed with ulcerative pancolitis five
ed use of chromoendoscopy whenever colonos- years ago and was maintained on mesalamine
copy is performed with standard-definition white 4.8 g/day and azathioprine 2mg/kg/day until
light endoscopy. The SCENIC group also suggest- six months ago when he lost his insurance and
ed chromoendoscopy be used in conjunction with stopped all medical therapy. On examination,
high definition white light endoscopy. However, he appears uncomfortable, HR 95 beats/minute
option D is incorrect because optimization of and temperature 37.9 °C. His abdomen is not
therapy should be achieved prior to repeating distended, but he has tenderness to palpation dif-
the endoscopic evaluation, even if chromoendos- fusely, worse in the left lower quadrant. His labs
copy is performed. Finally, the patient has mild reveal a WBC of 8,000/mL, Hgb 8.1 g/dl, albumin
disease with a history of good clinical response to 2.7 g/dl, and CRP 18 mg/L (normal: less than 0.3).
mesalamine therapy in the past; therefore, resum- What is the most appropriate next step?
ing mesalamine therapy is appropriate and escala-
tion to a biologic or thiopurine is not indicated at A. Send stool sample to test for Clostridium dif-
this time. Combining oral and rectal mesalamine ficile toxins
therapies has been shown to produce earlier and B. Initiate infliximab 5mg/kg
more complete relief of rectal bleeding compared C. Initiate IV methylprednisolone 300mg/day
with oral or rectal therapy alone. D. Obtain an abdominal ultrasound
E. Discontinue his prophylactic heparin, given
REFERENCES his anemia and rectal bleeding
Cairns, S.R., et al., Guidelines for colorectal cancer
screening and surveillance in moderate and high CORRECT ANSWER: A
risk groups (update from 2002). Gut, 2010. 59(5):
p. 666-89. RATIONALE
Laine, L., et al., SCENIC international consen- The patient’s presentation is consistent with acute
sus statement on surveillance and management of severe ulcerative colitis. Superimposed infections
dysplasia in inflammatory bowel disease. Gastro- should be excluded in all patients; this typically
enterology, 2015. 148(3): p. 639-651.e28. involves obtaining stool samples to assess for
Bisschops, R., et al., Chromoendoscopy versus Clostridium difficile infection and other GI patho-
narrow band imaging in UC: a prospective ran- gens. Intravenous systemic corticosteroid is the
domised controlled trial. Gut, 2018. 67(6): p. initial therapy of choice, typically with a dose of 60
1087-1094. mg/day of methylprednisolone or equivalent. If
Iacucci, M., et al., A Randomized Trial Com- no clear clinical response after 72 hours of cortico-
paring High Definition Colonoscopy Alone With steroid therapy, infliximab or cyclosporine rescue
High Definition Dye Spraying and Electronic therapy should be considered. Patients with IBD
Chapter 13 — Inflammatory bowel disease 363
have a three to four-fold increase in risk of venous through the electronic health records system. She
thromboembolism. This risk is even higher in the indicates that the red lesions in the pictures de-
setting of active disease, hospitalization, and cor- veloped two days ago and are limited to her lower
ticosteroid therapy. All hospitalized IBD patients extremities. They are raised and painful. She has
should be placed on pharmacologic DVT prophy- noted no change in her bowel symptoms and denies
laxis. In addition, rectal bleeding and anemia are fever or any other accompanying symptoms.
not contraindications for DVT prophylaxis.
Which of the following is an appropriate next step
REFERENCES in management?
Kornbluth, A. and D.B. Sachar, Ulcerative colitis prac-
tice guidelines in adults: American College Of Gas- A. Increase sulfasalazine to 4 g/day
troenterology, Practice Parameters Committee. Am J B. Add azathioprine to her current dose of
Gastroenterol, 2010. 105(3): p. 501-23; quiz 524. sulfasalazine
Grainge, M.J., J. West, and T.R. Card, Ve- C. Obtain a skin biopsy of the lesions
nous thromboembolism during active disease and D. Switch her therapy from sulfasalazine to
remission in inflammatory bowel disease: a cohort mesalamine
study. Lancet, 2010. 375(9715): p. 657-63. E. Start prednisone 60 mg daily
Kappelman, M.D., et al., Thromboembolic risk
among Danish children and adults with inflamma- CORRECT ANSWER: D
tory bowel diseases: a population-based nation-
wide study. Gut, 2011. 60(7): p. 937-43. RATIONALE
The described lesions are mostly consistent with
erythema nodosum (EN). EN is a form of pan-
Question 10 niculitis and represents the most common cutane-
A 51-year-old female diagnosed with mild left-sided ous manifestation associated IBD, occurring in up
ulcerative colitis two months ago was placed on to 10 percent of the cases. When EN is associated
sulfasalazine 2 g/day and daily folic acid with sig- with IBD, it usually parallels disease activity and
nificant improvement in her symptoms. At her last management should focus on treating the IBD.
clinic visit a week ago, she was having two formed EN can also be seen in association with several
non-bloody stools per day without urgency and her other conditions and drugs (see table below).
labs revealed a normal CBC, CRP and normaliza- Thus, not every EN presentation in IBD patients
tion of the previously elevated fecal calprotectin. is related to the bowel disease. This patient is in
Since then, she sent you the pictures shown below clinical remission and most likely endoscopic re-
mission given normalization of her inflammatory
markers. She has no other associated symptoms
to suggest any systemic disease or infection. Her
only change in medications was the initiation of
sulfasalazine two months ago at the onset of her
disease. So, it is reasonable to consider sulfasala-
zine as a potential cause of her EN. In the case of
sulfasalazine, the sulfonamide component of the
drug is the presumed trigger for EN, so switching
to a mesalamine agent is reasonable. There is no
indication to increase the sulfasalazine or add aza-
thioprine therapy at this time. While skin biopsy
can be helpful to confirm the diagnosis, in the ma-
364 Digestive Diseases Self-Education Program®
jority of cases, the diagnosis can be made based habits and no fever or chills. Labs reveal an
on the clinical presentation without the need elevated lipase level of 800 U/L and normal AST,
for biopsy, as in this case. In regard to manage- ALT, alkaline phosphatase and total bilirubin.
ment, EN is usually a self-limited condition and Her lipid panel and electrolytes are normal. She
typically resolves spontaneously without treat- denies any alcohol use. CT scan of the abdomen
ment. Therefore, the first line of therapy is often reveals edema in the peripancreatic fat. Choleli-
supportive care with compression bandages, thiasis was also noted, without choledocholithia-
elevation of the legs, and PRN NSAID use along sis or common bile duct (CBD) dilatation.
with addressing any potential treatable or modi-
fiable (i.e. medication) cause. In more severe In addition to the usual supportive care, which
cases, short courses of systemic corticosteroids of the following is appropriate with regard to
can be used, usually at 20 mg daily of predni- management of her condition?
sone. Higher doses are rarely required. Other
therapies that can be used include colchicine, A. She should have cholecystectomy before
dapsone, hydroxychloroquine, TNF-inhibitors, discharge
thalidomide and methotrexate. B. Discontinue budesonide
C. Discontinue 6-MP and start azathioprine
REFERENCE after recovery from the current episode
Garcia-Porrua, C., et al., Erythema nodosum: D. Discontinue 6-MP and consider initiating
etiologic and predictive factors in a defined biologic therapy after recovery from the
population. Arthritis Rheum, 2000. 43(3): p. current episode
584-92. E. Discontinue budesonide and 6-MP and
initiate high dose intravenous corticosteroid
therapy
Question 11
A 45-year-old female with a recent diagnosis of CORRECT ANSWER: D
inflammatory ileocolonic CD presents for evalu-
ation. After reviewing all therapy options with RATIONALE
her, she agrees to proceed with a budesonide ta- This patient’s presentation is consistent with
per and 6-MP 1.5 mg/kg/day. Three weeks after drug-induced pancreatitis. Among all the IBD
initiating therapy, she presents to the emergency therapies, thiopurines are the drugs most fre-
room with sharp epigastric pain radiating to the quently implicated as a cause of acute pancreati-
back. She reports nausea and vomiting in asso- tis. This reaction is usually a dose-independent
ciation with the pain, but no changes in her bowel idiosyncratic reaction, and is considered a
Chapter 13 — Inflammatory bowel disease 365
contraindication for the use of any thiopurine patient underwent cystoscopy with left ureteral
agent. Less frequently, 5-ASA can also cause stent placement.
acute pancreatitis and in extremely rare reports,
metronidazole and steroid therapy have been He presents today for follow up and is wonder-
implicated. In this patient, the acute pancreati- ing what he can do to reduce the risk of further
tis is most likely caused by the 6-MP therapy. stone formation. Which diet should be recom-
Therefore, both 6-MP and azathioprine use mended to this patient?
would be contraindicated. The patient should
discontinue 6-MP and plan initiation of a differ- A. Low oxalate, low calcium and low fat diet
ent class of therapy once she recovers from her B. Low oxalate, high calcium and low fat diet
pancreatitis. C. Low oxalate, low calcium and high fat diet
The clinical presentation is not consistent D. High oxalate, low calcium and low fat diet
with gallstone pancreatitis given her normal E. High oxalate, high calcium and low fat diet
liver enzymes and lack of choledocholithiasis
or CBD abnormalities on abdominal imaging. CORRECT ANSWER: B
Therefore, cholecystectomy is not indicated.
With no significant changes in her Crohn’s RATIONALE
symptoms, there is no indication for high dose Patients with CD are prone to develop neph-
IV corticosteroid therapy. However, given the rolithiasis. The most frequent types of stones
need to discontinue 6-MP as well as the potential seen in CD patients are calcium oxalate and
lag time until she recovers and is able to initiate uric acid stones. Calcium oxalate stones result
a more definitive CD therapy, budesonide should from hyperoxaluria associated with disruption
be continued. of the enterohepatic circulation and bile acid
malabsorption, as in patients with distal ileal
REFERENCES disease or a history of ileal resection. As more
Ramos, L.R., et al., Inflammatory Bowel Disease bile acids enter the colon, they competitively
and Pancreatitis: A Review. J Crohns Colitis, bind with calcium, allowing more free oxalate
2016. 10(1): p. 95-104. to be absorbed into the circulation and eventu-
Chaparro, M., et al., Safety of thiopurine ally excreted through the kidneys where it binds
therapy in inflammatory bowel disease: long- to calcium and forms calcium oxalate stones.
term follow-up study of 3931 patients. Inflamm Altered colonic permeability and GI decoloniza-
Bowel Dis, 2013. 19(7): p. 1404-10. tion of Oxalobacter formigenes have also been
implicated in the risk of calcium oxalate stone
formation. The patient described in the case is
Question 12 most likely suffering from calcium oxalate stones
A 31-year-old man with a history of stricturing given his prior ileocecectomy. In contrast, uric
ileocolonic Crohn’s disease requiring ileocecec- acid stones are caused by low urine pH and low
tomy four years ago returns to your clinic. He urine volume, both of which can develop in the
is currently maintained on adalimumab mono- setting of increased stool output, particularly
therapy, 40 mg every 14 days. He recently in patients with an end ileostomy or significant
visited the emergency room for evaluation of colonic resection. Management strategies differ
left flank pain and blood in the urine. A CT scan slightly depending on the expected type of stone.
revealed bilateral nephrolithiasis with a six mm For calcium oxalate stones, in addition to hydra-
non-obstructing right renal stone and a four mm tion, dietary modifications are important to en-
left ureteral stone with mild left hydronephro- hance the formation of insoluble calcium oxalate
sis. The urology service was consulted, and the complexes to be excreted in the stool. Patients
366 Digestive Diseases Self-Education Program®
should restrict dietary fat and oxalate and increase TAVE 2) with combined clinical and endoscopic
calcium intake. Cholestyramine, a bile salt binder, remission rates at eight weeks of 18.5 percent vs
can reduce fat malabsorption and enhance oxalate 8.2 percent and 16.6 percent vs 3.6 percent for pa-
excretion in the stool. Calcium supplementation tients receiving tofacitinib vs placebo in OCTAVE
can also be of use. For uric acid stones, manage- 1 and OCTAVE 2, respectively. Furthermore, the
ment involves hydration and alkalization of the treatment effect was similar between patients who
urine using potassium bicarbonate or potassium previously failed anti-TNF therapy and those who
citrate. were anti-TNF naïve. In the maintenance trial
(OCTAVE-Sustain), patients who responded to
REFERENCE the induction phase were randomized to receive
Corica, D. and C. Romano, Renal Involvement in tofacitinib at 5 mg BID, 10mg BID or placebo.
Inflammatory Bowel Diseases. J Crohns Colitis, Remission at week 52 was achieved in 34.3 per-
2016. 10(2): p. 226 35. cent, 40.6 percent and 11.1 percent in each group,
respectively. From a safety standpoint, nasophar-
yngitis, arthralgias and headaches were among
Question 13 the most commonly reported adverse effects in
A 55-year-old man with a history of ulcerative the treatment groups during the UC clinical trials.
pancolitis diagnosed six years ago presents for a Of the more serious adverse effects, herpes zoster
second opinion regarding treatment options. He infection was reported in 18 patients and non-mel-
failed several 5-ASA formulations, azathioprine, anoma skin cancer was reported in five patients
adalimumab monotherapy, and combination receiving tofacitinib during all three clinical trials.
therapy with infliximab and 6-MP. After discussing No other malignancies were reported. Increases
all the therapy options with the patient, he elected in levels of LDL, HDL and cholesterol were also
to start tofacitinib. Which of the following is true noted, with maximum effects generally observed
regarding tofacitinib therapy in ulcerative colitis? within the first six weeks of therapy. Hence, a lipid
panel assessment is recommended within four to
A. It often takes more than 16 weeks to see a eight weeks of initiation of therapy. Just as with
response other immunosuppressive medications, live vac-
B. Assessment of lipid parameters is recom- cines are contraindicated while on tofacitinib. In
mended in all patients after four to eight addition to UC, tofacitinib has also proven to be
weeks of tofacitinib therapy effective in managing rheumatoid arthritis (RA)
C. Live-attenuated vaccines are not contraindi- and psoriatic arthritis. It is currently FDA ap-
cated while receiving tofacitinib proved for all three conditions. It can therefore be
D. The most common infection seen while on this considered in patients with concomitant diagnoses
drug is influenza. of UC and RA or psoriatic arthritis.
E. It is contraindicated in patients with concomi-
tant rheumatoid arthritis REFERENCE
Sandborn, W.J., et al., Tofacitinib as Induction
CORRECT ANSWER: B and Maintenance Therapy for Ulcerative Colitis. N
Engl J Med, 2017. 376(18): p. 1723-1736.
RATIONALE
Tofacitinib is an oral, small molecule janus kinase
inhibitor that was recently approved by the FDA Question 14
for induction and maintenance of remission in UC A 45-year-old female with a history of refractory
patients. Its efficacy for induction of remission ulcerative colitis underwent total proctocolec-
was shown in two large RCTs (OCTAVE 1 and OC- tomy with ileal pouch anal anastomosis 15 years
Chapter 13 — Inflammatory bowel disease 367
ago. Prior to her surgery, she failed mesalamine bowel inflammation. Oral budesonide is an op-
therapy, azathioprine, and infliximab. She has tion for patients who do not respond to antibiotic
done well since her colectomy with only three epi- therapy, but is not indicated for this patient at
sodes of pouchitis over the past 15 years, managed this time. Vedolizumab, adalimumab and 6-MP
with short courses of antibiotics. She presents to are not indicated at this time given the absence of
establish care at your clinic and indicates she has evidence of Crohn’s disease. Mesalamine supposi-
recently experienced an increase in stool fre- tories can be used in cases of cuffitis, but cuffitis is
quency and urgency, as well as lower abdominal not present in this patient.
pain. She denies fever or chills, recent antibiotic
use, travel or sick contacts. She takes loperamide REFERENCES
as needed to reduce bowel frequency and ibu- Wu, H. and B. Shen, Pouchitis and pouch dysfunc-
profen daily for low back and knee pain that has tion. Gastroenterol Clin North Am, 2009. 38(4): p.
recently increased. Pouchoscopy reveals diffuse 651-68.
inflammation of the pouch and the pre-pouch Hata, K., et al., Pouchitis after ileal pouch-anal
ileum, up to five cm. No strictures were seen, and anastomosis in ulcerative colitis: Diagnosis, man-
the rectal cuff mucosa appeared normal. Biopsies agement, risk factors, and incidence. Dig Endosc,
from the pouch and the pre-pouch ileum revealed 2017. 29(1): p. 26-34.
small intestinal mucosa with moderate to severe
chronic inflammation and edema. No granulomas
were seen. You started a course of ciprofloxacin Question 15
and metronidazole. What additional measure is You are seeing a 25-year-old female who initially
recommended? presented for evaluation of bloody diarrhea and
abdominal pain. Her initial laboratory results are
A. Stop NSAID use shown in the table below:
B. Start vedolizumab
C. Start budesonide Further evaluation with ileocolonoscopy revealed
D. Start adalimumab and 6-MP evidence of mild to moderate left sided colonic
E. Start mesalamine suppositories
Test Result Normal range
CORRECT ANSWER: A
White cell count 5,100/L 4.8-11.8
Hemoglobin 8.3 g/dl 10.5-15
RATIONALE
The clinical presentation is most consistent with MCV 72fL 77-96
pouchitis, that may have been triggered by NSAID Platelet 233 bil/L 130-460
use. The presence of pre-pouch ileitis limited Albumin 3.1 g/L 3.2-4.6
to the distal 10 cm of the pre-pouch ileum in ALT 71 U/L 7-37
the setting of diffuse pouchitis is more likely an
AST 63 U/L 0-30
equivalent to backwash ileitis seen in patients
Alkaline phosphatase 69 U/L 37-132
with pancolitis, as opposed to Crohn’s disease of
the small bowel. NSAIDs may also be a cause of Total bilirubin 0.6 mg/dal 0.0-1
ileal inflammation proximal to the pouch. From C-reactive protein 1.3 mg/dl 0.0-0.8
a management standpoint, this patient should be Fecal calprotectin 455 mcg/g <= 50
treated for her pouchitis with antibiotics. Dis-
Ferritin 6 ng/ml 12-300
continuing NSAID use is also recommended, as
Vitamin B12 260 pg/ml 200-1240
NSAID use is a risk factor for the development of
pouchitis and may be a direct cause of her small Folate 3 ng/ml 3.5-17.5
368 Digestive Diseases Self-Education Program®
inflammation (marked erythema, friability and sociation between celiac disease and IBD, with re-
erosions) with a normal terminal ileum. Biopsies ports of increased prevalence of IBD among celiac
of the inflamed mucosa revealed histopathologic disease patients and vice versa. The diagnosis can
changes consistent with chronic inflammation be challenging because of the overlap in clinical
without granulomas. MR enterography did not and laboratory features, but it should be suspected
show any small bowel disease. Based on these in IBD patients who have persistent symptoms de-
findings, she was given a diagnosis of left-sided spite medical therapy. Another feature to suggest
ulcerative colitis and started on combined oral possible celiac disease in this case are the nutri-
and topical mesalamine therapy. A complete liver tional deficiencies that appear to be out of propor-
disease work-up revealed normal anti-nuclear tion to her colonic disease, and the unexplained
antibodies, antimitochondrial antibodies, and elevation of liver transaminases that can be seen
anti-smooth muscles antibodies, negative hepati- in patients with celiac disease. If celiac disease
tis C and B serologies, and normal ceruloplasmin is suspected, the first step in approaching the
level, urine copper and alpha 1 antitrypsin phe- diagnosis is serologic testing with total IgA and
notype testing. Liver ultrasound did not show IgA-tTG. This is usually followed by upper endos-
any significant abnormalities. On follow up two copy with duodenal biopsies to assess the degree
months after initiation of mesalamine therapy, the of inflammation. VCE may have a role in assess-
patient reported no further rectal bleeding and her ing celiac disease extension and identification of
abdominal pain was significantly improved, but complications. It may also have a role in assessing
she continued to have diarrhea. Repeat labora- for Crohn’s disease as a potential cause for her
tory studies showed her C-reactive protein had persistent diarrhea if her celiac disease testing is
normalized, and fecal calprotectin was down to 54 negative. However, celiac serology testing should
mcg/g, but she had persistent microcytic anemia still be obtained first. Glucose hydrogen breath
despite oral iron supplements, as well as elevation testing is usually used to assess for small intestinal
of ALT and AST. Flexible sigmoidoscopy while on bacterial overgrowth, which is another potential
mesalamine therapy revealed significant improve- cause for her persistent diarrhea, and can be con-
ment with only patchy erythema and no ulcers, sidered if her celiac disease assessment is nega-
erosions or friability. tive. MRCP is unlikely to assist in diagnosing her
condition, as her liver enzymes abnormalities are
Which of the following is the most appropriate not consistent with a cholestatic pattern.
next step to investigate the cause of her persistent
diarrhea? REFERENCE
Kelly, C.P., et al., Advances in diagnosis and man-
A. Schedule video capsule endoscopy (VCE) agement of celiac disease. Gastroenterology, 2015.
B. Schedule EGD with duodenal biopsies 148(6): p. 1175-86.
C. Obtain total immunoglobulin A (IgA) level and
IgA tissue transglutaminase antibodies (IgA-tTG)
D. Schedule her for magnetic resonance Question 16
cholangiopancreatography (MRCP) A 17-year-old female with a history of penetrat-
E. Obtain glucose hydrogen breath test ing and stricturing small bowel Crohn’s disease
complicated by enteroenteric and enterovesical
CORRECT ANSWER: C fistulas presents for follow up after recent surgical
resection. She was diagnosed one year prior to her
RATIONALE surgery and has had six hospitalizations, recurrent
Patients with IBD can have concomitant celiac urinary tract infections, failure to thrive, requiring
disease and studies have actually suggested an as- total parenteral nutrition that was complicated by
Chapter 13 — Inflammatory bowel disease 369
a central line infection. She received prolonged mended over endoscopy-guided therapy. Recently
systemic corticosteroids and attempted steroid- published guidelines by the American Gastro-
sparing therapies with 6-MP 1.5mg/kg/day for at enterology Association (AGA) provide guidance
least four months with no clinical response, then for the choice of therapy in this setting, recom-
switched to infliximab monotherapy but was only mending anti-TNF therapy and/or thiopurine as
able to receive three doses due to her infectious first-line therapy over other agents. The evidence
complications and hospitalizations. In addition, supporting the use of anti-TNF monotherapy vs
she had an infusion reaction with her third dose, thiopurine monotherapy, vs combination therapy
requiring discontinuation of the infusion and a with an anti-TNF agent and a thiopurine is either
dose of IV methylprednisolone. lacking or of very low-quality. Thus, the choice of
She is now four weeks post-operative and feels therapy should be made based on individual risk
well with no abdominal pain, diarrhea, or rectal assessment, therapy experience prior to surgery,
bleeding. She denies fecal urgency, dysuria or and other risk-benefit considerations. For this
pneumaturia. She has good energy and has regained young patient with severe penetrating disease
most of the weight she lost over the past year. She is and a prior reaction to infliximab, combination
currently completing her senior year of high school. therapy with an anti-TNF agent and azathioprine
She is nonsmoker but says her father smokes. She appears to be the most appropriate therapy and
has no family history of IBD or GI malignancies. She the only acceptable choice among the listed op-
has been off all IBD therapy since her surgery. tions. Antibiotics (nitroimidazoles) have been
suggested as a second-line alternative in patients
Which of the following is recommended at this with concerns about adverse effects associated
time? with anti-TNF therapies and thiopurines. The
evidence supporting any potential role for mesa-
A. Start oral mesalamine 4 g per day and sched- lamine in post-operative prophylaxis is very low
ule colonoscopy six months after surgery and, given the lack of efficacy in luminal CD, the
B. Schedule colonoscopy six months after sur- AGA guidelines suggest against use of mesalamine
gery and initiate therapy only if evidence of for post-operative prophylaxis. The guidelines
disease recurrence also recommend against the use of budesonide
C. Start budesonide 9 mg daily and schedule and probiotics because of substantial uncertainties
colonoscopy six months after surgery regarding their effectiveness. There have been no
D. Start adalimumab and azathioprine 2-2.5mg/ clinical trials evaluating the use of methotrexate
kg/day now and schedule colonoscopy six to for post-operative prophylaxis and therefore, it
12 months after surgery should not be recommended as first-line therapy.
E. Start methotrexate 25 mg subcutaneous injec- Regardless of whether early prophylactic
tions weekly now and schedule colonoscopy pharmacological therapy is initiated or not, all
six to 12 months after surgery patients should undergo repeat colonoscopy six
to 12 months after surgical resection to evaluate
CORRECT ANSWER: D for endoscopic disease recurrence, with initiation
or optimization of therapy should recurrence be
RATIONALE confirmed.
This patient is considered at high risk for disease
recurrence following her surgical resection, due REFERENCE
to the young age of onset and her complicated Nguyen, G.C., et al., American Gastroenterological
presentation with rapid progression to surgery. Association Institute Guideline on the Manage-
Exposure to second-hand smoke may be adding ment of Crohn’s Disease After Surgical Resection.
to her risk. Therefore, early prophylaxis is recom- Gastroenterology, 2017. 152(1): p. 271-275.
370 Digestive Diseases Self-Education Program®
Is also suggested to avoid anti-TNFs in patients A. Reduce the infliximab interval to every six
with a family history of demyelinating disease weeks
given the potential genetic predisposition. All B. Start vedolizumab 300 mg at weeks 0, two, six
anti-TNF agents can be used safely during preg- and then every eight weeks
nancy. For patients with latent tuberculosis (TB), C. Start ustekinumab IV induction dose based on
it is preferable to avoid initiating anti-TNF treat- her weight, then 90 mg subcutaneous injec-
ment until completion of the latent TB therapy. tions every eight weeks
However, in more urgent cases including poorly D. Switch azathioprine to methotrexate
controlled IBD, it acceptable to initiate anti-TNF E. Start adalimumab 160 mg at week 0, then 80
therapy after a minimum of four weeks of latent mg at week two, then 40 mg every two weeks
TB treatment. Patients with recent or active
solid malignancy should avoid anti-TNF therapy CORRECT ANSWER: A
given the potential impact on disease progression.
However, in patients with a history of solid cancer RATIONALE
treated more than five years ago, initiating anti- This patient has evidence of active luminal
TNF treatment is not contraindicated. Finally, Crohn’s disease along with increased activity of
TNF inhibitors should be avoided in patients with her perianal fistula. Therapeutic drug monitoring
moderate to severe heart failure. A history of reveals a low drug level (less than 5mcg/ml) with
myocardial infarction with intact cardiac function no detectable anti-drug antibodies. This sug-
is not a contraindication. gests a non-immune-mediated pharmacokinetic
failure and in such cases, it is recommended to
REFERENCE optimize the index drug before switching to a dif-
Lichtenstein, G.R., et al., ACG Clinical Guideline: ferent agent. Optimization of the index drug can
Management of Crohn’s Disease in Adults. Am J be achieved by increasing the drug dose, reducing
Gastroenterol, 2018. 113(4): p. 481-517. the dosing interval, or adding immunomodulator
therapy. This patient is already receiving combi-
nation therapy with azathioprine and infliximab,
Question 19 therefore increasing the infliximab dose or re-
You are seeing a 24-year-old female with a his- ducing the dose interval are appropriate inter-
tory of penetrating Crohn’s colitis with perianal ventions. There is no indication to switch from
phenotype. She is currently maintained on inflix- azathioprine to methotrexate. Vedolizumab, an
imab 5mg/kg every eight weeks and azathioprine anti-integrin, and ustekinumab, an IL12/IL23 in-
2 mg/kg/day. She indicates she has noticed hibitor, are both therapeutic options if the patient
increased bowel frequency, urgency and drainage continues to have active disease despite optimiza-
from her perianal fistula. Infectious etiologies tion of her infliximab therapy. Adalimumab is
are excluded, and MRI of the pelvis confirms the another TNF-inhibitor, and would be an appropri-
presence of a perianal fistula with no abscesses. ate choice if the patient had a high titer of anti-
Ileocolonoscopy shows mild-moderate active drug antibodies suggesting an immune-mediated
colitis involving the rectum and segments of the pharmacokinetic failure
transverse and ascending colon. Therapeutic
drug monitoring reveals a trough infliximab drug REFERENCE
level of 3.5 mcg/ml with no detectable anti-inflix- Vande Casteele, N., et al., American Gastroentero-
imab antibodies. logical Association Institute Technical Review on
the Role of Therapeutic Drug Monitoring in the
In addition to antibiotic treatment, which of the Management of Inflammatory Bowel Diseases.
following would you recommend? Gastroenterology, 2017. 153(3): p. 835-857.e6.
372 Digestive Diseases Self-Education Program®
REFERENCE
Shin, S.J., et al., Non-steroidal anti-inflammatory
drug-induced enteropathy. Intest Res, 2017. 15(4):
p. 446-455.
therapy for the last year. Prior to infliximab, he the Role of Therapeutic Drug Monitoring in the
failed mesalamine and azathioprine therapies. He Management of Inflammatory Bowel Diseases.
now presents with bloody diarrhea and abdominal Gastroenterology, 2017. 153(3): p. 835-857.e6.
pain for the past two weeks. Laboratory assess-
ment reveals microcytic anemia with elevated
C-reactive protein. Stool studies are negative for Question 22
any infectious process. Therapeutic drug moni- A 26-year-old female recently diagnosed with
toring reveals a trough infliximab level of 15mcg/ ileocolonic Crohn’s disease just completed induc-
ml with no detectable antibodies to infliximab. tion of adalimumab therapy and received her first
Ileocolonoscopy reveals a normal terminal ileum maintenance dose three days ago. She presents for
and moderate to severe inflammatory changes an unscheduled clinic visit for evaluation of a dif-
involving the entire colonic mucosa. You start a fuse scaly rash involving her abdomen, back and
prednisone taper at 40 mg daily with good clinical upper extremities that began the night before.
response. Which of the following is most appropri-
ate next step in management?
CORRECT ANSWER: C
You suspect drug-induced psoriasis. Which of the fol-
RATIONALE lowing statements is true regarding this condition?
This patient has moderate to severe disease
activity despite adequate trough infliximab levels A. This condition is seen more frequently in men
(more than 5mcg/ml). This suggests a mechanis- B. It only occurs in ulcerative colitis patients, not
tic failure of infliximab and therefore switching to those with Crohn’s disease
a drug of different class is recommended. Con- C. Anti-TNF should be stopped immediately
tinuing the same dose of infliximab, reducing the D. Switching to a different anti-TNF agent is
infusions intervals, or switching to adalimumab, associated with recurrence of skin lesions in
another TNF-inhibitor, are not appropriate op- 90-100 percent of patients
tions. Methotrexate has not been shown to be E. It most commonly involves the palms of
effective in inducing or maintaining remission for hands, soles of feet, and scalp
patients with ulcerative colitis. Vedolizumab is an
anti-integrin agent with proven efficacy in moder- CORRECT ANSWER: E
ate to severe ulcerative colitis and is an appropri-
ate choice for this patient RATIONALE
A wide range of dermatological conditions has
REFERENCE been reported in association with anti-TNF thera-
Vande Casteele, N., et al., American Gastroentero- py, including psoriasis, ezcema, vitiligo, lichenoid
logical Association Institute Technical Review on reactions, vasculitis, erythema nodosum, and
374 Digestive Diseases Self-Education Program®
topical mesalamine therapy and required an oral change outside of class. This reactive approach
prednisone taper. He was unable to fully taper off to therapeutic drug monitoring (TDM) is sup-
of prednisone and was initiated on infliximab 5 ported by the recent American Gastroenterological
mg/kg monotherapy. He did well initially and was Association technical review on TDM. The goal
able to taper off of prednisone. However, he now infliximab trough level is not entirely clear, and
has recurrent symptoms. He is negative for C. dif- studies are ongoing, but is currently suggested to
ficile and undergoes a flexible sigmoidoscopy with be 5µg/mL or more, though patients with fistuliz-
the following findings in the recto-sigmoid colon: ing Crohn’s disease may require levels 15µg/mL
or more. The patient has already tested negative
for C. difficile and repeat testing is not indicated.
If there is no evidence of viral cytopathic effect on
biopsies, there is no role for serum CMV markers.
Although vedolizumab is an option for induction
and maintenance of moderate to severe UC, the
need to change therapies would be better in-
formed by the results of the infliximab drug level/
antibody assay, as dose intensification of inflix-
imab, if appropriate, might allow the patient to
recapture response from this agent. Azathioprine
is slow in onset of action, and given the severity of
the inflammation on endoscopy, this would not be
Biopsies from the colon show moderate chronic the most appropriate next step.
active colitis with no viral cytopathic effect.
Which of the following is the next best step in his REFERENCES
management? Vande Casteele N, Herfarth H, Katz J, et al. Amer-
ican Gastroenterological Association Institute
A. Repeat C. difficile assay Technical Review on the Role of Therapeutic Drug
B. Check serum CMV IgM assay Monitoring in the Management of Inflammatory
C. Check Infliximab level/antibody Bowel Diseases. Gastroenterology 2017;153:835-
D. Change to vedolizumab 857.e6.
E. Add azathioprine Feagan BG, Rutgeerts P, Sands BE, et al. Ve-
dolizumab as induction and maintenance therapy
CORRECT ANSWER: C
for ulcerative colitis. N Engl J Med 2013;369:699-
710.
RATIONALE
This patient has moderate to severe ulcerative
colitis (UC), and has relapsed while on infliximab Question 29
monotherapy. A serum infliximab level/anti- A 28-year-old man with Crohn’s disease pres-
body will determine whether he has an adequate ents with abdominal pain, distension and
level of infliximab and whether he has developed increased bowel movement frequency, and is
anti-drug antibodies. If his drug level is low and found to have a severe ileal stenosis. He sub-
he does not have anti-drug antibodies, he would sequently undergoes ileocecal resection with
benefit from dose intensification. If he has a low direct anastomosis. Prior to surgery, he had
drug level in the presence of high antibodies to inf- been treated with recurrent courses of cortico-
liximab, he might benefit from within class change steroids in addition to azathioprine 2.5 mg/kg/
in therapy (e.g., to adalimumab). If he has good day. In the post-operative time period, you plan
infliximab levels , he would likely benefit from a to initiate an anti-TNF agent.
Chapter 13 — Inflammatory bowel disease 379
Which of the following is the most evidence-based fracture and will require surgery.
next step in monitoring this patient? When you see her back in clinic six months
after surgery, you review the results of a DEXA scan
A. Fecal calprotectin every six months ordered by her primary care physician. The scan
B. CRP every six months demonstrated osteoporosis of the spine. Which of
C.Drug levels and antibodies every six months the following factors, if present, is most strongly
D. Colonoscopy at six months associated with osteoporosis in a patient with IBD?
E. CT scan at six months
A. Age more than 30 years
CORRECT ANSWER: D B. Female gender
C. Disease duration of more than 10 years
RATIONALE
D. More than three months of steroid use
In the post-operative setting, the POCER (Post-Oper-
E. Smoking
ative Crohn’s Endoscopic Recurrence) trial random-
ized patients to active care (colonoscopy six months
CORRECT ANSWER: D
after resection) vs. standard care (no colonoscopy).
The active arm had significantly less endoscopic RATIONALE
recurrence and greater complete mucosal healing. Risk factors for osteoporosis in the general
Other non-endoscopic methods of monitoring and population include female gender, thin frame,
surveillance do not have the same high level of evi- postmenopausal status, smoking/alcohol use and
dence, including fecal calprotectin, CRP and CT scan. corticosteroid use. In the general population,
No prospective trial of therapeutic drug monitoring the United States Preventive Services Task Force
(TDM) and dose adjustment in the post-operative (USPSTF) recommends screening for osteoporosis
setting has been conducted. In the TAXIT random- in women age 65 years and older and in younger
ized controlled trial of clinical dosing of infliximab women whose fracture risk is greater than that of
vs. dosing based on trough concentration, there was a 65-year-old white woman who has no additional
no difference in the primary endpoint of clinical and risk factors.
biological remission at one year, but all patients were There is no specific age recommendation
optimized to reach a target trough level before being at which to begin screening for osteoporosis in
randomized to dosing based on their clinical features IBD patients. The prevalence of osteoporosis in
or continued dosing based on trough level. IBD patients is approximately 15 percent and is
strongly affected by age. In IBD populations, men
REFERENCES and women are at equal risk. The strongest risk
De Cruz P, Kamm MA, Hamilton AL, et al. Crohn’s factor in the IBD population is corticosteroid use.
disease management after intestinal resection: a Screening is recommended in patients with IBD
randomised trial. Lancet 2015;385:1406-17. who have used oral corticosteroid therapy for lon-
Vande Casteele N, Ferrante M, Van Assche G, ger than three consecutive months at dose of 7.5
et al. Trough concentrations of infliximab guide mg or more of prednisone per day.
dosing for patients with inflammatory bowel dis-
ease. Gastroenterology 2015;148:1320-9.e3. REFERENCES
Targownik LE, Bernstein CN, Leslie WD. Inflam-
matory bowel disease and the risk of osteoporosis
Question 30 and fracture. Maturitas 2013;76:315-9.
A 63-year-old woman with a longstanding history Farraye FA, Melmed GY, Lichtenstein GR,
of ulcerative colitis, currently in remission on me- et al. ACG Clinical Guideline: Preventive Care in
salamine therapy, is admitted to the hospital after Inflammatory Bowel Disease. Am J Gastroenterol
falling at home. She has an intertrochanteric hip 2017;112:241-258.
380 Digestive Diseases Self-Education Program®
Question 31 Question 32
An 18-year-old man presents with a two-month A 23-year-old woman with penetrating colonic
history of diarrhea (four to five bowel movements/ Crohn’s disease with perianal phenotype is now in
day) with urgency and intermittent rectal bleed- clinical and endoscopic remission on combination
ing. He denies any weight loss. He denies any therapy with infliximab and methotrexate. She is
abdominal pain. He is a former cigarette smoker interested in becoming pregnant. She asks about
who quit about six months ago. As part of his her medications. Which of the following would be
work up, he undergoes colonoscopy and is found most appropriate in regard to management of her
to have mild to moderate proctosigmoiditis. Crohn’s disease during pregnancy?
Which of the following treatment regimens would A. Stop methotrexate prior to conception and
be most appropriate at this time in order to induce continue infliximab
and maintain remission? B. Stop both methotrexate and infliximab prior
to conception, use no maintenance medica-
A. Resume smoking tions during pregnancy
B. Oral and topical mesalamine C. Continue both methotrexate and infliximab
C.Budesonide MMX D. Stop methotrexate now and plan to start mer-
D. Oral mesalamine captopurine during her pregnancy
E. Oral corticosteroids E. Start prednisone now and continue during
pregnancy to prevent relapse
CORRECT ANSWER: B
CORRECT ANSWER: A
RATIONALE
RATIONALE
While smoking cessation is associated with the
Methotrexate is contraindicated during pregnancy
development of ulcerative colitis, smoking is not
related to neural tube defects associated with its
a recommended treatment for the disease. In
use. Data have consistently shown that infliximab
left-sided ulcerative colitis, a meta-analysis of
is not associated with congenital abnormalities.
four RCTs using combination treatment with rec-
Additionally, infliximab has not been associated
tal 5-aminosalicylate enemas at a dose of 1 g/day
with other complications in the newborn. Dis-
combined with oral 5-aminosalicylate at a dose
continuation of therapy is associated with a risk
of at least 2.0 g/day was more effective than oral
of relapse of Crohn’s disease. Active disease is
5-ASA alone for induction of remission (relative
associated with increased risk of small for gesta-
risk RR = 0.65; 95 percent CI 0.47-0.91). These
tional age and/or pre-term delivery. Initiation
agents, unlike corticosteroids or budesonide
of mercaptopurine is not recommended during
MMX, can be used for both induction and main-
pregnancy, related to the rare risk of pancreatitis
tenance of remission.
at initiation. However, if a woman is stable on
maintenance mercaptopurine prior to conception,
REFERENCE
it is now recommended that she continue therapy
Ford AC, Khan KJ, Achkar JP, et al. Efficacy of
during pregnancy to prevent disease relapse.
oral vs. topical, or combined oral and topical
5-aminosalicylates, in Ulcerative Colitis: system-
REFERENCES
atic review and meta-analysis. Am J Gastroenterol
Lichtenstein GR, Feagan BG, Mahadevan U, et al.
2012;107:167-76; author reply 177.
Pregnancy Outcomes Reported During the 13-
Year TREAT Registry: A Descriptive Report. Am J
Gastroenterol 2018.
Chapter 13 — Inflammatory bowel disease 381
proceed with a trial of tofacitinib for his ulcerative therapy. You recommend initiation of an anti-TNF
colitis. Which of the following complications has agent in addition to the azathioprine. Which of
been associated with tofacitinib use? the following preventive measures should also be
recommended?
A. Non-melanoma skin cancer
B. Herpes zoster infection A. DEXA scan
C.Hyperlipidemia B. Pneumococcal vaccine (PPSV-23 and PCV-23)
D. Increases in CPK (creatinine phosphokinase) C. Intranasal influenza vaccination (FluMist®)
E. All of the above D. Human papilloma virus (HPV) vaccine
E. All of the above
CORRECT ANSWER: E
CORRECT ANSWER: B
RATIONALE RATIONALE
Tofacitinib is an orally administered small molecule In patients with chronic conditions such as
that is a non-selective inhibitor of the Janus kinase Crohn’s disease, annual influenza vaccine is
enzyme. The OCTAVE 1 (n=598) and OCTAVE 2 indicated, but should be given by the inactivated
(n=541) induction trials were conducted to as- influenza vaccine. In general, live vaccines should
sess the efficacy of tofacitinib 10 mg orally twice be avoided for at least four weeks prior to initia-
daily compared to placebo. Patients enrolled had tion, and during treatment with biologic therapy.
moderately-to-severely active UC and had failed The newest statement from the Infectious Disease
conventional therapies (half of them had previously Society of America (IDSA) recommends pneumo-
failed anti-TNF agents). The primary endpoint was coccal vaccination with both the PPSV-23 vaccine
remission (total Mayo score of two or less, no sub- and the PCV-23 vaccine. The timing of the vac-
score more than1 and rectal bleeding subscore of cines is based upon which is given first and the
0) at eight weeks. A significantly higher proportion age of the patient. Human papilloma virus vaccine
of patients in both induction trials achieved remis- is recommended by the FDA in all women and
sion with tofacitinib 10 mg twice daily compared men age 11-45 years (note that the age range has
to those who received placebo. While there was no increased). Immunosuppression may increase
difference overall between tofacitinib and placebo the risk for cervical dysplasia from HPV, and thus
groups for serious adverse events, tofacitinib has attention to this recommendation is warranted in
been associated with all of the above complications. women. In those who have been on corticosteroids
The hyperlipidemia is associated with increases in for more than three months, assessment of bone
both LDL and HDL, although the cardiovascular mineral density and maximizing vitamin D levels
implications are not known. is indicated.
REFERENCE REFERENCES
Sandborn WJ, Su C, Sands BE, et al. Tofacitinib as Long MD, Gulati A, Wohl D, et al. Immunizations
induction and maintenance therapy for ulcerative in Pediatric and Adult Patients with Inflammatory
colitis. N Engl J Med 2017;376:1723-1736. Bowel Disease: A Practical Case-based Approach.
Inflamm Bowel Dis 2015;21:1993-2003.
Kane S, Khatibi B, Reddy D. Higher inci-
Question 36 dence of abnormal Pap smears in women with
A 47-year-old man with colonic inflammatory inflammatory bowel disease. Am J Gastroenterol
Crohn’s disease has required two courses of 2008;103:631-6.
corticosteroids (totaling less than three months Kornbluth A, Sachar DB. Ulcerative colitis
of use) over the past year, in spite of azathioprine practice guidelines in adults: American College Of
Chapter 13 — Inflammatory bowel disease 383
Gastroenterology, Practice Parameters Committee. 0.92 and specificity was 0.94-0.97). Treatment for
Am J Gastroenterol 2010;105:501-23; quiz 524. CDI consists of oral vancomycin or fidaxomicin, or
Farraye FA, Melmed GY, Lichtenstein GR, fecal microbial transplantation (FMT) in refracto-
et al. ACG Clinical Guideline: Preventive Care in ry cases. For severe CDI in the general population,
Inflammatory Bowel Disease. Am J Gastroenterol the success rate of vancomycin is 78.5 percent,
2017;112:241-258. fidaxomicin success is similar to that of vanco-
mycin but with lower recurrence rates, and for
FMT is 83-94 percent for recurrent CDI. Recent
Question 37 guidelines from the Infectious Disease Society of
A 23-year-old woman with a three-year history
American no longer recommend metronidazole as
of ulcerative rectosigmoiditis has been in clinical
first line therapy for C. difficile.
and endoscopic remission for two years on oral
mesalamine 2.4 g daily and mesalamine enemas
REFERENCES
nightly. She presents to the clinic with new-onset
Reddy SS, Brandt LJ. Clostridium difficile infec-
watery, non-bloody diarrhea for two weeks. She
tion and inflammatory bowel disease. J Clin Gas-
denies any recent travel or sick contacts. Her
troenterol 2013;47:666-71.
examination is unremarkable. What is the next
Berg AM, Kelly CP, Farraye FA. Clostridium
best step?
difficile infection in the inflammatory bowel dis-
A. Treat with course of ciprofloxacin ease patient. Inflamm Bowel Dis 2013;19:194-204.
B. Initiate oral corticosteroids Bagdasarian N, Rao K, Malani PN. Diagnosis
C. Obtain a stool sample for and treatment of Clostridium difficile in adults: a
Clostridium difficile PCR systematic review. Jama 2015;313:398-408.
D. Perform a colonoscopy
E. Recommend fecal transplant
Question 38
A 66-year-old woman presents to your office with
CORRECT ANSWER: C
a two-month history of recurrent, watery, non-
bloody diarrhea. She has recently been diagnosed
RATIONALE
with osteoarthritis and had been taking ibupro-
IBD is an independent risk factor for the de-
fen 3 times daily for knee pain. Stool studies are
velopment of C. difficile infection (CDI), with a
obtained and are negative for infection, but are
three-fold increased risk compared to the general
positive for fecal leukocytes.
population. The traditional risk factors for CDI are
antibiotic exposure, hospitalization, older age, and What is the most likely diagnosis?
co-morbid conditions. However, IBD patients with
CDI are often younger, have community acquisi- A. Ulcerative colitis
tion, ongoing corticosteroid use, colonic involve- B. Crohn’s disease
ment with IBD, and may have no history of recent C. Microscopic colitis
antibiotic exposure. Corticosteroid use, with or D. Celiac Disease
without additional immunosuppressive agents, E. Small intestinal bacterial overgrowth
triples the risk of CDI in the IBD population. ,
Multistep algorithms using polymerase chain CORRECT ANSWER: C
reaction (PCR) for the toxin gene(s) or single-step
PCR on liquid stool samples have the best test RATIONALE
performance characteristics for CDI (for multi- The incidence rate of collagenous colitis is approx-
step: sensitivity was 0.68-1.00 and specificity was imately four per 100,000 person-years and the
0.92-1.00; for single step: sensitivity was 0.86- incidence of lymphocytic colitis is five per 100,000
384 Digestive Diseases Self-Education Program®
person-years. The incidence rate increases with Which of the following treatments is indicated at
rising age. Microscopic colitis refers collectively this time?
to both lymphocytic colitis and collagenous colitis.
The peak incidence is between 60 and 70 years, A. Chronic antibiotic use
with a greater risk among women. Colonoscopy B. Mesalamine suppositories
with biopsies is required to make the diagnosis, as C. Anti-TNF agent
endoscopically the colon appears normal. D. Probiotic
NSAIDs are a known trigger for microscopic E. Oral prednisone
colitis, along with proton pump inhibitors,
statins and selective serotonin reuptake inhibi- CORRECT ANSWER: B
tors. There are associations with celiac disease
and thyroid disease, and clinical assessment for RATIONALE
these disorders should be considered once the Historically, approximately 50 percent of patients
diagnosis is made if not done previously. Treat- will have no symptoms of pouchitis after total
ment consists of withdrawal of any potential abdominal colectomy and ileal pouch anal anasto-
offending trigger along with anti-diarrheal agents mosis (IPAA) for ulcerative colitis. The remainder
(e.g., bismuth or loperamide) or budesonide. If will experience pouchitis, with 11 percent having
there is no response to budesonide, other treat- one to two episodes per year, 24 percent having
ment options that have been used include ami- occasional symptoms, 10 percent with constant
nosalicylates and cholestyramine, although less symptoms, and four percent will be refractory.
efficacy data exist for these therapies. This patient has evidence of inflammation of the
cuff (cuffitis), which should be treated with topi-
REFERENCES cal therapies such as mesalamine suppositories.
Tong J, Zheng Q, Zhang C, et al. Incidence, preva- There is no evidence of Crohn’s disease of the
lence, and temporal trends of microscopic colitis: pouch, therefore an anti-TNF agent would not be
a systematic review and meta-analysis. Am J Gas- indicated at this time, nor would steroid therapy.
troenterol 2015;110:265-76; quiz 277. Antibiotics have demonstrated efficacy for treat-
Gentile NM, Khanna S, Loftus EV, Jr., et al. ment of acute pouchitis, particularly ciprofloxacin
The epidemiology of microscopic colitis in Olmsted and metronidazole. The probiotic VSL-3 has been
County from 2002 to 2010: a population-based shown to prevent recurrence of pouchitis in pa-
study. Clin Gastroenterol Hepatol 2014;12:838-42. tients who have recently undergone total procto-
Pardi DS. Microscopic colitis. Clin Geriatr colectomy with IPAA, and for prevention of recur-
Med 2014;30:55-65. rent pouchitis after treatment with antibiotics.
REFERENCES
Question 39 Stahlberg D, Gullberg K, Liljeqvist L, et al. Pouchi-
A 24-year-old man underwent total abdominal tis following pelvic pouch operation for ulcerative
colectomy with ileal pouch anal anastomosis for colitis. Incidence, cumulative risk, and risk fac-
refractory ulcerative colitis 12 months ago. He tors. Dis Colon Rectum 1996;39:1012-8.
now presents with two weeks of diarrhea, gas, Shen B. Acute and chronic pouchitis--patho-
urgency, and intermittent blood in his stool. He genesis, diagnosis and treatment. Nat Rev Gastro-
does not take any medication regularly other enterol Hepatol 2012;9:323-33.
than loperamide. A pouchoscopy is performed
and demonstrates a normal neo-terminal ileum,
normal pouch, but ulceration and erythema in the Question 40
area of the cuff for two to three cm. A 33-year-old woman presents with a six-week
Chapter 13 — Inflammatory bowel disease 385
history of progressively worsening diarrhea, ur- is superior to monotherapy with either agent in
gency and rectal bleeding. She undergoes colonos- ulcerative colitis. Gastroenterology 2014;146:392-
copy as part of her work-up, which shows ulcer- 400.e3.
ative pancolitis. She is started on oral prednisone
in addition to oral and topical mesalamine. She is
unable to taper off of prednisone due to recurrent Question 41
symptoms. You discuss various classes of medica- You start seeing a 19-year-old man with a one-year
tions with her for the treatment of moderate to history of ileal inflammatory Crohn’s disease who
severe ulcerative colitis. is transitioning from pediatric to adult manage-
ment of his Crohn’s. His mother accompanies
Which of the following therapy or therapies has him to the clinic visit. He has been treated with
demonstrated the greatest efficacy to achieve intermittent courses of budesonide for his disease,
corticosteroid-free remission at 16 weeks for mod- but has not been maintained on other therapies.
erate to severe ulcerative colitis? His mother is concerned about the risk of hepa-
tosplenic T-cell lymphoma. Which of the follow-
A. Infliximab alone ing is a known risk factor for hepatosplenic T-cell
B. Azathioprine alone lymphoma?
C. Infliximab and azathioprine combination
D. Infliximab and adalimumab combination A. Ustekinumab
E. Azathioprine and methotrexate combination B. Male gender
C. Vedolizumab
CORRECT ANSWER: C C. Methotrexate
E. Corticosteroids
RATIONALE
Based on the UC Success study, infliximab com- CORRECT ANSWER: B
bined with azathioprine is the best choice. In UC
Success, a total of 239 patients were included in RATIONALE
the efficacy analyses. Baseline characteristics were Hepatosplenic T-cell lymphoma (HSTCL) is a very
similar between treatment groups. Corticosteroid- rare neoplasm, accounting for less than 1 percent
free remission at week 16 was achieved by 39.7 of non-Hodgkin’s lymphoma. Approximately 10-
percent (31 of 78) of patients receiving infliximab/ 20 percent of these HSTCLs are associated with
azathioprine combination compared with 22.1 per- chronic immunosuppression. In patients with
cent (17 of 77) receiving infliximab alone (P =.017) inflammatory bowel disease, there is an associa-
and 23.7 percent (18 of 76) receiving azathio- tion with use of thiopurines, anti-TNF agents, and
prine alone (P =.032). Mucosal healing at week with combination therapy with anti-TNF agents
16 occurred in 62.8 percent (49 of 78) of patients and thiopurines. In a systematic review of HSTCL
receiving infliximab/azathioprine, compared with in patients with IBD, there were a total of 36 pa-
54.6 percent (42 of 77) receiving infliximab (P = tients with HSTCL. Of these, 20 received therapy
.295) and 36.8 percent (28 of 76) receiving aza- with infliximab and a thiopurine and 16 received a
thioprine (P =.001). Serious infections occurred in thiopurine as monotherapy for IBD. Four pa-
two patients (1 patient receiving infliximab alone, tients who had been treated with infliximab and
and 1 patient receiving azathioprine alone). a thiopurine also received adalimumab. One of
these patients had been given infliximab, adalim-
REFERENCE umab, and natalizumab. Of 31 patients of known
Panaccione R, Ghosh S, Middleton S, et al. Com- gender, only two were female. Twenty-seven of the
bination therapy with infliximab and azathioprine 30 patients of known age were younger than 35
386 Digestive Diseases Self-Education Program®
REFERENCES
Pugliese D, Guidi L, Ferraro PM, et al. Paradoxical
psoriasis in a large cohort of patients with inflam-
matory bowel disease receiving treatment with
anti-TNF alpha: 5-year follow-up study. Aliment
Pharmacol Ther 2015;42:880-8.
What is the best management of this rash? Freling E, Baumann C, Cuny JF, et al. Cu-
mulative Incidence of, Risk Factors for, and
A. Stop infliximab and start ustekinumab Outcome of Dermatological Complications of
B. Stop methotrexate and start azathioprine Anti-TNF Therapy in Inflammatory Bowel Dis-
D. Increase dose of infliximab ease: A 14-Year Experience. Am J Gastroenterol
E. Increase dose of methotrexate 2015;110:1186-96.
F. Add UV light therapy
Chapter 13 — Inflammatory bowel disease 387
REFERENCES RATIONALE
Kornbluth A, Sachar DB. Ulcerative colitis practice Although the causes of this variability in drug
guidelines in adults: American College Of Gastro- clearance of anti-TNF agents are incompletely un-
enterology, Practice Parameters Committee. Am J derstood, several factors are known to be critical.
Gastroenterol 2010;105:501-23; quiz 524. Specifically, factors associated with more rapid
Yuhara H, Steinmaus C, Corley D, et al. Meta- clearance of anti-TNF agents include: the pres-
analysis: the risk of venous thromboembolism in ence of anti-drug-antibodies, greater degree of
patients with inflammatory bowel disease. Ali- systemic inflammation as evidenced by the serum
ment Pharmacol Ther 2013;37:953-62. C-reactive protein (CRP) (higher levels associated
Nguyen GC, Sam J. Rising prevalence of ve- with increased clearance) or plasma TNF con-
nous thromboembolism and its impact on mortali- centration, serum albumin concentration (lower
ty among hospitalized inflammatory bowel disease levels associated with increased clearance), dis-
patients. Am J Gastroenterol 2008;103:2272-80. ease type (ulcerative colitis greater drug clearance
https://www.gastro.org/practice/quality-initia- than Crohn’s disease), body mass index (increased
tives/IBD_Measures.pdf. BMI associated with increased drug clearance),
Ananthakrishnan AN, Cagan A, Gainer VS, et gender (drug clearance in males greater than that
al. Thromboprophylaxis is associated with reduced in females), and concomitant therapy with im-
post-hospitalization venous thromboembolic events munosuppressive agents have all emerged as key
in patients with inflammatory bowel diseases. Clin determinants of pharmacokinetics.
Gastroenterol Hepatol 2014;12:1905-10. This impacts use of anti-TNF for patients
hospitalized with severe ulcerative colitis, as the
Question 45 therapeutic efficacy may be altered by rapid drug
A 19-year-old obese woman (BMI 35) is hospitalized clearance. Retrospective data have demonstrated
for a flare of ulcerative colitis. She was diagnosed six a reduction in three-month colectomy rate with
months ago, but has been unable to taper off oral higher dosing of infliximab in individuals with
corticosteroids in spite of the addition of mesala- these risk factors. Further prospective studies are
mine (4.8 g/ daily) and mercaptopurine at 1.5 mg/ needed to guide effective dosing strategies.
kg/day. Upon admission to the hospital, stool for
C. difficile is negative, and flexible sigmoidoscopy REFERENCES
shows severe proctosigmoiditis up to approximately Khanna R, Levesque BG, Sandborn WJ, et al.
Chapter 13 — Inflammatory bowel disease 389
Therapeutic Drug Monitoring of TNF Antagonists their blood. To date, there are no clear guidelines
in Inflammatory Bowel Disease. Gastroenterol regarding when to hold anti-TNF medications in
Hepatol (N Y) 2014;10:478-489. pregnant women. It is important to keep in mind
Gibson DJ, Heetun ZS, Redmond CE, et al. An that maintaining disease remission in the mother
accelerated infliximab induction regimen reduces is key, and the timing of holding anti-TNF therapy
the need for early colectomy in patients with acute should be individualized for each patient.
severe ulcerative colitis. Clin Gastroenterol Hepa- Importantly, it does NOT appear that children
tol 2015;13:330-335.e1. born to mothers on anti-TNF agents have a re-
duced response to inactivated vaccines. A total of
179 women completed the vaccine survey within
Question 46 PIANO (Pregnancy and Neonatal Outcomes) in
A 31-year-old woman with ulcerative colitis on IBD Study; 26 were biologic unexposed, 153 were
adalimumab monotherapy completes a success- exposed to a biologic agent). There was no signifi-
ful pregnancy with delivery of a healthy baby boy cant difference in proportions of infants with pro-
at 39 weeks. She was continued on adalimumab tective antibody titers against HiB born to exposed
up until week 36 and plans to restart therapy one mothers (n = 42, 71 percent) vs unexposed moth-
week after delivery. ers (n = 8, 50 percent) (P = .41). There was no dif-
ference in the proportion of infants with protective
Which of the following is recommended with re- antibody titers to tetanus toxoid born to exposed
gard to vaccinations for mother or child? mothers (80 percent) vs unexposed mothers (75
percent) (P = .66). The median concentration
A. All vaccines should be avoided in the first six of infliximab in cord blood did not differ signifi-
months of life for baby cantly between infants with vs without protective
B. No recommended changes to standard vaccine antibody titers to HiB (P = .30) or tetanus toxoid
schedule for mother or baby (P = .93). Mild reactions were observed in 7 of 40
C. Avoid hepatitis B vaccine in baby for first six infants who received rotavirus vaccine and whose
months (typically given prior to discharge) mothers had been exposed to biologic therapies.
D. Mother should avoid TDaP* vaccine (typically While reactions to the live rotavirus vaccine were
updated prior to discharge) mild, it is recommended that this vaccination be
E. Rotavirus vaccine (typically given at four postponed or avoided until anti-TNF clearance is
months) should be postponed or avoided obtained or six months has passed.
*Tetanus, Diphtheria, Pertussis
REFERENCES
CORRECT ANSWER: E Hashash JG, Kane S. Pregnancy and Inflamma-
tory Bowel Disease. Gastroenterol Hepatol (N Y)
RATIONALE 2015;11:96-102.
It is important to make sure that children born to Beaulieu DB, Ananthakrishnan AN, Martin C,
mothers who have received infliximab, adalimum- et al. Use of Biologic Therapy by Pregnant Women
ab, or golimumab after the 20th week of gestation With Inflammatory Bowel Disease Does Not Affect
do not receive any live vaccinations during their Infant Response to Vaccines. Clin Gastroenterol
first six months of life. These newborns are im- Hepatol 2018;16:99-105.
munosuppressed because the anti-TNF agent has
crossed the placenta into their blood. Newborns
have an immature reticuloendothelial system and Question 47
are unable to clear antibodies, and infliximab, A 32-year-old man is diagnosed with primary scle-
adalimumab, or golimumab will be detected in rosing cholangitis due to elevated alkaline phos-
390 Digestive Diseases Self-Education Program®
phatase and MRI/MRCP consistent with small edema, and is diagnosed with class III heart fail-
duct disease. He also has a history of loose stools ure during a hospitalization. He is diuresed during
for the past three to four years. He has not seen the hospitalization and has marked improvement
any blood in his stool and his weight has been sta- in respiratory status and lower extremity edema.
ble. He undergoes colonoscopy and is diagnosed As he still has active Crohn’s, you need to initiate a
with mild pan-ulcerative colitis. He has mild back- steroid-sparing therapy for his disease as well.
wash ileitis on examination of the terminal ileum.
He is placed on oral mesalamine for therapy. Which of the following agents would be contra-
Which of the following is your recommendation indicated for treatment of Crohn’s disease at this
for surveillance colonoscopy in this patient? time due to his class III heart failure?
reports of heart failure after therapy with a tu- analysis was 14 (95 percent CI, 5-34) cancers/100
mor necrosis factor antagonist. Ann Intern Med years patient follow-up. A 2016 systematic review
2003;138:807-11. and meta-analysis of 10 randomized trials found
an increased likelihood of detecting dysplasia with
chromoendoscopy compared with other tech-
Question 49 niques (RR 1.37, 95 percent CI 1.04-1.79), but on
A 49-year-old man with a 20-year history of ulcer- subgroup analysis this effect was confirmed only
ative colitis undergoes his biannual surveillance for chromoendoscopy compared with standard
colonoscopy. He has been maintained on oral white light endoscopy (RR 2.12, 95 percent CI
5-ASA and has required intermittent courses of 1.15-3.91)
prednisone during the course of his disease. More The SCENIC international consensus panel re-
recently, he has been in endoscopic and clinical viewed the available evidence and recommended
remission. His colonoscopy under high-definition the following: For patients with endoscopically
white light is unremarkable. Biopsies are obtained invisible dysplasia (confirmed by a second GI pa-
and he has evidence of flat (non-polypoid) low thologist) referral to an endoscopist with expertise
grade dysplasia in his sigmoid colon. Biopsy re- in IBD surveillance by using chromoendoscopy
sults are confirmed by a second GI pathologist. with high definition colonoscopy is suggested.
There were no studies comparing surveillance
What is the next best step in his management? colonoscopy to colectomy in this group with en-
doscopically invisible dysplasia. However, should
A. Repeat high-definition colonoscopy at one a patient elect to proceed with colectomy, a total
year colectomy would be indicated rather than a par-
B. Repeat high-definition colonoscopy at two tial colectomy.
years
C. Referral to colorectal surgery for sigmoid REFERENCES
colectomy Laine L, Kaltenbach T, Barkun A, et al. SCENIC
D. Referral to colorectal surgery for total international consensus statement on surveillance
proctcolectomy and management of dysplasia in inflammatory
E. Repeat colonoscopy with chromoendoscopy bowel disease. Gastroenterology 2015;148:639-
651.e28.
CORRECT ANSWER: E Iannone A, Ruospo M, Wong G, et al. Chro-
moendoscopy for Surveillance in Ulcerative Colitis
RATIONALE and Crohn’s Disease: A Systematic Review of
“Flat dysplasia” is defined as endoscopically invis- Randomized Trials. Clin Gastroenterol Hepatol
ible dysplasia. One meta-analysis on the cancer 2017;15:1684-1697.e11.
risk of low-grade dysplasia in chronic ulcerative
colitis included 20 surveillance studies totaling
Question 50
508 flat low grade dysplasia or low grade dysplasia
Which of the following factors is associated with
with dysplasia-associated lesions or masses. The
an increased risk of colectomy in patients with
studies predated the use of current technology
ulcerative colitis?
with image enhanced endoscopy or even high res-
olution endoscopy. The authors reported a 9-fold A. Age less than 40 years
risk of developing cancer (odds ratio [OR] 9.0, 95 B. Extensive colitis
percent CI, 4.0-20.5) and a 12-fold risk of develop- C. Deep ulcerations
ing any advanced lesion (OR 11.9, 95 percent CI, D. High CRP and ESR
5.2-27). The absolute risk of cancer in this meta- E. All of the above
392 Digestive Diseases Self-Education Program®
REFERENCES
Saha S, Zhao Y-Q, Shah S, et al. Menstrual Cycle
Changes in Women with Inflammatory Bowel Dis-
ease: A Study from the Ocean State Crohn’s and
Colitis Area Registry. Inflamm Bowel Dis 2014;
20: 534-540.
Gawron LM, Sanders J, Steele KP, and Flynn
AD. Reproductive Planning and Contraception
for Women with Inflammatory Bowel Diseases.
Inflamm Bowel Dis. 2016: 22(2): 459-464.
Rungoe C, Simonsen J, Riis L, et al. Inflam-
matory bowel disease and cervical neoplasia: a
population-based nationwide cohort study. Clin
Gastroenterol Hepatol. 2015; 13(4): 693-700.
394 Digestive Diseases Self-Education Program®
Answers & critiques
CHAPTER 14
Gastrointestinal cancers
Seth D. Crockett, MD MPH, Sonia Kupfer, MD and
Christina Ha, MD
395
396 Digestive Diseases Self-Education Program®
noma, but not squamous cell carcinoma. A. Referral to a thoracic surgeon for an esopha-
E. Esophageal adenocarcinoma is more common gectomy.
in Black men compared to White men. B. Referral to a surgical oncologist for esopha-
gectomy with colon interposition graft.
CORRECT ANSWER: A C. Endoscopic mucosal resection and radiofre-
quency ablation of the Barrett’s segment.
RATIONALE D. Endoscopic submucosal dissection of the en-
The incidence of esophageal adenocarcinoma tire Barrett’s segment.
has increased dramatically over the past several E. Repeat surveillance upper endoscopy in
decades, and is much more common in men one year.
compared to women, such that esophageal cancer
is the least common GI cancer among women. In CORRECT ANSWER: C
developed countries such as the US, esophageal
adenocarcinoma is more common than squamous RATIONALE
cell carcinoma. Tobacco is a risk factor for both The patient in the stem has carcinoma in situ only,
adenocarcinoma and squamous cell carcinoma, with no evidence of deep invasion or lymph node or
but is more strongly associated with squamous cell other metastases. The risk of lymph node metasta-
cancer. Esophageal adenocarcinoma is more com- sis in this scenario is very small, and survival rate
mon in Whites than in Blacks. is excellent. Regardless of age or comorbidities,
the best management of this condition includes
REFERENCES endoscopic mucosal resection (EMR) of any visible
Siegel R, Naishadham D, and Jemal A. Cancer nodularity within the Barrett’s segment, accom-
Statistics, 2012. CA Cancer J Clin 2012;62: panied by radiofrequency ablation. Assuming the
10–29. EMR specimen(s) does not show invasive cancer,
Rubenstein JH, Shaheen NJ. Epidemiology, this procedure is considered curative. Thus, despite
Diagnosis, and Management of the patient’s stated preference for aggressive care,
Esophageal Adenocarcinoma. Gastroenterology. the risks of an esophagectomy in this scenario
2015 Aug;149(2):302-17. (especially with the patient’s comorbidities) would
Hur C, Miller M, Kong CY, Dowling EC, outweigh any perceived benefit. Additionally, while
Nattinger KJ, Dunn M, Feuer EJ. Trends in ESD does have a role in management of early
esophageal adenocarcinoma incidence and esophageal neoplasia, it is associated with a higher
mortality. Cancer. 2013 Mar 15;119(6):1149-58. stricture rate, and resection of the entire Barrett’s
segment would be unnecessary.
Question 3 REFERENCES
A 64-year-old man has been diagnosed with long Gamboa AM, Kim S, Force SD, et al. Treatment
segment Barrett’s esophagus with a focus of car- allocation in patients with early-stage esopha-
cinoma in situ on biopsy specimens. He also has a geal adenocarcinoma: Prevalence and predictors
history of morbid obesity, diabetes, and emphyse- of lymph node involvement. Cancer. 2016 Jul
ma. Cross-sectional imaging of the chest and abdo- 15;122(14):2150-7.
men is negative for metastases and an endoscopic Kim HP, Bulsiewicz WJ, Cotton CC, et al. Focal
ultrasound showed superficial nodular mucosa in endoscopic mucosal resection before radiofrequency
the distal esophagus, but did not demonstrate any ablation is equally effective and safe compared
enlargement of local lymph nodes. The patient ex- with radiofrequency ablation alone for the eradi-
presses a preference for aggressive treatment. The cation of Barrett’s esophagus with advanced neo-
most appropriate management would include: plasia. Gastrointest Endosc. 2012 Oct;76(4):733-9.
Chapter 14 — Gastrointestinal cancers 397
RATIONALE
Question 4 H. pylori infection is by far the most important
Which racial or ethnic groups have the highest risk factor for gastric cancer worldwide. Less com-
risk of gastric adenocarcinoma in the US? mon risk factors for gastric cancer include Lynch
syndrome, Peutz-Jeghers syndrome, Menetrier’s
A. African Americans and Non-Hispanic Whites disease, and germline mutations in the CDH gene
B. Asians, American Indians, and Non-Hispanic (encoding E-cadherin). However, there is some
Whites evidence that selenium, as well as high consump-
C. Middle-Easterners and Latinos tion of fruits and vegetables may have protective
D. Asians, Latinos, and African Americans effects against gastric cancer.
E. South Asians and Middle-Easterners
REFERENCES
CORRECT ANSWER: D de Martel C, Forman D, Plummer M. Gastric can-
cer: epidemiology and risk factors. Gastroenterol
RATIONALE Clin North Am. 2013 Jun;42(2):219-40.
Gastric cancer is a leading cause of cancer death world- Giardiello FM, Welsh SB, Hamilton SR, et al.
wide, and Helicobacter pylori infection causes almost Increased risk of cancer in the Peutz-Jeghers
90 percent of non-cardia cancers, which is the pre- syndrome. N Engl J Med. 1987 Jun 11;316(24):
dominant type. Rates of gastric cancer are highest in 1511-4.
Hispanics (from Central and South America in particu- Qiao YL, Dawsey SM, Kamangar F, et al. Total
lar) and East Asian populations (particularly recent im- and cancer mortality after supplementation with
migrants), as well as African Americans. Non-Hispanic vitamins and minerals: follow-up of the Linxian
Whites have the lowest rate of gastric cancer. General Population Nutrition Intervention Trial. J
Natl Cancer Inst. 2009 Apr 1;101(7):507-18.
REFERENCES
Islami F, DeSantis CE, Jemal A. Incidence Trends
of Esophageal and Gastric Cancer Subtypes by Question 6
Race, Ethnicity, and Age in the United States, 1997- A 55-year-old Latino man undergoes upper endos-
2014. Clin Gastroenterol Hepatol. 2018 Jun 11. copy for dyspepsia symptoms, during which several
Lui FH, Tuan B, Swenson SL, Wong RJ. small nonbleeding antral ulcers are seen. Biopsies
Ethnic disparities in gastric cancer incidence of the antrum and corpus are taken during endos-
and survival in the USA: an updated analy- copy, and the histopathology review demonstrates
sis of 1992-2009 SEER data. Dig Dis Sci. 2014 the following finding (see path image). Which of
Dec;59(12):3027-34. the following is true regarding this patient?
398 Digestive Diseases Self-Education Program®
Question 7
Which of the following is true regarding screening
for gastric cancer?
Image from Wikimedia commons A. Screening for gastric cancer in the general US
population is not recommended
A. Cessation of nonsteroidal anti-inflammatory B. Screening for gastric cancer is ineffective, even
medications is the most important factor in in high risk populations
ulcer healing C. Serum E-cadherin is a biomarker for gastric
B. Appropriate therapy can reduce his risk of cancer, with a sensitivity of 85 percent
intestinal-type gastric cancer D. Latino patients should be offered annual upper
C. The gastric injury is likely caused by iron endoscopy starting at age 50 years to screen
deposition for gastric cancer.
D. Partial gastrectomy is the treatment of choice E. During endoscopic screening, five random
for this condition gastric corpus biopsies should be obtained ac-
E. Repeat endoscopy in three months should be cording to the Sydney protocol.
performed to confirm ulcer healing
CORRECT ANSWER: A
CORRECT ANSWER: B
RATIONALE
RATIONALE Neither the US Preventive Services Task Force,
The pathology image shows an immunohistochemis- nor any specialty societies in the US recommend
try stain that is positive for H. pylori, which increas- screening for gastric cancer, outside of certain
es the risk of both peptic ulcer disease and gastric high risk populations. These populations in-
adenocarcinoma. Therefore, therapy with an appro- clude those with a strong family history of gastric
priate antimicrobial and acid suppression combina- cancer, individuals with familial adenomatous pol-
tion regimen active against H. pylori infection is yposis or Lynch syndrome, or perhaps those with
important to facilitate ulcer healing and to eradicate extensive intestinal metaplasia (IM). Screening for
this carcinogen. H. pylori is more associated with in- gastric cancer does take place in Japan (where the
testinal type gastric cancer than the diffuse type. Re- incidence of gastric cancer is much higher) and is
peat EGD is not routinely recommended to confirm associated with the detection of more early tumors
ulcer healing in patients with peptic ulcer disease, in asymptomatic patients. Serum E-cadherin lev-
unless there is concern for associated cancer. els are not used for gastric cancer detection. The
Sydney protocol was designed to detect H. pylori
REFERENCES infection, and involves sampling from specific
Nardone G, Staibano S, Rocco A, et al. Effect of areas in both the antrum and corpus. A related
Helicobacter pylori infection and its eradication technique, gastric mapping, is typically used for
on cell proliferation, DNA status, and oncogene surveillance of gastric IM and also requires sam-
expression in patients with chronic gastritis. Gut. pling of specific areas in the antrum, incisura, and
1999 Jun;44(6):789-99. corpus (and placing biopsies in separate jars).
Chapter 14 — Gastrointestinal cancers 399
hole biopsies of the lesion reveal uniform small heterogeneous mass in the head of the pancreas,
round cells with rare mitotic figures, a desmoplas- with enlargement of local lymph nodes. Which of
tic reaction and immunohistochemistry staining the following is not a risk factor for this disease?
is positive for synaptophysin and neuron-specific
enolase. What explains the diarrhea symptoms A. Female sex
this patient has? B. Smoking
C. African American race
A. Secretion of vasoactive intestinal peptide into D. Older age
the ileum E. Obesity
B. Concomitant irritable bowel syndrome
C. Yersinia infection CORRECT ANSWER: A
D. Crohn’s disease
E. Metastasis to the liver RATIONALE
This case represents a classic presentation of
CORRECT ANSWER: E pancreatic cancer, over 90 percent of which are
adenocarcinomas. Known risk factors for pancre-
RATIONALE atic adenocarcinoma include male sex, smoking,
This patient has a carcinoid tumor in the ileum, African American race, obesity and older age, as
which is the most common site of small intestinal well as a history of chronic pancreatitis. A strong
carcinoids. Histopathologic features of carcinoid family history of pancreatic cancer can also indi-
tumors include uniform small cells, a dense des- cate a genetic predisposition.
moplastic reaction, and IHC that can be positive
for chromogranins A, B, and C, synaptophysin and REFERENCE
neuron-specific enolase. In contrast to hindgut Midha S, Chawla S, Garg PK. Modifiable and non-
carcinoids, small intestinal carcinoid tumors modifiable risk factors for pancreatic cancer: A
frequently have metastasized at the time of diag- review. Cancer Lett. 2016 Oct 10;381(1):269-77.
nosis. While non-metastatic lesions do not cause
systemic symptoms because the liver inactivates
serotonin delivered via the portal vein, hepatic Question 12
metastasis allows for serotonin to be delivered All of the following conditions increase the risk of
into the systemic circulation and exert its effects pancreatic cancer, EXCEPT:
via the carcinoid syndrome.
A. Mutation in the cationic trypsinogen gene pro-
REFERENCE tease serine 1
Strosberg J. Neuroendocrine tumours of the small B. Lynch syndrome
intestine. Best Pract Res Clin Gastroenterol. 2012 C. Peutz-Jeghers syndrome
Dec;26(6):755-73. D. Familial atypical multiple mole melanoma
syndrome
E. Blue rubber bleb nevus syndrome
Question 11
A 67-year-old African American woman presents CORRECT ANSWER: E
with anorexia, weight loss and yellowing of the
eyes noticed by her spouse. Serum direct bilirubin RATIONALE
and alkaline phosphatase levels are elevated. She All of these conditions are associated with an in-
has a body mass index of 32 kg/m2 and is a former creased risk of pancreatic cancer, with the excep-
smoker. A CT scan shows biliary dilatation and a tion of blue rubber bleb nevus syndrome.
Chapter 14 — Gastrointestinal cancers 401
REFERENCES REFERENCES
Syngal S., Brand R.E., Church J.M., et al: ACG Brugge W R, Lewandrowski K, Lee-Lewandrowski
clinical guideline: genetic testing and management E et al. Diagnosis of pancreatic cystic neoplasms:
of hereditary gastrointestinal cancer syndromes. a report of the cooperative pancreatic cyst study.
Am J Gastroenterol 2015; 110: pp. 223. Gastroenterology. 2004;126:1330–1336.
Rulyak S.J., Lowenfels A.B., Maisonneuve P., Vege SS, Ziring B, Jain R, Moayyedi P; Clinical
et al: Risk factors for the development of pancre- Guidelines Committee; American Gastroenterology
atic cancer in familial pancreatic cancer kindreds. Association. American gastroenterological associa-
Gastroenterology 2003; 124: pp. 1292. tion institute guideline on the diagnosis and man-
agement of asymptomatic neoplastic pancreatic
cysts. Gastroenterology. 2015 Apr;148(4):819-22.
Question 15
A 53-year-old woman presents to gastroenterology
clinic after a CT scan of the abdomen (ordered by Question 16
her primary care physician to evaluate an umbili- Which of the following factors increase the risk of
cal hernia) identified a three cm cystic lesion in cholangiocarcinoma?
the tail of the pancreas. She is asymptomatic and
denies any upper abdominal pain. An endoscopic A. Hereditary pancreatitis
ultrasound is performed, which demonstrates B. Caroli’s disease
dilation of the pancreatic duct. Fine needle aspira- C. Exposure to iodinated contrast dye
tion of the cyst finds that the intracystic fluid CEA D. Alcohol use
level is 260 ng/mL. E. Primary biliary cholangitis
Zhu AX, Lauwers GY, Tanabe KK. Cholangiocarci- A. Persistent Hepatitis B infection from vertical
noma in association with Thorotrast exposure. transmission increases the risk of HCC by 2 fold.
J Hepatobiliary Pancreat Surg. 2004;11(6):430-3. B. Aflatoxin B1 exposure and hepatitis B infection
have a synergistic effect on the risk of HCC
C. Hepatitis B virus can lead to HCC only in the
Question 17 presence of cirrhosis
Which of the following is true regarding the epide- D. Hepatitis B virus integrates into host RNA to
miology of hepatocellular cancer (HCC)? inhibit tumor suppressor gene translation
Question 18 Question 19
Which of the following is correct regarding the A 70-year-old man is referred for evaluation of
pathogenesis of hepatocellular carcinoma? fatigue, weight loss, and elevated LFTs. An MRI
404 Digestive Diseases Self-Education Program®
REFERENCES Question 22
Heimbach JK, Kulik LM, Finn RS, et al. Which racial or ethnic group has the highest
AASLD guidelines for the treatment of he- reported incidence and mortality from colorectal
patocellular carcinoma. Hepatology. 2018 cancer in the US?
Jan;67(1):358-380.
Jacobson IM, Lim JK, Fried MW. American A. Latinos
Gastroenterological Association Institute Clini- B. African Americans
cal Practice Update-Expert Review: Care of Pa- C. Non-Hispanic whites
tients Who Have Achieved a Sustained Virologic D. Alaskan Natives
Response After Antiviral Therapy for Chronic E. Asians
Hepatitis C Infection. Gastroenterology. 2017
May;152(6):1578-1587. CORRECT ANSWER: D
RATIONALE
Question 21 Alaskan natives have both the highest inci-
You perform a colonoscopy for a patient who un- dence and mortality for colorectal cancer in
derwent sigmoid resection for stage two colorec- the US, markedly higher than any other
tal cancer one year ago. The colonoscopy reveals ethnic group.
one diminutive adenoma in the cecum, which
you remove with a cold snare. REFERENCES
Surveillance, Epidemiology, and End Results
When should you recommend the next (SEER) Program, 2016.
colonoscopy? National Center for Health Statistics, Cen-
ters for Disease Control and Prevention, 2016.
A. Ten years Perdue DG, Haverkamp D, Perkins C,
B. Five years Daley CM, Provost E. Geographic variation
C. Three years in colorectal cancer incidence and mortality,
D. One year age of onset, and stage at diagnosis among
E. Six months American Indian and Alaska Native people,
1990-2009. Am J Public Health. 2014 Jun;104
CORRECT ANSWER: C Suppl 3:S404-14.
RATIONALE
According to the Multi-Society Task Force Question 23
on Colorectal Cancer, colonoscopy should be A 26-year-old man is referred to GI clinic with
performed one year after resection, and again microcytic anemia. He is asymptomatic apart
three years later, in order to decrease the risk of from mild fatigue, and denies any GI symptoms.
metachronous colorectal cancer. Specifically, he denies abdominal pain, constipa-
tion or diarrhea, vomiting, or any hematochezia
REFERENCE or melena. He reports no known family history
Kahi CJ, Boland CR, Dominitz JA. Colonoscopy of GI conditions, but is adopted. Physical exam
Surveillance After Colorectal Cancer Resection: is normal. The patient was recently seen by oph-
Recommendations of the US Multi-Society Task thalmology, and was found to have an abnormal
Force on Colorectal Cancer. Gastroenterology. fundoscopic exam showing the following find-
2016. 150(3):758–768.e11. ings in both eyes:
406 Digestive Diseases Self-Education Program®
REFERENCES
Image was originally published in the Retina
Image Bank® website. Author: Maturi R. Photog-
rapher: Steele T. Retina Image Bank. 2012; “Bear
Tracks” File no: 2967. © the American Society of
Retina Specialists.
Retina Image Bank® Bertario L, Bandello F, Rossetti C, et al. Con-
genital hypertrophy of retinal pigment epithelium
The appropriate next step in management is: (CHRPE) as a marker for familial adenomatous pol-
yposis (FAP). Eur J Cancer Prev. 1993 Jan;2(1):69-75.
A. Schedule diagnostic upper endoscopy and
colonoscopy
B. Referral back to PCP for management of Question 24
diabetes A 33-year-old woman is seen for a well woman vis-
C. Genetic testing for defects in MLH1, MSH2, it, and reports a family history of colorectal cancer
MSH6, and PMS2 genes in her mother, who was diagnosed at age 55 years,
D. Referral for genetic counselling and died of the disease at age 58 years. When
E. Laboratory testing including iron studies, total should she begin colorectal cancer screening?
IgA and tissue trans-glutaminase IgA levels
A. Now (age 33 years), then annually
CORRECT ANSWER: A B. Age 50 years, then every five years
C. Age 45 years, then every 10 years
RATIONALE D. Age 40 years, then every five years
The fundoscopic image depicts congenital hy- E. Age 35 years, then every 10 years
pertrophy of the retinal pigment epithelium
(CHRPE), colloquially referred to as “bear tracks”. CORRECT ANSWER: D
CHRPE is characterized by multiple hyperpig-
mented round or oval-shaped spots on the retina, RATIONALE
typically bilaterally. CRHPE typically does not Current guidelines from the US Multisociety Task-
cause symptoms and is found during routine eye force recommend that CRC screening begin earlier
examinations. As a clinical sign, CHRPE has a for patients with a family history of CRC in a first
positive predictive value of approximately 90 per- degree relative who was diagnosed under the age
cent for Familial Adenomatous Polyposis (FAP). of 60 years. Screening should begin at age 40 or
Therefore, an endoscopic evaluation is warranted, 10 years prior to earliest age of diagnosis, and
especially in light of unknown family history and continue every five years thereafter.
likely iron deficiency anemia suggesting occult GI
bleeding. Both upper and lower endoscopy should REFERENCE
be performed given the high frequency of duode- Rex DK, Boland CR, Dominitz JA et al. Colorectal
nal adenomas and fundic gland polyps in patients Cancer Screening: Recommendations for Physi-
with FAP. It would not be appropriate to perform cians and Patients From the U.S. Multi-Society
genetic testing or refer to genetic counselling with- Task Force on Colorectal Cancer. Gastroenterol-
out better phenotypic characterization, and the ogy. 2017 Jul;153(1):307-323.
Chapter 14 — Gastrointestinal cancers 407
in the gastrointestinal tract, the clinical picture gery is not established but should be done in young
does not fit with this condition. adulthood. In patients in whom gastrectomy is not
currently pursued, endoscopic surveillance should
REFERENCE be done according to the Cambridge protocol with
Syngal S, Brand RE, Church JM, et al. ACG clini- a minimum of 30 gastric biopsies. The other mo-
cal guideline: Genetic testing and management of dalities listed here are not effective for treatment or
hereditary gastrointestinal cancer syndromes. Am surveillance in HDGC.
J Gastroenterol. 2015 Feb;110(2):223-62
REFERENCES
van der Post RS, Vogelaar IP, Carneiro F, et
Question 29 al. Hereditary diffuse gastric cancer: updated
A 32-year-old male presents for discussion of his clinical guidelines with an emphasis on germ-
family history and genetic test results. The patient line CDH1 mutation carriers. J Med Genet. 2015
has no personal history of cancer and reports no Jun;52(6):361-74.
gastrointestinal symptoms. The patient reports Syngal S, Brand RE, Church JM, et al.
that his mother developed lobular breast cancer at ACG clinical guideline: Genetic testing and
age 35, his maternal uncle died of diffuse gastric management of hereditary gastrointestinal
cancer at age 42 years and his maternal grand- cancer syndromes. Am J Gastroenterol. 2015
mother also had gastric cancer at age 52 years, Feb;110(2):223-62.
although the histology of his grandmother’s cancer
was not known. The patient’s mother underwent
genetic testing that revealed a mutation in CDH1. Question 30
The patient had genetic testing and was found to A 52-year-old female presents with rectal bleed-
carry the same mutation. ing. She undergoes a colonoscopy that reveals 30
polyps throughout the colon. These polyps range
What is the best next step in management? in size from three mm to 20 mm. In addition,
a three cm mass is found in the cecum that is
A. Computed tomography of the abdomen partially circumferential. Pathology of the polyps
and pelvis reveals tubular and tubulovillous adenomas and
B. Upper GI study biopsies of the mass show a villous adenoma. The
C. Refer to surgery for total gastrectomy patient reports no family history of colon polyps or
D. Endoscopic ultrasound cancer. There is no family history of other cancers.
She undergoes genetic testing with a multigene
CORRECT ANSWER: C panel and is found to carry two mutations in the
MutYH gene.
RATIONALE
This patient carries a pathogenic variant in CDH1, In addition to referring the patient to surgery for
consistent with a diagnosis of hereditary diffuse colectomy, what other testing should be done?
gastric cancer (HDGC). The patient’s family history
of diffuse gastric cancer and lobular breast cancer A. Endometrial biopsy
under age 50 is consistent with this diagnosis. The B. Computed tomography of the abdomen
lifetime risk of diffuse gastric cancer in HDGC is 70 C. Upper endoscopy with side viewing
percent for men and 56 percent for women. Risk of duodenoscopey
lobular breast cancer for women is 42 percent. Risk D. Endoscopic ultrasound
reducing total gastrectomy is recommended for
CDH1 mutation carriers. The optimal age for sur- CORRECT ANSWER: C
410 Digestive Diseases Self-Education Program®
without dysplasia, the hepatic flexure polyp is a The distance between the adenocarcinoma and
tubular adenoma and the sigmoid polyp is hyper- resection margin is a 0.5mm without evidence of
plastic. lymphovascular invasion.
When should the patient return for surveillance What is the next best step in management?
colonoscopy?
A. Colonoscopy in three months
A. One year B. Colonoscopy in one year
B. Three years C. Refer for surgical resection
C. Five years D. PET scan
D. Ten years
CORRECT ANSWER: C
CORRECT ANSWER: B
RATIONALE
RATIONALE A malignant polyp is defined as a polyp with can-
Surveillance intervals after polypectomy depend on cer invading through the muscularis mucosa and
size, pathology and polyp number assuming com- into the submucosa. Unfavorable histologic fea-
plete resection of the polyp and an adequate bowel tures of malignant polyps include: poorly differen-
preparation. A three year interval is recommended tiated, presence of angiolymphatic invasion and/
for the following findings on baseline colonoscopy: or positive margin of resection (defined as tumor
three to 10 tubular adenomas, one or more tubular less than one to two mm from the transected mar-
adenomas 10mm or larger, one or more villous ad- gin or tumor cells present at the margin). In cases
enomas, adenoma with high-grade dysplasia, sessile of a malignant polyp with unfavorable histological
serrated polyps 10mm or larger, sessile serrated features, surgery is recommended. With favorable
polyp with dysplasia or traditional serrated ad- features, endoscopic resection is curative. There is
enoma. In this case, the most advanced finding was a controversy as to whether sessile malignant polyps
sessile serrated polyp greater than 10mm; therefore, should all be treated surgically, or if endoscopic
the surveillance interval should be three years. resection is feasible in the case of favorable histo-
logic features. In the case, there were unfavorable
REFERENCE features, and surgery is the treatment of choice.
Lieberman DA, Rex DK, Winawer SJ, et al. Guide- There is no role for PET scan in the evaluation of
lines for colonoscopy surveillance after screening malignant polyps.
and polypectomy: A consensus update by the US
Multi-Society Task Force on Colorectal Cancer. REFERENCE
Gastroenterology 2012; 143:844. Volk EE, Goldblum JR, Petras RE, Carey WD,
Fazio VW. Management and outcome of patients
with invasive carcinoma arising in colorectal pol-
Question 33 yps. Gastroenterology. 1995 Dec;109(6):1801-7.
A 55-year-old male undergoes his first colonos- ASGE Standards of Practice Committee,
copy for screening. He reports no gastrointestinal Fisher DA, Shergill AK, et al. Role of endoscopy in
symptoms and no family history of colorectal the staging and management of colorectal cancer.
cancer. A 20mm pedunculated polyp is found in Gastrointest Endosc. 2013 Jul;78(1):8-12.
the rectosigmoid colon and resected completely National Comprehensive Cancer Network.
using snare polypectomy with cautery. Pathology Colon Cancer (Version 3.2018). https://www.
of the polyp reveals poorly differentiated adeno- nccn.org/professionals/physician_gls/pdf/colon.
carcinoma with invasion into the submucosa. pdf Accessed September 24, 2018.
412 Digestive Diseases Self-Education Program®
REFERENCES
Question 35 Giardiello FM, Allen JI, Axilbund JE Guidelines
A 51-year-old female diagnosed with colon cancer on genetic evaluation and management of Lynch
in the ascending colon one year ago returns for syndrome: a consensus statement by the US
surveillance colonoscopy. She underwent right Multi-society Task Force on colorectal cancer. Am
hemicolectomy and was found to have no lymph J Gastroenterol. 2014 Aug;109(8):1159-79.
node involvement. The tumor was described as Syngal S, Brand RE, Church JM, et al.
Chapter 14 — Gastrointestinal cancers 413
ACG clinical guideline: Genetic testing and TR, Robertson DJ, Rex DK. Colonoscopy Surveil-
management of hereditary gastrointestinal lance after Colorectal Cancer Resection: Recom-
cancer syndromes. Am J Gastroenterol. 2015 mendations of the US Multi-Society Task Force
Feb;110(2):223-62. on Colorectal Cancer. Am J Gastroenterol. 2016
Mar;111(3):337-46.
Question 36
A 64-year-old man presents for colonoscopy due Question 37
to new iron deficiency anemia and constipation. A A 40-year-old female comes to the office to estab-
circumferential mass is found in the sigmoid colon lish care. She reports a history of freckling on her
that precludes passage of the colonoscope. Biopsies lips and buccal mucosa that has faded since child-
of the mass confirm adenocarcinoma. A computed hood (see image). She reports a history of small
tomography colonography (CTC) is performed and bowel obstruction at age 20 years that required
confirms an almost completely obstructing mass in surgery to remove a large jejunal polyp. Since
the sigmoid, but no other polyps are visualized. The then, she reports about 10 cumulative polyps in
patient undergoes a sigmoid resection and returns the small intestine that have been removed by
for follow-up three months later. double balloon enteroscopy. Pathology of the
small intestinal polyps showed an arborizing
What is the best next step in management? pattern of smooth muscle proliferation. She had
genetic testing that showed a germline mutation
A. Colonoscopy now in the STK11 gene. Video capsule endoscopy, EGD
B. Colonoscopy in nine months and colonoscopy performed six months were nor-
C. CTC now mal without polyps.
D. CTC in nine months
E. fecal immunochemical testing now
CORRECT ANSWER: A
RATIONALE
The patient had an obstructing colon cancer
that precluded complete colonoscopy at diagno-
sis. Therefore, current guidelines recommend a
complete colonoscopy three to six months after
surgical resection to assess for synchronous le-
sions. Even though the patient had a CTC pre-
operatively, colonoscopy three to six months
later is still considered prudent. Small and flat
lesions can be missed or not reported by CTC,
but are highly relevant in a patient with colorec- What is the next best step in her management?
tal cancer. Fecal immunochemical testing is not
recommended for surveillance after colorectal A. CT enterography
cancer resection. B. Colonoscopy
C. MRI/MRCP
REFERENCE D. Small bowel follow through exam
Kahi CJ, Boland CR, Dominitz JA, Giardiello FM,
Johnson DA, Kaltenbach T, Lieberman D, Levin CORRECT ANSWER: C
414 Digestive Diseases Self-Education Program®
RATIONALE
Question 38 This patient likely has familial adenomatous
A 24-year-old undergoes upper endoscopy for polyposis (FAP) and colonoscopy should be per-
dyspeptic symptoms. She reports six months of formed to evaluate for colorectal adenomas. In
epigastric abdominal discomfort and abdominal addition to colorectal and duodenal adenomas, in-
fullness. She denies nausea, vomiting, heartburn dividuals with FAP can develop numerous fundic
or weight loss. There is no family history of gas- gland polyps in the stomach. Fundic gland polyps
trointestinal malignancies. Laboratory evaluation are not considered to have malignant potential,
reveals normal comprehensive metabolic panel. even with the presence of low-grade dysplasia.
Complete blood count shows mildly decreased he- Gastrectomy is not indicated for management of
moglobin and hematocrit. H. pylori stool antigen fundic gland polyps. However, it should be noted
testing was negative. The patient was treated with that prevalence of gastric cancer appears to be ris-
once daily proton pump inhibitor therapy. She ing in FAP and careful evaluation for lesions with
returns two months later and reports no improve- malignant potential should be undertaken. Fundic
ment in her symptoms. An upper endoscopy is gland polyps are also associated with proton pump
Chapter 14 — Gastrointestinal cancers 415
inhibitor use, though the patient in this question What is the most likely diagnosis?
had only taken this medication for a short period
of time and this is unlikely to explain the large A. Juvenile polyposis syndrome
number of gastric polyps. B. Menetrier’s disease
C. Cronkhite-Canada syndrome
REFERENCES D. Peutz-Jeghers syndrome
Syngal S, Brand RE, Church JM, et al. ACG clini- E. Gastric lymphoma
cal guideline: Genetic testing and management of
hereditary gastrointestinal cancer syndromes. Am CORRECT ANSWER: A
J Gastroenterol. 2015 Feb;110(2):223-62.
Mankaney G, Leone P, Cruise M, LaGuardia L, RATIONALE
O’Malley M, Bhatt A, Church J, Burke CA. Gastric This patient has juvenile polyposis syndrome
cancer in FAP: a concerning rise in incidence. Fam (JPS) with large gastric hamartomatous
Cancer. 2017 Jul;16(3):371-376. polyps. JPS is due to germline mutations
in BMPR1A and SMAD4. Individuals with
SMAD4 mutations are more likely to have pol-
Question 39 yps and cancer in the stomach and are at risk
A 27-year-old man presents for an episode of he- for hereditary hemorrhagic telangiectasias
matemesis. He reports a history of about 20 cu- (HHT). The incidence of upper GI cancers in
mulative colonic polyps starting at age 15 years. JPS is about 21 percent. Hamartomas in JPS
The pathology of these polyps shows glandular have characteristic glandular dilation with an
dilation and inflammatory cell infiltrates. The inflammatory infiltrate. Menetrier’s disease
patient reports no history of skin freckling, hair is a non-hereditary condition that presents
loss, nail changes or weight loss. His family his- with thickened gastric folds and characteristic
tory is significant for a father with gastric cancer foveolar hyperplasia with tortuous (cork-
age 45 years, paternal uncle with gastric cancer screw) and cystically dilated foveolar glands
age 48 years, paternal uncle with colon cancer on histology. Cronkhite-Canada syndrome is
age 46 years. Upper endoscopy is performed and a non-hereditary syndrome characterized by
shows large polypoid masses in the gastric fun- hamartomas in the gastrointestinal tract with
dus (see image). protein-losing enteropathy, hair loss and nail
growth problems. Peutz-Jeghers syndrome
is a hereditary syndrome due to mutations
in the STK11 gene that predisposes to ham-
artomas, especially in the small intestine, as
well as numerous other cancers. Patients with
Peutz-Jeghers also have characteristic freck-
ling on the mouth and buccal mucosa. Gastric
lymphoma can present with thickened folds,
but the pathology is not consistent with this
diagnosis.
REFERENCE
Syngal S, Brand RE, Church JM, et al. ACG clini-
cal guideline: Genetic testing and management
of hereditary gastrointestinal cancer syndromes.
Am J Gastroenterol. 2015 Feb;110(2):223-62.
416 Digestive Diseases Self-Education Program®
Question 41
What is the most likely finding on histological A 35-year-old female presented with rectal bleed-
examination of the polyp? ing. Colonoscopy showed approximately 25 pol-
yps throughout the colon (see images). Eighteen
A. Solid nests of monomorphic endocrine cells polyps ranging in size from eight to 12mm were
B. Dilated glands lined by normal oxyntic removed from the ascending colon. One 20mm
mucosa polyp was removed from the transverse colon.
C. Spindle cells Five polyps were removed from the descend-
D. Glandular dilation and inflammatory cell ing colon ranging in size from six to 10mm. One
infiltrates 25mm polyp was removed from the rectosigmoid
colon. The polyps in the ascending and descend-
CORRECT ANSWER: A ing colon were sessile serrated polyps. The polyp
in the transverse colon was a traditional serrated
RATIONALE polyp and the rectosigmoid polyp was a tubulo-
This patient has a small neuroendocrine tumor villous adenoma. (See two images on the follow-
(e.g., carcinoid) of the stomach in the setting ing page.)
of autoimmune atrophic gastritis and perni-
cious anemia. Neuroendocrine tumors appear What is the best next step in management?
as nests of endocrine cells and are derived from
enterochromaffin-like cells. Type 1 neuroendo- A. Refer for colectomy now
crine tumors are well-differentiated, often mul- B. Genetic testing for APC mutation now
tifocal and are associated with chronic atrophic C. Colonoscopy in six to 12 months
gastritis. Small tumors, as found in this patient, D. FIT-DNA stool testing in six to 12 months
can be resected endoscopically. Type 2 neuroen-
docrine tumors are associated with conditions CORRECT ANSWER: C
Chapter 14 — Gastrointestinal cancers 417
population. Mutations in PRSS1 are associated with Numerous studies confirm an association between
autosomal hereditary pancreatitis, a syndrome that new-onset non-insulin-dependent diabetes and
also carries increased risk of pancreatic cancer, development of pancreatic cancer, especially in
but will present at an earlier age with recurrent individuals who are elderly, have low BMI, experi-
pancreatitis. Lynch syndrome due to mutations in ence weight loss or do not have a family history
mismatch repair genes carries an estimated nine to of diabetes. The other tests would not be the best
11-fold increased risk of pancreatic cancer. choice for evaluation of the pancreas.
REFERENCE REFERENCE
Syngal S, Brand RE, Church JM, et al. ACG clini- National Comprehensive Cancer Network. Pancre-
cal guideline: Genetic testing and management of atic Adenocarcinoma (Version 2.2018). https://
hereditary gastrointestinal cancer syndromes. Am www.nccn.org/professionals/physician_gls/pdf/
J Gastroenterol. 2015 Feb;110(2):223-62 pancreatic.pdf Accessed September 24, 2018
Question 43 Question 44
A 62-year-old female developed a painful and A 34-year-old male with a history of ulcerative
swollen nodule on her right calf that resolved colitis diagnosed four years ago and maintained
spontaneously. She had recurrence of a similar on 5-ASA presents with right upper quadrant
nodule on her left calf three months later. She pain. Laboratory evaluation shows elevated
reports generalized fatigue, bloating and eight alkaline phosphatase with otherwise normal liver
lbs. weight loss in last three months. Her labora- chemistries. MRCP shows features consistent with
tory tests are normal, except for a fasting glucose primary sclerosing cholangitis. When should the
of 110. She denies jaundice, nausea or vomiting, patient have his next colonoscopy?
dysphagia or change in her bowel habits. She had
a colonoscopy that was normal two years ago. She A. Now
has no family history of cancer. B. One year
C. Four years
What is the next best step in her management? D. Six years
of patients with gastric cancer. Enteropathy-asso- United States and Western Europe, rates of squa-
ciated T cell lymphomas are associated with celiac mous cell carcinoma have declined, while rates of
disease, not H. pylori. Finally, patients with famil- adenocarcinoma are increasing. Tumor length of
ial adenomatous polyposis have an increased risk of less than or equal to two cm has been shown to be
gastric cancer and screening with upper endoscopy a predictor of long-term survival. Endoscopic ul-
is warranted, though there are no recommenda- trasound with fine needle aspirate has been shown
tions to screen for H. pylori in these patients. to be more sensitive for nodal staging of esopha-
geal cancer than computed tomography.
REFERENCE
Chey WD, Leontiadis GI, Howden CW, Moss SF. REFERENCE
ACG Clinical Guideline: Treatment of Helico- National Comprehensive Cancer Network. Esoph-
bacter pylori Infection. Am J Gastroenterol. 2017 ageal Cancer (Version 3.2018). https://www.
Feb;112(2):212-239. nccn.org/professionals/physician_gls/pdf/esoph-
ageal.pdf Accessed September 24, 2018.
Question 49
A 42-year-old man presents with dysphagia for Question 50
three months. He reports progressive symptoms Which of the following conditions is NOT associ-
primarily with solid food. He also reports a 10 lb. ated with an increased risk of small bowel adeno-
weight loss. He has no history of heartburn. He carcinoma?
has a 10 pack-year smoking history but quit 10
years ago. His BMI is 35 kg/m2. Upper endoscopy A. Lynch syndrome
reveals a mass in the distal esophagus. The mass B. Crohn’s disease
is four cm in length with a diameter of 1.2 cm. C. Celiac disease
Biopsies confirm adenocarcinoma. D. Familial adenomatous polyposis
E. Common variable immune deficiency
Which of the following is true about his condition?
CORRECT ANSWER: E
A. Tumor diameter is a predictor of long-term
survival RATIONALE
B. EUS-FNA is more sensitive for nodal staging Adenocarcinoma of the small intestine accounts
than computed tomography for 30-40 percent of small bowel cancers. Other
C. Rates of esophageal adenocarcinoma are de- small bowel malignant tumors include neuro-
creasing endocrine tumors, lymphomas, and sarcomas.
D. Tobacco use is the strongest risk factor Risk factors for small bowel adenocarcinoma
include Lynch syndrome, Crohn’s disease, celiac
CORRECT ANSWER: B disease and familial adenomatous polyposis. The
relative risk for small bowel adenocarcinoma in
RATIONALE patients with Crohn’s disease of the small bowel
Esophageal cancers are classified as squamous cell is about 30. In celiac disease, the relative risk of
carcinoma or adenocarcinoma. Tobacco and alco- small bowel adenocarcinoma is 60-80-fold. Celiac
hol use are strong risk factors for squamous cell disease is also associated with enteropathy-asso-
carcinoma, whereas obesity, high body mass index ciated T cell lymphomas. Patients with inherited
and Barrett’s esophagus are strong risk factors cancer syndromes including Lynch syndrome and
for adenocarcinoma. Tobacco use is considered a FAP are also at risk for small bowel adenocarci-
moderate risk factor for adenocarcinoma. In the noma. There is a 100-fold increased risk in Lynch
422 Digestive Diseases Self-Education Program®
syndrome, and FAP is also associated with a 50-90 indications. It is important to identify lesions
percent lifetime risk of duodenal adenomas, of with malignant potential. In this case, the le-
which 3-10 percent will develop into adenocar- sion is most consistent with a mucinous cystic
cinoma. Common variable immunodeficiency neoplasm (MCN) that has malignant potential.
(CVID) is associated with increased risk of small MCNs are found almost exclusively in middle-
bowel lymphoma, but has not been identified as a aged women. They are most commonly located
risk factor for adenocarcinoma. in the body or tail of the pancreas and typically
do not communicate with the pancreatic duct.
REFERENCES Fluid aspirate on MCNs shows high CEA and
Pan SY, Morrison H. Epidemiology of cancer of variable amylase levels. Pathology shows co-
the small intestine. World J Gastrointest Oncol. lumnar epithelium surrounded by ovarian-type
2011 Mar 15;3(3):33-42. stroma. Resected MCNs without cancer do not
Syngal S, Brand RE, Church JM, et al. require postoperative surveillance based on a
ACG clinical guideline: Genetic testing and large systematic review that showed no syn-
management of hereditary gastrointestinal chronous lesions or recurrence in the absence of
cancer syndromes. Am J Gastroenterol. 2015 invasive cancer. Therefore, there is no role for
Feb;110(2):223-62. imaging or EUS. Serum CEA levels are not used
for diagnosis or surveillance of MCNs.
Question 51 REFERENCES
A 62-year-old female presents with bilateral lower Elta GH, Enestvedt BK, Sauer BG, Lennon AM.
abdominal pain and constipation. Computed ACG Clinical Guideline: Diagnosis and Man-
tomography of the abdomen and pelvis was nor- agement of Pancreatic Cysts. Am J Gastroen-
mal except for a five cm cystic lesion in the tail of terol. 2018 Apr;113(4):464-479.
the pancreas. The patient underwent EUS that Nilsson LN, Keane MG, Shamali A et
confirmed a five cm cystic lesion with septations al. Nature and management of pancreatic
that did not appear to be in communication with mucinous cystic neoplasm (MCN): a system-
the pancreatic duct. Fluid aspiration of the cyst atic review of the literature. Pancreatology
showed CEA of 490ng/mL and amylase of less 2016;16:1028–36.
than250U/L. The patient underwent surgical resec-
tion of the lesion and pathology showed columnar
epithelium surrounded by ovarian-type stroma Question 52
without high-grade dysplasia or pancreatic cancer. Which of the following conditions is NOT
associated with increased risk of
What is the best next step in management? cholangiocarcinoma?
REFERENCE
Tyson GL, El-Serag HB. Risk factors for cholan-
giocarcinoma. Hepatology. 2011 Jul;54(1):173-84.
424 Digestive Diseases Self-Education Program®
Answers & critiques
CHAPTER 15
425
426 Digestive Diseases Self-Education Program®
mic control, decreases infectious complications, through dietary intake. Iron deficiency is an effect
reduces the need for surgery, and reduces mortal- of untreated celiac disease.
ity. As to the site at which to feed, the alternatives
include postpyloric (mainly, nasojejunal) and pre- REFERENCE
pyloric (nasogastric) tube placement. The former Shepherd SJ, Gibson PR. Nutritional Inadequacies
usually requires the assistance of an endoscopist of the Gluten-Free Diet in Both Recently-diag-
or a radiologist, and this may result in a delay in nosed and Long-term Patients with Celiac Disease.
commencing enteral nutrition. This delay may Journal of Human Nutrition and Dietetics. 2012;
have an impact on the clinical outcome because 26, 349-358.
it is now believed that enteral nutrition should
commence as soon as possible after adequate fluid
resuscitation in order to maximize clinical benefit. Question 4
In contrast, a nasogastric feeding tube can usually A 57-year-old Caucasian female presents with
be inserted immediately and with ease, such that stable, decompensated cirrhosis due to primary
prepyloric feeding can be started without delay. biliary cholangitis. Her cirrhosis is complicated
by large ascites managed with 20 mg of furose-
REFERENCE mide and 50 mg of spironolactone. She has small
Crockett S, Wani S, et al. American Gastro- esophageal varices, and hepatic encephalopathy
enterological Association Institute Guideline controlled with rifaximin and lactulose. She also
on Initial Management of Acute Pancreatitis. has a history of osteoporosis and chronic kidney
2018;154(4)1096-1101. disease with a creatinine of 1.6 mg/dL. Aside from
her chronic autoimmune illnesses, she has contin-
ued to try and optimize her health, and has a body
Question 3 mass index (BMI) of 28kg/m2. She wants to know
Patients with celiac disease, subscribing to a strict more about how to optimize what she is eating.
gluten free diet are at particular risk for which The recommended dietary energy and protein
nutrient deficiency? intake for this patient is:
energy intake of 35 to 40 kcal/kg and a protein A. As soon as she wakes up in the morning
intake of 1.2 to 1.5 g/kg per day protein is recom- B. Around lunchtime
mended. Lower calorie requirements, like 8-10 C. Before dinner
kcal/kg per day are believed to be safe in criti- D. Evening snack before bedtime
cally ill, hospitalized patients. Caloric intake for
patients with a body mass index of less than 30 CORRECT ANSWER: D
is recommended to be between 35 and 40 kcal/
kg per day, with opportunity to modify to a lower RATIONALE
caloric goal of 25-35 kcal/kg/day in patients with This question was evaluated in a trial by admin-
a BMI of 30-40, and 20-25 kcal/kg/day in patients istering a high protein nutritional supplementa-
with a BMI greater than 40. The increased protein tion to patients with cirrhosis from either 9 a.m.
goal of 1.2 to 1.5 g/kg per day is recommended to until 7 p.m. or from 9 p.m. to 7 a.m. Patients were
maintain an otherwise negative nitrogen balance administered the same supplement with caloric,
in patients with cirrhosis. This same recommenda- carbohydrate, and protein balance. Total body
tion is made for all patients with cirrhosis, regard- protein was evaluated at three, six and 12 months.
less of body mass index. Dietary protein does not There was not statistically significant improve-
contribute to hepatic encephalopathy, however ment in total body protein in patients adminis-
patients with cirrhosis demonstrate evidence of tered the supplement during the daytime, but
increased lean body mass loss. Healthy people those patients who received the supplementation
require 25-30 kcal/kg/day and 0.8-1.2 g/kg per in the evening experienced a statistically signifi-
day of protein. cant 2kg improvement in muscle mass over the 12
months of the study.
REFERENCE
EASL Clinical Practice Guidelines on Nutrition REFERENCE
in Chronic Liver Disease. European Association Tsuchia M, Sakaida I, et al. The Effect of a Late
for the Study of the Liver. Journal of Hepatology. Evening Snack in Patients with Liver Cirrhosis.
2019. 70, 172-193. Hepatology Research. 2005; 31(2): 95-103.
Question 5 Question 6
A 57-year-old Caucasian female presents with A 64-year-old man with amyotrophic lateral
stable, decompensated cirrhosis due to primary sclerosis had a PEG tube placed 12 hours ago. A
biliary cholangitis. Her cirrhosis is complicated routine KUB film shows evidence of air under the
by large ascites managed with 20 mg of furose- diaphragm. The team wants to call surgery for
mide and 50 mg of spironolactone. She has small further management of a perforation and leak.
esophageal varices, and hepatic encephalopathy The patient has a white blood cell count of 6.7 g/
controlled with rifaximin and lactulose. She also dL, hemoglobin of 11.2 g/dL, and a normal lactate.
has a history of osteoporosis and chronic kidney What is your next step?
disease with a creatinine of 1.6 mg/dL. Aside from
her chronic autoimmune illnesses, she has contin- A. Remove the PEG tube
ued to try and optimize her health, and has a body B. Call the surgical team for urgent exploratory
mass index (BMI) of 28kg/m2. laparotomy
C. Call the gastroenterology team for an urgent
When should you recommend that this patient upper endoscopy
consume a healthy snack such as a complex carbo- D. Perform a CT scan with oral contrast
hydrate or protein-based snack? E. Monitor patient’s clinical status
428 Digestive Diseases Self-Education Program®
CORRECT ANSWER: D
Question 14
During pregnancy this micronutrient taken in ex- RATIONALE
cess can lead to adverse effects on the pregnancy Remember, hyperosmolarity related to hypergly-
such as spontaneous abortion and birth defects: cemia shifts fluid from the intracellular space to
432 Digestive Diseases Self-Education Program®
Question 21 A. Cholecystokinin
A 24-year-old woman is being seen in the B. Leptin
emergency department with an acute episode C. Ghrelin
of hematemesis. She tells the admitting doc- d. Glucose-dependent insulinotropic
tor that her hematemesis occurred after several polypeptide
episodes of emesis. She denies nausea currently,
and she has not had any further episodes of CORRECT ANSWER: C
Chapter 15 — Nutrition, obesity and eating disorders 435
REFERENCE
Date Y, Murakami N, et al. The role of the gastric Question 24
afferent vagal nerve in ghrelin-induced feeding A 37-year-old man from central Tennessee
and growth hormone secretion in rats. Gastroen- presents with progressive worsening of abdom-
terology. 2002; 123 (4):1120. inal pain, nausea, and tingling of his mouth
after a heavy meal. He has had a difficult time
distinguishing which foods in particular give
Question 23 him these symptoms, but symptoms are typi-
What magnitude of weight loss has been shown cally experienced two to five hours after a large
to reduce obesity-related complications? meal. His diet is rich in meats, and he explains
that he is an avid outdoorsman and enjoys pre-
A. Five to 10 percent paring what he hunts. You belive his symptoms
B. 10-15 percent may be explained by exposure to which of the
C. 15-20 percent following ticks?
D. 25 percent or more
A. Deer tick
CORRECT ANSWER: A B. Wood tick
C. Lone star tick
RATIONALE D. Brown dog tick
Behavior modification remains the foundation
of obesity treatment and is crucial for long- CORRECT ANSWER: C
term weight loss maintenance. A modest weight
loss of five to 10 percent body weight has been RATIONALE
repeatedly shown to reduce obesity-related com-
plication. The AGA has published a white paper This patient likely has a recently recognized
detailing an approach to help gastroenterolo- food allergy sydrome known as Alpha-Gal al-
gists incorporate obesity management into their lergy. This is an IgE mediated hypersensitivity
practice that emphasizes support for patient, to the non-primate, mammalian carbohydrate
continuous assessment, intervention, and re- galactose-alpha-1,3-galactose. This is contracted
assessment. after a bite from the Lone star tick, Amblyomma
americanum. The tick introduces the alpah-gal
REFERENCES protein from animals which it has previous fed
Acosta A, Streett S, Kroh MD, et al. White Paper from. The hallmark is a hypersensitivity reaction
AGA: POWER - Practice Guide on Obesity and two to five hours post exposure, and includes
Weight Management, Education, and Resources. reactions to lamb, beef or pork.
Clinical gastroenterology and hepatology : the
official clinical practice journal of the American REFERENCE
Gastroenterological Association. 2017;15(5):631- Platts-Mills TA, Schuyler AJ, Hoyt AE, Com-
649.e610. mins SP. Delayed Anaphylaxis Involving IgE to
436 Digestive Diseases Self-Education Program®
Galactose-alpha-1,3-galactose. Current allergy and Which of the following is the next best step?
asthma reports. 2015;15(4):12.
A. Recommend low fat diet supplemented wit
medium chain triglyceride (MCT) oil
Question 25 B. Total Parenteral Nutrition
Which of the following is one of the best validated C. Obtain liver ultrasound
screening indicators for malnutrition risk? D. Placement of gastric feeding tube and start
isotonic formula
A. Patient reports unintentional weight loss E. Clear liquid diet
B. Patient reports following a low fat,
low carbohydrate diet CORRECT ANSWER: A
C. Patient has advanced liver disease
D. Patient reports a recent flu-like illness RATIONALE
Chylous ascites has been detected in approximate-
CORRECT ANSWER: A ly one to 6.6 percent of patients after colorectal
cancer surgery and up to 11 percent after pancre-
RATIONALE atic surgery. This is diagnosed with a triglyceride
The only well-validated measure of malnutrition in level greater than 110mg/dL, cholesterol level less
the above mentioned list is unintentional weight loss. than 200mg/dL, and presence of chylomicrons in
The other items can certainly be found in the history the peritoneal fluid.
of a patient with malnutrition, but are not in and of Conservative management of chylous ascites
themselves validated measures of malnutrition risk. consists of either high protein, low fat diet that is
rich in MCT oil or total parenteral nutrition. Both
REFERENCE options successfully resolve the chylous ascites in
Detsky AS, et al. What is Subjective Global Assess- a majority of patients. TPN is typically reserved
ment of nutritional status? JPEN Journal Paren- for those that cannot tolerate an oral diet or can-
teral Enteral Nutrition. 1987; 11(1):8-13. not meet their nutritional requirements orally or
enterally.
Medium chain triglycerides as opposed to long
Question 26 chain fats (LCF) reduce the production of chyle.
A 52-year-old man is status post a right hemico- Bile salts create micelles with LCF allowing easier
lectomy for colon cancer three weeks ago. He pres- access for pancreatic enzyme hydrolysis and ab-
ents to his follow up visit complaining of progres- sorption in the intestinal mucosa. Once absorbed,
sive abdominal distention. He does not complain the fatty acids and monoglycerides are re-esterified
of abdominal pain. On physical exam, his abdo- into triglycerides combining with cholesterol,
men is distended and there is a fluid wave and protein and other substances to form chylomicrons,
shifting dullness noted. A diagnostic paracentesis which enter the lymphatic system through lacteals.
is performed. The results are as follows: MCTs are directly absorbed into intestinal cells and
transported as free fatty acids and triglycerides to
Serum lactic dehydrogenase: 356 U/L the liver via the portal vein effectively bypassing the
Serum total protein: 6.0 g/dL lymph system. Parenteral lipids are phospholipids
Pleural fluid glucose: 42 mg/dL designed for intravenous infusion. They are deliv-
Pleural fluid lactic dehydrogenase: 242 U/L ered directly into the blood stream and as a result
Pleural fluid total protein: 2.9 g/dL do not pass through the lymph system as chyle.
Pleural fluid triglycerides: 210 mg/dL Therefore, intravenous lipids are not contraindi-
Pleural fluid cholesterol: 90 mg/dL cated in patients with chyle leaks.
Chapter 15 — Nutrition, obesity and eating disorders 437
REFERENCES RATIONALE
Sriram K, Meguid RA, Meguid MM. Nutritional Overfeeding causes increased CO2 production, in-
support in adults with chyle leaks. Nutrition creases the respiratory drive, and requires higher
2016;32(2):281-6. rates of intermittent mandatory ventilation on
Weniger M, D’Haese JG, Angele MK et al. mechanical ventilation.
Treatment options for chylous ascites after major The patient formula was providing enough
abdominal surgery: a systemic review. The Ameri- energy to meet 41.5kcal/kg (Normal maintenance
can Journal of surgery. 2016;211:206-213. caloric goal 25-35 kcal/kg).
Baek S-J, Kim S-H, Kwak J-M et al. Incidence This is calculated by the following formula:
and risk factors of chylous ascites after colorectal (Formula strength x rate x hours of feeding) /
cancer surgery. The American Journal of surgery. (patients’ weight (kg) (1.5 x 60cc/hr x 24 hours) /
2013;2016:555-559. 52 kg = 41.5 kcal/kg.
McCray S PC. When Chyle Leaks: Nutrition The respiratory quotient (RQ) obtained from indi-
Management Options. Practical Gastroenterology rect calorimetry (IC) is defined by the ratio of carbon
2004;17:60-76. dioxide production (VCO2 ) to oxygen consumption
(VO2 ). The physiologic range for the RQ exists be-
tween 0.67 to 1.3. Overfeeding results in lipogenesis,
Question 27 which increases the VCO2 and the RQ respectively.
A 66-year-old woman with emphysema is admit- A higher RQ leads to increases in minute ventilation,
ted to the medical intensive care for acute hypoxic alveolar ventilation and VO2. Those patients af-
and hypercapnic respiratory failure. She is 5 feet fected most are those who are nutritionally depleted,
4 inches tall and weighs 52 kg. ABG on admission stressed, or hypermetabolic, or those who have lim-
revealed the following: pH 7.24, pCO2 67, pO2 53, ited pulmonary reserve for eliminating CO2.
bicarb = 31. A CT of the chest showed multiple bi- In the absence of fever, the disease-specific
lateral patchy opacities consistent with multifocal variations in energy expenditure may be abolished
pneumonia. The patient was intubated, started on with deep sedation among medical and surgical
empiric vancomycin and cefepime, steroids, nebu- ICU patients.
lizer treatment, and midazolam for sedation. An
orogastric tube is placed and tube feeding is initi- REFERENCES
ated and brought to goal (Osmolite 1.5 @ 60cc/ Guenst JM, Nelson LD. Predictors of total par-
hr) by 72 hours of intubation. On day four, the enteral nutrition-induced lipogenesis. Chest
patient’s respiratory rate increases and starts to 1994;105:553-9.
breath over the ventilator. A chest x-ray shows the Liposky JM, Nelson LD. Ventilatory response
multifocal opacities have improved. Repeat ABG to high caloric loads in critically ill patients. Crit
shows pH 7.27, pCO2 64, pO2 75, bicarb = 34. Care Med 1994;22:796-802.
Haugen HA, Chan LN, Li F. Indirect calorime-
Which of the following is the next best step? try: a practical guide for clinicians. Nutr Clin Pract
2007;22:377-88.
A. Increase sedation A skanazi J, Rosenbaum SH, Hyman AI, et al.
B. Increase tidal volume Respiratory changes induced by the large glu-
C. Change antibiotics to cover for anaerobic bacteria cose loads of total parenteral nutrition. JAMA
D. Decrease feeding rate 1980;243:1444-7.
E. Check for residual tube feeding formula in Raurich JM, Ibanez J, Marse P, et al. Resting
stomach energy expenditure during mechanical ventilation
and its relationship with the type of lesion. JPEN J
CORRECT ANSWER: D Parenter Enteral Nutr 2007;31:58-62.
438 Digestive Diseases Self-Education Program®
state. Furthermore, patients with intestinal anas- Which catheter is the optimal choice for replace-
tomotic leakage are at risk for hyponatremia. Vol- ment in this patient?
ume and sodium are lost into the peritoneal cavity
and third space. Therefore, the patient requires A. Tunneled silicone catheter
increased sodium content (75 percent to 100 per- B. Implanted port
cent of normal saline) to correct for sodium losses. C. Peripherally inserted central catheter
This patient is at risk for refeeding syndrome D. Midline
given his state of malnutrition. Refeeding syn- E. Internal jugular triple lumen catheter
drome is characterized by an abrupt decrease in
serum potassium, magnesium and/or phosphorus CORRECT ANSWER: A
after the introduction of a consistent nutrient
source. This results from pancreatic stimulation RATIONALE
with increased insulin and decreased glucagon This patient is at high risk for thrombosis given her
secretion, which causes intracellular shift of the history of a DVT and active malignancy and is now
electrolytes. Parenteral nutrition should be started admitted for a catheter related bloodstream infection
with reduced calories and appropriate supplemen- after five weeks of use. A tunneled silicone catheter is
tation of electrolytes and vitamins. the optimal choice in this patient because they have
the lowest rates of infection and thrombosis. They
REFERENCES are durable for long-term use and commonly used
Kaser SA, Nitsche U, Maak M, et al. Could hy- for delivery of chemotherapy. Tunneled catheters en-
ponatremia be a marker of anastomotic leakage ter through a subcutaneous tissue tract and contain a
after colorectal surgery? A single center analysis bonded extravascular Dacron cuff near the exit site,
of 1,106 patients over 5 years. Langenbecks Arch which provides a barrier to external infection.
Surg 2014;399:783-8. In comparison to tunneled catheters, subcuta-
McCray S WS, Parrish CR. Much ado about neous ports are associated with an increased risk
refeeding. Practical Gastroenterology 2005; of catheter related blood stream infections.
23:26-44. Although a trained physician or nurse-led vascu-
Mehanna HM, Moledina J, Travis J. Refeeding lar access teams can conveniently place peripherally
syndrome: what it is, and how to prevent and treat inserted central catheters at the bedside, they have a
it. BMJ 2008;336:1495-8. significantly increased risk of deep vein thrombosis
when compared to other central venous catheters.
Midline catheter tips do not reach the superior
Question 31 vena cava. Parenteral nutrition formulas often have
A 66-year-old woman with metastatic gastric high osmolarity, which can cause thrombophlebitis
cancer and history of left leg deep vein throm- and thrombosis if delivered to a vessel with low
bosis on warfarin presents with one day of fever, flow rates. The superior vena cava and right atrium
chills and nausea. The patient was started on par- is the optimal location for delivery of a high osmo-
enteral nutrition five weeks ago after discovery of lality formula (greater than900 mOsm/L) which
a high-grade malignant bowel obstruction at her have rapid blood flow of 2-6L/min.
distal jejunum. She has been receiving FOLFOX There is increased risk of catheter thrombosis
through her implanted port. She takes clear and infection with multi-lumen catheters.
liquids for comfort, but is otherwise dependent
on the PN. Peripheral and central blood cultures REFERENCES
are positive for methicillin resistant staphylococ- Micic D, Semrad C, Chopra V. Choosing the Right
cus aureus. She is started on vancomycin and the Central Venous Catheter for Parenteral Nutrition.
port is removed. Am J Gastroenterol 2019;114:4-6.
442 Digestive Diseases Self-Education Program®
Buchman AL, Opilla M, Kwasny M, et al. Risk ing System of Balthazar grade D. The existing
factors for the development of catheter-related evidence supports the superiority of enteral
bloodstream infections in patients receiving home nutrition in mild and severe acute pancreatitis
parenteral nutrition. JPEN J Parenter Enteral if patients cannot tolerate oral feedings. TPN
Nutr 2014;38:744-9. is indicated only when enteral nutrition is not
Herc E, Patel P, Washer LL, et al. A Model to possible, or the patient is not able to meet their
Predict Central-Line-Associated Bloodstream In- nutritional requirements via enteral feedings.
fection Among Patients With Peripherally Inserted There is no difference in tolerance or clinical
Central Catheters: The MPC Score. Infect Control outcomes between patients fed via the nasogastric
Hosp Epidemiol 2017;38:1155-1166. as opposed to the nasojejunal route. Provision
Hammes M, Desai A, Pasupneti S, et al. of enteral nutrition to patients with severe acute
Central venous catheters: incidence and predic- pancreatitis helps maintain intestinal integrity by
tive factors of venous thrombosis. Clin Nephrol preventing permeability, maintaining functional
2015;84:21-8. tight junctions and promoting the role of com-
Madsen H FE. The Hitchhiker’s Guide to Par- mensal bacteria. In studies that compared enteral
enteral Nutrition Management for Adult Patients. versus parenteral nutrition in acute pancreati-
Practical Gastroenterology 2006:46-68. tis, there was faster resolution of SIRS, shorter
duration of time to resolution of the disease
and reduced overall complications in patients
Question 32 who received enteral nutrition. In studies that
A 62-year-old women with history of alcoholic compared enteral versus parenteral nutrition in
chronic pancreatitis is admitted with nausea, acute pancreatitis, there was faster resolution of
vomiting and epigastric discomfort radiating to the SIRS, shorter duration of time to resolution of
back. Laboratory workup is significant for hemo- the disease and reduced overall complications in
globin 10.2 g/dL, WBC 17,200/ml, serum amylase patients who received enteral nutrition. In studies
134 IU/L, serum lipase 88 U/L and serum LDH that compared enteral versus parenteral nutrition
380 IU/L. CT abdomen shows a calcified pancreas in acute pancreatitis, there was faster resolution
with peripancreatic fluid collection. Analgesia and of SIRS, shorter duration of time to resolution of
lactated ringers are started. A nasojejunal tube is the disease and reduced overall complications in
placed after three days due to inability to tolerate patients who received enteral nutrition. In studies
oral diet and standard formula is started. Twenty- that compared enteral versus parenteral nutrition
four hours after advancing the formula, the patient in acute pancreatitis, there was faster resolution
begins to have worsening abdominal cramping and of SIRS, shorter duration of time to resolution of
steatorrhea. Infectious diarrhea workup is negative. the disease and reduced overall complications in
Which of the following is the next best step? patients who received enteral nutrition.
She is not tolerating the standard polymeric
A. Hold tube feeds for 24 hours formula of tube feeds due to pancreatic insuf-
B. Switch to an elemental formula ficiency and therefore needs to switch to either
C. Switch to parenteral nutrition a formula that contains peptides and MCTs and
D. Addition of glutamine to the enteral formula if still intolerant, the next step is a nearly fat-
free elemental formula. Although glutamine has
CORRECT ANSWER: B a trophic effect on gut integrity, it has not been
shown to alter mortality, infections or length of
RATIONALE hospital stay in critically ill patients and therefore
This patient has severe acute pancreatitis ac- supplemental glutamine is not recommended in
cording to a Ransons Criteria of 3 and CT grad- this patient.
Chapter 15 — Nutrition, obesity and eating disorders 443
REFERENCES RATIONALE
Kalfarentzos F, Kehagias J, Mead N, et al. Enteral This patient likely has vitamin A and K deficiency
nutrition is superior to parenteral nutrition in due to improper and longstanding use of chole-
severe acute pancreatitis: results of a randomized styramine.
prospective trial. Br J Surg 1997;84:1665-9. Vitamin A deficiency causes conjunctival
Windsor AC, Kanwar S, Li AG, et al. Com- xerosis, Bitot’s spots and night blindness, which
pared with parenteral nutrition, enteral feeding explains her frequent injuries in the night. The
attenuates the acute phase response and im- easy bruising over the traumatized areas is likely
proves disease severity in acute pancreatitis. Gut caused by vitamin K deficiency which is respon-
1998;42:431-5. sible for posttranslational gamma carboxylation of
Jabbar A, Chang WK, Dryden GW, et al. Gut im- clotting factors (II, VII, IX, X) and anticoagulant
munology and the differential response to feeding proteins C and S.
and starvation. Nutr Clin Pract 2003;18:461-82. Cholesytramine is a sequestrant used to treat
Kagnoff MF. Immunology of the intestinal bile acid diarrhea. These agents are positively
tract. Gastroenterology 1993;105:1275-80. charged indigestible resins that bind bile acids in
DeWitt RC, Kudsk KA. The gut’s role in metabo- the intestine to form an insoluble complex that is
lism, mucosal barrier function, and gut immunol- excreted in the stool, thus preventing their secre-
ogy. Infect Dis Clin North Am 1999;13:465-81, x. tomotor actions on the colon. However, by binding
bile acids, they can inhibit fat emulsification and
micelle formation, which interferes with fat-soluble
Question 33 vitamin absorption. These medications should be
A 34-year-old woman presents with pain in her taken one hour before or four hours after meals. Co-
left foot. Six months ago, she had a cholecystec- lesevalem is another sequestrant that can be taken
tomy for recurrent biliary colic. Shortly after, she that does not interfere with vitamin absorption.
developed diarrhea, which responded well to cho- Hemoglobin a1c is used to evaluate for diabe-
lestyramine. She takes it four times per day with tes mellitus. While the foot x-ray was positive for a
meals. Over the last month, she has stubbed her fracture, other typical findings for charcot ar-
toe on the bedroom and hallway furniture multi- thropathy including osteopenia, loss of joint space,
ple times in the night when getting up to go to the and sclerosis were not present.
bathroom. She lives with her boyfriend who often Domestic abuse can be considered, but her
works late and has put a strain on their relation- areas of bruising are atypical for this presentation.
ship. Her left foot has multiple areas of scabbing Vitamin B12 deficiency can cause paresthesias
and purpura, especially her big toe, which is also of hands and feet, loss of vibration sense and posi-
swollen and painful with passive flexion. She has a tion sense, which could explain the foot injury, but
bruise over her right forehead and dry/thickened does not explain the eye manifestations and bruis-
appearing conjunctiva on eye exam. X-ray of the ing of the head.
left foot shows fracture of her left toe.Which of Referral to an ophthalmologist is not indicated
the following is the next best step? at this time.
Slowman-Kovacs SD, Braunstein EM, Brandt mocysteine levels but normal methylmalonic acid
KD. Rapidly progressive Charcot arthropathy fol- levels. Both methylmalonic acid and homocyste-
lowing minor joint trauma in patients with diabetic ine levels decline rapidly after treatment and can
neuropathy. Arthritis Rheum 1990;33:412-7. help with monitoring response to vitamin B12
Boosalis M. Vitamins. Contemporary Nutrition repletion.
Support Practice: A Clinical Guide. Philadelphia, Vitamin B12 is released from proteins ingested
PA: Gottschlich MM, 1998:145-163. by the action of hydrochloric acid and pepsin in
Dukowicz AC, Lacy BE, Levine GM. Small intes- gastric secretions. The free B12 binds R binder
tinal bacterial overgrowth: a comprehensive review. (transcobalamin I), which is secreted by the
Gastroenterol Hepatol (N Y) 2007;3:112-22. salivary glands. This complex travels to the small
intestine where pancreatic proteases hydrolyze
R protein to free B12, which then goes on to bind
Question 34 intrinsic factor (IF). IF is made by the gastric
A 44-year-old man presents to clinic with one parietal cells. The IF-B12 complex moves into the
month of fatigue and painful parasthesias. He ileum where it attaches to a specific IF receptor
underwent a biliopancreatic diversion with and is absorbed. After absorption, the vitamin is
duodenal switch procedure for weight manage- bound to transcobalamin II, a carrier protein that
ment seven months ago. He lost significant transports vitamin B12 to the liver, bone marrow
weight within the first four months, but has since and erythrocytes.
stabilized. His laboratory workup is significant The biliopancreatic diversion with duodenal
for Hgb 8.4 g/dL, MCV 112 fL and serum vitamin switch involves a restrictive and malabsorptive
B12 level 120 pg/mL. component. During this surgery, 60 percent of
the stomach is resected therefore limiting expo-
Which of the following confirms the diagnosis of sure to hydrochloric acid and reducing intrinsic
B12 deficiency? factor production. The duodenal stump is closed.
The main site of vitamin B12 absorption, the
A. Increased methylmalonic acid and homocyste- proximal ileum is transected and anastomosed
ine levels to the distal small bowel about 50cm proximal
B. Increased methylmalonic acid and normal ho- to the ileocecal valve. The distal ileal segment is
mocysteine levels anastomosed to the remaining stomach. This pa-
C. Normal methylmalonic acid and homocysteine tient will likely need lifetime vitamin B12 supple-
levels mentation.
D. Normal methylmalonic acid and increased
homocysteine levels REFERENCES
Stabler SP. Vitamin B12 deficiency. N Engl J Med.
CORRECT ANSWER: E 2013;368(2):149-160.
Carmel R. Cobalamin (Vitamin B12). Modern
RATIONALE Nutrition in Health and Disease. Philadelphia, PA:
Measurements of methylmalonic acid total ho- Lippincott Williams & Wilkins, 2006:482-497.
mocysteine levels or both is helpful in making the Moestrup SK. New insights into carrier binding
diagnosis of vitamin B12 deficiency in patients and epithelial uptake of the erythropoietic nutri-
who have not yet been treated. The levels of both ents cobalamin and folate. Curr Opin Hematol
methylmalonic acid and total homocysteine are 2006;13:119-23.
markedly elevated in almost all patients with Xanthakos SA. Nutritional deficiencies in obe-
vitamin B12 deficiency. In contrast, patients with sity and after bariatric surgery. Pediatr Clin North
isolated folate deficiency will have elevated ho- Am 2009;56:1105-21.
Chapter 15 — Nutrition, obesity and eating disorders 445
Jeppesen PB, Sanguinetti EL, Buchman A, et al. day whereas a high output fistula drains greater
Teduglutide (ALX-0600), a dipeptidyl peptidase than 500ml/day.
IV resistant glucagon-like peptide 2 analogue, The recommended daily allowance (RDA) for
improves intestinal function in short bowel syn- protein in healthy adults is 0.8 g/kg body weight
drome patients. Gut 2005;54:1224-31. per day. High output fistulas usually require
Jeppesen PB, Gilroy R, Pertkiewicz M, et al. 30-35 kcal/kg of energy and 1.5-2.5 g/kg/day
Randomised placebo-controlled trial of teduglu- of protein for achievement of positive nitrogen
tide in reducing parenteral nutrition and/or intra- balance and wound healing. Studies have shown
venous fluid requirements in patients with short decrease in mortality in patients who are provided
bowel syndrome. Gut 2011;60:902-14. 30-35 kcal/kg for management of enterocutane-
Pevny S, Maasberg S, Rieger A, et al. Experi- ous fistulas. Low output fistulas typically require
ence with teduglutide treatment for short bowel 20-30 kcal/kg/day and 1-1.5g/kg/day of protein.
syndrome in clinical practice. Clin Nutr 2018. These patients also often need increased provision
of trace minerals and vitamins such as vitamin
C, copper , selenium and zinc to promote wound
Question 36 healing. Although this patient is starting on TPN
A 59-year-old man is admitted to the surgery because it has been shown to decrease enteral
service after findings of a colocutaneous fistula. fluid secretions by 30-50 percent, the goal will be
Two weeks ago he underwent partial colectomy to transition as soon as possible to enteral nutri-
with primary anastomosis for stage II colon ad- tion. It is recommended that patients receive at
enocarcinoma of his descending colon. Over the least 20 percent of their calories via the enteral
last three days, he noticed leakage of fecal matter route if possible to maintain mucosal integrity,
coming from his skin in the left lower quadrant. hormone signaling and immune function.
He lost seven pounds since his procedure and
currently has a BMI of 21kg/m2. Since admission, REFERENCES
the fistula has put out 400-600ml of fluid daily. Gribovskaja-Rupp I, Melton GB. Enterocutaneous
The primary service wishes to pursue conserva- fistula: proven strategies and updates. Clin.Colon
tive management by making the patient NPO and Rectal Surg. 2016;29(2):130-7.
starting parenteral nutrition. Sheldon GF, Gardiner BN, Way LW, et al.
Management of gastrointestinal fistulas. Surg
Which combination of energy and protein require- Gynecol Obstet 1971;133:385-9.
ments is most appropriate for this patient? Badrasawi M, Shahar S, Sagap I. Nutritional
Management in Enterocutaneous Fistula. What is
A. Total Energy: 20-25 kcal/kg/day, Protein: 2.5g/ the evidence? Malays J Med Sci 2015;22:6-16.
kg/day Council NR. Recommended Dietary Allow-
B. Total Energy: 15-20 kcal/kg/day, Protein: 2.0g/ ances. Washington, DC: National Academy Press,
kg/day 1989.
C. Total Energy: 30-35 kcal/kg/day, Protein: 1.5g/ Dudrick SJ, Maharaj AR, McKelvey AA. Artifi-
kg/day cial nutritional support in patients with gastroin-
D. Total Energy: 30-35 kcal/kg, Protein: 0.8 g/kg testinal fistulas. World J Surg 1999;23:570-6.
CORRECT ANSWER: C
Question 37
RATIONALE A 32-year-old with history of Roux-en-Y gastric
This patient has a high output colocutaneous fis- bypass five months ago presents to clinic with
tula. A low output fistula drains less than200ml/ complaints of a rash. Two weeks ago, she noticed
Chapter 15 — Nutrition, obesity and eating disorders 447
a dry scaly rash over both her legs. She tried bound to albumin and 30 percent bound to alpha-
moisturizing ointments without relief. She also macroglobulins. Physiologic conditions that cause
noticed frequent dandruff in the last few days. hypoalbuminemia may impair hepatic release of
Her appetite has been poor since the surgery zinc, which may cause a falsely low serum level.
and is concerned that she is losing the weight Vitamin C is typically absorbed in the jejunum
too fast. Examination shows contiguous dry and and ileum through sodium, energy dependent
scaly rash over her anterior legs and cracking and active transport. Its absorption is typically not
peeling at her fingertips. Labs are significant for affected by roux-en-y gastric bypass surgery and
copper 1.28 ug/mL(0.75-1.45 ug/mL), zinc 0.62 usually presents with anemia, bleeding gums,
ug/mL (0.66-1.10 ug/mL), selenium 102 ng/mL impaired wound healing, fatigue and depression
(80-142 ng/mL), albumin 3.1 g/dL (3.5-5.0g/dL), rather than causing skin manifestations.
triene:tetraene ratio 0.24 (0.01-0.038).
REFERENCES
A deficiency in which nutrient most likely explains John S, Hoegerl C. Nutritional deficiencies after
this patient’s symptoms? gastric bypass surgery. J Am Osteopath Assoc
2009;109:601-4.
A. Copper Lopes TI, Geloneze B, Pareja JC, et al. Blood
B. Zinc Metabolome Changes Before and After Bariatric
C. Vitamin C Surgery: A (1)H NMR-Based Clinical Investiga-
D. Essential fatty acids tion. OMICS 2015;19:318-27.
Wene JD, Connor WE, DenBesten L. The
CORRECT ANSWER: D development of essential fatty acid deficiency in
healthy men fed fat-free diets intravenously and
RATIONALE orally. J Clin Invest 1975;56:127-34.
This patient has essential fatty acid deficiency Mogensen K. Essential Fatty Acid Deficiency.
(EFAD) after roux-en-y gastric bypass surgery. Practical Gastroenterology 2017;164:37-44.
Monitoring of macro- and micronutrients af- Barr LH, Dunn GD, Brennan MF. Essential fatty
ter this procedure is important as it can lead to acid deficiency during total parenteral nutrition.
multiple deficiencies, including essential fatty Ann Surg 1981;193:304-11.
acids. Patients with EFAD often complain of a dry, Hamilton C, Austin T, Seidner DL. Essential
scaly rash, impaired wound healing and immune fatty acid deficiency in human adults during par-
dysfunction. The minimal amount of fat to pre- enteral nutrition. Nutr Clin Pract 2006;21:387-94.
vent EFAD is 100g per week. In the absence of the MJ D. Zinc. Present Knowledge in Nutrition.
essential fatty acids (Linoleic acid and alpha-lino- 8th ed. Washington DC: ILSI Press, 2001:329-
lenic acid), oleic acid is metabolized to eicosatrie- 343.
onic acid (triene) and there is reduced production Prelack K, Sheridan RL. Micronutrient supple-
of eicosatetraenoic acid (tetraene). This results in mentation in the critically ill patient: strategies for
an elevated triene:tetraene ratio. A ratio greater clinical practice. J Trauma 2001;51:601-20.
than 0.2 is diagnostic. V R. Micronutrient Recommendations for
Although, copper deficiency is possible after Wound Healing. Support Line 2004;24:3-9.
roux-en-y gastric bypass, this patient’s levels are
within normal limits.
Although zinc deficiency can cause dry, scaly Question 38
rashes, it is unclear if the patient has a true zinc A 28-year-old woman presents with symptoms
deficiency given coexistent hypoalbuminemia. The of abdominal pain, distention, nausea without
liver releases zinc into circulation with 70 percent vomiting and early satiety. She reports a weight
448 Digestive Diseases Self-Education Program®
loss of approximately 20 pounds over the past six A. Hold the tube feeds
months. Her physical exam is notable for a BMI B. Check his stools for Clostridium difficile
of 14.6 and a distended abdomen, but otherwise infection
unremarkable. Laboratory evaluation reveals C. Switch to a lower volume hyperosmolar formula
hyponatremia, hypokalemia, hypomagnesemia, D. Start a probiotic
hypophosphatemia, and low serum creatinine. E. Start Imodium therapy
Abdominal imaging reveals a large gastric bezoar.
CORRECT ANSWER: B
Which of the following diagnoses is most likely?
RATIONALE
A. Anorexia nervosa In ICU patients receiving enteral nutrition, diar-
B. Pyloric stenosis rhea is common and can be severe resulting in
C. Celiac disease dehydration, electrolyte disturbances and perianal
D. Jejunal intussusception skin breakdown. Despite that, enteral nutrition
E. Functional dyspepsia should not be reflexively held but instead continued
while evaluating the cause of the diarrhea. Most
CORRECT ANSWER: A nosocomial diarrhea is self-limited. Common eti-
ologies of diarrhea in enterally fed patients include
RATIONALE infections such as C. difficile, medications (proton
The notable features that suggest a diagnosis of pump inhibitors, antibiotics, selective serotonin
anorexia are the BMI, the laboratory findings and reuptake inhibitors etc.) and lack of fiber in the
the gastric bezoar on imaging which suggests de- enteral formula. Hyperosmolar formulas are less
layed gastric emptying. Delayed gastric emptying volume and are more calorically concentrated but
can occur in the setting of restricted oral intake and can also cause an osmotic diarrhea. Although there
weight loss. A potential mechanism for the gastro- is a lack of data that probiotics decrease diarrhea in
paresis may be smooth muscle atrophy due to long- this patient population, the most consistent benefit
standing protein calorie malnutrition. The primary of probiotics has been the decrease in infectious
mode of treatment is nutritional rehabilitation. morbidity in select patient populations (transplan-
tation, trauma, pancreatectomy) where it has been
REFERENCE shown to decrease infectious complications. Soluble
Bern et al. Gastrointestinal manifestations of eat- fiber supplementation is recommended in patients
ing disorders. JPGN. 2016;63:77-85. that have diarrhea with enteral feedings. Therefore,
the management of this patient should consist of
a thorough search for a cause of diarrhea includ-
Question 39 ing checking for C. difficile, assessing for potential
A 66-year-old man is in the ICU after esophagecto- medication side effects and possibly supplementing
my for esophageal adenocarcinoma. He underwent with soluble fiber.
PEG tube placement last week for inadequate oral
intake due to dysphagia. He had been tolerating his REFERENCE
enteral feedings with a standard polymeric formula McClave, SA, Taylor BE, Martindale RG et al.
until last night when the nursing staff noted the Guidelines for the provision and assessment
development of voluminous diarrhea. His perianal of nutrition support therapy in the critically ill
area is erythematous due to diarrhea. patient: society of critical care medicine (SCCM)
and American society for parenteral and enteral
What is your first step in managing this patient’s nutrition (ASPEN). J Parenter Enteral Nutr.
diarrhea? 2016;4(2):159-211.
Chapter 15 — Nutrition, obesity and eating disorders 449
has had difficulty with nausea and vomiting, she as shuffling gait and mental sluggishness. Zinc de-
has numbness and tingling in her extremities and ficiency causes impaired wound healing, immune
she is experiencing ataxia. On physical exam, she dysfunction, night blindness and skin lesions.
has a sensory and motor neuropathy. Her Hgb is Copper deficiency causes a myeloneuropathy, ane-
13.5 g/dl, MCV 93 fL, WBC 5.7x109/L and platelets mia and neutropenia. Selenium deficiency causes
250x109/L. cardiomyopathy and skeletal muscle dysfunction.
Essential fatty acid deficiency is rare in bariatric
What is the most likely cause of this patient’s surgery patients and causes a dry scaly rash, hair
symptoms? loss, hair depigmentation and poor wound heal-
ing. Therefore, the most likely explanation for this
A. Copper deficiency patient’s symptoms of ataxia, peripheral neuropa-
B. Selenium deficiency thy, in the setting of nausea and vomiting without
C. Zinc deficiency anemia is thiamine deficiency.
D. Thiamine deficiency
E. Vitamin B12 deficiency REFERENCES
Mechanick JI,Youdim A, Jones DB. Clinical
CORRECT ANSWER: C Practice Guidelines for the Perioperative Nutri-
tional, Metabolic, and Nonsurgical Support of the
RATIONALE Bariatric Surgery Patient—2013 Update: Cospon-
Vitamin and mineral deficiencies are common after sored by American Association of Clinical Endo-
bariatric surgery. The most common deficiencies are crinologists, The Obesity Society, and American
thiamine, vitamin B12, vitamin D, iron and copper. Society for Metabolic & Bariatric Surgery. Obesity.
There is also more awareness recently of subclinical 2013;21(1):S1-27.
micronutrient deficiencies in obese patients preop- Saltzman E, Karl JP. Nutrient deficien-
eratively. Thiamine deficiency has been reported cies after gastric bypass surgery. Ann Rev Nutr
in 0-29 percent of patients preoperatively and in 2013;33:183-203.
one to 49 percent of patients after bariatric surgery.
Symptomatic thiamin deficiency occurs weeks to
years after surgery and can occur after all bariatric Question 43
procedures. The most commonly reported deficiency A 57-year-old woman presents with a primary
syndrome is Wernicke’s encephalopathy. This is a complaint of intermittent dysphagia complicated by
neuropsychiatric disorder consisting of abnormal occasional food impactions. She undergoes EGD and
eye movements, ataxia and altered mental func- the endoscopist notes linear furrows and stacked
tion. Risk factors for thiamine deficiency in bariatric circular rings in her esophagus. Biopsies from the
patients include persistent vomiting, avoidance of proximal and distal esophagus revealed up to 25
eating and noncompliance with supplementation. eosinophils per hpf. She would like to investigate
Thiamine deficiency or beriberi has two entities. Dry non-pharmacologic treatments. Which of the follow-
beriberi consists of symmetrical peripheral neuropa- ing diets is most effective in treating this disease?
thy with both sensory and motor impairments. Wet
beriberi affects the heart and can result in cardio- A. Six food elimination diet (SFED)
megaly, cardiomyopathy, heart failure and tachycar- B. Elemental diet
dia. Post bariatric surgery patients should take a B C. Allergy testing directed elimination diet
complex vitamin containing 50mg thiamine once to D. Gluten free diet
twice daily in addition to a multivitamin. E. Cow’s milk elimination diet
Vitamin B12 deficiency presents with a macro-
cytic anemia, yellow skin, neurologic findings such CORRECT ANSWER: B
Chapter 15 — Nutrition, obesity and eating disorders 451
A. Lisdexamfetamine Question 46
B. Sleeve gastrectomy A 23-year-old woman presents to clinic to discuss
C. Orlistat therapy her diet. She has recently become a vegan and is
D. Sibutramine asking about the safety of this diet and any needed
E Phentermine-topiramate nutritional supplementation.
decreased serum cholesterol levels, decreased risk ability to see well at night and onset of non bloody
of heart disease and hypertension and a decreased diarrhea. He also noted to have frequent “colds”
risk of diabetes. Vegetarians also have lower BMIs and minor cuts and bruises which take long to
than non-vegetarians and a lower risk of cancer. heal. He has been married but unable to have
The vegetarian diet is lower in saturated fats and children. Physical examination reveals a bald male
higher in fiber, magnesium, potassium, vitamin with erythematous rash on his feet. Neurological
E and C as well as folate. The vegetarian diet and examination reveals normal tone and gait is ob-
more commonly, the vegan diet often contains served normal. Laboratory tests show normal TSH
lower intake of vitamin B12, calcium, vitamin D, and T4, normal CBC but low alkaline phosphatase.
zinc and long chain fatty acids. Lacto-ovo-vege- Stool studies are negative.
tarians obtain adequate vitamin B12 from dairy
foods or eggs. Vegans will need to obtain vitamin What supplementation is appropriate?
B12 from foods fortified with B12 or vitamin B12
supplements since no unfortified plant foods con- A. Biotin
tain significant amounts of vitamin B12. B. Zinc
The iron in plant food is nonheme iron, which is C. Thiamine
more susceptible to inhibitors and enhancers of D. Vitamin A
iron absorption. Organic acids such as vitamin
C can increase the absorption of iron. In addi- CORRECT ANSWER: B
tion, avoiding inhibitors of iron such as phytates,
calcium and polyphenolics (found in tea, coffee, RATIONALE
cocoa) will also enhance absorption. Incidence of This patient has zinc deficiency. Zinc deficiency
iron deficiency in vegetarians is similar to non- is one of the major factors contributing to burden
vegetarians and most experts do not recommend of disease in developing countries. Populations
supplementation unless a patient is found to be in South Asia, South East Asia, and sub-Saharan
iron deficient. Africa are at greatest risk of zinc deficiency. Low
serum zinc levels can be found in patients with
REFERENCES Crohn’s disease and subsequent small bowel
Position of the American Dietetic Association: veg- malabsorption, short bowel syndrome, hookworm
etarian diets. J Am Diet Assoc. 2009;109:1266-1282. infestation, pancreatic insufficiency, exclusive
Agnoli C, Baroni L, Bertini I et al. Position pa- parental nutrition, strict vegetarian diets, and an-
per on vegetarian diets from the working group of orexia nervosa. Common features include alope-
the Italian society of human nutrition. Nutr Metab cia, dysgeusia, acrodermatitis or bullous pustular
Cardiovasc Dis 2017;27(12):1037-1052. dermatitis(erythematous, pustular, vesiculobul-
barlesions), and low alkaline phosphatase. Zinc
deficiency is also associated with an increased
Question 47 incidence of night blindness, diarrhea, delayed
A 65-year-old male who recently migrated from wound healing and upper respiratory tract infec-
South Africa was referred to you by the primary tions. Delayed sexual maturation, oligospermia,
care physician for evaluation of new onset diar- hypogonadism and impotence are not uncommon.
rhea and weight loss. He has a history of alco- Medications including penicillamine, various di-
holism and known pancreatic insufficiency. He uretics, and sodium valproate can inhibit absorp-
reports that for many years he has decreased tion of zinc.
appetite because of difficulty with smell and taste. Biotin deficiency can present with similar hair
As a teenager he remembers having occasional and skin findings but additionally patients have
red blisters on his face. As he aged, he reports in- hypotonia, ataxia, seizures, conjunctivitis and
454 Digestive Diseases Self-Education Program®
hearing loss. Patients with thiamine deficiency comparison, roux-en-Y gastric bypass has 60-75
have nystagmus, peripheral neuropathy and percent expected loss, vertical sleeve gastrectomy
ataxia/ Vitamin A deficiency presents with night a 50-70 percent weight loss (3 year data) and
blindness and skin manifestations but in the ab- laproscopic adjustable gastric banding the lowest
sence of diarrhea. with 30-50 percent weight loss. It is important to
note that although biliopancreatic diversion with
REFERENCES duodenal switch provides the greatest weight loss
Ogawa Y,Kinoshita M,Shimada S,Kawamura T, and remission of diabetes, it will also cause more
Zinc and Skin Disorders. Nutrients. 2018 Feb 11 metabolic complications compared with the other
Assessment of marginal zinc status in humans.J surgeries. Also noted the roux-en-Y gastric bypass,
Nutr. 2000;130(5S Suppl):1350S. vertical sleeve gastrectomy and biliopancreatic
Prasad AS Zinc and immunity.Mol Cell Bio- diversion with duodenal switch all improves meta-
chem. 1998;188(1-2):63. bolic disease with vertical sleeve gastrectomy often
US Department of Health and Human Ser- used in a first step of a staged approach for weight
vices (HHS) and US Department of Agriculture loss which can later be converted to biliopancre-
(USDA): Dietary guidelines for Americans – 2015- atic diversion with duodenal switch procedure.
2020, eight edition(2015).
REFERENCES
Bays HE, Seger JC, Primack C, McCarthy W, Long
Question 48 J, Schmidt SL, Daniel S, Wendt J, Horn DB, West-
A 43-year-old female with a BMI 50kg/m2 and man EC: Obesity algorithm, presented by the Obe-
a history of hypertension and diabetes comes to sity Medical Association. www.obesityalgorithm.
you for advice on weight loss procedures. She has org. 2016-2017.
spent years trying to lose weight by diet and exer- Rubino F, Nathan DM, Eckel RH et al. Meta-
cise with no success. You review her labs which are bolic surgery in the treatment for type 2 diabetes:
unremarkable except HgA1C of 8.3 percent. She a joint statement by international diabetes organi-
has a referral for the bariatric surgeon but comes zations. Diabetes care. 2016;39:861-877.
to you investigating other options but wants to
know which bariatric procedure will provide the
most weight loss with greatest improvement in her Question 49
diabetes. Based on her question, what procedure Leptin secretion from adipocytes is an important
do you recommend? hormone in regulating obesity. Which of the fol-
lowing mechanisms is true of leptin?
A. Roux-en-Y gastric bypass
B. Sleeve gastrectomy A. Leptin secretion is decreased in overfeeding
C. Biliopancreatic diversion with duodenal switch B. As insulin levels rise, leptin levels decreases.
D. Gastric banding. C. Leptin upregulates food intake by signals to
the hypothalamus
CORRECT ANSWER: C D. Leptin is secreted in direct proportion to the
amount of fat present.
RATIONALE
According to the OMA Obesity algorithm (pg148) CORRECT ANSWER: D
expected loss in percent excess body weight at two
years for biliopancreatic diversion with duodenal RATIONALE
switch is 70-80 percent with greatest amount of Leptin is secreted by adipocytes in direct propor-
weight loss and resolution of metabolic disease. In tion to the amount of fat present. The levels fall
Chapter 15 — Nutrition, obesity and eating disorders 455
in response to fasting, not overeating. As insulin D. She has developed a gastro-gastric fistula and
levels fall, leptin secretion also decreases. Leptin need immediate endoscopy.
forms a negative feedback loop that down regu- E. She has developed dumping syndrome
lates food intakes at the level of the hypothalamus.
Hence obesity is thought to be leptin resistant and CORRECT ANSWER: A
insulin resistant state.
RATIONALE
REFERENCES Internal hernias are common with Roux-en-Y and
Perry B, Wang Y. Appetite regulation and weight biliopancreatic diversion with duodenal switch pro-
control: The role of gut hormones. Nutrition and cedures. They are usually accompanied by intermit-
Diabetes. tent, post prandial pain and emesis but sometimes
Dardeno TA, Chou SH, Moon HS, Chamber- patients present only with pain. It occurs because
land JP, Fiorenza CG, Mantzoros CS. Leptin in of herniation through a defect in the mesentery
human physiology and therapeutics.Front Neuro- created during the surgery with herniation through
endocrinol. 2010;31(3):377. Epub 2010 Jun 17. the transverse mesocolon defect the most com-
mon. Internal hernias have been described in up
to five percent of patients undergoing laparoscopic
Question 50 bariatric surgery. Diagnosis can be challenging both
A 35-year-old female comes to your office for clinically and radiographically since episodes are
evaluation of intermittent abdominal pain. About intermittent. Internal hernias can be difficult to de-
two years ago she had a roux-en-Y gastric bypass tect radiographically because they are intermittent.
for weight loss which has been effective: her BMI Several studies have shown that the “mesenteric
is now 27kg/m2 which is reduced from an initial swirl” sign on CT scan is the best indicator of an in-
BMI of 48kg/m2 and previously uncontrolled ternal hernia following gastric bypass. If suspected,
diabetes has now resolved. She complains that emergent laparoscopy is needed if attack is acute.
recently after eating, she experiences extreme Although gallstones and gallbladder disease
crampy pain and sometimes may vomit digested can be an early or late complication following bar-
contents. She denies loose bowel movements. She iatric surgery, and although her WBC is border-
has had a recent CT scan which is read as “unre- line elevated, her transaminases are normal. This
markable”. She also had an EGD remarkable only patient also does not have right upper quadrant or
for gastritis and evidence reflux. Today however epigastric post prandial or nocturnal pain com-
she has had increased abdominal pain. Vital signs mon with gallbladder or gallstone disease. Pa-
show BP 128/82 mm Hg and HR 103 bpm. Physi- tients with incisional hernia experience pain at the
cal examination produces mid abdominal tender- incision site and often can be palpated depending
ness; however she has no tenderness at the site of on the body habitus. Gastro-gastric fistula is sus-
the surgical scar. pected when there is suboptimal weight loss and
often weight regain and recurrence of metabolic
Labs show WBC 11,600 per mL, AST 43 IU/L, disease. These patients may also have non healing
ALT 46 IU/L, alkaline phosphatase 136 IU/L. gastric ulcers. This is not the case in this patient
What is your concern? who has optimal weight loss with no regain,
resolution of metabolic disease and recent normal
A. She has an internal hernia and need immediate EGD. Dumping syndrome is more common before
surgical consult two years post op with common symptoms of
B. She has an attack of acute cholecystitis facial flushing, light headedness, fatigue, reactive
C. She has developed an incisional hernia which hypoglycemia and post prandial diarrhea. This
may need repair patient had none of these symptoms.
456 Digestive Diseases Self-Education Program®
hypertension controlled on lisinopril comes to you recommended to increase aerobic activity to 300
for advice on weight loss. He has always main- minutes a week (five hours) of moderate intensity
tained a BMI of 24.5kg/m2 until about one year or 150 minutes a week of vigorous intensity exer-
ago when gradually gained weight as he started cises plus resistance training two days a week.
university where he reports to high level of stress
and poor eating habits. BMI is now 30kg/m2, his REFERENCES
body fat is 25 percent and his waist circumfer- U.S. Department of Health and Human Services.
ence is 40 inches. His current exercise program Physical Activity Guidelines for Americans. Office
includes 30 minutes of brisk walking three days a of Disease prevention and Health Promotion,
week and he lifts weights twice a week. Recently 2008. https://health.gov/paguidelines/2008/
he decided to join and gym and has a trainer to summary.aspx
help him lose weight. Slentz CA, Duscha BD, Johnson JL, Ketchum
K, Aiken LB, Samsa GP, Houmard JA, Bales CW,
How would you advise him to develop a good exer- Kraus WE Effects of the amount of exercise on
cise program for weight loss? body weight, body composition, and measures of
central obesity: STRRIDE--a randomized con-
A. Moderate intensity exercise 75min/week plus trolled study. Arch Intern Med. 2004;164(1):31.
two days a week muscle strengthening activi-
ties
B. Moderate intensity exercise 150min/week
plus two days a week muscle strengthening
activities
C. Moderate intensity exercise 300min/week
plus no muscles strengthening activities
D. Vigorous intensity exercise 150min/week
plus two days a week muscle strengthening
activities
CORRECT ANSWER: D
RATIONALE
This patient went from normal weight to class 1
obesity (BMI 30-34.9). By measurements of body
fat and waist circumference he is now classified as
obese (greater than or equal to 25 percent and 40
inches or more respectively).
USDHHS recommends for substantial health
benefits adults do at least 150min (two hours
and 30 minutes) a week of moderate intensity or
75 minutes (one hour and 15 minutes) a week of
vigorous intensity or an equivalent combination of
moderate and vigorous intensity aerobic activity.
Adults should do muscle strengthening activi-
ties of moderate or greater intensity involving all
major muscle groups on two or more days a week.
For extensive benefits such as weight loss, it is
458 Digestive Diseases Self-Education Program®
Answers & critiques
CHAPTER 16
Issues in pediatric
gastroenterology
Nancy McGreal, MD and Mary Kathleen Rogers Boruta, MD
459
460 Digestive Diseases Self-Education Program®
Which of the following conditions would be high- perianal pain. His primary care provider per-
est on your differential diagnosis? formed laboratory studies which ruled out enteric
pathogens, but did show evidence of anemia with
A. Inflammatory bowel disease hemoglobin 9.7g/dL, thrombocytosis with plate-
B. Celiac disease lets 535x109/L, ESR 27 and CRP 5.76. His physi-
C. Giardiasis cal examination demonstrates a palpable mass in
D. E. Coli 0157:H7 the right lower quadrant as well as one draining
E. Irritable bowel syndrome perianal fistula without evidence of abscess. Up-
per endoscopy is endoscopically and histologically
CORRECT ANSWER: C normal. Colonoscopy demonstrates diffuse, scat-
tered inflammation with aphthous ulcers, ery-
RATIONALE thema and contact bleeding throughout the colon.
The abrupt onset of symptoms, exposure to The ileocecal valve appears to be mildly stenosed
potential pathogens in a freshwater lake as well but is able to be intubated and shows moderate
as other afflicted individuals strongly suggests inflammatory changes with erythema, edema and
an infectious etiology rather than chronic condi- aphthous ulceration. MR enterography reveals a
tions such as inflammatory bowel disease, celiac right lower quadrant phlegmon with mild inflam-
disease or irritable bowel syndrome. Given the matory stenosis. Based upon this information,
mild severity and lack of rectal bleeding, E. coli you diagnose stricturing ileocolonic and perianal
0157:H7 is less likely, making giardiasis of greater fistulizing Crohn’s disease. What is your first line
concern. Giardiasis is a protozoal infection spread of medical treatment for this young gentleman?
through the fecal oral route most commonly via
contaminated food or water. The incubation A. Oral and rectal mesalamine
period is typically one to two weeks after ingestion B. Thiopurine
of a contaminated substance. Stool testing can be C. Methotrexate
performed to evaluate for ova and parasites and D. Ciprofloxacin and metronidazole
microscopy. More sensitive testing using nucleic E. Biologic therapy
acid amplification testing (NAAT) for Giardia can
be performed for quicker analysis. The first-line CORRECT ANSWER: E
treatment is metronidazole adjusted to the child’s
weight. Alternate therapies may include tinida- RATIONALE
zole, nitazoxanide, mebendazole, albendazole, and The patient in the above scenario presents with
paromomycin. a phenotype of moderate to severe stricturing
ileocolonic and perianal fistulizing Crohn’s disease
REFERENCES with associated phlegmon which warrants treat-
Dunn et al. StatPearls [Internet]. Treasure Island ment with biologic therapy. Studies have demon-
(FL): StatPearls Publishing; 2018 Jun 26. strated that biologic therapy initiated early in the
Bagarazzi M, Bell L (1998). Chapter 35: Para- course of complicated Crohn’s disease portends
sites. In Clinical Pediatric Gastroenterology (pp247- the best prognosis. Young gentleman and indi-
253). Philadelphia, PA: Churchill Livingstone. viduals of African-American descent have been
documented to have more aggressive courses in
Crohn’s disease. Cochrane meta-analyses have not
Question 2 shown benefit of mesalamine agents in patients
A 17-year-old African-American male presents to with Crohn’s disease. Thiopurines, 6-mercapto-
the office with a two-month history of abdomi- purine and as azathioprine, have been associated
nal pain, bloody diarrhea, anorexia, fatigue and with the development of hepatosplenic T-cell
Chapter 16 — Issues in pediatric gastroenterology 461
lymphoma in young gentleman with IBD and are You decide to admit him to the hospital for further
preferred to be avoided if possible. Methotrexate, evaluation. Which of the following is an appropri-
while useful for the treatment of luminal Crohn’s ate consideration of management of acute severe
disease, is not as efficacious as biologic therapy in colitis?
the setting of fistulizing Crohn’s disease. It also
has risks of hepatotoxicity and teratogenicity in A. Add a probiotic
young individuals of childbearing age which need B. Treat for C. difficile in the setting of negative
to be considered in decision making. Antibiotics, testing because it is a common complicating
such as ciprofloxacin and metronidazole, can be factor in IBD
utilized as adjuvant therapy in the setting of mild C. Consider flexible sigmoidoscopy if there is a
colonic Crohn’s disease but would not be effica- lack of response to IV corticosteroids to rule
cious to mitigate the inflammation of moderate to out complicating cytomegalovirus infection
severe Crohn’s disease as monotherapy. (CMV)
D. Start exclusive enteral nutrition therapy
REFERENCES E. If there is a lack of response to IV corticoste-
Dotson J et al. Inflamm Bowel Dis 2017 roids within five to seven days consider rescue
May;23(5):767-774. therapy with vedolizumab
Ford, AC et al. Am J Gastroenterol. 2011
Apr;106(4):617-29. CORRECT ANSWER: C
De Zoeten EF, Pasternak BA, Mattei P, Kramer
RE, Kader HA. J Pediatr Gastroenterol Nutr. 2013 RATIONALE
Sep;57(3):401-12bpm. The medical management of pediatric and adoles-
cent acute severe colitis is similar to adult man-
agement algorithms. In a multicenter study of
Question 3 children with ulcerative colitis, 15 percent de-
An 18-year-old male diagnosed with mild left- veloped acute severe colitis within three months
sided ulcerative colitis two weeks ago presents of diagnosis. Further population-based studies
to the office with worsening diarrhea, abdominal have demonstrated that nearly 30 percent of
pain, hematochezia, urgency and tenesmus. After children diagnosed with ulcerative colitis may
his biopsy results were available he was started on require hospitalization within the first three years
treatment with dose optimized oral mesalamine. of disease onset. It is always appropriate to rule
When his symptoms worsened one week later, out complicating enteric infections especially
mesalamine enemas were added to his regimen C. difficile in the setting of worsening ulcerative
for synergistic topical and oral rectal therapy. On colitis. In an individual who was recently started
physical examination, he is afebrile but tachy- on mesalamine preparations, either oral or rectal,
cardic with a heart rate of 115 bpm and borderline it is appropriate to consider whether or not they
hypotensive with a blood pressure of 100/85 mm are experiencing a hypersensitivity reaction to the
Hg. Clinically, he appears to be dehydrated with mesalamine agent which worsens their disease.
dry mucous membranes. Abdominal exam is In this setting, discontinuation of the mesalamine
noteworthy for tenderness in the bilateral lower therapy is appropriate. IV corticosteroids in the
quadrants. Laboratory studies are concerning for form of solumedrol or hydrocortisone are the
a white blood cell count 15.4K/mL, hemoglobin mainstay of rescue therapy. Studies indicate that
8.0 g/dL (previously 11.0 two weeks ago), platelets patient should demonstrate an improvement with-
554K/mL, ESR 59 and CRP 2.43. Stool studies in three to five days. In the pediatric population,
are negative for complicating enteric pathogens it is recommended that practitioners monitor the
including bacterial stool culture and C. difficile. Pediatric Ulcerative Colitis Activity Index (PUCAI)
462 Digestive Diseases Self-Education Program®
which measures abdominal pain, rectal bleeding, ents report that she has had symptoms of reflux
stool consistency, number of bowel movements since birth. She was started on a proton pump
per day, nocturnal stooling and impact on activity inhibitor by her primary care doctor which has im-
level. A flexible sigmoidoscopy is recommended if proved her fussiness but not resolved the volume
there is no improvement with IV corticosteroids of emesis. Her appetite remains stable and there
by day three to rule out CMV infection. A PUCAI is no report of growth failure. On physical ex-
score greater than 45 points after the third day amination, there is notation of a small, palpable
of intravenous corticosteroids indicates the need mass in her abdomen. Her parents report that
for consideration of rescue therapy. Second line she is stooling normally and deny any diarrhea or
salvage therapy should not be delayed beyond the constipation. Laboratory studies demonstrate hy-
fifth day of IV corticosteroid treatment. Rescue pochloremic and hypokalemic metabolic alkalosis.
options include infliximab as a recommended She is not anemic, nor are there any abnormalities
therapy for both induction and maintenance of of her liver or kidney function.
remission. Due to hypoalbuminemia, it is sug-
gested that an induction dose of 10 mg/kg may be Which diagnosis which would you be most con-
preferred in the setting of acute severe ulcerative cerned about at this juncture?
colitis. Calcineurin inhibitors including cyclo-
sporine and tacrolimus can also be considered in A. Typical infantile gastroesophageal reflux
individuals who are not previously exposed to bio- B. Pyloric stenosis
logics or immune modulators. The data regard- C. Cyclic vomiting syndrome
ing accelerated infliximab dosing in the setting D. Celiac disease
of acute severe ulcerative colitis remains mixed. E. Inflammatory bowel disease
In the adult literature, the short-term colectomy
rate is decreased, however, at two years colectomy CORRECT ANSWER: B
rates have been no different between individuals
who received accelerated infliximab dosing in the RATIONALE
setting of acute severe ulcerative colitis than those Infantile hypertrophic pyloric stenosis results in
who received standard dosing or 10 mg/kg at 0, obstruction of the pyloric channel. The etiology of
two and four weeks. Vedolizumab is an anti-inte- the condition is unknown but may be associated
grin therapy which is gut specific but slow acting. with failure of nitric oxide synthetase. It is typi-
At the current time, it is not recommended as an cally diagnosed in early infancy most commonly
induction agent in the setting of either children between two to eight weeks of age. While many
or adults hospitalized with acute severe ulcerative infants have symptoms of acid reflux, infants with
colitis. Preliminary studies have suggested that pyloric stenosis have more profuse and projectile
individuals may be able to be induced with cyclo- emesis sometimes with evidence of bile. There
sporine followed by vedolizumab only as a mainte- can be a palpable mass on examination which has
nance agent. been described as an “olive” in the mid abdomen.
Classic laboratory studies indicate hypochloremic
REFERENCE and hypokalemic metabolic acidosis. Ultrasonog-
Turner D et al. JPGN 2018; 67(2): 292-310. raphy is regarded as the diagnostic gold stan-
dard measure which demonstrates evidence of a
hypertrophied pylorus showing a muscle thickness
Question 4 of 4 mm or more in a pyloric channel of 16 mm or
A six-week old female, term infant presents to the more. While typical infantile GERD can present
emergency department with symptoms of recur- with symptoms of recurrent regurgitation, it is
rent, non-bloody, non-bilious vomiting. Her par- not usually associated with laboratory abnormali-
Chapter 16 — Issues in pediatric gastroenterology 463
B. Draw a serum H. pylori IgG utilizing a urea breath test or stool antigen test.
C. Request the child submit a stool specimen for Prior to performing noninvasive studies to confirm
H. pylori stool antigen now eradication children should be off of proton pump
D. Advise the mother to discontinue PPI therapy inhibitors for at least two weeks and antibiotics for
for two weeks and return for a urea breath test at least four weeks.
E. Schedule an upper endoscopy with biopsy with
immunohistochemistry or culture REFERENCE
Jones N et al. JPGN 2017; 64(6): 991-1003
CORRECT ANSWER: E Kotilea K et al. Pediatr Drugs 2018; 20: 337-351.
RATIONALE
H. pylori infection is not uncommon in the pediat- Question 7
ric population. Seroprevalence rates from stud- Which of the following is TRUE regarding Clos-
ies have suggested it that affects 33-37 percent of tridioides difficile (C. difficile) infection in infants
children. This varies depending upon worldwide and children?
location as well as socioeconomic status. Higher
rates of infection are typically associated with low A. The incidence of C. difficile infection in
to low-middle class populations and common in children in the United States has declined over
areas or countries with a severe income inequality. the last few decades
Epidemiologic studies have demonstrated that the B. Antibiotics are not an important risk factor of
prevalence of H. pylori infection in both children initial C. difficile infection in children
and adults appears to be declining and developed C. C. difficile testing in infants less than one year
countries. This may be related to changes in gastric of age is not recommended due to high rates of
microbiota, economic development and/or the non-pathogenic colonization
increase in incidence of immunological disorders D. C. difficile in children usually only occurs once
in children. Therefore, it would not be appropriate and recurrent C. difficile infection is not seen
to dismiss this as a possible diagnosis especially E. Fecal microbiota transplant cannot be utilized
in light of the pertinent family history. A variety in children with recurrent C. difficile
of noninvasive tests are available including stool
antigen testing as urea breath testing. Studies have CORRECT ANSWER: C
demonstrated that H pylori infection is not associ-
ated with symptoms in the absence of peptic ulcer RATIONALE
disease in the pediatric population. For this reason, Clostridioides (formerly Clostridium) difficile (C.
a test and treat strategy utilizing noninvasive difficile) is a spore forming, anaerobic gram-pos-
methods is not recommended. Current pediatric itive bacillus. Affect humans either from contact
guidelines recommend that the diagnosis of H. in the environment or through fecal oral transmis-
pylori infection be based upon histopathology con- sion. The bacteria produces two toxins, toxin A
sistent with H. pylori positive gastritis plus at least and toxin B, which are believed to be a primary
one other type test and/or culture. Therefore, the component and causing infection. Studies have
recommended diagnostic testing would be an upper demonstrated that children can be infected with
endoscopy examination. Serologic testing for H. C. difficile and in fact the incidence has increased
pylori IgG would not necessarily construe current in the United States in the pediatric population.
infection and may represent prior resolved infec- Exposure to antibiotics appears to be a significant
tion and is not recommended as a diagnostic test. risk factor for initial C. difficile infection. Those
It is recommended that eradication of H. pylori be most commonly implicated are third and fourth
assessed four weeks after completion of treatment generation cephalosporins, clindamycin and fluo-
Chapter 16 — Issues in pediatric gastroenterology 465
roquinolones. When considering testing a child past medical history noteworthy for anxiety and
for C. difficile diarrhea it is important to recognize depression. She has been seen by two previous
that testing and infants less than one year of age gastroenterologists with normal laboratory test-
is not recommended due to high rates of coloniza- ing, endoscopic evaluation and cross-sectional
tion. In the current literature, it is estimated that imaging in the form of ultrasound, CT scan and
37 percent of infants were less than one month of MRI of the abdomen and pelvis. Her family feels
age are colonized with C. difficile and are asymp- frustrated and desperate that no one has identi-
tomatic. Young infants do not possess receptors fied an etiology of her symptoms. You perform
for the toxins generating C. difficile diarrhea and additional testing which reveals normal fecal
are therefore asymptomatic. By two years of age, calprotectin, studies for porphyria and have ruled
toddlers possess similar microbiota profiles to out median arcuate ligament syndrome. She has
adults and therefore may be at risk. The clinical missed 20 days of school and her parents request
presentation is similar to adults with diarrhea homebound teaching and narcotic medication.
which may or may not be associated with hema- Which of the following is the most appropriate
tochezia or abdominal tenderness. American management strategy?
Academy of Pediatrics guidelines recommend oral
metronidazole for initial treatment and first recur- A. Obtain an EKG and consider a trial of a
rence of mild to moderate disease. If a child has tricyclic antidepressant in the form of
severe C. difficile infection or a second recurrence amitriptyline or nortriptyline
of C. difficile, oral vancomycin is the preferred B. Prescribe a long acting narcotic to manage pain
agent. For recurrent C. difficile pulse tapered C. Recommend indefinite homebound schooling
vancomycin is an option as well as consideration D. Suggest surgical laparoscopy for an
of fidaxomicin in children over the age of six years. unidentified source of abdominal pain
Fecal microbiota transplant has been studied in E. Repeat upper endoscopy, colonoscopy,
children with recurrent C. difficile who have not CT and MRI to reassure parents
responded to standard antibiotic regimens and
found to be efficacious. CORRECT ANSWER: A
REFERENCES RATIONALE
D’Ostroph A et al. Infection and Drug Resistance Chronic recurrent abdominal pain that is functional
2017: 365-375. in nature is an exceedingly common condition in
Crews J and Kaplan S (2018). Clostridioi- pediatrics. It is a diagnosis of exclusion and due
des (formerly Clostridium) difficile infection in diligence should occur with additional testing espe-
children: Clinical features and diagnosis. In M cially in the presence of alarm symptoms of overt
Torchia (Ed.) UptoDate. Retrieved October 12, gastrointestinal symptoms, growth failure or other
2018 from https://www.uptodate.com/contents/ extraintestinal manifestations. When children
clostridioides-formerly-clostridium-difficile-infec- present with symptoms of recurrent abdominal
tion-in-children-treatment-and-outcome pain, it is appropriate to evaluate for conditions
such as inflammatory bowel disease, celiac disease,
H. pylori infection, chronic pancreatitis, forms of
Question 8 carbohydrate malabsorption, small bowel bacte-
A 14-year-old girl presents with symptoms of rial overgrowth syndrome and unusual and rare
chronic abdominal pain for four years. It is peri- etiologies as described in the scenario above if
umbilical in nature and not associated with fevers, appropriate. When other medical etiologies have
chills, nausea, vomiting, diarrhea or abdominal been ruled out, it is appropriate to review with
pain. Her growth is stable. She has a significant families the management of functional, chronic
466 Digestive Diseases Self-Education Program®
recurrent abdominal pain which may include medi- ogies may include nonsteroidal anti-inflammatory
cal therapeutics in the form of antispasmodics, use, corticosteroid use as well as individuals being
tricyclic antidepressants, gabapentin, pregabalin or critically ill in the pediatric intensive care unit are
topiramate. Some patients may require multidisci- all risk factors for the development of gastritis.
plinary therapy including psychological counseling, Psychological stress has not been associated with
biofeedback or other multimodality therapies. In the development of physiologic changes of gastri-
general, the psychosocial isolation resulting from tis and there have been no documented studies
remaining out of school is not in the best interest to indicate in association with serotonin reuptake
of most children with these conditions. Narcotic inhibitors. While children with excess psychosocial
medications are also not recommended due to stress may experience abdominal pain in the form
issues related to dependence and the possibility of of visceral hypersensitivity it does not necessarily
the development of narcotic bowel syndrome. correlate with histological changes of gastritis. An-
tibiotic use in children has not been associated with
REFERENCES the development of gastritis.
Rajindrajith S, Zeevenhooven J, Devanarayana
NM, Perera BJC, Benninga MA. REFERENCE
Expert Reviews in Gastroenterolology and Sierra D et al. Pediatrics in Review 2018; 39(11):
Hepatology. 2018 Apr;12(4):369-390. 542 – 547.
Question 9 Question 10
A common cause of gastritis in the pediatric A five-year-old boy is admitted to the hospital with
population is: a three-day history of abdominal pain, arthralgias
and a purpuric rash which developed over the but-
A. Helicobacter pylori infection tocks and lower extremities. His parents report
B. Anxiety that prior to the onset of his symptoms he had
C. ADHD medications what appeared to be a viral illness. Laboratory
D. Serotonin reuptake inhibitors (SSRIs) studies disclose mild leukocytosis with a normal
E. Antibiotics hemoglobin and platelet count. ESR is signifi-
cantly elevated to 54. Urinalysis shows evidence
CORRECT ANSWER: A of both proteinuria and hematuria. ANA, rheu-
matoid factor and complement levels are normal.
RATIONALE He is diagnosed with Henoch-Schönlein Purpura
Gastritis, inflammation of the gastric intestinal mu- (HSP). Despite nonsteroidal anti-inflammatory
cosa can occur in children. The incidence of peptic medications (NSAIDS), his abdominal pain is
ulcer disease in the pediatric population is estimat- not improving and nephrology would like to limit
ed to range from two to eight percent. The most exposure to NSAIDs due to concern for renal in-
common etiologies of gastritis in children are due volvement. What is the next appropriate step for
to H. pylori infection, corticosteroid use and immu- managing his abdominal pain?
nosuppressive medications. Studies have demon-
strated that there can be a predilection towards the A. Narcotic pain medication
development of gastritis based upon Race ethnicity. B. Corticosteroids
Non-Hispanic black and Mexican American popu- C. Infliximab
lations have been shown to have a higher rate of D. Cyclophosphamide
gastritis. Among the answers presented, H. pylori E. He does not require further medical therapy as
infection is the most common etiology. Other etiol- HSP usually resolves on its own
Chapter 16 — Issues in pediatric gastroenterology 467
with a male to female ratio of 2:1. The most for how she should best be followed. Which of
common presentation is in children two years of the following do you recommend?
age and location in the distal 100 cm of the ter-
minal ileum. Approximately 50 percent of diver- A. Because she is a known APC gene mutation
ticulae contain heterotopic gastric tissue which carrier, you begin annual surveillance
results in acid secretion and subsequent bleed- colonoscopies and upper endoscopy now
ing. Meckel’s diverticulum can be diagnosed B. Perform small bowel follow-through to
with 85-90 percent sensitivity with a radionucle- evaluate for the possibility of small bowel
otide 99 m TC pertechnetate scan demonstrating polyps
evidence of ectopic gastric mucosa in the right C. Recommend the child undergo abdominal
lower quadrant. Treatment require surgical re- ultrasound and AFP testing every six months
section. While inflammatory bowel disease can until the age of five due to risk of
contribute to lower GI bleeding, the abrupt onset hepatoblastoma in the setting of APC
and acute nature makes this less likely. H pylori mutations
and intussusception are typically associated with D. Schedule a thyroid ultrasound because there
abdominal pain. Hemorrhoids are an uncom- is an associated risk of thyroid cancer in the
mon occurrence in young individuals and are setting of FAP
generally not the cause of painless rectal bleed- E. Hemoccult her stools annually to decide
ing until at least adolescence. when you will begin endoscopic screening
and surveillance.
REFERENCES
Turk D, Michaud L (1998). Chapter 346.3b. CORRECT ANSWER: C
Lower Gastrointestinal Bleeding. In Walker’s
Pediatric Gastrointestinal Disease (pp 1314- RATIONALE
1315). Hamilton, Ontario: BC Decker. FAP is the most common inherited polyposis
Lirio R et al. Gastrointestinal Endoscopy syndrome in the pediatric population. The ge-
Clinics of North America 2016; 26: 63-73. netic underpinnings of FAP are due to a trun-
cated mutation in the adenomatous polyposis
coli (APC) gene. It is inherited in an autosomal
Question 12 dominant fashion, however, de novo mutations
A two-year-old otherwise healthy girl presents may occur in 20-30 percent of cases. In classic
to your office with her parents. Her father was FAP, innumerable colon polyps carpet the colon.
recently diagnosed with familial adenomatous Individuals with attenuated FAP may have fewer
polyposis syndrome (FAP) after he presented colon polyps. In families with history of FAP
with rectal bleeding and underwent ileocolonos- and known genetic mutation, related family
copy. He reports that his colonoscopy showed members can be referred for mutation testing.
innumerable polyps throughout his colon but no If no genetic mutation is known, it is recom-
evidence of dysplasia or colon cancer. Her father mended that children begin endoscopic surveil-
subsequently underwent genetic testing which lance between the ages of 10-12 years. Biannual
revealed a positive APC gene mutation variant endoscopic screening is recommended after the
associated with FAP. He has been referred for age of 10-12 years with intensification of screen-
colectomy. Their daughter was seen by a geneti- ing if the number of polyps significantly increas-
cist and underwent genetic testing which showed es or demonstrates worrisome histopathology.
that she possesses a similar APC gene mutation While gastric and duodenal lesions can occur in
as her father. Her parents are exceedingly wor- children with FAP, current guidelines recom-
ried about her prognosis and seek your advice mend beginning upper endoscopic surveillance
Chapter 16 — Issues in pediatric gastroenterology 469
terology (pp 81-87). Philadelphia, PA: Churchill distal tracheoesophageal fistula is the most com-
Livingstone. mon comprising 8.5 percent of cases. Most chil-
McCray S et al. Practical Gastroenterology dren with such malformations undergo surgical
2016 pp 56-66. repair in early infancy. Depending upon the type
and length of atresia, as well as fistula location,
there can be varied surgical outcomes. Long-term
Question 14 studies demonstrate that GERD is experienced by
A 20-year-old male with a history of congenital many patients. One study revealed ongoing GERD
type C tracheoesophageal fistula diagnosed at in 35-50 percent of children with associated
birth status post repair presents to your office to repairs. Heartburn remains present in up to 46
transition his care from his pediatric gastroen- percent of adults. There does appear to be an in-
terology provider to your office. Which of the fol- creased risk of Barrett’s esophagus, and therefore
lowing is a common long-term complication after long-term antacid therapy may be indicated both
repair of a tracheoesophageal fistulae that both for symptom relief and suppression of Barrett’s
adult and pediatric gastroenterologists should be esophagus if it is identified endoscopically. Up to
aware of? 75 percent of patients with congenital esophageal
atresia and tracheoesophageal fistula experience
A. GERD and future risk of Barrett’s esophagus esophageal dysmotility as documented by manom-
B. Eosinophilic esophagitis etry. Many children have difficulty with feeding
C. Schatzki’s ring disorders and need to learn how to eat slowly and
D. Zenker’s diverticulum swallow appropriately. Esophageal strictures
E. Esophageal inlet patch may occur postoperatively in six to 40 percent of
patients. Many patients require serial dilations
CORRECT ANSWER: A postoperatively. Respiratory complications are
also common after such surgical repairs. These
RATIONALE include tracheomalacia, recurrent pulmonary in-
Congenital esophageal atresia (EA) and/or tra- fections in childhood, bronchial reactivity as well
cheoesophageal fistula (TEF) are common con- as persistent pulmonary function abnormalities
genital abnormalities of the aerodigestive tract. particularly in some individuals where there can
The incidence of congenital esophageal atresia be a long-standing history of aspiration. There is
is estimated to be one in 3000-4500 live births. no definitive increased risk of eosinophilic esopha-
There are five different types of congenital esoph- gitis, Schatzki’s ring, Zenker’s diverticulum or
ageal atresia with associated tracheoesophageal esophageal inlet patch with EA or TEF.
fistula. See diagram below. As illustrated in this
vignette, type C proximal esophageal atresia with REFERENCES
Kovesi T et al. Chest 2004; 126(3): 915-925
Burjonrappa SC et al. E J Pediatr Surg 2011; 21(1):
25-29 Long AM et al. J Pediatr Surg 2017; 52(2):
226-220.
Burge K et al. Brit J Surg 2013; 100: 515-52.
Question 15
A 14-year-old girl presents to the office with symp-
toms of gas, bloating, abdominal discomfort and
intermittent diarrhea. She has no other antecedent
Chapter 16 — Issues in pediatric gastroenterology 471
medical problems and her growth is adequate. She bowel intestinal brush border over time. This
denies unexplained fevers, chills, nausea or vomit- is rare below the age of five years and more
ing. During dietary history review, you identify a common in teenagers and young adults. It
link between ingestion of dairy and her symptom- is also more common in individuals of non-
atology, prompting the possible diagnosis of lactose European descent.
intolerance. 4. Secondary lactose intolerance -this may be a
sequelae of small bowel injury due to etiology
Which of the following testing would you perform such as giardiasis, Crohn’s disease or celiac
to confirm her diagnosis? disease. This typically resolves with restitu-
tion of the small bowel epithelial border.
A. Milk skin prick testing
B. Lactose breath hydrogen testing The most common symptoms are as described
C. Stool eosinophil count above with a constellation of abdominal pain,
D. Increase volume of lactose in diet and evaluate diarrhea and bloating. At times there can be as-
if symptoms worsen. sociated nausea and vomiting with the ingestion of
E. Milk IgE RAST testing dairy-containing products.
A variety of diagnostic modalities are avail-
CORRECT ANSWER: B able including measurement of stool reducing
substances and stool pH. Stool reducing sub-
RATIONALE stances will be increased in the setting of lactase
Lactase non-persistence (ie; lactose intolerance) deficiency in the stool pH reduced below a level
is a common gastrointestinal condition affecting of 5.5 to six. Lactose breath testing is also avail-
up to 70 percent of the world’s population due to able. Breath samples are measured every 15-30
decline in lactase expression over time. Lactose minutes for up to three hours. The diagnosis of
is the main carbohydrate in human and mamma- lactose intolerance is substantiated by an increase
lian milk. It is the primary source of nutrition for in exhaled hydrogen greater than 20 parts/mm
mammalian infants. Lactose is metabolized by the from baseline. False negatives can occur in the
small bowel enzyme called lactase which resides setting of recent antibiotic use. The gold standard
on the small bowel brush border within entero- for diagnosing lactase deficiency is measurement
cytes. Lactase deficiency can be divided into four of duodenal disaccharidases during upper en-
main classifications: doscopy examination. Due to the nature of this
invasive testing, it is not necessarily performed as
1. Developmental lactase deficiency - Due to a first-line test unless there are other symptoms
maturational delay, premature infants born that need to be evaluated. It is very common in
before 34 weeks of life may have a deficiency clinical practice to recommend a lactose-free diet
of lactase to digest lactose and may have a and assess response; increasing lactose in the diet
transient form of lactose intolerance until they would not be appropriate. If symptoms resolve
reached an appropriate chronological age. this is both diagnostic and therapeutic at the
2. Congenital lactase deficiency (alactasia) - This same time. Cow’s milk allergy is different from
is an incredibly rare autosomal recessive dis- lactose intolerant pathophysiologically. Cow’s
order in which infants are unable to tolerate milk allergy is mediated by the IgE portion of the
lactose presenting with loose watery diarrhea, immune system and is a true allergy rather than
failure to thrive and electrolyte imbalances. an intolerance. Individuals diagnosed with true
3. Lactase non-persistence - The most com- cow’s milk allergy in infancy and childhood need
mon of lactose intolerance is associated with to avoid proteins and calcium milk including
decrement of lactase enzymes across the small albumin, lactate albumin and over albumin. Some
472 Digestive Diseases Self-Education Program®
children may outgrow this in childhood but others cal remission with normal laboratory, stool and
may have persisting issues related to a true cow’s radiographic studies. After small bowel surgi-
milk allergy. As such they should be followed cal resection, small bowel bacterial overgrowth
by an allergist in the long-term. Milk skin prick syndrome is a common complication in the setting
testing, stool eosinophil count and milk IgE RAST of Crohn’s disease and other chronic gastrointes-
testing are not indicated as there are tests for milk tinal conditions. It is always prudent to consider
allergy or cow milk protein soy intolerance. the possibility of recrudescence of Crohn’s disease
contributing to active symptoms after surgical
REFERENCE resection. When inflammatory disease develops
Heine et al. World Allergy Organization Journal after surgical resection it is most likely to occur at
2017; 10: 41-49. the site of surgery and anastomosis. In this set-
ting it is appropriate to rule out infectious etiolo-
gies contributing to flare symptoms and consider
Question 16 colonoscopy to reevaluate the colonic mucosa as
A 17-year-old young woman with a 10-year his- well as the surgical anastomosis. It is prudent to
tory of stricturing ileal Crohn’s disease status post inquire about NSAID use in all IBD patients due
ileocecal resection five years ago presents to your to the risk of flare. In this patient, enteric infection
office for evaluation and management of bloating, as already ruled out making C. difficile unlikely.
abdominal discomfort and diarrhea. She denies Lactose intolerance is less likely given her report
unexplained fevers, chills, nausea, vomiting, joint of tolerance to dairy.
pain, skin rashes or oral ulcers. Her growth is
adequate and she has normal menses. Stool studies REFERENCE
are negative for enteric pathogens. Since her sur- Quigley E et al. Infect Dis Clin N Am 2010; 24:
gery she has been maintained on adalimumab 40 943-959.
mg subcutaneously every two weeks with labora-
tory testing indicating a normal CBC, CMP, ESR,
CRP, fecal calprotectin, and therapeutic drug levels Question 17
without antibody formation. MR enterography is A two-week-old ex-23-week premature infant
normal. Re-staging upper endoscopy and colonos- being cared for her in the neonatal intensive care
copy are endoscopically and histologically normal. unit receiving infant formula develops symptoms
She denies NSAID use and sensitivity to dairy. of feeding intolerance, abdominal distention, loose
stool and hematochezia. The infant has manifest-
What is your most likely diagnosis? ed some regurgitation but no projectile vomiting.
There is no report of bilious emesis. Plain film
A. Small bowel bacterial overgrowth syndrome of the abdomen demonstrates evidence of ileus
B. NSAID enteritis with pneumatosis. The infant is overall clinically
C. C diff infection stable. Laboratory studies demonstrate anemia
D. Lactose intolerance and thrombocytosis.
E. Active Crohn’s disease
What is the most likely diagnosis?
CORRECT ANSWER: A
A. Pyloric stenosis
RATIONALE B. Inflammatory bowel disease
The correct answer is small bowel bacterial C. Juvenile polyp
overgrowth syndrome. Active Crohn’s disease D. Necrotizing enterocolitis
is unlikely diagnosis given evidence of surgi- E. C. difficile infection
Chapter 16 — Issues in pediatric gastroenterology 473
In the setting of acetaminophen toxicity is criti- brae. Abdominal ultrasound does not indicate
cal to measure serial electrolytes, liver enzymes hepatosplenomegaly. Based upon these findings
and acetaminophen levels at least every 12 hours. what is the most likely diagnosis of cholestatic
Mental status and physical examination should liver disease in an infant?
be assessed for evolution of acute liver failure. Of
the treatment options offered, if there has been A. Biliary atresia
documented ingestion of greater than 150 mg/ B. Alagille’s syndrome
kg within less than four hours of initial presenta- C. Rotor syndrome
tion, treatment with activated charcoal can be D. Dubin-Johnson syndrome
considered unless there are symptoms of gastro- E. Congenital CMV infection
intestinal obstruction, altered mental status or
airway issues. Individuals who present four to 24 CORRECT ANSWER: B
hours after ingestion should have an acetamino-
phen level checked and be treated with empiric RATIONALE
N-acetylcysteine. In patients who present more The vignette describes a child with Alagille
than 24 hours post ingestion or are at risk of liver syndrome characterized by triangular facies,
injury and failure, consideration should be given cholestatic jaundice, heart murmur and butterfly
to chronic acetaminophen poisoning. Treatment vertebrae. In the differential diagnosis it is critical
protocols typically utilize N-acetylcysteine for 21- to rule out other causes of cholestatic liver dis-
72 hours. Liver transplantation is rarely required ease including TORCH syndromes, biliary atresia,
in children or adolescents with acetaminophen alpha-1 antitrypsin deficiency, drug induced liver
poisoning compared to adults. Children should disease and Alagille disease. The presence of con-
have close monitoring of liver enzymes, coagula- genital malformations raises concern for a genetic
tion factors and mental status. anomaly. In the setting of abnormal facies, chole-
static jaundice and heart murmur, biliary atresia
REFERENCE as well as Alagille’s syndrome are considerations.
Heard K, Burns M. Management of acetaminophen Vertebral abnormalities are more closely associat-
poisoning in children and adolescents. In J.F. Wiley ed with Alagille’s syndrome making this the most
(Ed). UptoDate. Retrieved November 14, 2018 from likely diagnosis.
https://www.uptodate.com/contents/manage- Alagille syndrome is an autosomal dominant
ment-of-acetaminophen-paracetamol-poisoning- disorder caused by heterozygous mutations in
in-children-and-adolescents JAG1 and NOTCH2 genes in the Notch signaling
pathway. Histologically it results in bile duct pau-
city. The hallmark features of Alagille’s syndrome
Question 19 include characteristic triangle facies, congenital
A two-month-old term infant presents with cardiac disease, vascular anomalies, renal anoma-
symptoms of difficulty gaining weight. Physical lies and skeletal malformations. Ninety-six percent
examination demonstrates an alert infant with of individuals diagnosed with Alagille’s syndrome
mildly dysmorphic triangular facies and scleral will have an identified mutation in the JAG1 or
icterus. A 2/6 systolic ejection murmur is auscul- NOTCH2 gene. They will manifest with cholestatic
tated. No evidence of hepatosplenomegaly. The liver disease and often symptoms of pruritus. Dur-
child appears mildly jaundiced. Laboratory studies ing their evaluation it is critical that in addition to
demonstrate ALT 80 U/L, AST 70 U/L, alkaline laboratory studies, they undergo cardiology evalu-
phosphatase 200 U/L, total bilirubin 2.6 mg/dL, ation, renal ultrasound, radiographic assessment
direct bilirubin 1.7 mg/dL, GGT 258 U/L. Chest of the spine, nutritional assessment and genetic
x-ray shows evidence of possible butterfly verte- testing as detailed above. Twenty to 30 percent of
Chapter 16 — Issues in pediatric gastroenterology 475
patients with Alagille’s syndrome will ultimately the intrahepatic bile ducts. The etiology remains
require liver transplantation. unknown but it is suspected that it may be related
While biliary atresia is in the differential to a form of perinatal infection. Affected infants
diagnosis the clinical features described are more typically present with jaundice in the first months
characteristic of Alagille’s syndrome. Rotor syn- of life and direct hyperbilirubinemia. Charac-
drome and Dubin-Johnson syndrome are condi- teristic hallmarks on liver biopsy include portal
tions of indirect hyperbilirubinemia rather than tract edema, proliferation of bile ducts, bile duct
direct hyperbilirubinemia. Congenital CMV infec- plugs and fibrosis. Surgical management consists
tion, as part of the TORCH syndromes, should of hepatoportoenterostomy known as the Kasai
be considered but is less likely in the absence of procedure. This involves dissection of the biliary
microcephaly and petechiae. remnant proximal to the level of the porta hepa-
tis anastomosed to a Roux-en-Y loop of jejunum
REFERENCE to reinstate bile duct flow. The most common
Mitchell E at al. Clin Liver Dis 22(2018); complication after hepatoportoenterostomy is as-
625-641. cending cholangitis. This most commonly occurs
within two years of surgery and may affect more
than half of children who have undergone Kasai
Question 20 procedure. It is characterized by fever and eleva-
A two-year-old infant with a history of biliary tion of hepatic transaminases as well as alkaline
atresia status post Kasai procedure presents to phosphatase. Young children are prone to many
the emergency department with the abrupt onset infectious illnesses, however, in febrile children
of fussiness, irritability and tactile temperature. who have undergone a Kasai procedure, ascend-
Evaluation reveals temperature 102.7°F, tachycar- ing cholangitis should always be a critical concern.
dia and borderline hypotension. Laboratory stud- Most commonly this is caused by gram-negative
ies indicate white blood cell count 15.4 x 109/L , organisms such as Pseudomonas. It is appropri-
ALT 245 U/L, AST 238 U/L, alkaline phosphatase ate to provide broad-spectrum antibiotic coverage
490 U/L, total bilirubin 4.6 mg/dL, direct biliru- while blood cultures are pending.
bin 4 mg/dL, and indirect bilirubin 0.6 mg/dL. While portal hypertension and liver failure are
Liver ultrasound demonstrates evidence of prior possible sequelae of biliary atresia, the acute presen-
surgical hepatoportoenterostomy but no other tation with fever more likely supports a diagnosis of
acute changes. Blood cultures are pending. ascending cholangitis. The presence of choledocho-
Which of the following is the most likely lithiasis is uncommon in children including those
diagnosis? with biliary atresia making this unlikely. Pancreati-
tis is not necessarily associated with biliary atresia
A. Cholangitis and also less common in children.
B. Portal hypertension
C. Liver failure REFERENCE
D. Choledocholithiasis Murray K and Jonas M (1998). Chapter 48: Biliary
E. Pancreatitis Atresia. In Clinical Pediatric Gastroenterology (pp
331-347). Philadelphia, PA: Churchill Livingstone.
CORRECT ANSWER: A
RATIONALE Question 21
Extrahepatic biliary atresia is a disorder that A 17-year-old girl presented to her primary care
results in progressive destruction of the extrahe- physician with symptoms of lethargy, malaise and
patic biliary system with variable involvement of fatigue. She has no significant past medical histo-
476 Digestive Diseases Self-Education Program®
ry and takes no medications. On physical exami- between boys and girls, however, there is a higher
nation she was not jaundiced and her sclerae were incidence of females in adolescence. It may pres-
anicteric, however, hepatomegaly was present ent subacutely with vague symptoms of malaise,
with a liver edge palpable 3 cm below the costal lethargy and sometimes joint pain and rash.
margin. Screening laboratory tests performed by Children with subacute disease may not have
her primary care provider disclosed AST 285 U/L, clinical stigmata of liver disease with the exception
ALT 363 U/L, alkaline phosphatase 82 U/L, Total of hepatosplenomegaly. Approximately one half
bilirubin 1.0 mg/dL, direct bilirubin 0.4 mg/dL, of children with autoimmune hepatitis present
albumin 3.2 g/dL. Laboratory testing for hepatitis with fulminant acute disease which can progress
A, B, C, Epstein-Barr virus and Cytomegalovirus to liver failure. Autoimmune hepatitis is divided
are negative. Abdominal ultrasound shows mild into type I and type II depending upon circulating
hepatomegaly with normal liver echogenicity. She autoantibodies. Type I autoimmune hepatitis is
is referred to your office for further hepatology more common and associated with positive anti-
evaluation. Your laboratory testing shows: ceru- smooth muscle antibodies as well as antinuclear
loplasmin 38 mg/dL, total serum IgA 80 mg/dL, antibodies. Type II autoimmune hepatitis is less
tissue transglutaminase IgA 8, alpha 1 antitrypsin common and associated with the presence of anti-
level 118 mg/dl, ANA positive 1:640, anti-smooth LKM antibodies.
muscle antibody positive and total serum IgG Histologically, the classic findings of autoim-
2184 mg/dL. You determine the need to perform mune hepatitis include a portal inflammatory in-
a liver biopsy. Based upon the serologic findings filtrate consisting of lymphocytes and plasma cells
in this workup what will liver histopathology most extending beyond the limiting plate and around
likely demonstrate? adjacent periportal hepatocytes. This is referred
to as piecemeal necrosis or interface hepatitis. In
A. PAS-positive, diastase resistant granules in the severe cases necrosis may be present. PAS posi-
hepatocytes tive, diastase resistant granules in the hepatocytes
B. Concentric fibrosis around the interlobular would be indicative of alpha-1 antitrypsin deficien-
bile ducts cy which has been ruled out based upon serologic
C. Microvesicular and macrovesicular steatosis level and genotype testing. Concentric periportal
with hepatic copper content greater than fibrosis otherwise known as onion skinning would
250 µg dry weight be a classic finding in primary sclerosing cholan-
D. Portal inflammatory infiltrate consisting of gitis. While there can be overlap syndromes of
lymphocytes and plasma cells extending across autoimmune hepatitis and sclerosing cholangitis,
the limiting place into the periportal hepatocytes the adolescent in the vignette above does not have
E. Bile duct proliferation, portal tract fibrosis and features of cholestasis. A finding of bile duct pro-
bile duct plugs. liferation, portal tract fibrosis and bile duct plugs
would be suggestive of a cholestatic disorder and
CORRECT ANSWER: D is a classic finding of biliary atresia. The patient’s
laboratory testing is not indicative of cholestasis
RATIONALE and it would be unusual for such a diagnosis to be
The adolescent and the above scenario is most made in adolescence.
likely presenting with autoimmune hepatitis
based upon her serologic evaluation disclosing a REFERENCES
positive ANA, anti-smooth muscle antibody and Roberts E. Liver International 2011 31(10):
elevated total serum IgG. Autoimmune hepatitis 1424-1431.
can present in childhood at any age. In younger Le Bail B. Clinics and Research in Hepatology
children the incidence appears to be equivalent and Gastroenterology 2012; 36(3): 248-252.
Chapter 16 — Issues in pediatric gastroenterology 477
Her iron levels are normal. Which nutrient defi- that the patient had passage of meconium within
ciency would you be most concerned about? the first 24 hours of life. Around 2.5 years of age,
he successfully toilet trained. However, after
A. Vitamin B12 insufficiency starting kindergarten around six months ago, the
B. Niacin deficiency patient developed decreased bowel movement
C. Ascorbic acid deficiency frequency and developed fecal incontinence. He
D. Riboflavin deficiency is currently having one hard, large volume bowel
E. Vitamin A deficiency movement every week with daily fecal inconti-
nence. He does report discomfort with bowel
CORRECT ANSWER: A movements. His examination demonstrated a
large rectal stool burden.
RATIONALE
Vitamin B12 insufficiency is a common complica- What would be the recommended next step in
tion of ileal resection after Crohn’s disease due management?
to loss of absorptive surface area in the ileum.
There appears to be a higher risk with ileal resec- A. Initiation of an osmotic laxative
tions greater than 20 cm. Individuals with mild B. Initiation of a stimulant laxative
or relatively recent vitamin B12 deficiency may C. Bowel cleanout
be asymptomatic or simply report mild fatigue. D. Increased dietary fiber
In more chronic cases, vitamin B12 deficiency E. Behavioral therapy
can affect demyelination of nerves and results
in paresthesias, depression and poor memory. CORRECT ANSWER: C
Niacin deficiency, also known as pellagra, is a rare
nutritional deficiency. Clinical symptoms consist RATIONALE
of dermatitis, diarrhea and dementia. Ascorbic Functional constipation is found in approximately
acid deficiency (Vitamin C) results in scurvy. Indi- 14 percent of children. The Rome IV criteria
viduals present with bony abnormalities, petechial for functional constipation in children is two or
hemorrhage of the skin, myalgias gingivitis and more of the following occurring at least one per
other bone abnormalities. Riboflavin deficiency week for a minimal of one month: 1. Two or fewer
presents with photophobia, glossitis, angular defecations in the toilet per week for a child at a
stomatitis and growth abnormalities. Vitamin developmental age of four-years-old, 2. At least
A deficiency would also be less likely given her one episode of fecal incontinence per week, 3.
overall well appearance and stably managed History of retentive posturing, 4. History of hard
Crohn’s disease. Clinical symptoms of vitamin A or painful bowel movements, 5. Large fecal mass
deficiency would include xeropthalmia as well as in rectum, 6. History of large diameter bowel
night blindness. movements. This patient meets criteria for func-
tional constipation. He does not have alarming
REFERENCE features to suggest another etiology leading to his
Tershakovec (1998). Chapter 13. Vitamin Defi- constipation symptoms. Functional constipation
ciency: In Clinical Pediatric Gastroenterology (pp commonly starts in children at the time of toilet
95-102). Philadelphia, PA: Churchill Livingstone. training or starting school with either pain with
defecation, fear of toileting or social pressures
leading to retention. Treatment for this typically
Question 26 consists of disimpaction with either oral agents or
A six-year-old male is seen in clinic due to enemas followed by establishing a maintenance
chronic fecal incontinence. His father reports regimen. Osmotic laxatives like polyethylene
Chapter 16 — Issues in pediatric gastroenterology 481
glycol 3350 are typically used prior to stimulant along the intestine during early gestation, result-
laxatives. ing in a distal portion of the intestine being agan-
glionic. Rectal and distal sigmoid involvement is
REFERENCES seen in around 85 percent of cases, with the other
Borowitz, et al. Treatment of childhood consti- 15 percent involving more proximal intestine. It
pation by primary care physicians: efficacy and can rarely involve the entire colon and small intes-
predictors of outcomes. Pediatrics. 2005; 115: tine. Ganglion cells inhibit local smooth muscles,
873-877. resulting in the characteristic inability for agangli-
Hyams, J, et al. Childhood functional gastroin- onic bowel to relax. This lack of inhibition gives
testinal disorders: Child/adolescent. Gastroenter- rise to the absence of rectoanal inhibitory reflex
ology. 2016; 150: 1456-1468. (RAIRs) during anorectal manometry. The lack
Mugie, S. et al. Epidemiology of constipation of inhibition also produces a transition zone on
in children and adults: a systematic review. Best contrast enema, with the distal aganglionic bowel
Pract Res Clin Gastroenterol. 2011; 25: 3-18. being narrow and the more proximal bowel con-
taining ganglia being dilated.
Lack of meconium passage in the first 48 hours
Question 27 of life raises concern for Hirschsprung’s disease.
A two-month-old male presents with abdominal Other causes for possible failure to pass meco-
distention and poor appetite. His family notes nium include cystic fibrosis, anorectal malforma-
that the patient has chronic difficulties with con- tion, small left colon syndrome, meconium plug
stipation, reporting that they have to use a glyc- syndrome and megacystis-microcolon-intestinal
erin suppository to help him have a bowel move- hypoperistalsis syndrome.
ment every two to three days. The family reports
that he even needed a suppository in the newborn REFERENCE
nursey at day of life three due to lack of passage of Kenny, S et al. Hirschsprung’s disease. Semin
meconium. Pediatr Surg. 2010; 19 (3): 194-200.
Wyllie R. et al. Pediatric Gastrointestinal and
What finding would you expect to during evalua- Liver Disease. 4th edition. Elsevier Saunders,
tion to explain the patient’s symptoms? Philadelphia, 2011.
D. Esophageal manometry the family has recently tried adding rice cereal
E. Upper intestinal endoscopy to the milk. What is the most likely cause for the
patient’s presentation?
CORRECT ANSWER: E
A. Cow’s milk protein allergy
RATIONALE B. Rotavirus infection
Eosinophilic esophagitis (EoE) is an allergy C. Food protein induced enterocolitis syndrome
mediated disease with eosinophilic infiltration of (FPIES)
the esophageal mucosa with resulting symptoms D. Metabolic disease
of esophageal dysfunction. EoE has a prevalence E. Intestinal malrotation
of 29.5 cases per 100,000 in children and 43.4
cases per 1000,000 in adults, with the disease CORRECT ANSWER: C
being common in patients with another under-
lying atopic condition (asthma, food allergies, RATIONALE
eczema, allergic rhinitis). Younger children Food protein inducted enterocolitis syndrome
commonly present with vomiting, oral aversion (FPIES) is a non-IgE cell mediated food allergy
or abdominal discomfort. Older children com- that presents at a young age commonly with
monly present with dysphagia, food impaction vomiting, diarrhea and dehydration after exposure
and abdominal discomfort. EoE is diagnosed by to the food allergen. In severe cases, exposure to
upper intestinal endoscopy, with greater than 15 the allergen can lead to shock. The prevalence of
eosinophils per 40x field found in the esophagus FPIES is relatively low, with one series demon-
while on proton pump inhibitors (PPIs) being di- strating three cases per 1000 births. The most
agnostic for the condition. This patient is noted common allergens for FPIES include cow’s milk,
to have a history of food allergy with oral aver- soy, rice, oats and fish. Symptoms for cow’s milk
sion and possible episodes of food impactions, or soy based FPIES typically present less than
raising concern for EoE. six months of age with infant formulas, with
solid food based symptoms starting five to seven
REFERENCE months of age. The diagnosis of the condition
Dellon, E. and Hirano, I. Epidemiology and natu- is typically made by withdrawing the suspected
ral history of eosinophilic esophagitis. Gastroen- allergen and seeing if symptoms resolve. Allergy
terology. 2018; 154(2): 319-332. testing is of limited diagnostic value. Treatment
consists of managing hemodynamic instability
following exposure, with maintenance focusing
Question 29 on strict avoidance of triggers. FPIES is typically
A five-month-old unvaccinated female presents to a self-limited process, with most children hav-
the emergency department due to vomiting and ing resolution of reactions between one to five
diarrhea. On evaluation, the patient has promi- years after symptom onset. For this patient, she
nent dehydration with hypotension requiring IV has characteristic episodes of vomiting, diarrhea
rehydration. The family reports that this is their and hypotension following feedings of expressed
second visit to the emergency room in the last breastmilk mixed with rice. With this, rice should
two weeks for similar symptoms. With the last be eliminated from the patient’s diet.
episode, the patient had resolution of symptoms
within 24 hours of presentation. They deny any REFERENCE
significant vomiting or diarrhea at baseline. The Nowak-Wegrzyn, A. et al. International consensus
patient normally fed with expressed breastmilk. guidelines for the diagnosis and management of
To try to help the infant sleep through the night, food protein-induced enterocolitis syndrome: Ex-
Chapter 16 — Issues in pediatric gastroenterology 483
ecutive summary-workgroup report of the adverse cross over the vertebral column. Treatment typi-
reactions to foods committee, American Academy cally consists of a Ladd’s procedure to reduce any
of Allergy, Asthma & Immunology. J Allergy Clin volvulus, divide Ladd’s bands, broaden the base
Immunol. 2017; 139(4): 1111-1126. of the mesenteric vascular pedicle and perform an
appendectomy.
Question 30 REFERENCES
You are asked to evaluate a two week old female Adams, S. and Stanton, M. Malrotation and
in the clinic due to recurrent vomiting. The family intestinal atresias. Early Hum Dev. 2014; 90(12):
notes that the patient has intermittent episodes of 921-925.
vomiting throughout the day. They describe the Wyllie R. et al. Pediatric Gastrointestinal and
emesis as having a “green” color with the occa- Liver Disease. 4th edition. Elsevier Saunders,
sional appearance of formula. During some of the Philadelphia, 2011.
episode, the infant is noted to appear uncomfort-
able with abdominal distention. In clinic today,
the patient is feeding without distress. What is Question 31
the most likely diagnosis for this patient? A seven-year-old female presents to clinic due to
recurrent vomiting. She is noted to have a one-
A. Pyloric stenosis year history of recurrent vomiting. On further
B. Gastroesophageal reflux questioning, she reports that the vomiting typi-
C. Cow’s milk protein allergy cally occurs every three to four weeks and lasts for
D. Intestinal malrotation around one day. The family describes her waking
E. Food protein induced enterocolitis syndrome up in the morning vomiting, with symptoms spon-
(FPIES) taneously resolving later in the day. During the
episodes, she will have greater than 10 episodes of
CORRECT ANSWER: D non-bloody and non-bilious emesis. What treat-
ment options might be offered to this patient?
RATIONALE
This patient presents with intermittent bilious A. Amitriptyline
emesis with associated abdominal pain and dis- B. Cyproheptadine
tention, raising concern for an obstructive process. C. Propranolol
Since the symptoms are intermittent, with the D. L-carnitine and Coenzyme Q10
patient able to easily feed in between episodes, E. All of the above
pyloric stenosis is unlikely. During the fifth week
of gestation, the midgut herniates into the yolk CORRECT ANSWER: E
sac and undergoes sequential growth, elongation
and rotation. With normal rotation, the small RATIONALE
bowel mesentery is fixed to the posterior body Cyclic vomiting syndrome occurs is around 0.2-1.0
wall resulting in minimal opportunity for torsion. percent of the population. The condition typically
With intestinal malrotation, the lack of intestinal starts between 3.5 to seven years of age but can
fixation increases the risk for volvulus. In patients occur in adulthood. The ROME IV criteria for the
with malrotation, volvulus is more common in condition is 1. Occurrence of two or more periods
the neonatal age range, with untreated volvulus of intense, unremitting nausea and paroxysmal
potentially leading to ischemia and bowel necro- vomiting lasting hours to days within a six month
sis. The diagnosis is typically made by an upper period, 2. Episodes are stereotypical, 3. Episodes
GI series demonstrating the duodenum failing to are separated by weeks to months with return to
484 Digestive Diseases Self-Education Program®
following swallowing an object around two hours pediatric foreign body ingestions. Gastrointest
ago. The patient is asymptomatic with no drool- Endosc Clin N AM. 2016; 26(1): 99-118.
ing or apparent discomfort. On X-ray, a 20mm
radiopaque foreign body with a double halo sign is
see in the region of the proximal esophagus. What Question 34
management is needed at this time? A 22-month-old male presents to urgent care for
evaluation after being found in his parents’ garage
A. Observation in urgent care with repeat drinking from an unlabeled bottle. The patient is
X-ray in two hours crying throughout the examination and has a rash
B. Emergent endoscopic removal of the around his mouth. He does not appear to be able
foreign body to handle his secretions, with the patient drool-
C. Inpatient admission for observation ing throughout the examination. What long-term
D. Surgery consultation complication is the patient at highest risk for
E. Upper GI series with water soluble contrast developing?
REFERENCES The high risk for injury with ingestion led to the
Hyams, J, et al. Childhood functional gastrointes- mandatory recall of the common Buckyballs (rare
tinal disorders: Child/adolescent. Gastroenterol- earth magnets) in 2014. If two or more magnets
ogy. 2016; 150: 1456-1468. are in the esophagus or stomach, urgent endo-
Wilsey, M. et al. The role of upper gastroin- scopic removal is indicated. If the foreign body is
testinal endoscopy in the diagnosis and treatment distal to the stomach, serial monitoring to ensure
of caustic ingestion, esophageal strictures and progression of the magnets is recommended in an
achalasia in children. Gastrointest Endosc Clin N asymptomatic patient. In a symptomatic individ-
Am. 2001; 11(4): 767-787. ual, like the patient in this clinical vignette, urgent
surgical evaluation is needed.
Question 35 REFERENCE
Gastroenterology is consulted about a three-year- Kramer, R. et al. Management of ingested foreign
old male who presented to the emergency depart- bodies in children: a special clinical report of the
ment due to abdominal pain and distention. On NASPGHAN Endoscopy Committee. J Pediatr
examination, the patient is febrile with notable Gastroenterol Nutr. 2015; 60(4): 562-574.
discomfort to palpation over the abdomen. On
abdominal X-ray, four circular, radiopaque objects
that are in a linear arrangement are seen in the re- Question 36
gion of the stomach without evidence of intestinal A three-month-old male presents in clinic with
obstruction or free air. recurrent spitting. The mother notes that the in-
fant is currently on a standard cow’s milk formula.
What is the next step in management? With approximately half of his feedings, he is hav-
ing effortless regurgitation of a small amount of
A. Inpatient observation formula without a bilious appearance. She reports
B. Endoscopic retrieval of foreign body intermittent crying when she places him in his
C. Urgent surgical evaluation crib after feedings. His pediatrician has had the
D. Outpatient evaluation with serial abdominal family to thicken the formula with rice cereal and
X-rays try positional changes without improvement. The
E. Proceed with abdominal CT patient is growing well with no abnormal find-
ings on examination. His stool sample is guaiac
CORRECT ANSWER: C negative. What is the most likely etiology for the
infant’s symptoms?
RATIONALE
When ingested, magnets can attract between dif- A. Eosinophilic esophagitis
ferent layers of bowel. This attraction can lead to B. Pyloric stenosis
localized ischemia with risk for necrosis, perfora- C. Intestinal malrotation
tion and enteroenteric fistula formation. Many D. Physiologic gastroesophageal reflux
toys contain rare earth magnets like neodymium E. Delayed gastric emptying
that have increased attractive force compared to
conventional magnets. These magnets appear in CORRECT ANSWER: D
X-rays as circular radiopaque objects typically in
a linear distribution in small numbers and circu- RATIONALE
lar in larger numbers. The risk for bowel injury Regurgitation is common in infancy with 51 per-
from magnets is significantly higher with rare cent of infants having daily emesis at 0 to three
earth magnets compared to conventional ones. months of age and 67 percent having daily em-
Chapter 16 — Issues in pediatric gastroenterology 487
esis at four months of age. For infants 0 to three a two year history of a burning sensation in the
months of age, 14 percent are perceived as having epigastric region that occurs several times a week.
a “problem” with gastroesophageal reflux due to The patient also reports having abdominal full-
discomfort, frequent emesis and back arching. ness and early satiety associated with the episodes
For individuals in the age range of the clini- of discomfort. Due to the discomfort, the patient
cal vignette, 41 percent of infants have spitting will rarely avoid eating full meals and reports in-
with most feedings. Infants have a higher rate of creased belching. She denies any difficulties with
reflux symptoms due to a weaker lower esopha- bowel movements, reporting one Bristol 4 stool
geal sphincter as baseline, shorter intra-thoracic daily. The patient also denies vomiting or chest
esophagus, higher rate of supine positioning, and discomfort. What is the most appropriate diagno-
large fluid volume when compared to adults. The sis for this patient?
most common causes of gastroesophageal reflux
symptoms in children are physiologic gastro- A. Irritable bowel syndrome
esophageal reflux and cow’s milk protein allergy. B. Abdominal migraines
This patient does not have red flags to suggest a C. Functional dyspepsia
different etiology for his symptoms. Management D. Gastroesophageal reflux
of reflux in this age group starts with parental re- E. Peptic ulcer disease
assurance, upright positioning after feedings and
thickening feedings with oatmeal or rice cereal. CORRECT ANSWER: C
For refractory symptoms, a trial of a partially hy-
drolyzed or amino acid based formula for possible RATIONALE
cow’s milk protein allergy is recommended prior Chronic abdominal pain is common in the pedi-
to initiation of a proton pump inhibitor or H2 atric age range, with symptoms occurring in up
blocker. to 19 percent of children greater than four years
of age. Functional abdominal pain disorders are
REFERENCES the primary etiologies for the discomfort in the
Martin, A. et al. Natural history and familial majority of children. Based on ROME IV guide-
relationships of infant spilling to 9 years of age. lines, functional abdominal pain disorders consist
Pediatrics. 2002; 109(6): 1061-1067. of functional dyspepsia, irritable bowel syndrome,
Nelson, S. et al. Prevalence of symptoms of gas- abdominal migraines and functional abdominal
troesophageal reflux during infancy. A pediatric pain not otherwise specified (NOS). This patient
practice-based survey. Pediatric Practice Research reported epigastric burning discomfort, raising
Group. Arch Pediatr Adolesc Med. 1997; 151(6): concern for functional dyspepsia. The criteria
569-572. for functional dyspepsia is one or more symp-
Rosen, R. et al. Pediatric gastroesophageal toms for at least four days per month for greater
reflux clinical practice guidelines: joint recom- than two months of: 1. Postprandial fullness, 2.
mendations of the North American Society for Early satiety, 3. Epigastric pain or burning not
Pediatric Gastroenterology, Hepatology, and associated with defecation, 4. Symptoms cannot
Nutrition and the European Society for Pediatric be explained by another condition. This patient
Gastroenterology, Hepatology and Nutrition. J meets the above criteria. In addition, she also
Pediatr Gastroenterol Nutr. 2018; 66(3): 516-554. reports difficulties with finishing meals and post-
prandial belching suggesting postprandial distress
syndrome subtype of functional dyspepsia. She
Question 37 denies stooling difficulties, making irritable bowel
A 14-year-old female seeks evaluation due to syndrome unlikely. She does not have episodic
chronic abdominal discomfort. She reports having symptoms to suggest abdominal migraines.
488 Digestive Diseases Self-Education Program®
REFERENCE REFERENCES
Hyams, J, et al. Childhood functional gastrointes- Krishnasamy, S. and Abell, T. Diabetic gastropare-
tinal disorders: Child/adolescent. Gastroenterol- sis: principles and current trends in management.
ogy. 2016; 150: 1456-1468. Diabetes Ther. 2018; 9(suppl 1): S1-S42.
Saliakellis, E. and Fotoulakim M. Gastroparesis in
children. Ann Gastroenterol. 2013; 26(3): 204-211.
Question 38 Waseem, S. et al. Spectrun of gastroparesis
A 17-year-old female with type 1 diabetes mellitus in children. J Pediatr Gastroenterol Nutr. 2012;
presents to clinic due to abdominal discomfort. 55(2): 166-172.
She describes epigastric fullness that worsens with
eating. Associated with eating, the patient reports
nausea with intermittent non-bilious emesis. She Question 39
describes the emesis as having the appearance An 18-month-old male infant presents to an ur-
of previous eaten food. She does report a five gent care with vomiting and intermittent episodes
pounds weight loss. Her recent endocrinology of abdominal pain. The abdominal pain episodes
appointment indicated that she had a hemoglobin are intermittent, with the toddler appearing to
A1c of 12.4 percent. have severe pain and drawing his legs up towards
his abdomen followed by relative calm. During
What is the most appropriate long-term the episodes, he is having prominent bilious em-
treatment option? esis. On examination, the patient has a palpable
mass in the right upper quadrant but is otherwise
A. Erythromycin stable. A rectal examination was guaiac positive
B. Improved glycemic control without visible blood. Abdominal ultrasound
C. Ondansetron describes a “target-sign” mass in the right side of
D. Metoclopramide the abdomen. The family reports that this is the
E. Cognitive behavioral therapy longest ongoing episode of abdominal pain that
the patient has experienced. What is the most ap-
CORRECT ANSWER: B propriate next step in management?
RATIONALE A. Abdominal CT
The prevalence of gastroparesis is unclear in the B. Diagnostic laparoscopy
pediatric age range. For adults, the prevalence for C. Observation
definite gastroparesis is 37.8 for women and 9.6 D. Air contrast enema
for men per 100,000 population. Gastroparesis in E. Colonoscopy
adults is most commonly caused by diabetes mel-
litus (around 1/3), post-surgical complications, CORRECT ANSWER: D
neurologic conditions or idiopathic. For pediat-
rics, 70 percent of cases are idiopathic in natures RATIONALE
with just 4 percent being associated with a compli- Intussusception is the most common cause of me-
cation from diabetes mellitus. Diabetic gastropa- chanical intestinal obstruction in children ages 6 to
resis is thought to be caused by visceral autonomic 36 months, seen in up to 50 percent of cases. This
neuropathy. Initial treatment involves improving condition is caused by invagination of one portion
glycemic control with prokinetics and antiemetic of the gastrointestinal system into the lumen of an
for ongoing symptoms. This patient has classic adjacent segment. With this, the proximal intes-
symptoms of gastroparesis with reported post- tine is the intussusceptum, with the distal recipient
prandial fullness, abdominal pain and vomiting. intestine is the intussuscipiens. The invagination
Chapter 16 — Issues in pediatric gastroenterology 489
typically occurs in an antegrade order, with ileoco- been having multiple episodes of emesis starting
lic involvement seen in 80 percent of cases. This today. On examination, the patient has promi-
invagination is caused by the unbalanced forces nent discomfort with palpation or percussion over
created when a normal peristaltic wave encounters the periumbilical region. Screening labs demon-
a focal deformity in the bowel wall (lead point) strate a white blood cell count of 14,000 cells/mL
or a region with different mechanical properties. with an elevated C-reactive protein. A plain film
Focal lead points are found in only 10 percent of abdominal X-ray did not demonstrate evidence for
children, with the most common lead point being obstruction. While waiting for results, the patient
a Meckel’s diverticulum. While adults are found now reports more intense right lower quadrant
to have lead points in over 90 percent of cases. discomfort. What is the most likely etiology for
Ultrasound is the gold standard modality in detect- the patient’s symptoms?
ing intussusception. Non-operative management
with hydrostatic or pneumatic enemas under imag- A. Functional abdominal pain
ing guidance can resolve the condition in 80 to 95 B. Crohn’s disease
percent of physiologically stable cases. Surgical re- C. Intussusception
duction is recommended in patients with evidence D. Acute appendicitis
of perforation, pneumoperitoneum, physiologic E. Volvulus
instability or failure to fully reduce after three to
four non-operative attempts. Recurrence of intus- CORRECT ANSWER: D
susception is seen in five to 15 percent of pediatric
patients after non-surgical reduction. RATIONALE
This patient has a high clinical probability for Acute appendicitis is the most common surgi-
intussusception based on his age and intermittent cal condition found in the population, with the
symptoms of likely intestinal obstruction. His lifetime risk for diagnosis being 9 percent. Chil-
abdominal ultrasound highlighted the character- dren ages 10-14 have the highest incidence at
istic “target-sign” lesion for the condition. Since 169.6 cases/100,000 for diagnosis of appendicitis.
the patient is physiologically stable, non-operative Typical symptoms for the diagnosis include peri-
reduction with an air contrast enema would be the umbilical pain followed by right lower quadrant
most appropriate initial treatment. pain, nausea and vomiting. Other factors sug-
gesting the condition include a positive Rovsing
REFERENCES sign, psoas sign, or obturator sign on examination
Applegate K.E. Intussusception in children: as well as laboratory findings of white blood cell
evidence-based diagnosis and treatment. Pediatr count greater than 10,000 cells/mL, sterile pyuria
Radiol. 2009; 39(Suppl 2):S140-S143. and an elevated c-reactive protein. The Pediatric
Wyllie R. et al. Pediatric Gastrointestinal and Appendicitis Score (PAS) seen below is one of the
Liver Disease. 4th edition. Elsevier Saunders,
Philadelphia, 2011. Diagnostic Finding Value
Cough or Percussion tenderness 2
Fever 1
Question 40
RLQ tenderness 2
A 9-year-old male with a history of chronic ab-
Anorexia 1
dominal pain presents to the emergency depart-
Nausea and vomiting 1
ment due to acute worsening of periumbilical
Migration of pain 1
abdominal discomfort. His father reports new
onset of fever to 102F this afternoon. He also Leukocytosis (WBC >10,000) 1
notes that the patient refused to eat lunch and has Polymorphonuclear neutrophilia 1
490 Digestive Diseases Self-Education Program®
scoring systems used in pediatrics to assess risk of months of 1. Repeated regurgitation and rechew-
appendicitis. With this system, a score of five or ing or explusion of food that begins soon after
less is not suggestive of appendicitis. A score six or eating and stops with sleeping, 2. Not proceeded
more suggests appendicitis, with a score seven to 10 by retching, 3. No other clear etiology for symp-
indicating a high probability of the condition toms. This patient is at higher risk for rumination
(P 0.001). In this case, the patient has a PAS of syndrome with her developmental differences.
nine, making appendicitis the most likely diagnosis. Her painless regurgitation after eating meets cri-
teria for the condition. Prolonged high resolution
REFERENCES esophageal manometry can identify specific sub-
Anderson, J. et al. Examining a common disease groups of rumination. Antroduodenal manom-
with unknown etiology: trends in epidemiol- etry can detect simultaneous contractions called
ogy and surgical management of appendicitis in R-waves that can be seen in some patients with
California, 1995-2009. Worl J Surg. 2012; 36 (12): rumination syndrome. Since regurgitation stops
2787-2794. with sleeping, pH/Impedance probes demonstrate
Samuel, M. Pediatric appendicitis score. J Pe- resolution of symptoms with sleep. The condition
diatr Surg. 2002; 37(6): 877-881. is primarily diagnosed clinically, with other stud-
ies performed as clinically indicated. Treatment
typically consists of behavioral management.
Question 41
A 14-year-old female with a history of cerebral REFERENCE
palsy presents for evaluation due to recurrent Hyams, J, et al. Childhood functional gastrointes-
regurgitation. By report, she is regurgitating tinal disorders: Child/adolescent. Gastroenterol-
food into her mouth several times daily following ogy. 2016; 150: 1456-1468.
meals. Her parents report that the regurgitation
does not appear to be painful. What diagnostic
finding could be seen in this patient? Question 42
A 8-year-old previously healthy female presents
A. Normal pH/Impedance probe findings following an isolated episode of rectal prolapse.
during sleeping The family notes that the prolapse spontane-
B. Esophageal dysmotility on upper GI series ously reduced within one hour of onset with the
C. Decrease in gastric emptying time patient not having any perceived discomfort. On
D. Esophageal eosinophilia on upper intestinal examination, the patient has a dilated rectum with
endoscopy increased stool burden but no other clear abnor-
E. Absence of “R-waves” on antroduodenal malities. Which condition is most likely contrib-
manometry uting to the patient’s symptoms?
Question 50
An exclusively breastfed 15-month-old female
presents for evaluation due to an elevated alkaline
phosphatase. On examination, the patient is not-
ed to have a persistently open fontanelle, beading
on the costochondral junctions, and progressive
lateral bowing of the femur and tibia. What vita-
min is most likely to be deficient in this patient?
A. Vitamin A
B. Vitamin C
C. Vitamin D
D. Vitamin E
E. Vitamin K
CORRECT ANSWER: C
RATIONALE
This patient is noted to have a persistently open
fontanelle, beading on the costochondral junc-
tions and bowing of the femur and tibia. These
findings are commonly seen with rickets. Other
possible physical exam can include decreased
growth velocity, frontal bossing, decreased muscle
tone and delayed motor milestones. Rickets is
classified as poor mineralization of the growth
plate due to abnormal serum calcium or phospho-
rus. Calcipenic rickets is caused by insufficient
vitamin D or insufficient intake/absorption of
calcium. Phosphopenic rickets is typically related
to renal phosphate wasting. To assess for rickets,
parathyroid hormone, calcium and phosphorus
levels should be obtained. If the parathyroid level
is elevated, then the vitamin D level should be as-
sessed. Treatment for vitamin D deficiency rickets
focuses on vitamin D supplementation. This
patient is at increased risk for rickets due to her
continuing to exclusively breastfeed.
REFERENCE
Lambert, A. and Linglart, A. Hypocalcaemic and
hypophosphatemic rickets. Best Pract Clin Endo-
crinol Metab. 2018; 32(4): 455-476.
Answers & critiques
CHAPTER 17
Question 1 REFERENCE
Which of the following patients is the most Leffler DA, Schuppan D. Update on serologic
appropriate candidate for TTG-IgA testing for testing in celiac disease. Am J Gastroenterol.
celiac disease? 2010. 105(12):2520-4.
497
498 Digestive Diseases Self-Education Program®
ance has not been shown to be associated with of IV therapy with a cephalosporin (C) or car-
villous blunting on endoscopic biopsy. bapenem in order to reach the CNS, which can be
affected by T. whipplei with no symptoms. There
REFERENCE is no role for Immunosuppressive therapy (E),
Pallav K, Leffler DA, Tariq S et al. Noncoeliac anti-parasitic therapy (B), or dietary therapy (A)
enteropathy: The differential diagnosis of villous in the treatment of Whipple’s Disease.
atrophy in contemporary clinical practice. Aliment
Pharmacol Ther. 2012;35(3):380-90. REFERENCES
Moos et al. Changing paradigms in Whipple’s
disease and infection with T. whipplei. Eur J Clin
Question 3 Microbiol Infect Dis. 2011; 30:1151-1158.
A 52-year-old male with past medical history of Feurle GE, Junga NS, Marth T. Efficacy
hypothyroidism presents with two months of of ceftriaxone or meropenem as initial thera-
watery diarrhea and weight loss. In addition, he pies in Whipple’s disease. Gastroenterology.
notes that his knee has been hurting more signifi- 2010;138(2):478-486.
cantly and he is having more regular headaches.
He undergoes upper endoscopy with duodenal
biopsy which reveals villous atrophy with PAS- Question 4
positive macrophages. Which of the following is Which of the following clinical situations has the
the most appropriate treatment for this patient? best indication for oral steroid therapy?
a rare disorder characterized by severe diarrhea, D. Small intestinal biopsies positive for Congo
weight loss secondary to malabsorption, villous Red Staining with birefringence under
atrophy of the small intestinal mucosa and the polarized light
presence of autoantibodies, primarily anti-entero- E. Small intestinal biopsies demonstrating
cyte and anti-goblet cell antibodies. Oral steroids, granulomas
either prednisone or budesonide, is considered
first-line therapy. Second-line therapies may CORRECT ANSWER: D
include mycophenolate mofetil, azathioprine, or
cyclosporine RATIONALE
In Answer A, the patient does have small bowel This patient has amyloidosis of the small bowel.
crohn’s disease, however new onset fevers and Amyloidosis of the small intestine can present
chills in a patient with fistulizing disease is con- with malabsorption, diarrhea, abdominal pain, or
cerning for an abscess formation or other infec- ulceration resulting in gastrointestinal bleeding.
tion. Steroids would not have an indication at this Rarely it presents with perforation of the bowel.
time. In Answer B, the patient has celiac disease Non-GI involvement of primary amyloidosis can
but has not met criteria for refractory celiac disease include restrictive cardiomyopathy, chronic kid-
as she has not undergone 6 months of a gluten free ney disease, peripheral neuropathy, weight loss, or
diet. Lastly, there is no role for steroid therapy in periorbital purpura.
Whipples disease, an infection with Tropheryma Amyloidosis of the small bowel is diagnosed
whipplei bacteria, as seen in answer C. by biopsy of the affected area. The gold standard
for diagnosing amyloidosis is Congo red staining
REFERENCE of biopsies typically obtained from the rectum or
Akram S, Murray JA, Pardi D et al. Adult au- duodenum that characteristically shows apple-
toimmune enteropathy: Mayo Clinic Roches- green birefringence under polarized light.
ter experience. Clin Gastroenterol Hepatol. Celiac disease is diagnosed by TTG-IGA (A),
2007;5(11):1282-1290. Autoimmune enteropathy can be associated with
auto-enterocyte antibodies (B), and Crohn’s dis-
ease and multiple infections including tuberculo-
Question 5 sis can be associated with granulomas (E). Gastric
A 70-year-old African-American male with signifi- hyperkinesis (C) is not associated with ulceration
cant past medical history of type 1 diabetes and of the small bowel.
restrictive cardiomyopathy presents to your office
with six weeks of abdominal discomfort and diar- REFERENCE
rhea. This has been associated with a 15-lb. weight Ebert et al. Gastrointestinal manifestations
loss. Most recently, he has some dark tarry stools of amyloidosis. Am J Gastroenterol. 2008;
for the past three days. An upper endoscopy and 103:776-787.
colonoscopy are grossly normal. You perform a
video capsule endoscopy which demonstrates an
extended area of ulcerated, edematous, friable mu- Question 6
cosa in the mid small bowel. Which of the follow- A 67-year-old man with a history of chronic
ing test results is most likely for this patient: kidney disease on dialysis presents with diarrhea
and a 20 pound weight loss. On endoscopy, the
A. Elevated TTG-IgA mucosa is granular with polypoid protrusions.
B. Positive Auto-Enterocyte Antibody Histologic examination is positive for Congo read
C. Accelerated transit time in the stomach on staining. Which of the following treatment options
SmartPill testing (Wireless Motility Capsule) is the most appropriate for this patient?
500 Digestive Diseases Self-Education Program®
A. Gluten Free Diet heart failure, sepsis, and recent myocardial infarc-
B. Immunosuppressive therapy tion. Traditional presentation includes severe,
C. Supportive Care and Anti-diarrheal therapy acute abdominal pain out of proportion to the
D. Proton Pump Inhibitor therapy exam. CT Angiography or MR Angiography are
E. Anti-Tumor Necrosis Factor Therapy the major diagnostic tools, as lab testing includ-
ing lactate may be delayed. Treatment is usually
CORRECT ANSWER: C emergent surgical revascularization. Exploratory
laparotomy may be required if peritonitis is pres-
RATIONALE ent on exam. There is a significant urgency to
The mainstay of therapy for small bowel amy- treatment as the mortality rate is over 50 percent
loidosis is supportive care. There is no role for and studies have shown that the greater that time
gluten free diet like there is for celiac disease from symptom presentation to intervention, the
or gluten intolerance (A), immunosuppressive intestinal viability decreases significantly to nearly
therapy (B), Acid Suppression (D), or Anti-TNF 18 percent after 24 hours (E).
therapy (E) On the other hand, chronic mesenteric isch-
emia can present as sitophobia (C), can be caused
REFERENCE by artherosclerosis (A), and can be tested using
Ebert et al. Gastrointestinal manifestations of Balloon Tonometry (B). Interventional treat-
amyloidosis. Am J Gastroenterol. 2008; ment, when necessary, includes percutaneous
103:776-787. transluminal mesenteric angioplasty on an elec-
tive basis (D).
Question 7 REFERENCES
Which of the following is true regarding acute American Gastroenterological Association Medical
mesenteric ischemia? Position Statement: Guidelines on Intestinal Isch-
emia. Gastroenterology. 2000;118(5):951-953.
A. Mesenteric atherosclerosis is the major cause. LoboMartinez E, Carvajosa E, Sacco O et al.
B. Balloon Tonometry testing can be used to Embolectomy in mesenteric ischemia. Rev Esp
distinguish the clinical significance of a Enferm Dig. 1993; 83:351-354.
radiologist finding of mesenteric ischemia.
C. Sitophobia is an associated symptom.
D. Elective percutaneous transluminal mesenteric Question 8
angioplasty is an effective treatment. At 36-year-old Caucasian female with past medi-
E. After 24 hours of symptoms, the ability to cal history of hypothyroidism presents to the
salvage healthy intestinal tissue is less than gastroenterology office for ongoing bloating. This
50 percent. bloating is associated with altered bowel habits
resulting in two to three bowel movements a day.
CORRECT ANSWER: E She has tried a Histamine-2 Antagonist agent and
Simethicone with limited results.
RATIONALE
Acute mesenteric ischemia is a serious, potentially The gastroenterologist sends blood testing with
fatal disease in which blood vessel is acutely oc- results indicated below:
cluded via emboli, thrombi, or potentially vaso-
constriction resulting in damage to the bowel. WBC 4.6K/mL
Risk factors include older age, hypercoagulable Hgb 13.4 gm/dL
states (i.e. Protein C deficiency), long standing Hct 40.2%
Chapter 17 — Small bowel disease 501
Question 15 REFERENCE
A 22-year-old male with a past medical history Khoshini R, Dai SC, Lezcano S et al. A systematic
of eosinophilic esophagitis on fluticasone inhaler review of diagnostic tests for small intestinal
440 mcg daily and chronic nausea controlled bacterial overgrowth. Dig Dis Sci. 2008; 53:
with Metoclopramide presents to the GI office for 1443-1454.
new onset bloating and loose stools. Blood tests
including a TTG-IgA and IgA level are within
normal limits. An upper endoscopy demonstrates Question 16
atrophic gastritis on biopsy while a colonoscopy A 62-year-old female with past medical history of
reveals diverticulosis of the transverse colon. He Non Hodgkins Lymphoma treated with chemo-
undergoes a hydrogen breath test which confirms therapy and radiation eight years ago presents to
the diagnosis of small intestinal bacterial your office with increased watery diarrhea. The
overgrowth. patient complains of increased bloating, disten-
sion, and some increased flatulence as well. Infec-
Which aspect of his past medical history tious stool studies have been negative. Endo-
most likely contributed to his development scopic evaluation and biopsies are negative. She
of SIBO: undergoes a CT Enterography demonstrating a 5
cm stricture in her ileum. She is treated with 10
A. Metoclopramide days of rifaximin which resolves the symptoms,
B. Diverticulosis of the large intestine but they return a few weeks after treatment.
C. Eosinophilic esophagitis
D. Atrophic gastritis Which of the following is the next best step in
therapy?
CORRECT ANSWER: D
A. Put patient on regimen of cycling antibiotics
RATIONALE B. Loperamide PRN
Small intestinal bacterial overgrowth is a patho- C. Referral to surgeon for resection of stricture
logic overgrowth of bacteria in the upper small D. Wireless motility capsule (SmartPill) testing
intestine. Risk factors for SIBO include the E. Cholestyramine
following: dysmotility of the gut (I.e. gastropare-
sis), primary systemic scleroderma, small bowel CORRECT ANSWER: C
diverticulosis, altered anatomy (i.e. small bowel
strictures, gastric bypass surgery), celiac disease, RATIONALE
Immunodeficiency (i.e. IgA deficiency), pancreatic This patient has recurrent small intestinal bacte-
insufficiency, and Gastric hypochlorhydria from rial overgrowth (SIBO), a pathologic overgrowth
either atrophy (D) or from medications such as of bacteria in the upper small intestine. Risk fac-
proton pump inhibitors. tors for SIBO include altered anatomy of the small
While medications that slow the bowel may be bowel: small bowel diverticulosis, strictures of
associated with SIBO, pro-motility agents such as the small bowel, gastric bypass, or blind limbs. If
metoclopramide (A) do not have a known associa- altered anatomy are the cause of recurrent SIBO,
tion. Diverticulosis of the large intestine (B) is not treatment should be geared towards reversal of
associated with SIBO, only diverticulosis of the these changes when possible. Resection of a 5 cm
small intestine. And while eosinophilic enteritis stricture is a reasonable next step.
or other inflammatory diseases of the small intes- Cycling antibiotics can be used when the
tine can contribute to SIBO, there is no associa- underlying cause of SIBO is non reversible (A).
tion with eosinophilic esophagitis (C). Loperamide (B) and cholestyramine (E) poten-
506 Digestive Diseases Self-Education Program®
tially might help manage symptoms, but will pleural and pericardial effusions, hypoalbumin-
not fix the underlying problem. There is no role emia hypogammaglobinemia, and potentially
for wireless motility capsule testing (D) in this fat-soluble vitamin deficiencies.
patient, particularly given the ileal stricture. Diagnosis is made by endoscopy where you
may see a creamy yellow-white appearance
of the jejunal villi. Endoscopic biopsy reveals
Question 17 dilated mucosal and submucosal lymphatic ves-
A 28-year-old female with a history of Turner sels. Stool testing may show an elevated alpha 1
syndrome presents and complaints of progres- antitrypsin.
sively worsening odiferous diarrhea that appears The cornerstone of treatment is a low-fat diet
to have float atop the water in the toilet. On supplemented with medium chain triglycerides.
review of symptoms she also reports worsening The goal is to minimize the engorgement of the
lower extremity edema. Infectious stool studies lymphatics. As medium chain triglycerides are
are negative, but she has a positive alpha-1 anti- directly absorbed, they do not cause further lym-
trypsin stool study. On endoscopy, she is found phatic dilation.
to have dilated lymphatics in the second part of
the duodenum. Biopsy results confirm dilated REFERENCE
lymphatics in the mucosal and submucosal lay- Alfano V, Tritto G, Alfonsi L et al. Stable reversal
ers. The next step in treatment is: of pathologic signs of primitive intestinal lym-
phangiectasia with a hypolipidic, MCT-enriched
A. Rifaximin x 10 days diet. Nutrition. 2000;16(4):303- 304.
B. Gluten free diet
C. Short bowel transplant
D. Total parenteral nutrition Question 18
E. Low fat diet supplemented with medium A 20-year-old male presents to the emergency
chain triglycerides room with dark stools for two days. He reports
that he has had ongoing abdominal discomfort
CORRECT ANSWER: E intermittently for the past month, but never
noticed a change in color of his stools until now.
RATIONALE He denies taking any medications. His weight
This patient has primary intestinal lymphangiec- has been stable. Physical exam reveals diffuse
tasia (PIL), also known as Waldmann’s disease. ulcerations in his mouth and on his scrotum.
PIL is a rare condition characterized by dilata- His hemoglobin is 9.6 g/dL on admission. Up-
tion of the intestinal lymphatics which blocks per endoscopy and colonoscopy are normal. A
effective fluid drainage leading to a subsequent video capsule endoscopy reveals deep, cratered
loss of lymph fluid into the GI tract. PIL is as- ulcerations in the jejunum.
sociated with von Recklinghausean syndrome,
Turner syndrome, and Hennekam syndrome. The most likely cause of this patients’ findings is:
Secondary intestinal lymphangiectasia can
present with similar symptoms, but is caused A. Crohn’s disease
by impaired lymphatic flow from lymphoma, B. Bechet’s disease
right sided heart failure, and thoracic duct outlet C. Surreptitious NSAID use
obstruction. D. Mast cell disorder
Major symptoms include a protein losing E. Takasayu arteritis
enteropathy that may result in lymph edema,
peripheral edema, serous effusion - particularly CORRECT ANSWER: B
Chapter 17 — Small bowel disease 507
discomfort, and loose stools. Infectious workup A. Small intestinal bacterial overgrowth
is negative. Endoscopic evaluation with duode- B. Small bowel lymphoma
nal biopsies is negative. A lactulose breath test is C. Refractory celiac disease
positive. Which of the following lab abnormalities D. Inadvertent gluten exposure
is not associated with this entity:
CORRECT ANSWER: D
A. Folate deficiency
B. Vitamin A deficiency RATIONALE
C. B12 deficiency This patient is a classic demographic patient for
D. Vitamin D deficiency celiac disease and had both positive serologic and
E. Low albumin histologic testing. In addition, they responded well
to initial treatment with a normalization of the
CORRECT ANSWER: A TTG-IgA. The most common cause of an elevated
TTG-IgA in a patient with celiac disease is dietary
RATIONALE indiscretion – either inadvertently or on purpose.
Small intestinal bacterial overgrowth (SIBO) is Small bowel lymphoma (A) is a known com-
a pathologic overgrowth of bacteria in the upper plication of long term celiac disease and would be
small intestine. A lactulose or glucose breath test unlikely at this time. Refractory celiac disease (C)
is the diagnostic test of choice for SIBO. SIBO can refers to persistent TTG-IgA elevation despite ad-
result in significant vitamin deficiencies including herence to a gluten free diet for six or more months.
fat soluble vitamins (A, D, E, K and B12). It may This patient previously responded to a gluten free
also be associated with protein losing enteropathy, diet making this diagnosis less likely. While SIBO
resulting in a low albumin. Folate (A) tends to be (A) does have a higher prevalence in patients with
increased due to bacterial production of folate, celiac, it will not elevate the TTG-IgA.
rather than a deficiency.
REFERENCES
REFERENCE Abdallah H, Leffler DA, Dennis M et al. Refrac-
Khoshini R, Dai SC, Lezcano S et al. A systematic tory celiac disease. Curr Gastroenterol Rep.
review of diagnostic tests for small intestinal bacte- 2007;9(5):401-5.
rial overgrowth. Dig Dis Sci. 2008; 53:1443-1454. Di Sabatino A, Corazza GR. Coeliac disease.
Lancet 2009;373(9673):1480-93.
Question 21
A 23-year-old female from the Middle East pres- Question 22
ents to your office to establish care for her celiac A 45-year-old female with past medical history of
disease. Upon reviewing her records, you see that anti-phospholipid syndrome and subsequent acute
she had TTG-IgA testing with a normal IgA level mesenteric ischemia presents to establish GI care.
and subsequent duodenal biopsies confirming the One year ago, she had an episode of acute mes-
diagnosis. Since her diagnosis two years ago, the enteric ischemia requiring resection of 140 cm of
patient reports maintaining a strict gluten free small bowel. She currently has an end ileostomy.
diet. Subsequent testing after starting this diet For the past three months, she has had significant
showed a normal TTG-IgA level. For the past ostomy output of over three liters per day despite
three months, she has increasing bloating and trying loperamide, atropine/diphenoxylate and a
loose stools. You repeat her TTG-IgA and find tincture of opium. She has been hospitalized twice
that it is elevated. The most likely cause of the for dehydration in this time period. She has had
patient’s increased TTG-IgA level is: negative infectious workup and negative endos-
Chapter 17 — Small bowel disease 509
copy and ileoscopy with biopsies. Which of the (baseline 13.8) with an MCV of 78 fl. Iron studies
following medications is most indicated for this demonstrate a low total iron and low ferritin. An
patient: endoscopy and colonoscopy are normal. On video
capsule endoscopy, you see an approximately
A. Tyrosine kinase inhibitor 3 cm, non-mucosal mass in the jejunum. The
B. Glucagon-like peptide-2 agonist lesion is surgically excised and no spread to the
C. Anti IGE humanized monoclonal antibody lymph nodes or metastasis to the liver is seen. On
D. Cromolyn sodium pathology review, the lesion tests positive for KIT
E. Anti-tumor necrosis factor therapy (CD 117 leukocyte antigen). What is his diagnosis:
Question 27 Question 28
A 25-year-old woman presents with a five-year A 30-year-old woman was diagnosed with celiac
history of watery, non-bloody diarrhea and recur- disease 36 months ago. She has remained compli-
rent bacterial pneumonias. Laboratory studies ant with a strict gluten-free diet including regular
show iron-deficiency anemia with low 25-OH Vi- visits with a dietician. She is currently asymptom-
tamin D. celiac serologies are negative, and small atic with normalization of tissue transglutaminase
bowel biopsies reveal near total villous atrophy, IgA antibody and duodenal biopsies.
increased intraepithelial lymphocytes and crypt
hyperplasia with absent plasma cells. What is the In addition to advising her to remain on a strict
most appropriate initial treatment strategy to ad- gluten-free diet, her management should include
dress the gastrointestinal symptoms? which of the following?
RATIONALE RATIONALE
This patient has gastrointestinal manifestations This patient’s celiac disease is well-controlled on
of common variable immune deficiency (CVID), a strict gluten-free diet. With normalization of her
which can present similarly to celiac disease. tissue transglutaminase antibody levels and duo-
Histologically, intestinal biopsies will reveal villous denal biopsies have normalized. Given this, there
atrophy, crypt hyperplasia and intraepithelial lym- is no role for surveying her small bowel with video
phocytosis similar to celiac disease. However, while capsule endoscopy to look for ulcerative jejunitis
plasma cells are increased in celiac disease, they are or lymphoma. In the absence of current diarrhea
absent in common variable immune deficiency. to suspect concomitant microscopic colitis, there
The initial treatment strategy for CVID typi- is no indication for colonoscopy. Although small
cally includes oral corticosteroids, either pred- intestinal bacterial overgrowth can be seen with
nisone or budesonide, while infliximab or other celiac disease, she has no current symptoms to
anti-tumor necrosis factor agents are reserved for suggest this diagnosis. With an established diag-
steroid-dependent or refractory disease. Other nosis of celiac and a normalized tissue transglu-
options including gluten-free diet and intravenous taminase antibody, there is no role for additional
immunoglobulin therapy do not improve the GI serologic testing. Patients with celiac disease,
symptoms associated with CVID. Rifaximin may even when well-controlled, are at higher risk for
treat any associated SIBO in the setting of CVID pneumococcal pneumonia due to underlying hy-
but is unlikely to cease the diarrhea as bacterial posplenism and hence should undergo vaccination
overgrowth is typically the consequence of the against Streptococcus pneumoniae.
luminal changes not the cause.
REFERENCE
REFERENCE Rubio-Tapia A, Hill ID, Kelly CP, et al. ACG
Agarwal S, Mayer L. Gastrointestinal manifesta- clinical guidelines: diagnosis and management
tions in primary immune disorders. Inflamm of celiac disease. Am J Gastroenterol. 2013 May;
Bowel Dis. 2010;16:703-711. 108(5): 656–677.
Chapter 17 — Small bowel disease 513
REFERENCE
Question 33 Frye JM1, Hansel SL, Dolan SG, Fidler JL, Song
A 34-year-old woman presents for consultation LM, Barlow JM, Smyrk TC, Flicek KT, Hara AK,
for bloating, abdominal pain and diarrhea for the Bruining DH, Fletcher JG. NSAID enteropathy:
past three months after a recent hip fracture. Labs appearance at CT and MR enterography in the age
demonstrate iron deficiency anemia with nega- of multi-modality imaging and treatment. Abdom
tive celiac serologies. Endoscopy reveals normal Imaging. 2015 Jun;40(5):1011-25. doi: 10.1007/
appearance with biopsies revealing partial villous s00261-015-0367-2.
atrophy with increased intraepithelial lympho-
cytes. A CT enterography reveals a diaphragm
like stricture in the proximal ileum. A subsequent Question 34
colonoscopy reveals normal appearing ileum and A 36-year-old woman from Texas presents to the
colon with biopsies from ileum revealing acute office for further evaluation of chronic diarrhea.
ileitis without architectural distortion. She reports a five-month history of post-prandial
diarrhea with foul-smelling stool associated with
Which of the following is the next best step in an unintentional 15-pound weight loss due to lack
management? of appetite, increased abdominal cramping and
bloating. History notes a trip to the Dominican
A. Start gluten-free diet Republic 11 years ago. Notable features on physi-
B. Six-week trial of budesonide cal exam include glossitis, pitting pedal edema,
C. Stop NSAIDs and a distended abdomen with hyperactive bowel
D. Trial of rifaximin sounds. Laboratory testing reveals hemoglobin of
E. Start infliximab 10.2 g/dL, hematocrit 33 percent with an MCV of
110 fL (elevated), folate 2.0 ng/ml and a slightly
CORRECT ANSWER: C decreased albumin of 3.1 g/dL with negative celiac
serologies, stool studies negative for infection, but
RATIONALE positive for fecal fat testing with 20 grams of fat
In a person with negative celiac serologies with- in 24-hours. An upper endoscopy was performed
out conclusive histology to support a diagnosis of with duodenal biopsies showing mild villous
celiac disease, trial of GFD is not recommended. blunting with increased numbers of plasma cells,
Similarly, a trial of budesonide and infliximab is eosinophils, and lymphocytes in the lamina pro-
not appropriate in this setting without findings pria. Which of the following is the most appropri-
to support a diagnosis of Crohn’s disease. The ate treatment?
findings of a diaphragm like stricture is specific to
NSAID enteropathy which is a likely cause given A. Metronidazole
the history of a recent hip fracture. The adverse ef- B. Tetracycline
fects of NSAIDs on the upper gastrointestinal tract C. Folic acid plus B12 plus tetracycline
have been well documented. Data suggest that D. Amoxicillin + clarithromycin
NSAIDs can also be harmful to the small intestine. E. Rifaximin
In addition to causing diaphragm-like strictures,
ulcerations, perforations and diarrhea, NSAID use CORRECT ANSWER: C
516 Digestive Diseases Self-Education Program®
along with progressive pedal edema, shortness of after she returned from a year-long service mis-
breath and ascites. On labs, his albumin is 1.5 g/ sion in Haiti. She has up to five bowel movements
dL and fecal alpha-1-antitrypsin clearance is 400 a day, and she describes her stool as malodor-
mg per 10ml (normal, less than 27mg per 100ml). ous and “greasy,” but not bloody. Her stool has
tested negative for bacterial pathogens, ova and
Which of the following findings would be expected parasites and Clostridium difficile toxin assay on
on upper endoscopy? two separate occasions in the month prior to her
clinic visit. Despite this, her primary physician
A. Diffusely ulcerated, friable mucosa treated her with a 10-day course of ciprofloxacin
B. Scalloped duodenal folds and metronidazole which improved her symptoms
C. Normal small intestinal mucosa but symptoms recurred two months after comple-
D. Multiple jejunal angioectasias tion of antibiotic therapy. Subsequent lab testing
E. Creamy yellow-white jejunal villi revealed negative tissue transglutaminase and
anti- endomysial antibodies. Her hemoglobin was
CORRECT ANSWER: E 10.2 g/ dL with an MCV of 105 fL. Her vitamin
B12 level was 1000 ng/L and folate was 3 ng/mL.
RATIONALE Her liver chemistries and serum creatinine were
This patient has protein-losing enteropathy due normal. An upper endoscopy was grossly normal,
his previous testicular malignancy that required but duodenal biopsies revealed normal villi with
radiation therapy to lymph nodes in his abdo- increased lymphocytes in the lamina propria. A
men leading to secondary lymphangiectasia from colonoscopy performed at the same time including
obstruction of lymphatic ducts in the abdomen. random colonic biopsies and ileal inspection, was
Protein losing enteropathy is characterized by normal.
hypoalbuminemia, which can lead to edema
and ascites due to decreased oncotic pressures, Which of the following is the most appropriate
hypogammaglobulinemia, and fat malabsorp- next step in her management?
tion. Luminal protein loss can be measured by
checking the fecal alpha- 1-antitrypsin clearance A. Initiate a gluten-free diet
as well as identification of the underlying cause. B. Perform HLA haplotyping
Classic endoscopic findings in lymphangiectasia C. Treat with tetracycline and folic acid
include creamy yellow-white jejunal villi indicat- D. Treat with albendazole
ing the dilated lymphatic vessels without evidence E. Perform lactulose hydrogen breath testing
of duodenal scalloping, ulceration, friability or
angioectasias. CORRECT ANSWER: E
REFERENCE RATIONALE
Umar SB, DiBaise JK. Protein-losing Enteropa- While the travel history would suggest a risk
thy: Case Illustrations and Clinical Review. Am J of tropical sprue, the high B12 and response
Gastroenterol. 2010;105:43-49. to ciprofloxacin and metronidazole with re-
currence of symptoms in a patient with a risk
factor for SIBO (long standing poorly controlled
Question 37 type 1 diabetes mellitus) and a relatively normal
A 34-year-old woman with a 20-year history of duodenal biopsies except for increased IELs
poorly controlled type 1 diabetes mellitus comes to would suggest underlying SIBO as the cause of
clinic complaining of several months of bloating, her symptoms. Breath testing would therefore
diarrhea and 10 lb weight loss that began shortly be the next best step.
518 Digestive Diseases Self-Education Program®
A. Fatty liver disease from TPN Which of the following will most likely establish
B. Repeated cephalic and basilic vein thrombosis the underlying diagnosis?
C. One episode of line-related sepsis from non-
fungal organisms A. Anti-enterocyte antibody testing
D. One episode of line-related fungemia B. Duodenal biopsy with Periodic Acid Schiff
E. One episode of severe dehydration staining
Chapter 17 — Small bowel disease 519
C. Rectal biopsy with Congo red staining Ebert E, Nagar M. Gastrointestinal manifestations
D. Fecal calprotectin testing of amyloidosis. Am J Gastroenterol. 2008;103(3):
E. Glucose hydrogen breath test 776-87.
CORRECT ANSWER: C
Question 40
RATIONALE An 81-year-old man is admitted for a post-polyp-
This patient most likely has gastrointestinal (AA) ectomy bleeding after removal of 5 mm polyp with
amyloidosis associated with chronic inflamma- a cold-snare during a surveillance colonoscopy
tory conditions of Crohn’s disease and ankylos- which demonstrated normal appearing mucosa
ing spondylitis. Gastrointestinal amyloidosis can except for the single polyp. He details a history
present with features of dysmotility, including of another episode of post-polypectomy bleed-
gastroparesis, dysphagia, gastroesophageal reflux ing with a prior procedure five years ago. He is
and intestinal pseudo-obstruction or diarrhea otherwise healthy and is on no medications except
with intestinal inflammation and concomitant Aspirin 81 mg which was stopped five days prior
bacterial overgrowth. Symptoms suggestive of a to the procedure. He does not drink alcohol and
protein-losing enteropathy with hypoalbumin- is a non-smoker. Labs demonstrate a hemoglobin
emia, edema, malabsorptive diarrhea, edema, of 10.2 g/dL, with albumin 3.4 g/dl and serum
and pleural/pericardial effusions can also occur. creatinine 1.1 mg/dl.
Diagnosis is made through biopsies ideally from
the duodenum or from the rectum with Congo red Which of the following test is the most appropriate
staining of mucosal biopsies showing the charac- next step to determine the reason for the post-
teristic apple-green birefringence under polarized polypectomy bleeding?
light. In AA amyloidosis – gamma globulins tend
to be elevated due to chronic inflammation from A. Bleeding time
associated conditions such as Crohn’s disease. B. Activated partial thromboplastin time (APTT)
Treatment is usually supportive with anti-diarrhe- C. Fibrinogen level
als, nutritional supplementation and addressing D. Rectal biopsy at the time of relook colonoscopy
the underlying condition. with Congo-red staining
Other options such as anti-enterocyte antibody E. Ristocetin cofactor activity
for autoimmune enteropathy and PAS staining
(for Whipple’s disease) are unlikely to establish CORRECT ANSWER: D
the diagnosis. There is no role for checking a fecal
calprotectin in a Crohn’s patient with a normal RATIONALE
MR enterography due to poor sensitivity for small This patient most likely has senile amyloidosis
bowel inflammation. Finally, the patient may have which is found in 10-36 percent of patients over
underlying small bowel intestinal overgrowth with 80 years old and mainly involves the heart and to a
Crohn’s disease and AA amyloidosis which may be lesser extent the GI tract. In one study, 41-44 per-
diagnosed by the breath test but would not yield cent of elderly patients had amyloid deposits in the
the underlying diagnosis of AA amyloidosis. sub-serosal veins localized to the small and large
intestine. This may present as recurrent gastroin-
REFERENCES testinal bleeding after a routine polypectomy.
Petre S, Sha A, Gilani N. Review article: gastro- Hence option D is the best option. Other options
intestinal amyloidosis – clinical features, di- cover testing for platelet function, von-Willebrand
agnosis and therapy. Aliment Pharmacol Ther. disease (Ristocetin cofactor activity), DIC (fibrino-
2008;27:1006-1016. gen levels) and coagulation cascade defects (APTT).
520 Digestive Diseases Self-Education Program®
extremities but progressing to involve the abdo- loss with large, foul-smelling four to five bowel
men and chest. The bone marrow and gastroin- movements per day. Physical examination is no-
testinal tract are common extracutaneous sites table for a febrile, cachectic-appearing male with
for mast cell accumulation – common GI tract numerous patches of hyperpigmentation of the
symptoms include nausea, vomiting and diarrhea skin, multi-site lymphadenopathy and a protuber-
often times related to gastric acid hypersecre- ant abdomen with shifting dullness on examina-
tion and may be increased due to excess circu- tion. He undergoes and EGD with small bowel bi-
lating histamine, which can be associated with opsies where PAS positive foamy macrophages are
gastrointestinal bleeding. Diagnosis of systemic observed. Which of the following additional stains
mastocytosis requires the presence of one major should be requested with the biopsy specimen?
criterion (presence of greater than 15 mast cells as
detected by tryptase staining) in the bone marrow A. Congo-red staining
or extracutaneous organs and one minor criterion B. Quantitation of eosinophils
or three minor criteria. Minor criteria for diagno- C. CD-117 staining
sis include: 1) atypical morphology in greater than D. Acid-fast bacillus staining
25 percent of mast cells, 2) codon 816 mutation of E. Trichrome staining
KIT (the gene for c-kit), 3) mast cells expressing
CD2 and/or CD25 in the bone marrow or extra- CORRECT ANSWER: D
cutaneous organs and 4) serum tryptase levels
greater than20ng/mL. RATIONALE
A skin biopsy alone will not make the diagnosis The patient is presenting with classic signs and
of systemic mastocytosis, while serum tryptase can symptoms of Whipple’s disease.
also be elevated in cases of acute myeloid leuke- Whipple’s disease is due to Tropheryma whip-
mia (AML), chronic myeloid leukemia (CML) and plei, a gram-positive bacillus, more commonly
myelo- dysplastic syndrome (MDS) so its diagnos- affecting Caucasian men between the ages of
tic utility is limited on its own. An octreotide scan 49-55 years of age exposed to the bacteria, which
is used to detect extra-pancreatic neuroendocrine is typically found in sewage, soil or fecal matter
tumors while a serum C1 esterase levels are useful in rural communities. This involves primarily the
in the work up of angioedema. gastrointestinal tract, but has numerous extrain-
testinal manifestations including neurologic,
REFERENCES cardiovascular, pulmonary, ophthalmologic and
Pardanani A. Systemic mastocytosis in adults: musculoskeletal sequelae. Small bowel biopsies
2012 update on diagnosis, risk stratification and should show characteristic foamy macrophages
management. Am J Hematol. 2012;87(4):401-11. with positive periodic acid-Schiff staining. How-
Horny HP, Metcalf DD, Bennett JM, et al. ever, these histologic findings need to be differ-
Mastocytosis. In: WHO classification of tumors entiated, in the appropriate clinical setting, from
of hematopoietic and lymphoid tissues. Swerdlow histologic findings of M. avium complex disease
SH, Campo E, Harris NL, et al. (Eds). Lyon IARC in which PAS- positive macrophages are also
Press, Lyon 2008; 54-63. found. Staining for acid-fast bacilli should differ-
entiate between the two diseases. Additionally, a
positive Tropheryma whipplei polymerase chain
Question 43 reaction testing can also confirm the diagnosis.
A 50-year-old Caucasian man who is a farmer by Trimethoprim-sulfamethoxazole for at least one
occupation, presents with eight-month history of year is the standard treatment regimen, however,
migratory arthralgias of the large joints followed doxycycline and hydroxychloroquine can be used
by more recent onset of headaches, 20 lb. weight for resistant cases.
522 Digestive Diseases Self-Education Program®
endoscopy reveals a furrowed esophagus with anecdotal success. There are now case reports and
concentric ring like appearance, mild erythema in abstracts reporting the possible efficacy of anti-
the antrum and a nodular, erythematous duode- alpha4-beta7 Integrin (vedolizumab) therapy in
num. The colonoscopy reveals ulcerated, inflamed ameliorating eosinophilic esophagitis. However,
mucosa in the recto-sigmoid. Antral biopsies are more data in large patient groups is necessary be-
negative for Helicobacter pylori and duodenal bi- fore advocating vedolizumab for the management
opsies reveal villous blunting with greater than 50 of eosinophilic gastroenteritis.
eosinophils per high-powered field. Additionally,
biopsies from the esophagus and recto-sigmoid REFERENCES
also demonstrate greater than 30 eosinophils per Sampson HA, Sicherer SH, Birnbaum AH. AGA
high-powered field. technical review on the evaluation of food allergy
in gastrointestinal disorders. Gastroenterology.
Which of the following would you recommend 2001;120(4):1026-40.
next for this patient? Nhu QM, Chiao H, Moawad FJ, Bao F, Konijeti
GG. The Anti-α4β7 Integrin Therapeutic Antibody
A. Cromolyn sodium for Inflammatory Bowel Disease, Vedolizumab,
B. Diphenhydramine Ameliorates Eosinophilic Esophagitis: a Novel
C. Vedolizumab Clinical Observation. Am J Gastroenterol. 2018
D. Elemental diet Aug;113(8):1261-1263.
E. Corticosteroids
sulfasalazine, and azathioprine are capable of autoimmune complex, small vessel vasculitis, with
inducing remission without the need for surgery multiorgan involvement. It is characterized by a
in many patients, whereas TNF-a antagonists and classic tetrad of nonthrombocytopenic palpable
thalidomide have proven useful in resistant and purpura, arthritis or arthralgias, gastrointestinal
complicated cases. and renal involvement. GI involvement may mimic
Crohn’s disease although the biopsies are usually
REFERENCES diagnostic. Most cases are self-limiting although
Hatemi G, Christensen R, Bang D, et al. 2018 oral steroids are indicated in patients with severe
update of the EULAR recommendations for the colicky abdominal pain usually started as predni-
management of Behçet’s syndrome. Ann Rheum sone or methylprednisolone at 1 to 2 mg/kg per day
Dis. 2018 Jun;77(6):808-818. for one to two weeks and then tapering to a stop in
Lopalco G, Rigante D, Venerito V, et al. Up- next one to two weeks. Steroids may prevent major
date on the Medical Management of Gastroin- complications such as gastrointestinal bleeding
testinal Behçet’s Disease. Mediators Inflamm. or intussusception. Immunosuppressive drugs
2017;2017:1460491. (cyclophosphamide, azathioprine, cyclosporine A,
and mycophenolate mofetil) in combination with
high-dose IV pulse steroids are recommended if
Question 48 there is no benefit from steroids alone.
A 26-year-old Caucasian male presented with
fever and sore throat for five days along with ery- REFERENCE
thematous, nonpruritic rash involving the extrem- Sohagia AB, Gunturu SG, Tong TR, Hertan HI.
ities and arthralgias. He subsequently develops Henoch-schonlein purpura-a case report and
right lower quadrant pain, aggravated with meals, review of the literature. Gastroenterol Res Pract.
and associated with watery diarrhea. Labs show 2010;2010:597648.
showed WBC: 14,900/microL, CRP: 12.6 mg/dL.
An MR enterography showed 20 cm of thickened
ileum. An upper endoscopy showed multiple ero- Question 49
sions in the duodenum and antrum while a colo- A 50-year-old woman with short bowel syndrome,
noscopy showed erythema and inflammation in as the result of repeated resections for Crohn’s dis-
the terminal ileum and cecum. Biopsies from both ease, is TPN-dependent and is frequently admit-
areas demonstrate evidence of leukocytoclastic ted to the hospital for line-sepsis. Despite dietary
vasculitis. Skin biopsy also showed leukocytoclas- alterations, acid-reducing medications and use of
tic vasculitis. Which of the following agents would anti-diarrheal medications such as Imodium and
you recommend next? Lomotil, she continues to have admissions for
severe dehydration.
A. Sulfasalazine
B. Prednisone What would be the next best step in the
C. Azathioprine management?
D. Infliximab
E. Vedolizumab A. Eluxadoline
B. Cholestyramine
CORRECT ANSWER: B C. Teduglutide
D. Octreotide
RATIONALE E. Prednisone
This is a case of Henoch-Schonlein purpura which
is a self-limited, systemic, non-granulomatous, CORRECT ANSWER: C
526 Digestive Diseases Self-Education Program®
REFERENCES
Miettinen M, Lasota J. Gastrointestinal stro-
mal tumors. Gastroenterol Clin North Am
2013;42:399-415.
Poveda A, del Muro XG, Lopez-Guerrero JA,
et al. GEIS 2013 guidelines forgastrointestinal
sarcomas (GIST). Cancer Chemother Pharmacol
2014;74:883-98.
528 Digestive Diseases Self-Education Program®